KAT 3

Lakukan tugas rumah & ujian kamu dengan baik sekarang menggunakan Quizwiz!

An engine is supplied with 1 kg of a hydrocarbon fuel with a heat of combustion of 50 MJ/kg. The engine can consume exactly 1 kg of fuel per minute. If this engine can produce 150 kW of power for one minute, the efficiency of the engine is:

20%. Efficiency (Ɛ) is given by: Ɛ = (Work output / Energy input) x 100% Ɛ = (150 kW x 60 s) / (50 MJ/kg x 1 kg) x 100% Ɛ = (150 kJ/s x 60 s) / (50 x 103 kJ) x 100% Ɛ = (3 x 60) / (103) x 100% Ɛ = 180 x 10-3 x 100% Ɛ = 18% (only 1 significant figure is allowed)

If the gauge pressure on a hospital oxygen tank reads 2 atm, what is the total hydrostatic pressure inside the tank?

3 atm Ptotal = P0+ Pgauge where P0 is the atmospheric pressure. Thus, Pgauge= Ptotal - P0.

A well-sealed flask found in a laboratory is said to contain a 3 M solution of an unidentified compound. The solution is measured to be 2.5 L and to have a density of 2 g/mL. The molar mass of this compound is closest to:

667 g/mol. D is correct. As the flask is well-sealed, we can assume that the molarity on the label is correct (and if it weren't, the question would be impossible to answer). (3 mol compound / 1000 mL) × (1 ml / 2 g) = 0.0015 mol / g, or 667 g/mol.

Which of the following molecules or ions is an amphoteric species?

HSO4-

A construction worker is taken to the hospital after an accident involving a nail gun. Upon examination, it is determined that a nail entered his torso at an angle that missed most vital organs and blood vessels. However, the nail lodged in his spine between two thoracic vertebrae and severed many of the frontmost nerves, though rear and middle nerves are unaffected. What symptoms will this injury likely cause for the worker?

He will lose mobility in parts of the body below the injury. Sensory, or afferent, tracts lie toward the rear (dorsal) side of the spinal cord. In contrast, motor, or efferent, tracts are positioned near the front (ventral) and lateral sides. If only the ventral nerves are severed, the worker should retain sensation in body parts below the injury, but will likely suffer problems with the mobility of both voluntary and involuntary muscles.

Which of the choices below accurately define(s) a type of identity? I. Gender identity: a person's perception of being male, female, or other II. Class identity: the socioeconomic group to which a person belongs III. Ethnic identity: the collection of a person's genetic physical features, such as skin tone and hair color IV. Racial identity: the non-physical aspects of a person's culture, such as traditions and language

I and II only A is correct. Both gender identity and class identity are properly defined here.

assist the bipolar cells by processing and organizing visual information. Doing so allows us to discern the edges of objects and detect visual contrasts.

horizontal cells

The total number of possible structural isomers of pentane is:

three. B is correct. Pentane has 3 structural isomers: n-pentane, isopentane, and neopentane.

Where is the F1 fraction of the enzyme located?

Inside the mitochondrial matrix

means adhering to a rule, such as a policy, standard, specification, or law.

compliance

She would start to be able to think logically using abstract ideas.

formal operational stage

What is the percent composition by mass of nitrogen in glutamine (C5H10N2O3)?

19% B is correct. Glutamine has a molar mass of 146 g/mol. The molar mass of nitrogen is 14 g/mol, so the two equivalents of nitrogen present in one mole of glutamine have a mass of 28 g. (28 / 146) × 100 = 19%. This can also be obtained by discerning that 28 is slightly less than one-fifth of 146, so our answer should be slightly less than 20%.

How many protons does Complex IV add to the proton gradient along the inner mitochondrial membrane?

2 Complex IV adds two protons to the intermembrane space per oxygen-reducing reaction. In contrast, Complex III pumps four protons.

Dodeca-2,4,6,8-tetraene contains how many fewer pi bonds than sigma carbon-carbon bonds?

7 B is correct. This twelve-carbon structure includes eleven sigma bonds between carbons, as well as four pi bonds. Remember, double and triple bonds must contain one sigma bond before they "add" bonds made from pi orbitals! Since the question asks for the difference between pi and sigma quantities, the answer is 11 - 4 or 7.

In which situation would a variable-ratio schedule be most advantageous over a variable-interval one?

A dealership owner needs to motivate his salesmen to sell as many cars as possible, as quickly as they can. B is correct. Variable-ratio schedules are the most effective at quickly producing large numbers of responses.

Which cell is LEAST likely to be lysed by a T cell upon presentation of an antigen in the context of MHC?

A dendritic cell presenting a peptide from E. coli Dendritic cells present antigen to helper T cells to activate the adaptive immune system. They are not lysed by the T cells to which they present.

Which of the following enzymes, structures, or processes is NOT required to produce a mature mRNA transcript?

A stop codon Stop codons are involved in the termination of translation, not transcription.

A deficiency in which of the following biological molecules will result in a halt in glycolysis?

ADP The net reaction for glycolysis is as follows: Glucose + 2 NAD+ + 2 ADP --> 2 pyruvate + 2 NADH + 2 ATP As evident here, glucose, NAD+, and ADP are reactants in this process. If one or more of these species are unavailable, glycolysis will be unable to proceed normally.

Which of the following neurotransmitters is NOT a catecholamine?

Acetylcholine Acetylcholine is neither a monoamine nor a catecholamine, unlike the other listed neurotransmitters.

holds that there is a causal connection between neural activation and dream content.

Activation-synthesis theory

Which of these statements is/are true? I. All transmembrane proteins are integral proteins. II. All transmembrane proteins are eukaryotic. III. All integral proteins are transmembrane proteins.

All transmembrane proteins are integral proteins.

The molecule below is treated with CrO3. Which functional group will NOT be present in the product?

An aldehyde Chromium trioxide (CrO3) is a strong oxidizing agent and is reacting with a molecule that contains tertiary, secondary, and primary alcohols. The primary and two secondary -OH groups will be oxidized into a carboxylic acid and two ketones, respectively. The tertiary alcohol will not be oxidized, as that would require the breaking of a C-C bond; it will thus remain as an unaltered alcohol in the product. However, the final product will contain no aldehydes.

Human cells are eukaryotic and can therefore utilize aerobic respiration to create ATP. However, one type of cell can only use glycolysis to obtain its energy. What is the identity of this cell?

An erythrocyte

Which molecule would most likely be soluble in a liquid N2 solution?

CCl4 N2 has nonpolar bonds, and the molecule as a whole is nonpolar. While CCl4 has polar bonds, it is nonpolar overall due to its symmetry. In fact, it is the only nonpolar answer choice listed.

Nitrogen primarily exists in the atmosphere as a diatomic gas. Which of the following is true about this form of nitrogen?

Diatomic nitrogen gas is relatively inert and can be used as the atmosphere in laboratory conditions to prevent unwanted side reactions.

nonpolar amino acids

Glycine, Alanine, Valine, Leucine, Isoleucine, Phenylalanine, Tryptophan, Methionine, Proline

Which of the following theories of personality places a high value on self-actualization?

Humanistic theory The humanistic approach to personality posits that individuals naturally strive for self-actualization, or the fulfillment of one's potential. This goal actually forms the top of Maslow's hierarchy of needs; in other words, humans have a psychological need for self-actualization and can work to achieve it once more basic needs have been met.

Confabulation

Korsakoff's syndrome

HDLs contain relatively less cholesterol than

LDLs

During prenatal development, what is observed in the stage of neurulation?

The ectoderm furrows beneath the notochord, forming the neural groove. A is correct. Neurulation occurs three to four weeks after fertilization and marks the formation of the developing nervous system. It is characterized by the furrowing of the notochord out of the ectoderm to produce the neural groove.

A drug that disrupts hydrogen bonding would most directly affect what level of protein structure?

Secondary Secondary structure includes the formation of alpha helices and beta-pleated sheets. Both of these structures are defined by patterns of hydrogen bonding.

What cell stage of spermatogenesis is the first one to be haploid?

Secondary spermatocyte Secondary spermatocytes are the products of meiosis I, where haploid cells are first formed.

polar amino acids

Serine, Threonine, Cysteine, Tyrosine, Asparagine, Glutamine

If a legally blind individual is unable to perceive visual contrasts and identify the edges of shapes, which structure in his or her visual pathway is most likely damaged?

The horizontal cells

Freud's psychoanalytic theory separates human personality into three main "structures," or components. Which of these choices accurately describes one of these components?

The superego drives the individual to accomplish moral aims, and in doing so, often counteracts the id. The superego is the "moral" component of human personality. It typically drives individuals to avoid behaviors that are morally wrong while striving to fulfill realistic goals.

bind to the enzyme-substrate complex.

Uncompetitive inhibitors

What is the electron configuration for Ni2+?

[Ar]3d8

A patient with persistent difficulties fighting off bacterial infections is found to have a genetic mutation that adversely impacts the speed of cytoskeletal reorganization in immune cells. This condition will most dramatically impact the function of:

Macrophages Macrophages must undergo rapid actin reorganization during the process of phagocytosis. If macrophages cannot use this behavior to engulf bacteria, it reflects a significant impairment in their function.

A mixture of propylamine and butanoyl chloride should create:

N-propylbutanamide. Mixing an acyl halide with an amine will promote a nucleophilic attack, yielding an amide as the product. Here, the two reagents will combine to form N-propylbutanamide, a seven-carbon amide.

Golgi apparatus functions

Packaging of proteins in vesicles for transport to other parts of the cell Targeting of proteins for excretion Production of lysosomes

Which of the following is TRUE about animal communication?

In psychology, there is disagreement as to whether or not animals can actually be taught a language.

Which of these statements must be true for Receptor A, a peptide hormone receptor?

It contains both hydrophobic and hydrophilic regions, and the hormone binds the hydrophilic regions. Peptide hormones are unable to cross the plasma membrane and must instead bind to transmembrane receptors. These receptors must include a hydrophobic region to span the membrane, as well as hydrophilic regions on either terminal to bind with the peptide hormone and intracellular machinery.

posits that physiological arousal happens first, and the brain subsequently interprets this experience as emotion.

James lange

Reaction 1: A ↔ B + C K1 = 1 ΔG1° = 90.2 kJ/mol Reaction 2: A ↔ 2 D + E K2 = 2 ΔG1° = -132.4 kJ/mol Reaction 3: B ↔ A + B K3 = 4 ΔG1° = 12.7 kJ/mol Calculate the equilibrium constant for the reaction 2B ↔ A + 2 D + E.

Keq = K2/(K3K1) D is correct. The overall reaction is equal to (Reaction 2 + Reaction 3 - Reaction 1). Keq of this process as a whole, then, is (K2K3) / K1.

A student must demonstrate a substitution reaction for his laboratory partner. Specifically, he needs to conduct this reaction in such a way as to include the nucleophile in the rate-determining step. Which starting material and nucleophile should the student use, respectively?

A primary alkyl halide and a strong nucleophile C is correct. The question stem describes an SN2 procedure. These processes, formally known as bimolecular nucleophilic substitution reactions, happen in one step and involve a "backside attack" by a strong nucleophile. Since the attacking atom must bind at the same time as the leaving group is removed, these reactions require an unhindered substrate and a powerful nucleophile.

enantiomer of chiral compound

a dash/wedge switches

A nation currently in stage 2 of a demographic transition will experience:

a decrease in mortality rate. Due to advancements in sanitation, medicine, and other fields, mortality rate begins to fall during stage 2. This marker will continue to decrease through stage 4.

In some high-poverty, segregated neighborhoods, people struggle to find work and be socially mobile because they know few others near them who can connect them with job openings. This situation best relates to:

a lack of social capital. C is correct. Social capital is the collective value of social networks and the connections that may arise from those networks. The benefits of social capital include access to job openings, which promote social mobility. The question stem describes a neighborhood that appears to be low in social capital.

when antibodies are created in response to an antigen

active immunity

Generally, a free carboxylic acid is considered more electrophilic than its corresponding:

amide.

part of the semicircular canals, which help us perceive rotational acceleration

ampullae

According to George Herbert Mead, the me is best characterized as:

an internalized collection of others' attitudes regarding one and how one should be, resulting in the socialized identity.

Which of the following processes will be stimulated in conditions of elevated ATP and decreased ADP? I. Glycolysis II. Gluconeogenesis III. The citric acid cycle

gluconeogenesis In times of high ATP, pyruvate kinase is inactivated, which stops glycolysis. Pyruvate is then shuttled into the gluconeogenic pathway. Low levels of ADP activate enzymes that function in gluconeogenesis, further promoting this process. Finally, ADP is required to activate many enzymes in the citric acid cycle; therefore low levels of ADP will inactivate this cycle.

lactate can be used to generate

glucose

Which of the following amino acids exhibits the largest number of resonance structures?

glutamate

The diverse category of eukaryotic non-coding RNA includes:

microRNA, snoRNA, and tRNA, but not mRNA or hnRNA. Non-coding RNA is that which is never translated into a peptide product. Since this definition encompasses a broad range of molecules, it is easier to consider which types of RNA do code for proteins. Both mRNA and hnRNA (another term for pre-mRNA) are coding molecules. Furthermore, microRNA, snoRNA (small nucleolar RNA), and tRNA are not; instead, they serve alternate functions in the cell.

have very high first ionization energies, since an electron must be taken from a complete octet.

noble gases

Space heaters frequently generate heat by exploiting the fact that running a current through resistance expels heat in proportion to the amount of resistance encountered. An HVAC specialist is designing a space heater using a coil of wire made of copper (ρ = 1.68 × 10-8 Ω m). However, tests of the prototype find that it does not generate enough heat. What change to the design would NOT increase the resistance of the system?

Adding a second coil of twice the length in parallel. This would decrease the system's resistance, as shown by the below calculation. But first it must be noted that, since it is twice as long, the second coil has twice the resistance of the first. Hence, 1/Requivalent = 1 / Rcoil1 + 1 / 2Rcoil1 1/Requivalent = 3 / 2Rcoil1 Therefore, Requivalent = ⅔ Rcoil1

The structure of malonic acid, an organic acid with a variety of uses in manufacturing, is shown below. If one mole of sucrose is able to depress the freezing point of a certain volume of water by 3.9°C, what would be the freezing point of the same volume of water when one mole of malonic acid is added?

-11.7°C

Keq is unaffected by

change in product and reactant concentration

There are 100 cats in a population that follows Hardy-Weinberg conditions. 4 cats display a recessive white coat color. How many of the cats are heterozygous?

32 4/100 cats are recessive, which means that q2 = .04 and q = 0.2. P must then be 0.8, and the number of heterozygotes is 2pq x 100.

A strain of E. coli bacteria divides every 6 minutes. A single cell is placed in a beaker and after one hour, it fills the beaker. At what time was the beaker half-full?

54 minutes If every bacterium divides every 6 minutes, then the total amount of bacteria doubles every 6 minutes.

Consider the figure below. If the molecule on the left were to isomerize into a pyranose, the -OH group attached to which carbon would form a hemiacetal with carbon 2?

6 A pyranose is a six-membered ring. For fructose, the leftmost molecule, to isomerize into such a structure, the farthest oxygen atom from the carbonyl must act as a nucleophile and attack. This reaction would yield a ring that includes carbons 2, 3, 4, 5, and 6, as well as the oxygen atom from the attacking -OH.

According to the model articulated by Erik Erikson, which example demonstrates an individual in the industry vs. inferiority stage of development?

A happy, healthy, 8-year-old male leads his soccer team in goals scored and excels in mathematics class. In Erikson's stages of development, the industry vs. inferiority stage (ages 6-12 years) is when a child learns how to use skills and intelligence to make it in the world and get what he or she desires. Unsuccessful resolution of this stage results in a sense of inadequacy and lack of self-esteem. This 8-year-old is apparently in the process of resolving this conflict successfully.

Which of these is NOT a biological monomer?

A nucleic acid A nucleic acid is a polymer composed of multiple nucleotide monomers.

In which of these stages do homologous chromosomes separate into distinct cells? I. Anaphase of meiosis III. Anaphase of meiosis IIIII. Anaphase of mitosis

Anaphase of meiosis I

Which of these molecules is a biological monomer?

Fructose

Which other metabolic processes are connected to the pentose phosphate pathway? I. Gluconeogenesis II. Glycolysis III. Cellular respiration IV. Nucleic acid synthesis

I. Gluconeogenesis II. Glycolysis III. Cellular respiration IV. Nucleic acid synthesis

Which of these choices matches a common diagnostic exam to the sense that it assesses? I. Balancing on one foot with closed eyes - proprioception II. Response to light touching of the fingers - somatosensation III. Touching one's nose or other body parts with closed eyes - vestibular sense

II only Somatosensation is a generalized term for any sensation of touch. Monitoring a patient's response to light touching of his or her fingers is thus a logical test for proper somatosensation in the extremities.

Which of the following modifications to an enzyme-catalyzed reaction would affect its maximum velocity? Select all that apply.

Increasing the enzyme concentration, adding an uncompetitive inhibitor, adding a noncompetitive inhibitor

Nucleophilic additions to aldehydes and ketones occur due to:

The electron-poor nature of the carbon in the C=O bond. Aldehydes and ketones contain C=O bonds with carbons that are electron-poor in nature, which makes them prone to nucleophilic additions.

In an experiment in physiology lab, Jonathan poisons a laboratory animal by inhibiting a particular enzyme. He observes that the animal displays hypotension, a lowered heart rate, and prolonged skeletal muscle contractions. The enzyme targeted was likely:

acetylcholinesterase

Ketone body generation can promote

acidosis

the vestibule is part of the

inner ear

The normal ratio of NADP+ to NADPH is:

low, because NADPH functions as a reducing agent.

basic amino acids

lysine, arginine, histidine

The expression of the lac operon:

must be initiated by the binding of allolactose to a repressor protein. The lac operon is an inducible system. When lactose is absent, a repressor binds to the operator region of the operon, blocking expression. When lactose is present, allolactose binds the repressor instead, allowing expression.

Boiling chips and vacuum distillation, respectively, are used in distillations to:

provide nucleation sites that give the liquid a place to start forming bubbles to prevent superheating; lower the boiling points of the substances to be distilled.

The exchange of allelic gene forms is involved in which of the following processes?

recombination

A magnetic field generated by a medical diagnostic apparatus causes a charge to move through half a circle of radius 2 × 10-3 m. If this field exerts a magnetic force of 5 × 10-2 N, how much work does it perform during this process?

0 J Magnetic fields always exert a force on a moving particle that is perpendicular to both the particle's velocity and the field itself. Remember, the equation for work is W = Fd cos θ . This relationship tells us that only the force exerted parallel to the object's motion contributes to the work, of which there is none here.

A solution of acetic acid has a pH of 6.2. What is the approximate molar concentration of protons in the solution?

8 x 10-7 M A pH of 6 corresponds to an H+ concentration of 10-6 M. Our pH is slightly higher, meaning that our solution is slightly less acidic. 8 x 10-7 is slightly less than 10-6.

Which of these scenarios constitutes a demonstration of the second law of thermodynamics?

A catabolic process may be coupled with an anabolic reaction, yielding a net increase in entropy. The second law of thermodynamics states that a spontaneous reaction or cyclic process must yield a net entropy increase. In other words, the entropy of the universe should be continually rising.

According to the expectancy-value theory, which college course will a student be most motivated to attend?

A course that she thinks she will perform fairly well in and is also strongly recommended for her major B is correct. The expectancy-value theory states that motivation is the product of both "expectancy," or how well we predict a certain activity will go, and "value," or our predicted benefit from completing the activity. Choice B describes a course in which the student predicts moderate success and a high amount of value.

Which of these experimental setups most clearly lacks external validity?

A procedure that found a relationship between infants in specific laboratory conditions, but may or may not apply to infants in typical households External validity refers to how well the experimental results may be generalized to other situations. A study that finds a specific correlation under certain conditions, but likely does not apply to other environments or to "real life," lacks external validity.

Which of these listed conditions would most likely result in nausea, dizziness, and vertigo?

A puncture within the semicircular canal that has caused leakage of the fluid inside

A runner is attempting to practice sprinting while listening to a language-learning podcast. Which type(s) of processing is this individual utilizing?

Listening to the podcast utilizes controlled processing. The type of processing required to complete an action depends on whether the task is simple and familiar or new and advanced. Here, we can infer that the language-learning podcast was an unfamiliar task that would require controlled processing.

The reaction of magnesium sulfide with hydrogen peroxide produces both magnesium sulfate and water. What is the balanced equation for this process, and which element is reduced?

MgS (s) + 4H2O2 (aq) 🡪 MgSO4 (aq) + 4H2O (l); oxygen A is correct. On the reactant side, magnesium and sulfide ions have +2 and -2 charges, respectively. Magnesium maintains this +2 charge on the product side, while sulfur changes its oxidation state to +6. Oxygen, however, initially has an oxidation state of -1, but becomes -2 as a component of water. For this reason, oxygen, not magnesium, is reduced.

Which of the following procedures would be best for extracting methanal from an aqueous solution?

Four successive extractions with 18.75 mL of ether Serial extractions are always more effective than single extractions. Note that the overall volume amounts in these choices are comparable.

The hydrolysis of ATP couples to other reactions that: I. are exergonic. II. are thermodynamically unfavorable. III. are reversible. IV. can involve oxidization.

II only A is correct. The hydrolysis of ATP is an exergonic reaction that generates energy to power unfavorable processes. By coupling the hydrolysis of ATP to another reaction, the net sum of the energies of the two reactions will be exergonic overall.

can either bind to the free enzyme at an allosteric site or bind to the enzyme-substrate complex, but typically show a preference for one state or the other.

Mixed inhibitors

A patient actively infected with human papillomavirus (HPV) was found to have high serum antibody levels. HPV is a non-enveloped DNA virus. Antibodies isolated from this patient would most likely show high affinity for:

L9, a viral capsid protein. The capsid is the only choice that could possibly have been exposed to the surrounding environment before the virus entered the cell. All other choices either are concealed within the viral capsid or would be produced much later in the viral life cycle. Antibodies are involved in extracellular defense, meaning that an effective immune response would need to consist of antibodies specific for an antigen exposed prior to cellular penetration.

Which of these methods constitutes a possible means of separating enantiomers?

None of the above D is correct. Enantiomers, which share physical and chemical properties, cannot be resolved using typical laboratory techniques. These include methods like distillation, recrystallization, and extraction, which are useless until the original species are reacted to form diastereomers. Enantiomers can be separated by reaction with a chiral compound, but glutaraldehyde (as its IUPAC name shows) is achiral.

Consider a reaction of the below molecule with hydrobromic acid. If a halogen atom were to add to this bond in Markovnikov fashion via reaction with HBr, to which labeled carbon would bromine attach?

None of the above answers are accurate. D is correct. While the diagram attempts to hide this, both ends of the double bond are equally substituted. In fact, both have the same substituents: one tert-butyl group and one alkyl group each. For this reason, neither end would be specifically prone to Markovnikov addition, in which the halogen atom adds to the more substituted of the two atoms.

A pharmacologist is testing the vasoconstrictive effects of caffeine on thermoregulation. He theorizes that, in cold weather, caffeine consumption can have a warming effect on the body by promoting the constriction of smooth muscle in arterioles. In what way is this hypothesis incorrect?

None of the above; this theory is perfectly accurate. Arteries, arterioles, veins, and venules contain walls with a layer of smooth muscle. When this muscle constricts in vessels near the body's surface, blood is shunted away from the colder external environment. In this fashion, vasoconstriction helps to keep blood near the body's warmer core.

Several researchers test out a new mass spectrometry apparatus using a known sample of 3-pentanone. They note several peaks, one of which corresponds to an m/z value of 15. How can this reading be interpreted?

One of the fragments has a mass-to-charge ratio of 15 amu/charge, likely a methyl group. Mass spectrometry peaks represent mass-to-charge ratios. More specifically, mass spec involves the fragmentation and ionization of the molecule in question. Typically, this ionization simply entails the removal of one electron, leaving the mass of the fragment virtually unchanged. Here, the molecule was likely broken so as to remove a methyl group, and that group was likely made to carry a charge of +1. As a result, we see an m/z reading of (12 + 1 + 1 + 1) / (1) or 15.

Which period of the periodic table contains the lowest-energy electrons that may be found in a d subshell?

Period 4 The d subshell begins at period 4 and increases in energy as you move down the table. Note that the d orbitals located in period 4 are termed "3d," not "4d," orbitals.

Which of these phenols has the lowest pKa?

Phenol, or a benzene ring attached to a single -OH group, is the most acidic of the compounds listed. All three of the other choices include electron-donating hydrocarbon groups. The presence of these substituents gives extra electron density to the conjugate base of each structure, making it more unstable and less likely to form in the first place. Remember, it is electron-withdrawing substituents that tend to make a structure more acidic.

While rock climbing, Eric notices a rapidly-moving object out of the corner of his eye. He quickly turns his head to see that a bird just flew around to the other side of the cliff. Relieved, Eric decides that this is not a threat. What type of appraisal is Eric performing?

Primary appraisal Cognitive appraisal of stress consists of two stages: primary and secondary. During primary appraisal, the person analyzes the potential stressor along with the surrounding environment to determine whether it represents a threat. Eric does this as he glances toward the moving object, realizes its identity, and decides that it is harmless to him.

includes the hypothesis that we attempt to solve problems while dreaming without being limited by the rules of reality.

Problem-solving dream theory

At what stage of meiosis are ova held at until puberty?

Prophase I All ova are held in this phase until it is their turn to develop in the ovary and ovulate.

Which of the following statements is/are true? I. The carbonyl carbon in an aldehyde is less sterically hindered than that of a ketone. II. The carbonyl carbon in a ketone is more positively charged than in that of an aldehyde. III. Aldehydes are more reactive than ketones. IV. A C=O bond is shorter than a C=C bond.

The carbonyl carbon in an aldehyde is less sterically hindered than that of a ketone, Aldehydes are more reactive than ketones, and A C=O bond is shorter than a C=C bond.

A location on an enzyme where binding of one or more substrate (reactant) molecules occurs, followed by the formation of products.

active site

Pinocytosis, sometimes called "cell drinking," involves the engulfment of extracellular fluid and any associated solutes by one or more vesicles. Pinocytosis is a form of: I. passive transport. II. active transport. III. phagocytosis. IV. endocytosis.

active transport and endocytosis

A pharmaceutical chemist is attempting to convert the molecule below into a primary alcohol for use in a nucleophilic substitution reaction. This could best be accomplished by:

adding the molecule to a dilute solution of LAH. B is correct. The easiest way to change this molecule into a decent nucleophile is to reduce it to a primary alcohol. The oxygen in a relatively unhindered -OH group has two lone pairs, attracting it to electropositive atoms. LAH (or LiAlH4) is a strong reducing agent that can easily convert aldehydes to their corresponding alcohols.

Immediately before sleep, EEG shows a shift to patterns of larger and slower waves called __________ waves.

alpha During periods of relaxation, while a person is still awake, brain waves become slower, increase in amplitude, and become more synchronous. These types of waves are called alpha waves.

is one that is bound to the carbon immediately adjacent to a carbonyl carbon. Only choice B includes a proton (two, actually) that fits this description.

alpha hydrogen

In its phosphorylated state, a receptor tyrosine kinase is stable for quite some time before it is ultimately dephosphorylated and ceases to propagate signals. Signal amplification can be achieved because:

ample time exists for multiple signaling proteins to become activated by the phosphorylated receptor. As long as the phosphorylated tyrosine residues exist to provide a viable binding site, many downstream signaling molecules may bind, unbind, and move on to exert their effects, allowing the process to repeat.

An unsaturated free fatty acid must contain: I. an alkene or alkyne. II. an ester. III. a carboxylic acid.

an alkene or alkyne and a carboxylic acid

The sugars shown above are most specifically described as:

anomers. A is correct. Anomers are a subtype of epimer that differ only at the anomeric carbon. This is true of the two sugars shown, as their anomeric hydroxyl groups point upward and downward, respectively.

Researchers building a synthetic hearing system want to create an auditory pathway as similar as possible to that of humans. The most appropriate order for this pathway would be:

auditory nerve → inferior colliculus → medial geniculate nucleus → auditory cortex. Once the organ of Corti detects physical vibrations and converts them to electrical signals, the auditory nerve delivers those signals to the brain, where they reach the inferior colliculus. Fibers then travel to the medial geniculate nucleus (a region of the thalamus). From here, they move to the auditory cortex for final processing.

All of the following concepts are examples of retrieval cues EXCEPT: A. the serial position effect. Show Explanation B. context-dependent memory. Show Explanation C. automatic processing. Show Explanation D. priming.

automatic processing

a form of unconscious functioning that is sometimes known as "muscle memory," explains why familiar tasks are easier than less familiar ones

automatic processing

a bias that is seen when people make decisions based on available information

availability heuristic

Propyne is exposed to H2 in a solution containing a chunk of solid platinum. If it reacts successfully, the original alkyne will:

be fully reduced to form propane. D is correct. Catalytic hydrogenation is a form of reduction. (Remember, in organic chemistry, reduction tends to involve the net gain of bonds to hydrogen!) Here, an alkyne is reduced to form the corresponding alkane, or singly-bound carbon structure.

As increasing amounts of NaCl are added to water:

boiling point will increase, melting point will decrease, and vapor pressure will decrease. Addition of solutes leads to boiling point elevation, freezing (melting) point depression, and vapor pressure depression.

A ray of ultraviolet light is moving from a medium with a refractive index of n1 to one with an index of n2. With regard to total internal reflection, the critical angle (c') can be expressed using which equation?

c' = sin-1(n2/n1) The critical angle is the angle of incidence above which total internal reflection will occur. In other words, this is the angle that yields an angle of refraction of 90°. (Larger incident angles will yield larger angles of refraction, causing the light to bend back into the original medium.) We can find our answer using Snell's law: n1sinθ1 = n2sinθ2, or n1sinθ1 = n2sin(90°). Since the sine of 90° is 1, this simplifies to n1sinθ1 = n2, which can be rearranged into sinθ1 = n2/n1. Finally, taking the inverse sine of both sides yields θ1 = sin-1(n2/n1). As θ1 represents the critical angle, we have our answer.

a cholesterol deficiency would cause

celll membranes to become too permeable, too rigid, and less secure in the lipid bilayer

positive focal lengths mean

converging systems

amino acids with sulfur

methionine and cysteine

used in the last step of glycolysis.

pyruvate kinase

chair conformations as epimers

the same but C1 has OH or H switched

Effective vaccines can provide lifelong immunity to certain pathogens without ever eliciting symptoms. Of the choices below, the molecule from which the most effective vaccine could be developed is:

whole poliovirus particles subjected to intense heat. D is correct. Using a whole viral particle increases the chance of presenting a viable antigenic domain to which antibodies may effectively bind. Intense heat is frequently used to denature viral enzymes and melt viral genetic material, eliminating its pathogenic potential.

Which of the following statements regarding chemical kinetics is/are always true? I. Reaction rate is concentration-dependent and temperature-independent. II. An exergonic reaction will be spontaneous only if it can overcome the associated activation energy barrier. III. The change in Gibbs free energy of a single-step process will be equal in magnitude for the forward and reverse reactions. IV. Enzymes are used to shift the equilibrium of a reaction to favor the product(s).

An exergonic reaction will be spontaneous only if it can overcome the associated activation energy barrier and The change in Gibbs free energy of a single-step process will be equal in magnitude for the forward and reverse reactions.

A reversible competitive inhibitor, known as physostigmine, is used to treat glaucoma. Based on this information, which of the following statements is true concerning the use of this medication?

An overdose of physostigmine can likely be reversed.

The mitochondrial membrane is not permeable to large or polar ions. Therefore, carnitine, a shuttling protein, is required to transport molecules such as fatty acids across this membrane. A carnitine deficiency would directly impact which of the following metabolic reactions?

The citric acid cycle B is correct. Carnitine is used to transport fatty acids into the mitochondria. Fatty acids are utilized to create acetyl-CoA for the citric acid cycle; without acetyl-CoA, the cycle cannot progress.

An appropriate metabolic response to low blood glucose levels is: I. decreasing the glycolytic rate. II. increasing the rate of glycogenolysis. III. increasing the rate of gluconeogenesis. IV. decreasing ketone production.

decreasing the glycolytic rate, increasing the rate of glycogenolysis, and increasing the rate of gluconeogenesis.

hexokinase occurs in the

cytoplasm

A new element, Z, is discovered and is found to contain 120 protons and 125 neutrons. What is the correct atomic notation for this element?

245/120 Z

An organic chemistry professor wants to demonstrate a substitution reaction to his students. If he needs this process to occur in a single step and invert the stereochemistry of the original reactant, what starting material should he use?

A primary alkyl halide, because it lacks steric hindrance The question stem describes an SN2 procedure. These processes, formally known as bimolecular nucleophilic substitution reactions, happen in one step and involve a "backside attack" by a strong nucleophile. Since the attacking atom must bind at the same time as the leaving group is removed, these reactions require an unhindered substrate.

What will be the major product of an aldol condensation?

An α,β-unsaturated carbonyl Consider an aldol condensation undergone with two aldehydes. First, one aldehyde forms a new C-C bond with another at the carbonyl carbon. That product then undergoes an elimination reaction to form the α,β-unsaturated aldehyde.

A student is trying to identify the group of an unknown molecule. Given that it forms a positive charge in water, which of the following tests would NOT be helpful to run?

Analysis of the solubility of the nitrate salt of the molecule C is correct. All nitrate salts are soluble, so this test would not provide any new information.

When running an SDS-PAGE procedure, researchers often add a reducing agent such as beta-mercaptoethanol (BME). What purpose does this serve?

BME denatures the protein's tertiary structure C is correct. Reducing agents are most often used in such procedures to reduce disulfide bridges into free thiol groups. As these bridges are part of the tertiary structure, C is a correct statement.

bind to enzymes at an allosteric site, regardless of whether the substrate is already bound.

Noncompetitive inhibitors

The reaction below depicts a theoretical chemical process that is being studied by a university aeronautics lab. 5X + 2Y 🡪 3Z What is the rate law for this reaction?

This answer cannot be determined with the information given. D is correct. Unless a reaction is specified as an elementary or single-step process, its rate law can only be determined experimentally.

Of the molecules below, which is optically active?

This compound has two chiral centers and lacks a plane of symmetry. While it may appear to be meso at first glance, mental rotation around the central bond shows that the hydrogen and bromine atoms will align in an asymmetrical, opposite fashion.

Which amino acid will provide the largest amount of energy when fully degraded?

Tyrosine Tyrosine has the most carbon atoms and therefore has the greatest ability to be oxidized and provide energy.

Psoriasis is characterized by the appearance of red or white scaly plaques on an individual's skin. This disorder, which involves the rapid growth of keratinocytes, can have severe effects due to inflammatory and thermoregulatory disruption. A potential treatment for psoriasis might include:

a topical agent that lengthens the interphase portion of the cell cycle. A is correct. The question stem mentions that psoriasis involves overly rapid growth of keratinocytes, or young epidermal cells. In fact, it is this rapid growth that produces thick plaques on the exterior of a sufferer's skin. Choice A involves a medication that lengthens interphase, thus increasing the amount of time between cell divisions and slowing growth. In addition, this medication is topical, or intended for application directly to the skin.

found in the citric acid cycle which takes place in the mitochondria.

citrate synthase

At the (-) end of a microtubule, depolymerization is prevented by:

anchoring of that end to an MTOC. All microtubules originate from microtubule-organizing centers, or MTOCs. These structures anchor the minus end of the microtubule to prevent its depolymerization. They also form the origin of the spindle apparatus during cell division.

An external stimulus triggers the simultaneous experience of physiological arousal and emotion.

cannon bard

In the formation of water from oxygen and hydrogen gas, consider the following bond enthalpies: H-H 436 kJ/mol O=O 499 kJ/mol O-H 463 kJ/mol This reaction can be classified as:

exothermic. The reaction must be written out and balanced to yield 2H2 + O2 → 2H2O. To break the bonds present in the reactants requires 2 (436) + 499 = 1371 kJ/mol, while forming the bonds in the products releases 4 (463) = 1852 kJ/mol. Since more energy was released than put in, this process is exothermic.

Low plasma glucose is known as

hypoglycemia

is an invaginated structure that develops after the morula, and includes cell growth.

gastrula

would not have second ionization energies that are so markedly higher than their first IE values.

halogens

universal emotions

happiness, sadness, contempt, surprise, fear, disgust, anger

The acid with the lower pKa is:

one with more inductive effect

Male patients who have sustained a traumatic lumbar spinal injury often present with a condition known as priapism, a prolonged and persistent erection. Most specifically, this condition results from damage to the:

sympathetic nervous system. B is correct. Erection is triggered by parasympathetic nervous stimulation. Since the parasympathetic and sympathetic systems are in constant opposition, damage to the sympathetic branch leads to unchecked activation of parasympathetic pathways.

fewer electron withdrawing groups makes it a

weaker acid

Which of these solvents would be LEAST useful for a nucleophilic substitution reaction involving 3-bromo-3-methylheptane?

Acetone, due to its lack of an -OH or -NH bond A is correct. Acetone is a polar aprotic solvent, meaning that it cannot donate hydrogen bonds. SN2 reactions proceed best when in the presence of these solvents, but the reaction referenced in the question stem is SN1. (We know this because the original alkyl halide is tertiary.) SN1 reactions are facilitated by the presence of polar protic solvents.

The first step of the oxidative phase of the pentose phosphate pathway is the oxidation of glucose 6-phosphate by glucose 6-phosphate dehydrogenase and NADP+. As expected, NADPH is a strong inhibitor of this reaction. What other substance would you expect to be an inhibitor of the first step of the oxidative phase?

Acetyl-CoA B is correct. NADPH is most often used for anabolic reactions and is a key part of fatty acid synthesis. Acetyl-CoA is combined with malonyl-CoA to form fatty acids. If the concentration of acetyl-CoA is high, the first step of the pentose phosphate pathway will be inhibited, as there will be no need for reactants of fatty acid synthesis.

It was discovered in the 1600s that people exposed to smallpox did not get sick from exposure to the similar cowpox pathogen. What type of immunity is this?

Active immunity

The quantum number that denotes the subshell and the angular momentum of an electron is the:

azimuthal quantum number. This value tells us whether a particular electron is found in an s, p, d, or f subshell. It is also said to describe the shape of a particular orbital.

The temperature at absolute zero is:

-273°C. Absolute zero represents a temperature of 0 K, which is not an option; however, remember that Kelvin temperature is equal to Celsius temperature + 273.

Which of the following molecules is most likely to be analyzed using ultraviolet-visible spectroscopy?

. 1,3,5,7-octatetraene While you do not need to understand the details of UV-Vis for the MCAT, you should know that it is typically used to assess the presence of highly conjugated systems. Conjugation refers to the presence of alternating double bonds within the structure of an organic molecule. From its name alone, we can see that 1,3,5,7-octatetraene has such a structure.

A cube-shaped package is accidently dropped off a ship into the underlying water. The package has a dimension of √2 m as the diagonal length on each face, and floats in the 40°C water with 0.4 m of its height remaining above the surface. What is the specific gravity of the package?

0.6 For test day it will be helpful to know the shortcut that the portion of an object submerged in water is the same as its specific gravity. Since the side length is 1 (using Pythagorean's theorem), 60% of the block is submerged, so its specific gravity is 0.6.

An organ pipe is open at both ends and exactly 1.5 m in length. What is the wavelength of the second harmonic of a wave that resonates in this pipe?

1.5 m B is correct. The equation for wavelength in an open pipe is λ = 2L/n, where n is the number of the relevant harmonic. For the second harmonic, n = 2, which reduces this equation to λ = L.

In a theoretical procedure, one-half of the uncharged nucleons in a chlorine-35 atom are isolated and weighed. Noting that the mass of a proton is 1.672 × 10-27 kg while that of a neutron is 1.675 × 10-27 kg, the total mass of this collected sample will be closest to:

1.5 × 10-26 kg. C is correct. For our purposes, we can treat protons and neutrons as though they have the same mass. However, this question references "uncharged nucleons," meaning neutrons. One 35Cl atom contains 18 neutrons, so half of that number gives us 9. Since the mass of one neutron is 1.67 × 10-27 kg, we can multiply by 9 to get 1.5 × 10-26 kg.

The carrier frequency for cystic fibrosis, an autosomal recessive disorder, in the Ashkenazi Jewish population is 1 out of every 24 individuals. A baby born to a couple from this demographic is tested for this mutation. A family history reveals that the mother's father was afflicted with cystic fibrosis. What is the approximate probability that the child will be a carrier of this disease?

1/50 C is correct. Since the mother's father expresses the autosomal recessive trait, he must have two copies of the mutation and will pass one on to his daughter, making her an obligate carrier. The husband has a 1/24 chance of also being a heterozygote. Even if both parents are carriers, there is only a 50% chance that the child will be one as well (along with as a 25% chance that he will have cystic fibrosis, and a 25% probability that he will be homozygous dominant). To get our answer, multiply (1/24)(1/2) = 1/48, which is closest to choice C.

Which of the following shifts would indicate the presence of a carboxylic acid proton in a 1H-NMR spectrum?

11.0 ppm Carboxylic acid protons appear in the 10-12 ppm range on an NMR spectrum. This represents a relatively downfield (deshielded) position.

A two-pulley system allows a dockworker to apply a 1000 N force over a distance of 10 m, while lifting a crate by a vertical distance of 7.5 m. If the crate is raised at a constant speed and friction and resistance are negligible, what is the weight of the crate?

1300 N A pulley is a simple machine where work (W = force*distance) must be conserved. Here, the work done by the machine must be equal to the work done on the crate. 10,000 J = F * 7.5m, so F = 1300 N.

How many π and σ bonds are present in the molecule below?

2 π and 7 σ Each bond in this molecule includes one σ sigma bond, yielding seven in total. Each double bond (here, two) has one π bond in addition to its σ bond.

1.5 M tartaric acid (a diprotic acid) is titrated with 0.5 M KOH to generate the following titration curve: If the initial volume of tartaric acid was 750 mL, what volume of KOH is required to reach point 2 on the curve?

2.25 L Point 2 is the first equivalence point, meaning that we need to add enough KOH to neutralize the first proton, or one-half of the amount required to fully neutralize tartaric acid. We can use N1V1 = N2V2: (3 N tartaric acid)(0.75 L) = (0.5 M KOH)(x L). Solving for x, we get a volume of 4.5 L. However, remember that this equation gives us the volume of base needed to fully neutralize both protons! To answer this question, we need to divide by two, giving us 2.25 L.

A scientist observes an emitted photon in the form of an infrared ray. He knows only that in its excited state, the principal quantum number of the associated electron was 6. What is the frequency of this ray of light? R = 2.18 × 10-18 J; h = 6.64 × 10-34 Js.

2.75 × 1014 Hz C is correct. To calculate the frequency, we must use the following equation: hf = R(1/n2final - 1/n2initial), in which nfinal = 3 and ninitial = 6. (We know that the final principal quantum number must be 3 because infrared light is emitted, placing this emission as part of the Paschen series.) Plugging in values yields an approximate frequency of f = 2.75 × 1014 Hz.

In the Krebs cycle, NAD+ and FAD react with substrates to form NADH and FADH2. Over the same period of time that nine molecules of FADH2 are produced, how many units of NADH will be synthesized?

27 molecules C is correct. During the Krebs, cycle for every molecule of acetyl-CoA, three units of NADH and one of FADH2 are produced. Therefore, if nine molecules of FADH2 are formed, 27 NADH molecules must be created as well.

What is the ratio of neutrons in radon-202 to that in radon-206?

29:30 The number of neutrons for each isotope can be found by subtracting the atomic number of radon (86) from the relevant mass number. For example, radon-202 contains (202 - 86) or 116 neutrons. This gives us a ratio of 116:120, which simplifies to 29:30.

Which of these alkyl halides, under SN1 conditions, would form the most stable carbocation?

3-bromo-3-methylhexane A is correct. The more alkyl substituents bound to a particular position, the more stable the potential carbocation. Remember, however, that SN1 reactions require the loss of a leaving group before such a charged species can form. Here, our leaving groups are halogen atoms. For this reason, we are looking for an answer in which the halogen is located on a highly substituted carbon. (If you can't tell from the IUPAC names, try drawing out the structures.) Choice A represents a tertiary alkyl halide.

A certain elastic coil has a spring constant of 100 N/m. How much must this spring be stretched in order to store the same amount of energy held by a 50 kg mass at rest one meter above the surface of the Earth?

3.1 m C is correct. The gravitational potential energy of the mass is equal to mgh, or 500 J. This value can be set equal to the elastic potential energy of a spring, U = ½ kx2. This simplifies to 500 J = ½ (100 N/m)(x2), or x2 = 10. The square root of 10 is closest to 3.1 m.

How many carbon atoms are contained within furanose and pyranose rings, respectively?

4, 5 Furanoses are five-membered rings that consist of four carbon and one oxygen atom, while pyranoses are six-membered rings made of five carbon atoms and one oxygen.

A number of astronauts want to use the swinging of a pendulum on a rover to measure the gravitational force on another planet. If a 1 m pendulum with a 0.1 kg mass at the end oscillates with a period of 0.31 s, what is the approximate acceleration due to gravity at the planet's surface, in terms of the gravity on Earth (g)?

40 g For a pendulum, period, length, and gravitational acceleration are related by the equation T = 2π√(L/g). Note that mass is not included in this formula; note also that 0.31 is very similar to 0.1π. Plugging in 0.1π s for period and 1 m for length gives us 0.1π = 2π √(1/x), or 0.1 = 2 √(1/x). 0.05 = √(1/x) 0.0025 = 1/x x = 400 m/s2 This is forty times more than the approximate acceleration on Earth, 10 m/s2. For this reason, the answer is 40 g.

A nurse injects a patient with 1 cm3 of saline solution at a flow of 0.5 cm3/s. The inside diameter of the needle is 0.04 cm. What is the approximate velocity of the saline exiting the needle?

400 cm/s Volume flow rate = A*v and the area of a circle is πr2 so we get 0.5 = π(0.02)2*v and thus v = 0.5/(3*4x10-4)= 0.5/12 x 10-4 = 400 cm/s.

Consider the species depicted below, with stereochemistry denoted in wedge-and-dash form. How many diastereomers of this molecule exist?

6 B is correct. A diastereomer is a stereoisomer with stereochemistry that varies from the original molecule at at least one, but not every, chiral center. The number of stereoisomers of a molecule is equal to 2n, where n denotes its number of stereocenters. Since this compound has three chiral centers, eight stereoisomers exist. However, two are enantiomers, leaving six diastereomers remaining.

The diagram below represents the electronic configuration of a selenium atom. It is implied that all shells other than the outermost shell, which contains 4 electrons, are full. With regard to this atom, what is the effective nuclear charge when n = 4?

6 B is correct. Effective nuclear charge can be found by subtracting the total number of electrons in all shells preceding the one in question from the nuclear charge (the number of protons). Selenium has an atomic number of 34. The atom pictured appears to be oxidized (Se2+), as we would expect the outermost shell to have 6 electrons if the atom were neutral. In other words, the diagram includes 32 electrons. Starting from the first ring, the number of electrons in each shell is, respectively, 2, 8, 18, and 4. (Note that this information corresponds to the atom's electronic configuration.) To calculate the effective nuclear charge when n = 4, then, we only need to find (34 - 28), or 6.

A researcher studying the efficacy of EcoRI decides to cleave a linear double-stranded DNA molecule with the restriction enzyme under non-optimal conditions. The dsDNA is 1000 base pairs long and contains two EcoRI recognition sites at base 12 and base 300. If the digested dsDNA is subjected to size-exclusion chromatography, what is the maximum possible number of elution peaks?

6 C is correct. Under non-optimal conditions, both complete and partial digestion will occur. We will have a total of three different DNA segments as a result of full digestion (let's call them A, B, and C). In addition, three segments can also result from partial digestion (A+B, B+C, and A+B+C). These six segments have different sizes, so they would produce six different elution peaks in SEC.

150 mL of 0.75 M ethylene glycol, one of the primary reagents used in the manufacture of polyester clothing, is diluted with 1.85 L of water. What is the final molarity of the solution?

6.0 × 10-2 M B is correct. Here, we should use M1V1 = M2V2, keeping in mind that V2 must include both the 1.85 L of water and the original 0.15 L of solute. M2 = M1V1 / V2 = (0.15 L × 0.75 M) / 2 L = ~0.06 M.

The energy required to move one mole of electrons through a potential difference of 1.0 V is closest to which value?

6.022 × 1023 eV

An orchid breeder crosses two types of orchid, one that is tall with a petite bloom and one that is short with a large bloom, with the goal of obtaining a tall orchid with a large bloom. The first cross yields all short flowers with petite blooms. What is the expected yield of tall orchids with large blooms if bred from the first generation?

6.25% A is correct. The first cross yields only short flowers with petite blooms, indicating that these are the dominant traits. If you were to cross two of these progeny, you could expect a result that follows normal Mendelian ratios for a dihybrid cross: 9:3:3:1. Here, the "1" represents the 1/16, or 6.25%, that will display both recessive traits.

A blue light with wavelength of 450 nm has what frequency?

6.7 x 1014 Hz For waves, v = fλ, and for light, velocity is represented by c and is equal to 3 x 108 m/s in a vacuum. 450 nm is equal to 450 x 10-9 m, which can also be written as 4.5 x 10-7 m. 3 x 108 m/s = f(4.5 x 10-7 m), so f = 3/4.5 x 1015 = 0.6 x 1015 = 6 x 1014 Hz.

In a particular biological system, 10 mM of substrate is present. If the initial velocity of the associated enzyme-catalyzed reaction is one-eighth of its Vmax, Km is equal to:

70 mM. The Michaelis-Menten equation states the following: Because the initial reaction rate is equal to one-eighth of its maximum value, we can substitute (1/8)Vmax for V0. (1/8)Vmax = (Vmax[S]) / (Km + [S]), which simplifies to 1/8 = [S] / (Km + [S]). Solving for Km shows that it is equal to 7[S], or 70 mM.

A man with blonde hair, blue eyes, and fair skin has a daughter with a woman with dark hair, brown eyes, and dark skin. (Assume that hair, eye, and skin color are each dependent on a single gene.) Based on their differences in these three traits, how many possible phenotypic combinations are possible for their child?

8 B is correct. We are given no information regarding dominance or recessivity, so we must take all potential allelic combinations into account. Three genes are present, with two possibilities each; thus, we can use 2n to find that there are eight different combinations.

In the following reaction, if 260 g of zinc is allowed to react with hydrochloric acid in excess, how much hydrogen gas should be produced? Zn + 2HCl → ZnCl2 + H2

8 g The reaction is already balanced. The molecular weight of zinc is approximately 65 g/mol, so 4 moles of Zn are originally present. Complete reaction would produce 4 moles of H2, or 8 g.

A far-off star is moving away from the earth at a speed of 1.0 × 108 m/s. The star appears to be blue/green when viewed through a telescope. If blue/green light has a frequency of 6.0 × 1014 Hz, what is the frequency of the light the star is actually emitting?

8.0 × 1014 Hz The Doppler effect applies to light as well as sound waves, with an associated equation of f' = f(V ± Vo / V ± Vs). If the star is moving away from Earth, the perceived frequency of light will be lower than the actual frequency ("red shift"), ruling out answers A and B. To decide between the remaining choices, remember that you are given the perceived frequency, not the original frequency. Plugging values into the equation gives us 6.0 × 1014 Hz = f [3.0 × 108 / (3.0 × 108 + 1.0 × 108)]. This simplifies to 6.0 × 1014 = 0.75f. Solving for the original frequency yields 8.0 × 1014 Hz.

A cylindrical tank filled with ethanol is situated as shown below, held above the ground by metal pylons. Note that the top of the tank is entirely open to the air above. If ethanol (specific gravity = 0.79) exits the tank through the small pipe near the bottom at 40 m/s, what is the distance marked "x"?

80 m B is correct. Since both ends of this system are subject to the same pressure (atmospheric pressure, to be specific), Bernoulli's equation can be simplified to velocity = √2gh. We can plug in our known value for velocity to obtain (40 m/s) = √(2)(10 m/s2)(h), or 1600 m/s = 20h. Thus, the value of "h" (which is equal to the height difference between the starting and ending points for the fluid) is 80 m. The specific gravity of ethanol is irrelevant.

A 10-kg sled rests on a 30° ramp with a coefficient of static friction of 0.5. An upward force is applied to the sled, parallel to the slope of the ramp, in incremental values until the sled begins to accelerate up the ramp. Approximately what minimum force is required to perform this action?

95 N C is correct. The block will begin to accelerate once all opposing forces have been overcome. Here, these forces are static friction (which opposes motion) and gravity (which opposes upward movement). Since the applied force needs only to minimally surpass equilibrium, we can set our net force equation up as follows: Fnet = Fapplied - (Fg + Fs) = 0 N. This simplifies to Fapplied = Fg + Fs. Now, we need to find the component of the gravitational force that is actually acting on the block, along with the normal force. To do so, we can break our values into components.

Which of the following are true of eukaryotic mRNA before it leaves the nucleus? I. A 3' cap is added. II. A 5' cap is added. III. Exons are spliced out of the sequence. IV. Introns are spliced out of the sequence.

A 5' cap is added and Introns are spliced out of the sequence.

Which of the following diseases would be LEAST logistically suited for a cohort study?

A rare genetic disease A cohort study begins with a set group of healthy individuals, then tracks changes in their health (or other variables) over time with the end goal of determining risk factors for specific diseases. This protocol would not work properly with the disease mentioned here; since those with a genetic condition have always had it, no healthy individuals are at risk for this disease. If a cohort were established, it would either consist of healthy individuals who never acquired the disease or sufferers who had it from the beginning; this would yield no new information.

The diagram below depicts the reverse of a glycolytic process. This reaction is catalyzed by which type of enzyme?

An aldolase The reaction is part of the glycolysis/gluconeogenesis pathway. As shown, it involves condensation of two three-carbon structures into a larger aldol. This reaction is mediated by an aldolase enzyme.

In a positive repressible operon, an activator protein binds to the DNA and the genes are actively transcribed. In such a case, the addition of which of the following would stop transcription?

An inhibitor C is correct. In a positive repressible operon, an inhibitor can attach to the activator, blocking the activator from binding to the DNA. This results in the downregulation of transcription.

A scientist looking to isolate membrane-bound organelles via centrifugation would NOT want to study:

Archaea. Archaea, once known as archaebacteria, are classified as prokaryotes. As such, they do not possess membranous organelles.

Cefoperazone is a β-lactam antibiotic used to treat infections by resistant strains of Pseudomonas. In what way is the structure of this compound similar to that of α-acetolactone?

Both contain at least one cyclic ring.

At which time are estrogen levels lowest for a woman?

During menstruation Estrogen and progesterone help to build the uterine lining. During menstruation, estrogen and progesterone levels are low.

Jason recently moved to Turkey from Australia. He is having a difficult time adjusting, and he often compares the culture in Turkey to what he was previously accustomed to, believing that his Australian culture is superior. Which of these terms best explains Jason's attitudes?

Ethnocentrism A is correct. Jason is displaying an ethnocentric ideology; in other words, he is judging another culture by the standards of his own, and generally believes that his own Australian culture is superior to others.

Which enzyme is exclusive to gluconeogenesis?

Fructose 1,6-bisphosphatase B is correct. Gluconeogenesis must overcome the three irreversible glycolytic steps. The enzymes used to achieve this are glucose 6-phosphatase, fructose 1,6-bisphosphatase, PEP carboxykinase and pyruvate carboxylase.

Cultural capital can include: I. educational degrees or certifications. II. networking connections gained from membership in a fraternity. III. public speaking ability. IV. retirement and college funds.

I and III only Cultural capital includes personal qualities or assets that enhance the social mobility of the individual in their possession. Education (whether in the form of a degree, certificate, or relevant experience) is a classic example of this form of capital (I). Public speaking ability is also a personal quality that can make a person more socially successful. Other examples of cultural capital include attractiveness, style of dress, and sense of humor (III).

Consider the following "bicycle," here an organic molecule composed of two fused rings. Following treatment of this species with aqueous tosylic acid, the resulting product would be accurately described as: I. an aldehyde. II. a ketone. III. a triol. IV. a diol.

I and III only Hydrolysis of the acetal will yield a free aldehyde and two alcohols (I). Since the compound already contained an alcohol group that is untouched by the acid hydrolysis, the final product is considered to be a triol (III).

Molecules that regulate blood calcium include: I. dihydroxycholecalciferol. II. calcitonin. III. parathyroid hormone.

I, II, and III This answer choice is correct. Dihydroxycholecalciferol is the active form of vitamin D, which works in the kidney to increase calcium reabsorption (I). Calcitonin decreases serum calcium levels (II) ; this action is opposed by parathyroid hormone (PTH) (III).

Which of the following will NOT convert CH3CH2OH to CH3COH? I. PCC II. K2Cr2O7 III. O3

II and III only K2Cr2O7 and O3 are both strong oxidizing agents and would convert the described alcohol to its associated carboxylic acid, adding two bonds to oxygen.

Which of the following are naturally occurring R-amino acids? I. Glycine II. Cysteine III. Aspartic acid IV. Proline

II only B is correct. Glycine is the only amino acid that is achiral. All other common residues have S configurations, with the exception of cysteine, the only common amino acid designated as R.

Which of the following statements about boiling point is accurate?

If the vapor pressure of water is 38 mmHg in a near-vacuum environment with a Patm of 0.05 atm, it will boil.

A light signal has a frequency of 7.2 x 1012 Hz. The position of this signal on the EM spectrum indicates that it must be which form of electromagnetic radiation?

Infrared radiation Visible light (ROYGBIV, 1015 Hz) is surrounded by infrared radiation (lower f, higher λ) on the left and UV light (higher f, lower λ) on the right. The range of infrared radiation is 1012-1014 Hz.

Two nucleotide triphosphates bond together and release a small molecule. In an aqueous solution, what is likely to happen to this molecule?

It will degrade into an inorganic phosphate. Two high-energy molecules like nucleotide triphosphates will release a molecule of pyrophosphate. This species will likely be hydrolyzed into inorganic phosphate, or Pi.

Which of the following cytoskeletal proteins contributes the most to the skin's resistance to stretching and tearing?

Keratin Keratin fibers extend across epithelial cells (in the skin, for example) to link adjacent cells via structures called desmosomes. As a type of intermediate filament, keratin possesses high tensile strength; as such, it can form a network of fibers that distribute mechanical stress among the cells of an epithelial layer. Keratin is therefore the cytoskeletal protein that contributes most to the skin's stretching ability.

Individuals that participate in the Atkins diet often report sweet-smelling breath and/or sweat, in addition to a metallic taste in the mouth. Based on this information, the Atkins diet most likely involves the metabolism of which molecule(s)?

Ketone bodies B is correct. The Atkins diet induces a state of physiological ketosis, which forces the body to utilize stores of fat for energy due to extremely low glucose intake. The sweet-smelling breath and perspiration come from the metabolism of ketone bodies, the primary metabolic molecule present during ketosis. The specific molecule responsible for the smell is acetone, which is both excreted in urine and exhaled.

A neuron is observed to display impaired neurotransmitter secretion. Upon further study, this is found to be caused by a defect in the vesicular transport of newly synthesized NTs from the cell body down the axon. A problem with which class of cytoskeletal accessory proteins is most likely at fault?

Kinesins, as they travel toward the + end of microtubules Kinesins attach to vesicles and travel toward the + end of microtubules, which extend toward the cell membrane from microtubule organizing centers (like the centriole) that are more central to the cell. Note that the + end tends to be the location at which dimers rapidly add to the existing structure.

During starvation, which tissue uses amino acids to maintain blood glucose levels?

Liver Gluconeogenesis takes place in the liver. This process can use many precursors of pyruvate, such as amino acids, to form glucose.

Hypersensitivity to light touch may indicate a disorder that involves:

Meissner's corpuscles.

What is a possible explanation for the high stability of acid anhydrides in comparison to esters?

None of the above; acid anhydrides are actually less stable than esters. Remember, stability is the opposite of reactivity. Thus, this question is asking why anhydrides are less reactive than esters, which is simply not true. In fact, after acyl halides, anhydrides are the most reactive of the carboxylic acid derivatives. This trend relates to the anhydride's leaving group, which is a carboxylate ion. This species displays resonance between the two oxygen atoms. Since this makes the group stable in solution, it can easily leave and the anhydride can participate in a variety of reactions.

Which solvent would be most suitable for the recrystallization of acetylsalicyclic acid (aspirin) immediately after its synthesis? Note that the known melting point for acetylsalicyclic acid is 136°C and the sample collected is displaying a melting point of 135.9°C.

None of the above; recrystallization either would not work or is not necessary for the purification of this sample. Solvents 1, 2, and 3 will not work here. The ideal solvent would be one in which acetylsalicylic acid is highly soluble in warm temperatures and insoluble in cold ones. However, note the melting point data, which shows that our sample has a narrow melting point within a tenth of a degree of the expected value. This implies that our obtained compound is very pure already and likely does not need to be recrystallized.

Americium-241 is commonly used in smoke detectors. The decay of this element produces alpha particles, which are caught by a detector. If smoke is present, it will prevent these particles from reaching the detector, triggering an alarm. As one americium-241 nucleus decays, it also produces:

Np237. Alpha decay causes an element to emit two protons and two neutrons, also known as a helium nucleus. As a result, americium will lose two protons to become the element Np. Its atomic mass will decrease by four due to the loss of four total nucleons.

Recent studies have discovered significant changes to bacterial flora in the gut that correlate with severe obesity. These changes affect the processing of nutrients in the intestines and may partially prevent obese individuals from losing weight. This discovery, if widely accepted, might have what implications on society's view of obesity?

Obesity might become increasingly medicalized. Medicalization is the process by which a condition is viewed as a disease (and thus deserving of help from the medical profession), rather than as something else, such as a habit, personality trait, character flaw, or lifestyle choice. Here, if obesity begins to be perceived as biological rather than a simple result of overeating, it would likely result in medicalization.

Which of these situations best exemplifies a proactive social movement?

Over ten thousand high school students across the nation who campaign for gender-neutral bathrooms in their institutions A proactive social movement is one that aims to enact change or "make progress." In other words, its members organize, whether formally or informally, an effort to change their society. In contrast, a reactive social movement resists this change, and its members desire to return to or keep the status quo. Here, the group of high school students is aiming to enact a change that has not yet been seen in a large scale, so they constitute a proactive movement.

what is required to develop a DNA fingerprint

PCR, restriction enzymes, and electrophoresis

A man reports to the ER with a knife wound. The surgeon on call notes that the cut goes down to a layer of loose connective tissue, but stops before reaching the hypodermis. The deepest part of the cut extends into which layer of skin?

Papillary layer The papillary layer is where the dermis begins with loose connective tissue. Both stratums are in the epidermis, and the subcutaneous layer is beyond the dermis.

Sonic hedgehog (SHH) is a developmental signaling hormone that is released from the notochord to create a concentration gradient that spans the dorsoventral axis. Higher concentrations of SHH are found ventrally, near the notochord, while lower concentrations are found dorsally. Based on this information, what type of signaling molecule is SHH?

Paracrine SHH is released during development when the notochord is still present. The described concentration gradient implies that it is released to neighboring cells.

The Internet provides never-before-seen access to information and opposing viewpoints. However, psychologists find that, in spite of this, people have become even more steadfast in their beliefs. How can this apparent contradiction be explained?

People are falling victim to the confirmation bias, causing them to overvalue information that agrees with their own viewpoints.

Both Person A and Person B must perform an athletic feat in front of a large crowd. Person A performs better than he typically does when practicing alone, while Person B falls on his face and embarrasses himself. With regard to social psychology, what may explain these events?

Person A had practiced the feat often and found it fairly simple, while Person B was less familiar with it and perceived it as difficult. The experiences of both of these individuals can be explained by social facilitation. According to this idea, individuals tend to perform better in front of groups when the task in question is fairly simple or familiar, but perform worse when they are less comfortable with what they are doing.

Reactions are favorable when they increase the overall entropy of a system. Which class of lipids will have the most positive entropy when dissolved in water? Assume that the water itself does not react with any of these choices to form new molecules.

Phospholipids C is correct. This question is asking for the molecule that interacts most favorably with water. In general, lipids are hydrophobic, meaning that they would form a low-entropy mixture in water; however, phospholipids are amphipathic due to their charged phosphate head and hydrocarbon tails. In other words, they are partially polar. As a result, compared to the three other molecules, phospholipids would have the most favorable (or "least unfavorable") interactions in aqueous solution.

Which of the following must be stored in an inert gas in order to prevent dangerous exothermic reactions with the atmosphere?

Potassium Potassium has a small first ionization energy, meaning that it desperately wants to lose one electron to reach the ideal noble gas configuration. As a result, it tends to react violently to yield K+.

Which of the following compounds contains a weak Brønsted-Lowry base?

Potassium acetate The acetate anion (C2H3O2-) is the conjugate base of acetic acid, a weak acid. Since acetate readily gains a proton to form acetic acid in solution, it is able to act as a Brønsted-Lowry base. Note that it is not strongly basic; in contrast, it is stabilized by resonance, making it somewhat weaker.

The latter half of the menstrual cycle, shown below, is characterized by endometrial proliferation and vascularization. During the final days of the cycle, assuming implantation has not occurred, uterine contractions facilitate expulsion of the endometrium Which of the following hormones, if administered to a pregnant female, would be most effective in preventing pre-term birth?

Progesterone

Which of these compounds contains the most acidic alpha proton?

Propanal When dealing with aldehydes and ketones, remember that the alpha protons (those bound to the carbon atom(s) adjacent to the carbonyl carbon) are especially acidic. Of the choices, propanal includes the alpha proton with the conjugate that is least destabilized by nearby electron-donating groups.

One cell line commonly used in biological research is the HeLa strain, which is derived from samples of a cancerous tumor from a woman named Henrietta Lacks. These cells are functionally immortal, in that they can divide indefinitely without dying, and were key to developing advances such as the polio vaccine. However, Mrs. Lacks was not informed that her cancer cells had been preserved before her death, and her family was surprised to find her medical records published without their knowledge years later. The case of Henrietta Lacks infringed on which principles of research ethics? I. Minimizing the risk of harm to subjects II. Protecting the privacy and confidentiality of subjects III. Obtaining informed consent

Protecting the privacy and confidentiality of subjects and Obtaining informed consent

Ion-exchange chromatography with a negatively-charged stationary phase is used to separate two polypeptides, A and B. Which of the following amino acids is LEAST likely to predominate in the macromolecule that elutes first?

R Ion-exchange chromatography separates compounds based on net charge. In this case, the stationary phase is negative, meaning that positive compounds will be retained in the column. In other words, these species will elute more slowly because they will interact more with the stationary phase. Therefore, we would expect the polypeptide that elutes first to contain fewer positively-charged residues than the one that elutes second. Arginine (R) is positively charged at physiological pH, so it is unlikely to be common in the polypeptide referenced.

Graff is selecting between two different pre-cut copper (ρ = 17.1 nΩ-m) wires. Wire A is 9 mm long, and has a cross-sectional area of 1 mm2. Wire B is 4.5 mm long, with a cross-sectional area of 2 mm2. How does the resistance of Wire B compare to Wire A?

RB = ¼ RA R = ρL/A. Wire B has half the length and double the area, so it will have ¼ the resistance.

In eukaryotes, the termination mechanism for protein synthesis requires which of these?

Release factors Release factors are proteins that recognize the stop codons and terminate protein synthesis. Note that AUG is the start codon; the three stop codons are UGA, UAA, and UAG.

A new species of desert iguana is discovered to possess structurally similar ears to those of humans. However, these reptiles do not have necks and never evolved to walk in curved or circular paths. As animals that only have the capacity to travel in straight lines, which structure do these iguanas most likely lack in comparison to humans?

Semicircular canals

Of the pairings below, which properly matches the type of learning with the broader category in which it is placed?

Sensitization - a type of nonassociative learning Sensitization is defined as an increase in the magnitude of a response after repeated exposures to the same stimulus. It is one of the two major types of nonassociative learning.

The number of unique human antibodies dramatically exceeds the number of human genes available to code for them. Which of the following mechanisms accounts for this fact?

Somatic (VDJ) recombination C is correct. VDJ recombination is the process by which genes for immunoglobulins are rearranged in developing lymphocytes to create novel gene products.

A medical student is asked to plate a certain type of bacteria and leave the plates overnight to allow for growth. The student places the bacteria on enriched media (containing organic sugars), then moves them into an incubator at 37 °C overnight with a loose seal. Upon his return, the student notes no new growth on the bacterial plates. Which of the following could have resulted in this observation?

The bacteria are phototrophs and require sunlight to obtain electrons. If the bacteria are phototrophs, they need exposure to sunlight to grow. Their placement in an incubator caused them to be devoid of sunlight.

Water is a rare substance in that the solid is less dense than the liquid at the freezing point, resulting in a solid form that floats on top of the liquid. Which of the following best explains this phenomenon?

The bent structure of the water molecule and ratio of covalently-bonded hydrogens to lone pairs of electrons on the oxygen atom maximizes the hydrogen bonding that occurs in the solid phase, producing a hexagonal structure with large empty spaces.

As swallowed food passes from the esophagus into the stomach, it must pass through which structure?

The cardiac sphincter The cardiac sphincter, named for its proximity to the heart, connects the esophagus to the stomach.

An electron orbiting a helium nucleus transitions from the n = 1 energy level to the n = 2 energy level. How does the electrical potential energy compare between these energy levels?

The electrical potential energy is greater at n = 2. Electric potential energy (measured in joules) is calculated using PE = Fd = (kQq/r2)r= kQq/r, and the signs of the charges matter! At n=1, the r is smaller, so the fraction becomes more negative (e.g. -4/2 > -4/1), which is smaller.

reciprocal of the y-intercept

VMax

According to the defensive attribution hypothesis, when is an individual most likely to blame the victim of an accident or crime?

When the accident seems entirely random and results in severe injury or death The defensive attribution hypothesis stems from the human fear that terrible consequences might happen to us simply due to chance. To avoid being scared by this idea, we tend to blame the victims of an accident or to draw distinctions between ourselves and them (as in, "She was behaving in a way that I never would, so this could never happen to me"). This victim-blaming is most prevalent when the incident has severe consequences and when we perceive ourselves as similar to the victim.

Social loafing explains why:

a government auditor is less thorough when assigned to the same project as other auditors.

Urbanization is the large-scale movement of people from rural areas to urban regions. Direct effects of urbanization include: I. a rise in the prices of goods and services in cities.II. the occurrence of urban "heat islands."III. a decrease in job opportunities in rural parts of the country.

a rise in the prices of goods and services in cities and the occurrence of urban "heat islands."

is a hollow ball of cells that develops after the morula.

blastula

As increasing amounts of NaCl are added to water:

boiling point will increase, melting point will decrease, and vapor pressure will decrease. Colligative properties include boiling point elevation, freezing (melting) point depression, and vapor pressure depression. As more solute is added, these properties will be seen with an increasing effect.

According to Charles' Law, all of the following statements are correct EXCEPT:

both A and C. Charles's law states that the volume of a gas is directly proportional to its temperature, assuming that other factors are held constant. Here, we are looking for statements that are either factually incorrect or do not relate to this law. Statement C is true, but it relates to Boyle's, not Charles', Law.

will increase the initial rate of the reaction, but will not increase the Vmax.

allosteric activator

In the presence of 2,3-biphosphoglyeric acid (2,3-BPG), the hemoglobin-O2 saturation curve is shifted to the right. This is an instance of:

allosteric regulation.

staphylo-

clustered

Matching one's attitudes, behaviors and beliefs to societal norms is known as:

conformity

converging lenses are

convex

On day 47, the professor leaves for his sabbatical, and the replacement teacher does not reward any students for hand-raising. This change results in:

extinction

The products of the pentose phosphate pathway are linked to

glycolysis, fatty acid synthesis, and glutathione reduction.

During anaphase I of meiosis:

homologous chromosomes separate, resulting in haploid cells. Homologous chromatids separate during anaphase of meiosis I, producing cells that are technically haploid. In contrast, sister chromatids separate during anaphase of meiosis II.

Chromosome instability is a phenomenon in which parts or even entire chromosomes are deleted or duplicated. This can result in an abnormal number of chromosomes in daughter cells, a condition known as aneuploidy. All of the following may cause chromosome instability EXCEPT:

homologous recombination

high plasma glucose is known as

hyperglycemia

the incus, malleus, and stapes are in the

middle ear

A single, healthy individual may possess:

multiple identities. B is correct. Identity can be defined as our relationship within categories or groups of people. For example, one person can have a racial or ethnic identity ("I am an African-American"), a gender identity ("I am male"), and a class identity ("I am a member of the middle class"), plus more, all at once.

Of the isomers below, that with the highest boiling point is:

octane. Boiling point is affected most by intermolecular forces and molecular weight. However, all four of these choices experience similar intermolecular forces (London dispersion forces only), and all weigh the same, as they are all isomers of C8H18. For this reason, we need to consider the extent to which each hydrocarbon chain is branched. The less branching a molecule displays — in other words, the more closely it resembles a straight chain — the more easily it will tend to "stack" atop like molecules and the higher its boiling point will be. Of the isomers listed, octane has the least branching and will have the highest boiling point.

Sound waves have a relationship between oscillation and wave propagation that is:

parallel. A is correct. Sound waves are known as longitudinal waves, because the oscillation of the particles of the wave (matter vibrating) is in the same direction or parallel, as the propagation of the wave itself. That is, the wave propagation is parallel to the oscillation of the medium.

is a particular way of talking to children, which involves slowing down speech, speaking in a higher pitch and giving pauses for them to respond.

parentese

would be injecting someone with antibodies produced by someone else

passive immunity

Light from a star passes through a lens, as shown in the following diagram. Note that both A and B are located one focal length from the lens. In the situation above, the image will be:

real and minimized. In this situation, the image is produced on the opposite side of the lens from the object. For lenses, this constitutes a real image. Next, to assess magnification, we should use the equation m = -di/do. Since the star can be assumed to be much farther from the lens than the image is, magnification must be a value close to zero. The image must be smaller than the object, making C the best choice.

Cystine is cleaved into two amino acids residues by:

reduction

Production of the enzymes found within peroxisomes

rough ER

is the only one of the three that involves conscious thought regarding the stimulus. Specifically, it states that physiological arousal is followed by cognitive appraisal of the situation, and the combination of these two factors creates the experience of a specific emotion.

schacter singer

neutral amino acids

serine, threonine, asparagine, glutamine

All of the below features are shared by individuals in REM sleep and those who are physiologically awake EXCEPT:

similar EOG readings. Electrooculograms, or EOGs, measure eye movement. Rapid eye movement is the most classic characteristic of REM sleep, but is not demonstrated by people who are physiologically awake.

The bacterial potassium channel operates in a similar manner to the eukaryotic K+ channel that is involved in action potential propagation. Like neurons, bacteria are sensitive to subtle osmotic changes, and the potassium ions that pass through this channel are not hydrated. The most plausible explanation for the exclusion of sodium ions from this bacterial channel is that:

sodium ions have a smaller atomic radius, allowing them to maintain a more tightly-coordinated sphere of hydration.

Rf values are highest near the

solvent front

Exhibition of moderate activity on an EMG is typical of which of these listed sleep stages?

stage 1, 2, and 3

A patient has suffered a head injury that is causing severe hearing loss. Sound appears to travel normally through his external auditory canal and tympanic membrane, but is not reaching the oval window of the organ of Corti. The damaged structure in this case is most likely the:

stapes. B is correct. The stapes, one of the three bones of the middle ear, precedes the organ of Corti. Its role is to transmit vibrations from the middle to the inner ear through the oval window of the cochlea. Damage to the patient's stapes would result in the symptoms described.

In an isolated cave, two bat species are discovered and found to share a distant ancestor. The ears of the two species seem to have adapted to find different sizes of prey using echolocation. When crossed, individuals of the distinct species are unable to produce a virile litter. This situation exemplifies:

sympatric speciation. Sympatric speciation is that which occurs without a physical barrier. A population that diverges into two separate species in a single cave certainly falls under this form of speciation.

A caveman sees a tiger and experiences fear at the exact instant when his heart begins to race. This situation relates best to:

the Cannon-Bard theory of emotion.

The preferred ion configuration of many elements on the periodic table is determined by:

the electron configuration of the nearest noble gas; elements will gain or lose electrons until they have the same valence electron configuration as this noble gas.

The tertiary structure of myoglobin is virtually identical to that of a single hemoglobin subunit. Hemoglobin and myoglobin differ in that:

the two molecules have different tissue distribution.

Circular dichroism (CD) spectroscopy is a versatile spectroscopic technique that is routinely employed to probe biomolecular secondary structure. If CD spectroscopy were used to follow the progress of a polymerase chain reaction, the resulting spectra would most likely:

vary depending on the particular phase of the thermal cycle, because heating disrupts the native double helix. C is correct. During the cooler phases of PCR, the two DNA strands are able to remain coordinated with each other via hydrogen bonding. As heat enters the system, the complementary strands dissociate and lose their characteristic helical secondary structure. This change would most likely be observable when using CD spectroscopy.

A nonviscous fluid is traveling through a network of tubes at equal height, but differing in their radii. Which graph best depicts the relationship between the hydrostatic pressure in a vessel and the square of the velocity through which fluid moves at that specific point?

velocity decreases as pressure increases

is the original cell formed by the fertilization of an ovum by a sperm cell.

zygote

Cellulose differs from starch in that cellulose includes a(n):

β-1,4 acetal linkage. B is correct. Cellulose contains β-1,4 acetal linkages between glucose molecules. Humans lack an enzyme to cleave these bonds, which explains why we cannot digest cellulose. Starch contains α-1,4 acetal linkages, which we can break down.

Based on its structure, what method of signaling is most probable?

Binding to intracellular hormone receptors in the cytosol that initiate gene transcription and regulation From the given structure, this is a steroid hormone, as it is relatively small and nonpolar. It will thus be able to traverse the plasma membrane and bind to intracellular receptors. Steroid hormones are long-acting, so we would expect this molecule to influence long-term gene expression rather that transient cAMP cascades or calcium influx.

Glycolysis, among other vital functions, serves to help regulate blood sugar. One hour after a meal, what is the expected saturation status of hexokinase and glucokinase with glucose?

Both enzymes will be saturated. Directly after a meal, sugars are absorbed into the blood, resulting in high plasma glucose levels. Hexokinase has a much higher affinity for glucose than its isozyme, glucokinase. However, at high glucose concentrations, both enzymes should be saturated with glucose.

What is the net charge of the peptide Arg-Ala-Phe-Leu at pH 8?

+1 The net charge at pH 8 is the sum of the side chain charges. There is one basic residue, Arg, providing a net +1 charge.

The formula ΔTb = Kbmi can be used to calculate boiling point elevation, a colligative property. Which of the following are possible units for Kb?

(°C ∙ kg) / mol

Which of these ΔG° values most likely corresponds to a reversible step of glycolysis?

-1.7 kJ/mol Glycolysis is a catabolic process with a negative net change in free energy. Reversible glycolytic steps refer to those reactions that do not serve as committed steps because they do not represent a large free energy change. In other words, the forward and reverse reactions of these processes have similar ΔG values, so they lack a strong tendency to move in one direction or the other. Thus, it is the choice with the ΔG° that is closest to zero that will be the most easily reversed.

The structure of malonic acid, an organic acid with a variety of uses in manufacturing, is shown below. If one mole of sucrose is able to depress the freezing point of a certain volume of water by 3.9°C, what would be the freezing point of the same volume of water when one mole of malonic acid is added?

-11.7°C Freezing point depression is a colligative property, meaning that it depends on the number of particles in solution. Sucrose is organic and does not dissociate in water, but carboxylic acids will lose a proton at a pH of 7. Malonic acid, then, dissociates into three particles: its deprotonated form and two free protons. Thus, one mole will depress this volume by three times the amount that sucrose affected it, or 11.7°C. Water typically freezes at 0°C, making our new freezing point -11.7°C.

To enhance the flavor of a homemade soda, 49 g of phosphoric acid (H3PO4) is mixed with 2 L of carbonated water. If the density of the water is 1.5 g / mL, what is the solution's molality?

0.17 m B is correct. Molality is measured in moles of solute per kilogram of solvent. To find moles of solute, we must take 49 g H3PO4 × (1 mol H3PO4 / 98 g) = 0.5 mol H3PO4. Next, use density to calculate the mass of the solvent: 2000 mL carbonated water × (1.5 g/mL) = 3000 g, or 3 kg. Finally, 0.5 mol / 3 kg = 0.17 m.

600 mL of 1.5 M benzene-1,4-dicarboxylic acid (C8H6O4), one of the primary reagents used in the manufacture of Kevlar, is diluted with 2.4 L of water. What is the final molarity of the solution?

0.30 M For dilutions, use M1V1 = M2V2, keeping in mind that V2 must include the 2.4 L of water as well as the original 0.6 L of solute. M2 = (M1V1) / V2 = (0.6 L × 1.5 M) / 3 L = 0.30 M. Note also that this solution is being diluted to five times its original volume, so its new concentration must be one-fifth its initial value.

What is one example of adaptation in relation to signal detection?

A man who resides above a bakery stops noticing the smell wafting from below.

Which of the following statements regarding chemical kinetics is/are always true? I. According to collision theory, reaction rate increases with increasing concentration. II. Raising the temperature will always increase the total amount of products formed. III. The activation energy for a forward reaction is equal to that of the reverse reaction. IV. Catalysts decrease the ΔG of a reaction.

According to collision theory, reaction rate increases with increasing concentration.

A beam of electrons is fired north and enters an infinite magnetic field that is pointed vertically downwards. What path will the electron beam follow after entering the magnetic field?

East The right hand rule gives us west but applies to positive charges moving, so you need to reverse the answer for a negative charge in a magnetic field, which gives us east.

Under the conditions given, which of the following neurons is most likely to fire an action potential?

An unmyelinated neuron that receives multiple EPSPs that summate temporally Myelination has no effect on neuron excitability. It only affects conduction velocity, which is irrelevant if the neuron never fires at all. Therefore, we can ignore the myelination aspect and choose an answer based on the described combination of input potentials. If more than one excitatory postsynaptic potential (EPSP) arrives in a very short interval, the neuron cannot return to a resting potential in the time between the arrivals. Therefore, multiple inputs over a short span of time increase the likelihood of reaching threshold, which is necessary for an action potential to fire.

If hexanamide is treated with ethanol, what would be the expected products?

Hexanamide The amine group is a stronger nucleophile than ethanol, and thus has less propensity to leave the amide. Amides are significantly more stable than esters.

Which of these enzymes is exclusive to gluconeogenesis?

Pyruvate carboxylase To proceed, gluconeogenesis must overcome the three irreversible steps that occur during glycolysis. The enzymes used to achieve this are glucose 6-phosphatase, fructose 1,6-bisphosphatase, PEP carboxykinase, and pyruvate carboxylase.

DNP is a toxin that destroys the proton gradient created by the electron transport chain. Where in the cell does DNP act?

Inner mitochondrial membrane C is correct. The electron transport chain is present on the inner mitochondrial membrane and pumps protons from the matrix into the intermembrane space. DNP is an uncoupling agent which allows protons to leak back across this membrane, therefore destroying the gradient.

Tom is conducting a biochemistry experiment. He knows that a certain enzyme is present in his test tube, along with unknown concentrations of substrate and a competitive inhibitor. After addition of a quantity of substrate, Tom might observe that:

At a low substrate concentration, addition of more substrate will cause the rate to increase linearly. When the Vmax is approached (at high substrate concentration), the rate will increase at a nonlinear pace. After Vmax is achieved, addition of substrate will not impact the rate at all.

Which of the following is true of a typical viral capsid?

Most viruses have only one or two genes coding for capsid proteins. Most viral capsids are composed of repeated identical monomers, reducing the need for a large genome containing multiple different genes for structurally distinct capsid proteins.

Nucleic acids that can catalyze biological reactions include:

RNA only. While we typically think of enzymes are protein-based (and most are), some forms of RNA do perform enzymatic functions. These molecules are known as ribozymes and play important roles in protein production and packaging.

Most sociologists divide demographic transitions into either four or five distinct stages. In which of these intervals does a society's fertility rate remain stable while its mortality rate drops?

Stage 2 B is correct. A society is in stage 2 immediately prior to full-scale industrialization. During this phase, the mortality rate drops due to advances in medical care. However, the fertility rate remains relatively stable, provoking a net population increase.

An individual who has somnambulism experiences which stages of sleep differently from a normal individual?

Stage 3 and 4 D is correct. Somnambulism (sleepwalking) occurs in individuals during Stages 3 and 4 of the sleep cycle.

Consider the figure below. If the two carbohydrates above formed a disaccharide via an α-1, β-2 linkage, which molecule would it be?

Sucrose The two molecules shown are fructose (left) and glucose (right). A disaccharide composed of these two monomers is known as sucrose.

A researcher compares two antibodies that recognize the same antigen, even though they are made by different animal species. How will these antibodies differ?

The antibodies will have different constant regions.

Recently, it was discovered that a Chinese man had only 44 chromosomes. Part of his karyogram is depicted below. It can be assumed that in this case, chromosomes 1 - 12 are normal. This man meets a nice girl with a wild-type genotype, but the couple is struggling with infertility. What may be an underlying cause?

The resulting zygote is aneuploid and unstable.

In the standard action potential found in a neuron, hyperpolarization represents:

excess potassium efflux. In the standard action potential, depolarization is sodium influx, repolarization is potassium efflux (outflow), and hyperpolarization is excess potassium efflux.

If a two-year-old says, "Water go," to which the parent responds, "Yes, the water goes down the drain." The parent is using which strategy?

expansion. Expanding is restating, in a linguistically sophisticated form, what a child has said.

Glucose can be used to generate

fatty acids

Proteins can be used to generate

fatty acids

embedded in the extracellular matrix. These are the cells that produce the fibers comprising connective tissues such as collagen.

fibroblast

All of the following statements about reinforcement are false EXCEPT:

fixed-ratio reinforcement schedules are less resistant to extinction than variable-interval schedules. A is correct. Fixed-ratio schedules are highly susceptible to extinction because the subject immediately notices when reinforcement stops. Variable-interval schedules, in contrast, result in slower extinction, since the subject continues to hope that a reward is coming.

The method of loci would be most helpful when transferring information:

from short-term memory into semantic memory. The method of loci is a memorization technique meant to facilitate the encoding of long-term memories. Most likely, this method would be used with facts or pieces of data that need to be remembered. These fragments of information would pass straight from short-term into semantic memory.

Acid A has a pKa < 4.7, while Acid B has a pKa > 5.8. In aqueous solution at 25 °C, Acid B:

has a conjugate base with a larger Kb than the conjugate of Acid A. The higher the pKa value, the weaker the acid, so Acid B must be weaker than Acid A. Comparatively weak acids have comparatively strong conjugate bases, and the stronger the base, the larger its Kb value.

cause chromosome instability

nondisjunction, multipolar spindles, merotelic attachment

which genes are most likely to be linked?

ones on the same leg of chromosome

is the point at which the nasal optic fibers cross paths. It does not relate to the question stem.

optic chiasm

represent the pathways used by optic fibers after the optic chiasm.

optic tracts

Contemporary sociology includes:

postmodernism, feminism, and critical theory. These are the hallmarks of modern sociology.

A higher-order cognitive process that may include the trial-and-error approach is known as:

problem solving

Alkali metals are defined by their: I. reactivity with water. II. ability to act as strong reducing agents. III. ability to form strong acids when bound to hydrogen. IV. ground-state valence shell configuration of ns1.

reactivity with water, ability to act as strong reducing agents, and ground-state valence shell configuration of ns1.

Seminal vesicle dysfunction in males will most likely result in:

reduced fertility.

The mutation responsible for this disease is most likely found in a gene that codes for:

intermediate filaments Progeria is caused by a mutation that causes the synthesis of defective nuclear lamins. In healthy cells, lamins are comprised of intermediate filaments that form the structural support for the nuclear membrane.

acidic amino acids

aspartic acid, glutamic acid

refers to impaired coordination of muscle movements, such as those required to walk

ataxia

Biological factors related to learning and memory include all of the following EXCEPT:

ataxia, or an impairment in an individual's ability to learn language. C is correct. Ataxia is actually defined as an inability to perform proper muscle movements. As such, it does not directly relate to learning or memory.

A "silent" dog whistle takes advantage of a characteristic of sensory systems known as:

the absolute threshold. The absolute threshold is the smallest level of energy required by an external stimulus to be detectable by the human senses, including vision, hearing, taste, smell and touch. A "silent" dogwhistle uses frequencies that dogs can detect but humans cannot.

According to impression management theory, the three selves that operate during self-presentation are:

the authentic self, ideal self, and tactical self.

Which step requires two additional enzymes to bypass during gluconeogenesis?

the last step conversion of phosphoenolpyruvate to pyruvate

Apoptosis is a sequential process that can be divided into three distinct stages: initiation, commitment, and execution. The commitment stage is marked by a measurable dissipation in the mitochondrial proton gradient, owing to the permeabilization of the outer mitochondrial membrane. The execution stage features the activity of cysteine proteases that degrade vital cellular components. Cysteine protease activation is most directly triggered by:

the release of cytochrome c into the cytosol.

truths about arteries

they have thick muscular walls they carry blood away from the heart they contain no valves

A patient enters a doctor's clinic and is diagnosed with a melanin deficiency. Symptoms experienced by this individual likely include:

unusually light-colored skin and eyes. C is correct. Melanin, a pigment derived from tyrosine, provides color to the eyes and skin. This pigmentation helps protect the skin from harmful and potentially cancer-causing UV rays. The patient described likely suffers from a form of albinism, a complete melanin deficiency marked by whitish skin and pink eyes.

In glycolysis, what is the net yield of ATP per molecule of pyruvate produced?

1 ATP molecule In its entirety, one "round" of glycolysis requires two ATP molecules and produces a total of four. In other words, glycolysis forms a net quantity of two ATP molecules per original molecule of glucose. However, glucose is split into two molecules of pyruvate, the end product of the glycolytic pathway. For this reason, only one ATP molecule is made per unit of pyruvate.

Which of the following bonds would exhibit the most ionic character?

A single bond between H and F C is correct. Ionic character is determined by the electronegativity difference between bonded elements. Therefore, the pair with the largest difference will exhibit the most ionic character. The easiest way to examine this is to look for the atoms' distance apart on the periodic table. Hydrogen and fluorine are on opposite sides, meaning that hydrogen is relatively electropositive, while fluorine is actually the most electronegative of the elements. These two atoms are farther away than any other pair, resulting in the bond with the most ionic character.

A population of beetles is taken from its mainland home to a neighboring island with very few shrubs. While mainland beetles create their nests in shrubbery, island beetles nest on the ground. After a long period of time, when the island beetles are reunited with their mainland relatives, it is observed that the two populations can no longer interbreed. Which form of speciation must have occurred?

Allopatric speciation B is correct. Allopatric speciation occurs when populations, or parts of the same population, are separated by a physical barrier.

Which of the following personality disorders is most overrepresented in the prison population compared to the population at large?

Antisocial personality disorder

Which of the following correctly orders these elements from the least to the most reactive with water?

Be, Li, Na, K C is correct. All of the listed molecules are alkali metals, with the exception of beryllium, an alkaline earth metal. Alkali metals react especially well with water due to their low ionization energies, and they become more reactive as you move down the group. Therefore, Be is the least reactive with H2O, while Li, Na, and K (in that order) are more so.

Insulin is a peptide hormone that exerts its effects through extracellular binding to a specific G protein-coupled receptor. If a competitive inhibitor of this receptor is administered intravenously to a patient, what should occur?

Blood glucose should increase. If the insulin receptor is inhibited, insulin will be unable to bind and provoke a physiological response. Insulin functions to decrease plasma glucose and promote its storage in the form of glycogen. Inhibition of this hormone should yield an increase in blood glucose levels.

Which of the combinations of reagents below will yield an aldehyde?

D is correct. Only primary alcohols, which are positioned at the ends of carbon chains, can be oxidized to form aldehydes. Additionally, this can only be accomplished using weak oxidants, such as PCC. A stronger oxidizing agent would cause a primary alcohol to gain two bonds to oxygen and form a carboxylic acid.

As the Ice Age altered global climates, mammoths evolved a thicker layer of fur. What kind of population selection is this?

Directional selection Directional selection is when an extreme phenotype is favored over others.

A simple alkene is shown below. What is a non-IUPAC name for this structure?

Ethylene B is correct. The diagram depicts the simplest possible alkene. While this molecule is termed "ethene" according to IUPAC nomenclature, its common name is ethylene.

Which of these enzymes act upon α-1,4 linkages in polysaccharides? I. Glycogen synthase II. Glycogen branching enzyme III. Glycosyltransferase IV. Glycogen phosphorylase

Glycogen synthase, Glycosyltransferase, and Glycogen phosphorylase

How do HDL molecules differ from LDLs?

HDLs have relatively more protein than LDLs. The given acronyms stand for "high-density lipoprotein" and "low-density lipoprotein." Since protein is generally more dense than cholesterol, it makes sense that HDL molecules would have relatively more protein.

Which of the following conditions is / are most likely to cause a substantial decrease in tidal volume? I. Acute asthma II. Late pregnancy III. Narcotic overdose IV. Pulmonary embolism

II and III only The substantial volume occupied by the gravid uterus does, in fact, decrease the ability of the lungs to expand, reducing tidal volume (II). Overdose on a narcotic decreases respiratory drive, resulting in a decrease in both the rate and the depth of breathing (III).

Self-concept and identity are very similar psychological concepts that are often thought to be synonymous. However, they do differ in a subtle way. How can this distinction best be summarized?

Identity relates to the groups and categories that we belong to, while self-concept is the entirety of our ideas about ourselves. B is correct. Self-concept refers to our full set of conscious ideas about ourselves, while identity is generally used in relation to the categories we belong to. In other words, our identity might be racial ("I am an Asian-American"), gender-based ("I am a woman"), etc. In contrast, our self-concept includes less group-based qualities like personality traits and our perceptions of our own bodies.

The structures of four common classes of immunoglobulin are show below. A laboratory technician wants to determine the antigenic profile of human red blood cells via a hemagglutination assay. Assuming that the technician has access to immunoglobulin molecules specific for human erythrocyte antigens A and B, which class of Ig would be the most useful laboratory reagent in this process?

IgM C is correct. Hemagglutination refers to erythrocyte aggregation that results from cross-linking by antibodies. In a laboratory setting, a blood sample that reacts positively with a given antibody will visibly thicken, informing the researcher of the presence of a relevant antigen. Since IgM offers the greatest number of available binding sites, it will be the most effective in causing aggregation of erythrocytes.

Historically, much of the black population of the U.S. has lived in segregated neighborhoods that were frequently built in undesirable locations, such as downwind from factories. Additionally, due to underrepresentation in the political system, noxious industries and toxic waste disposal plants have often been sited near black neighborhoods. As a result, black neighborhoods often display significantly higher levels of environmental toxins such as lead and radon, and residents possess elevated risk of cancers and developmental abnormalities. These health problems can be best described as the legacy of which social phenomenon?

Institutional discrimination The conditions described result from institutional discrimination, which affects an entire group in society and often relates to policies or actual laws.

A positively-charged particle with a mass of 1.67 × 10-27 kg and a charge of 1.6 × 10-19 C is traveling at 1 × 104 m/s in a 0.50 T magnetic field. What magnitude of magnetic force will this particle experience?

Insufficient information provided to calculate magnetic force. Since the angle between velocity and B field is not specified, the cross-product of velocity and B cannot be determined. Therefore the magnetic force on the particle cannot be determined.

A long wire is formed into a circular loop, such that the current travels in a clockwise direction. What is the direction of the induced magnetic field inside of the loop?

Into the loop Use your right hand to determine this. Stick your thumb in the direction of the current and curl your four fingers around the wire, representing the circular magnetic field emanating from the wire. Your fingers will curl out of the page outside the loop and into the page inside the loop.

During translation, what function does the enzyme peptidyl transferase serve?

It catalyzes the formation of an amide bond. Peptidyltransferase carries out the main function of the ribosome: to link amino acids together with an amide linkage (also known as a peptide bond). It is one of very few enzymes that are made up of rRNA rather than protein.

Which of these statements describe(s) the amino acid W at physiological pH? I. It has no net electrical charge. II. It is not a zwitterion. III. It is neither acidic nor basic.

It has no net electrical charge and It is neither acidic nor basic.

Which of the following atoms or ions would be expected to have the smallest radius?

K+ K+, Ar, and Cl- have the same number of electrons, but K+ has the most protons. These exert an attractive force on the electrons, giving K+ the smallest radius.

How could one best describe the learning theory of language development?

Language is learned as a direct result of operant conditioning. B is correct. The learning theory posits that language acquisition results from operant conditioning - in other words, from rewards and punishments. Essentially, parents and other adults react to their child's first words with excitement and praise, and this motivates the child to speak more often and with increasing complexity.

Though relatively slow, nucleophilic substitution reactions feature in a number of reaction schemes commonly employed in a laboratory setting. One such process, the Finkelstein reaction, involves the treatment of a primary alcohol with tosyl chloride, followed by addition of a halide salt. This series of steps permits the synthesis of a primary alkyl halide. Which of the following conditions would best optimize the speed and yield of the Finkelstein reaction?

Neutral pH with dimethylformamide as the solvent D is correct. Since the initial substrate is primary, it cannot form a carbocation; as a result, we know that the reaction described must proceed according to an SN2 mechanism. SN2 reactions progress most rapidly in polar aprotic solvents, of which the only ones listed are acetone and dimethylformamide. However, an acidic environment would protonate and reduce the available concentration of the halide anion in solution, slowing down the substitution. Optimally, the reaction should instead be conducted at neutral pH, as choice D mentions.

Which of the following enzymes is required to generate GTP from GDP?

Nucleoside-diphosphate kinase

The conversion of β-hydroxybutyrate to acetoacetate occurs via what type of reaction?

Oxidation Acetoacetate has a ketone (specifically, an acetyl group) where β-hydroxybutyrate has an alcohol. Since acetoacetate has more bonds between carbon and oxygen, it must be the more oxidized of the two compounds.

Consider the following reaction: C2H6 + O2 → CO2 + H2O If 4 moles of ethane gas is placed in a sealed chamber with 10 moles of oxygen gas, which species is the limiting reagent?

Oxygen gas You must first balance the reaction to obtain 2C2H6 + 7O2 → 4CO2 + 6H2O. Then, compare the ratio of moles:coefficient for the two reactants. Ethane's ratio (4/2) is larger than that of oxygen (10/7), so oxygen is limiting.

Which of these enzymes catalyzes an irreversible step in glycolysis?

Phosphofructokinase C is correct. Glycolysis has three irreversible steps. These are catalyzed by hexokinase, phosphofructokinase and pyruvate kinase.

The phosphorylation of proteins or DNA is an important regulatory mechanism. Which statement best explains the function of phosphorylation in regulation?

Phosphorylation of enzymes changes their conformations, thereby allowing cells to determine when the enzyme is active.

Which of the following processes would be most affected if an organism lost its ability to synthesize snRNAs?

Post-transcriptional modification B is correct. A major role of snRNAs involves the formation of the spliceosome. This cellular machinery is responsible for removing introns from pre-mRNA transcripts during post-transcriptional processing. If an organism stopped producing snRNA, this modification process would be most significantly impaired.

Which pathway does the metabolism of fatty acids most closely resemble?

Production of acetyl-coA during cellular respiration D is correct. Fatty acid metabolism involves the process of beta-oxidation, which ultimately splits fatty acid chains into acetyl-CoA. This molecule enters the Krebs cycle in the same way as acetyl-CoA that is produced via aerobic cellular respiration.

Is a halogen more likely to be reduced or oxidized, and why?

Reduced, because a halogen will gain one electron to fill its valence shell. A is correct. Halogens have seven valence electrons. To obtain a stable noble gas configuration, they must pick up an additional electron. When an element gains an electron, it is reduced.

A capacitor with flat, circular plates of radius r is used as a component in a laptop computer. The designers find that they must double the circuit's capacitance to deal with certain power fluctuations. Which of the following setups would accomplish this goal?

Replacing the capacitor with a new capacitor of radius (21/2)r Capacitance (in farads) can be calculated using the equation C = εA/d, where ε is the dielectric permittivity of the material between the plates (a measure of how readily electric charge is "felt" through the material), A is the plates' area, and d is the distance between the plates. Remember, the area of a circle is given by πr2. For that reason, increasing the plate radius by a factor of the square root of 2 would double the area and thus the capacitance.

What is the path taken by the electrical signal that causes heart contraction?

SA node → AV node → bundle of His → Purkinje fibers

Treatment of (S)-2-bromobutane with ethanol results in a largely racemic mixture. The mechanism was most probably:

SN1. SN1 produces a racemic mixture due to the identity of its intermediate, a carbocation (which lacks chirality at the center that is involved). The nucleophile can then attack either side of the carbocation.

Which of these procedural measures would contribute most to optimal PCR function and a high yield of the desired product?

Selection of a primer sequence that does not complement itself A is correct. If primers are complementary to each other (or if a primer is complementary to itself), primer nucleotides may anneal with one another to form a small, useless dimer. Selection of primer sequences that do not exhibit this characteristic will decrease the chances of promoting this side reaction, ultimately optimizing the desired PCR.

According to the cognitive developmental theory outlined by Jean Piaget, a child entering the preoperational stage would exhibit which of the following?

She would gain an understanding of object permanence, or the idea that things do not cease to exist when they move out of view. D is correct. The preoperational stage is Piaget's second stage of development. A grasp of the concept of object permanence signifies the end of the sensorimotor stage (ages 0-2) and the beginning of the preoperational stage (ages 2-7, approximately).

What is the critical angle at which total internal reflection (TIR) will occur as light moves from air (n = 1) to water (n = 1.33) to glass (n = 1.52)?

TIR will not occur. For TIR to occur, the light must be traveling from a high index of refraction to a low index. This situation is the opposite.

Why does polymerization occur much more rapidly at the (+) end of an actin polymer than at the (-) end?

The (+) ends of actin molecules display stronger intermolecular interactions than the (-) ends. While the rate of polymerization or depolymerization depends on the surrounding actin concentration, the (+) end tends to grow much more quickly than the (-) end. (+) actin is bound to ATP, while (-) actin is bound to ADP. The (+) terminal displays significantly more powerful intermolecular interactions than the (-) actin. When the (+) actin polymerizes, ATP is cleaved to become ADP. The ADP-bound actin at the minus end of the polymer interacts much more weakly, promoting slower, if any, polymerization.

Which of these examples best demonstrates the Gestalt principle of closure?

The NBA symbol does not actually show a person, but rather an outline made by two separate shapes. Our brains then perceive this as a basketball player.

Which fundamental law of physics prohibits an engine from having an efficiency that is greater than 100%?

The First Law of Thermodynamics A is correct. It is the first law that states that energy cannot be created or destroyed, only converted between forms. In other words, we cannot possibly obtain a larger energy output than was originally supplied. Such a scenario would be required to yield an efficiency of greater than 100%.

Chloroacetic acid has a pKa of 2.85, while formic acid has a pKa of 3.75. Which of the following statements is true?

The Kb of the formate anion is larger than the Kb of the chloroacetate anion. The higher the pKa value, the weaker the acid, meaning that formic acid must be less acidic than chloroacetic acid. Relatively weaker acids have relatively stronger conjugate bases, and the stronger the base, the larger its Kb value. Remember, the "-ate" suffix is commonly used for anions, which is how we know that formate is likely the anion (or conjugate base) derived from formic acid. The same is true for chloroacetate.

A researcher tries to express the human gene for insulin, screened from the human genomic library, in a bacterial cell. The translated protein fails to fold into its functional native conformation. What could explain this misfolding?

The genes in the human genomic library contain introns, which cannot be excised by the bacterial cell. Bacterial cells cannot carry out post-transcriptional modification of mRNA. Without this modification, introns will be translated along with exons, leading to incorrect folding of the expressed protein.

The diagram below shows a type of epithelial layer. This type of epithelium would most likely be found in which of the following locations?

The lining of the distal convoluted tubule Initially, this question may seem oddly specific. However, note that the diagram shows a single layer of epithelial cells. As substances can most easily diffuse through a single cell layer, this cross-section likely comes from an organ specialized for absorption or secretion. Of the options listed, the best fit for this function is the nephron, of which the distal convoluted tubule is a part. Specifically, the lining of the nephron consists of a simple cuboidal epithelium, which is what the diagram shows.

A protein is isolated from the blood of a patient undergoing a diagnostic test. After purification, the protein is run on an SDS-PAGE gel, where two distinct bands appear. What is a possible explanation for this result?

The protein contained two subunits of different molecular weights.

A sample of nonanoic acid is analyzed using nuclear magnetic resonance (NMR) spectroscopy. Which of this molecule's protons will exhibit a peak that is farthest upfield?

The proton(s) on carbon 9, which will display a shift of around 1.0 ppm

Severe combined immunodeficiency (SCID) was the first disease that was successfully treated using gene therapy. Unfortunately, a small number of patients later developed leukemia, representing one of the risks of gene therapy. How might these leukemias have developed?

The randomly integrated DNA may have activated an oncogene, causing leukemia. C is correct. One of the drawbacks of gene therapy is the vector randomly integrates the new DNA sequences. This could possibly lead to integration into a region that would activate an oncogene. As leukemia is a cancer of the white blood cells, this choice makes the most sense.

After reading her psychology book, an MCAT student decides that she will only solve problems using algorithms from this point forward. What are the advantages and disadvantages of this strategy?

The student will always reach a definitive answer, but algorithms are not very efficient and often require impractical amounts of time. D is correct. Algorithms follow a step-by-step method of problem-solving that is nearly foolproof with regard to finding a definitive solution. This is the method most commonly used by computers. However, it is disadvantageous because it can be an extremely time-consuming process, especially for complex problems.

A cellular abnormality results in the rapid production of sense RNA that is complementary to a large portion of a certain gene. What will most likely happen to the transcription of that gene?

Transcription of the gene will largely be unaffected. This question requires understanding of the complementary DNA strands involved in transcription. One strand is the sense (non-template) strand, while the other is the antisense (template) strand. mRNA is transcribed directly from the antisense strand, giving it the same base sequence as the sense strand. Thus, the production of sense RNA will not affect the corresponding antisense strand, which is the strand that acts as a template for transcription.

Isaac, a chemistry student, stumbles across a closed vessel containing n-heptane. He wishes to react this compound in a way that increases its boiling point. Of the following, what should Isaac do?

Utilize a free-radical reaction to halogenate the compound at one of its carbons Boiling point is mainly determined by intermolecular forces and molecular weight. Since unsubstituted alkanes are able to exert only London dispersion forces, they typically display low boiling points. However, if Isaac were able to halogenate n-heptane, he would be giving it a polar bond between the halogen atom and one carbon. This would allow the product to experience dipole-dipole forces, raising its boiling temperature.

Interestingly, a society's norms are often difficult to identify directly. In contrast, deviance (or violation of these norms) can be simpler to spot, making it a valuable tool to learn about socially accepted behavior. What cause-and-effect relationship below does NOT pair an example of deviance with the appropriate societal response?

Violation of a more is generally punished by legal restitution. A is correct. Mores, which specifically relate to moral norms, play a major role in dictating the accepted practices in society. Mores also relate to taboos; for example, a feeling of disgust at the idea of incest is considered a more. Violation of a more usually does not result in legal restitution.

Which of the following molecules is most likely to be filtered into the Bowman's capsule in the kidneys?

Vitamin C Vitamin C is water-soluble and is excreted in the urine.

Which of the following vitamins mainly serves as an antioxidant?

Vitamin E Vitamin E is most well-known for its antioxidizing capabilities.

A psychiatrist wishes to determine whether his new patient has an internal or external locus of control as soon as possible. Which question would be most productive for the psychiatrist to ask?

When you last did poorly on a test or other challenge, what reason did you give for that occurrence? D is correct. An individual's locus of control is based on his perception of his own level of involvement in, or control over, the events that occur in his life. A person with an external locus of control might answer with "I failed a test because the professor didn't like me," while someone with an internal locus might say "I didn't do well because I didn't study enough, but if I had studied, I could've changed the outcome."

The figure below depicts a proposed example of alternative splicing. A, B, and D represent exons, while C represents an intron. Is this situation possible?

Yes, because alternative splicing can remove exons in addition to introns. B is correct. During alternative splicing, a complex known as the spliceosome binds to the pre-mRNA and forms a loop, similar to that shown in the figure. Segments in the loop will be spliced from the mRNA sequence. Here, the fragments labeled B (an exon) and C (an intron) will be removed. Contrary to popular misconception, the removal of exons can occur during splicing. In fact, such removal is a primary reason why numerous mRNA transcripts can be generated from a single gene.

What is the electron configuration of Os3+?

[Xe] 4f145d5 When determining the electron configuration of a cation, remember that the highest-energy electrons should be removed first. As the 6s subshell possesses the highest principal quantum number, its electrons must also have the most energy of those involved. Thus, the two electrons in the 6s shell are removed first. Next, a single electron from the 5d shell must be taken off as well, as it has higher energy than the 4f electrons.

For the organic molecule shown below, stereochemistry has been deliberately omitted. Treatment of this species with aqueous sulfuric acid would yield a major product best described as:

a bicycle containing one 7-membered ring and one 6-membered ring. A is correct. Here, the primary product (given below) is an acetal formed by the reaction between the terminal aldehyde and two of the hydroxyl groups.

James was recently diagnosed with myopia, a condition in which the lens of the eye has too much converging power. To correct this situation, James should use:

a concave lens with a negative focal length. The question stem describes myopia as the result of excessive convergence of incoming light. For this reason, correction of this condition requires a diverging lens. Remember, for lenses, "convex" equates to converging, while "concave" denotes a diverging system. All diverging lenses have negative focal lengths, which relates to the idea that light never actually converges on the opposite side of the lens from the object.

If you wish to promote an E2 reaction, you should use:

a substrate with an anti-periplanar hydrogen atom, a strong base, heat, and a polar aprotic solvent. A is correct. E2 reactions are bimolecular eliminations. These reactions are analogous to SN2 mechanisms, as both are second-order and involve a one-step mechanism. Unlike SN2 reactions, however, E2 processes must involve a hydrogen that is "anti" to the leaving group. Additionally, use of a strong base tends to push a reaction toward E2, while E1 reactions can proceed with weaker basic species. Finally, like SN2 processes, E2 reactions prefer aprotic solvents.

Consider the figure below. Curve B most likely represents:

adiabatic expansion. Here, the volume is increasing, so Curve B must relate to some form of expansion. Adiabatic curves (which denote processes in which no heat is exchanged, but work is typically done on or by the system) are generally more steeply-sloping than isothermal curves.

The following reaction is useful for the decarboxylation of esters under certain synthetic circumstances. In reality, this process is driven by entropy. In addition to the major organic product depicted above, carbon dioxide and bromomethane are lost from the reaction vessel as volatile gases. With this information in mind, the first step of this reaction is best described as:

an SN2 reaction. The nucleophilic bromide ion attacks the terminal methyl group, displacing the carboxylate. This is the only plausible mechanism by which bromomethane can be formed. Conveniently, this also results in the formation of an intermediate that is capable of subsequent decarboxylation.

gender characteristics

biological sex, gender identity, sex-based social structures

-coccus

circular shape

begins with a specific group of healthy individuals and tracks changes in their health (or other variables) over time to determine their risk of developing one or more diseases. Here, an initial cohort is assembled from the region near the factory and monitored for five years.

cohort study

converging mirrors are

concave

The DSM-IV, published in 1994, renamed multiple personality disorder due to perceived inaccuracies in the name. One major issue is that sufferers of this disorder do not actually have many distinct "personalities"; they possess multiple "personality states" that cause them to lack a cohesive identity. The current name for this condition is:

dissociative identity disorder. C is correct. Multiple personality disorder is the old name for dissociative identity disorder (DID).

Which of these is NOT a form of nonverbal communication?

email; it is written

Xixi is a thirty-year-old woman who was recently laid off and does not feel that she can find another job soon. Although her family is concerned about her well-being, Xixi tries her best to be optimistic. She often uses phrases such as "I am sure the right job will come along" or "this is a good opportunity for me to work on myself." The contrast between Xixi's internal emotions and her external emotional display exemplifies:

impression management; Xixi is attempting to influence her family's perception of her and to align this perception with her goals of finding another job.

Robert and Jenny have recently been diagnosed with different psychological disorders. While Robert is anxious and self-conscious of his diagnosis, Jenny feels perfectly normal and writes off most of her symptoms as "just how she always has been." Robert is:

less likely than Jenny to have a personality disorder. A number of personality disorders, along with anorexia nervosa, exemplify ego-syntonic conditions. Ego-syntonic disorders are those that align with the patient's self-concept. In other words, instead of feeling sick or abnormal, the patient feels good and may even enjoy her disease, considering it to be part of her usual behavior. Jenny appears to be experiencing such feelings here.

In order for a human's second-order visual perception to work, the object must be:

moving. Second-order motion perception occurs through examining the changes in an object's position over time through feature tracking in the retina.

In the process of glomerular filtration, which molecules are excluded from entering the nephron? I. Platelets II. Sodium III. Glucose IV. Erythrocytes

platelets and erythrocytes

A quick jump of the eye from one position to another as it takes in visual stimuli is known as a(n):

saccade. The eyes scan the visual field by jumping quickly from one focus to another in saccades, rather than smoothly tracking from point to point.

Aspirin is routinely prescribed to patients at risk of myocardial infarction. Aspirin irreversibly inhibits an enzyme necessary for the synthesis of thromboxane A2, a prostaglandin derivative produced by activated platelets. Likely actions of thromboxane A2 include:

stimulation of vascular smooth muscle. B is correct. If aspirin is prescribed to reduce the risk of myocardial infarction (heart attack), it likely plays some role in inhibiting thrombosis. This implies that thromboxane A2 is involved in the progression of the clotting cascade. Stimulation of vascular smooth muscle will result in vasoconstriction, an early step in the coagulation process. This choice also makes sense, as we would expect aspirin to inhibit the constriction of blood vessels.

The slender giant moray (Strophidon sathete) is a species of large marine eel that is technically classified within the moray family. Like most oceanic creatures, the giant moray has adapted to have different intracellular solute concentrations than freshwater animals. If one of these marine eels were placed in a typical river, its cells would:

swell, as they would be hypertonic to their environment.

The giant panda nearly became extinct several years ago, but recovery efforts have kept its population stable. However, nearly all giant pandas are now genetically very closely related. This is due to:

the bottleneck effect. C is correct. The bottleneck effect occurs when a disaster or other event results in a loss of a large portion of a population. All future members of the population then descend from a small number of individuals, which drastically reduces genetic variability.

A middle-aged woman who must wear protective sunglasses in bright light most likely has a problem with which labeled structure?

the choroid

During most of the duration of an action potential, another stimulus, regardless of its strength, cannot cause the neuron to fire again. This is termed the absolute refractory period, and it can be attributed to:

the fact that sodium channels are inactivated.

Within a eukaryotic cell, the β-oxidation of fatty acids takes place in:

the mitochondria.

Which of the molecules shown above is LEAST likely to be located in a lipid raft surrounding a protein?

the most double bonds

Oxytocin, secreted by the posterior pituitary, increases the contractility and elasticity of the uterine wall. As a result, uterine contractions continue, eventually culminating in parturition (birth). Oxytocin production is most directly induced by:

uterine contraction. C is correct. Parturition is one of the rare examples of a positive feedback loop. Stimulation of oxytocin receptors located in the uterine wall increase uterine elasticity, which causes increased uterine stretch during subsequent contractions. This stimulates stretch receptors that feed back to the posterior pituitary gland to trigger more oxytocin production.

When selecting a buffer for PCR, it is most important to ensure that the solution contains:

magnesium cations A is correct. Magnesium is a cofactor for Taq DNA polymerase. This ion aids in the coordination of the pyrophosphate leaving group, facilitating phosphodiester hydrolysis and the subsequent addition of incoming nucleotide triphosphates.

Which of these statements accurately describes chirality or chiral compounds?

All meso compounds have multiple chiral centers but are not optically active overall. This is the definition of a meso compound. Note that compounds with a single stereocenter cannot be meso, as these molecules require symmetry to yield a structure that is achiral as a whole.

Proper eukaryotic DNA replication requires a number of enzymes. The elongation step of replication involves: I. DNA polymerase. II. Ter protein. III. initiation proteins. IV. the Dicer enzyme.

DNA polymerase

Which of the following are base units in the SI system? I. Kilometers II. Grams III. Seconds IV. Kilograms

III. Seconds IV. Kilograms

What is the function of HMG-CoA reductase?

It aids in the production of cholesterol by the smooth ER. HMG-CoA reductase acts as part of the mevalonate pathway, a cholesterol production process that occurs in the liver.

this point exists exactly halfway between the first and second pKa values, which are represented by the half-equivalence points on the curve above

approximate isoelectric point (pI)

is the tendency people have to overemphasize personal characteristics and ignore situational factors in judging others' behavior.

fundamental attribution error

Atorvastatin is a drug used to treat hypercholesterolemia. Which of the following is the most likely mechanism of action by which atorvastatin reduces serum cholesterol?

Atorvastatin inhibits HMG-CoA reductase, the rate-limiting enzyme in the cholesterol biosynthesis pathway. The rate-limiting step of a biosynthetic pathway is typically the best target for regulation, both exogenous and endogenous. Altering the activity of a rate-limiting enzyme produces the fastest and most pronounced change in metabolic output.

The following reaction takes place in an electrochemical cell: Zn (s) + 2 H+ (aq) → Zn2+ (aq) + H2 (g). If the reduction potential of Zn is -0.76 V, what is the emf of this cell?

+0.76 V Zinc is oxidized in the given reaction, as it loses electrons. The emf will thus be the opposite of the reduction potential of zinc

A person pushes a 60 kg gurney starting from rest to the top of a 20 m long hospital ramp and brings it to a complete stop. If the ramp rises at a 30 degree angle from the horizontal and reaches a height of 6.5 m, then what is the net work done?

0 kJ Just use energies! The gurney went from zero height to 6.5 m, and had an initial and final velocity of 0 m/s. The work done on the gurney is the change in gravitational potential energy, mgh, but gravity was doing equal and opposing work on the gurney the entire time hence why the change in kinetic energy is 0 J. So the net work done is 0J (since Wnet= ΔKE)

A thin-layer chromatography experiment is conducted with an alumina stationary phase and a hexane solvent. If the plate is 12 cm long, the solvent front traveled a total of 10 cm, and Sample C migrated exactly 4 cm, what would be its Rf value?

0.4 B is correct. Rf values are calculated by dividing the distance traveled by a sample by the distance traveled by the solvent front. In this scenario the equation looks like: Rf = 4 cm / 10 cm, which gives 0.4.

Consider the following reaction. 2A3 (g) ↔ A6 (g) If Kp = 10 and the partial pressure of A6 is 0.1 atm at equilibrium, what is the mole fraction of A3 at equilibrium?

0.5 B is correct. Setting up the equation 10 = [A6]/[A3]2 tells us that A3 = 0.1. From this value, we know that the mole fraction of the compound in question is 0.5.

The extinction coefficient for a particular protein is 0.685 Lmol-1cm-1. Which value is closest to the protein concentration if the absorbance is 7 and the path length is 1 cm?

10 M D is correct. The Beer-Lambert law states that absorbance = εCl, where ε denotes the extinction coefficient, C represents concentration, and l is the path length in cm. Solving for C gives 10 M.

A study of nitrogen-oxygen bonds reveals bond lengths on the order of 115 pm, 125 pm, and 135 pm in various molecules. What would be the expected bond length of the shorter bond in nitrite?

125 pm We must assume that the three bond lengths correspond to a triple, double, and single bond. Triple Bonds are the shortest, while single bonds are the longest. Nitrite has one double bond and one single bond, so the double bond is the shorter of the two (125 pm, as opposed to 135 pm).

Four species of bacteria are cultured separately in thioglycolate broth. Which labeled tubes depict organisms that are exclusively capable of using anaerobic respiration?

2 and 4 only The question is asking for the tubes that contain obligate anaerobes. We can infer that aerobic species would cluster close to the open top of the tube, while anaerobes would prefer to grow lower in the tube, away from oxygen. Sample 2 clearly consists of obligate anaerobes, as no cells are located anywhere near the air. In contrast, the bacteria in Sample 4 are spread evenly throughout the broth, but still do not gravitate toward the open top. This species is likely aerotolerant, meaning that it is an obligate anaerobe that does not die in the presence of oxygen.

Bacteria in the gut play an essential role in processing methane through the following reaction. CH4 (g) + SO42- (aq) 🡪 H2O (l) + HS- (aq) + HCO3- (aq) The above experiment is repeated, and again, 50 mg of methane and 60 mg of sulfate are mixed in a closed vessel. What is the theoretical yield of hydrosulfide ion?

20 mg HS- C is correct. According to the reasoning used for #1261, sulfate is the limiting reagent here. We can thus use the given amount of 0.06 g sulfate to find how much hydrogen sulfide should form. As always, though, we must first convert grams of sulfate to moles. 0.06 g SO42- × 1 mol/96 g = approximately 0.0006 mol. Since sulfate reacts to form hydrogen sulfide in a 1:1 molar ratio, 0.0006 mol of HS- will be formed. Finally, we must convert to grams of hydrogen sulfide. 0.0006 mol HS- × 1 mol/33.065 g = about 0.02 grams of HS-.

If the circuit had a power consumption of 7.5 W, and the resistance of A was 5 Ω, the resistance of B would be:

20 Ω The voltage and power are given, allowing us to find I using P = IV. I = 7.5 W / 15 V, or 0.5 A. Next, the resistance of resistor B can be found using the equation V = IR. As explained for #1558, the equivalent resistance for the parallel element containing resistor A is 2 Ω. Finally, we can find the resistance of B as follows: V = IRtotal, so Rtotal = V/I. (2 Ω + 8 Ω + B) = V/ I = 15 V / 0.5 A = 30 Ω. (10 Ω + B) = 30 Ω, so B = 20 Ω, choice D.

The diagram below represents the electronic configuration of an arsenic atom. It is implied that all shells except for the outermost one (which contains 4 electrons, as shown) are full. With regard to this atom, what is the effective nuclear charge on an n = 3 electron?

23 Effective nuclear charge can be found by subtracting the total number of electrons in all shells preceding the one in question from the nuclear charge (the number of protons). Arsenic has an atomic number of 33. The atom pictured appears to be oxidized (As+), as we would expect the outermost shell to have 5 electrons if the atom were neutral. In other words, the diagram includes 32 electrons, but we only need to consider those in the first two shells. Starting from the first ring, the numbers of electrons in those shells are, respectively, 2 and 8. (Note that this information corresponds to the atom's electronic configuration.) To calculate the effective nuclear charge when n = 3, then, we only need to find (33 - 10), or 23.

Which equation correctly represents the combustion of calcium?

2Ca (s) + O2 (g) --> 2CaO (s) C is correct. All combustion reactions should have oxygen, which is bimolecular, as a reactant. Since calcium has a +2 charge, it will combine with oxygen to form CaO, and one molecule of O2 will be required for every two molecules of Ca.

A new element ("L") is discovered and found to contain 240 protons and 120 neutrons. What is the correct atomic notation for this element?

360/240 L

An object is placed 40 cm to the left of a converging lens with a focal length of 20 cm. Where will the image appear?

40 cm to the right of the lens This question describes a converging lens and an object positioned more than one focal length away. In such a situation, a real and inverted image will form. Real images appear on the opposite side of the lens from the object, meaning that this image must fall on the right, not the left. To find an exact answer, use the formula 1/f = 1/di + 1/do. Plugging in 20 cm for f and 40 cm for do gives us a di value of 40 cm.

In a circuit, Resistor A and Resistor B are situated in series, with no branch points or other circuit elements between them. If 18 A of current passes through Resistor A, and if Resistor B has a resistance of 2.5 Ω, what is the voltage drop across Resistor B?

45 V Remember, V = IR is effective for individual resistors as well as the circuit as a whole. However, you must be careful to plug in values corresponding only to the resistor in question. Here, since Resistors A and B are in series, the same amount of current (18 A) passes through Resistor B as through Resistor A. Since we are given the resistance of Resistor B, we can simply multiply (18 A)(2.5 Ω) to yield 45 V.

At what time during development does an "embryo" become a "fetus"?

9 weeks An embryo becomes a fetus by definition at the 9th week of development. You must know the different developmental stages for an embryo and fetus.

If a sociologist argues that all so-called objective phenomena are open to debate because all meaning is subjective, what would they consider a "fact" to be?

A social construct

In times of glucose deprivation, the body can obtain energy by utilizing its stores of fat. This involves the production of ketone bodies, the metabolism of which is relatively inefficient. One particular ketone body can be detected by smelling one's breath. Which product is responsible for yielding sweet-smelling breath and sweat?

Acetone Acetone is formed from the breakdown of acetoacetic acid. Up to 30% of this product can be excreted from the body in the urine or through mechanisms such as exhalation. Unlike the other ketone bodies, acetone cannot be converted back into acetyl-CoA.

Which of the following neurotransmitters is found in large amounts in both the central and peripheral nervous systems?

Acetylcholine C is correct. In addition to playing many roles in the peripheral nervous system (such as muscle activation and synaptic transmission of autonomic nervous signals), acetylcholine is also used in the CNS in areas related to arousal, attention, and motivation.

Which of the following statements accurately describes the James-Lange theory of emotion?

An external stimulus triggers physiological arousal, which is then interpreted as emotion. B is correct. The James-Lange theory of emotion posits that physiological changes, like increased heart rate or breathing, cause a subsequent experience of emotion. For example, imagine that someone jumps in front of you in a haunted house. This theory predicts that your heart rate will speed up first, and you will feel fear as a result.

A scientist tags the cells in the invagination of a developing deuterostome blastula. The labeled cells are shown below as those marked by an arrow. Following another 8 weeks of development and organogenesis, which of the following tissues would still be labeled?

Anus The tagged cells represent the blastopore, so tissues that derive from the blastopore will also be labeled. In deuterostomes, the blastopore immediately gives rise to the anus.

A chemist attempting a complex synthesis procedure succeeds in creating the molecule shown below. Treatment of this compound with which of the following reagents would produce a species likely to return a positive Benedict's test?

Aqueous sodium hydroxide The molecule shown in the diagram is not a reducing sugar, which is necessary to produce a positive result for the test mentioned. Treatment with sodium hydroxide would likely promote the displacement of the azide group by a nucleophilic hydroxide ion. The resulting sugar would contain a free aldehyde and would thus return a positive Benedict's test.

Which of the following pairings properly matches the symptom of a psychological disorder with its description?

Catalepsy - rigid, immovable muscles and lowered sensitivity to stimuli D is correct. Catalepsy refers to the decreased, stiffened muscle movement that is often seen as a symptom of catatonic-type schizophrenia. Alternatively, this type of psychotic disorder can result in the exact opposite: dramatically increased motor activity or rates of speech.

Which sequence of events correctly depicts the conversion of DNA into protein in a eukaryotic cell?

DNA is transcribed into mRNA in the nucleus, then translated into an amino acid chain in the cytosol.

A human egg experiences autosomal non-disjunction during meiosis. Once fertilized, what could be a resulting chromosomal abnormality found in the embryo?

Edward's syndrome "Autosomal" tells us that the non disjunction is of a non-sex-chromosome. Edward's syndrome is trisomy 18, while the rest of the answer choices deal with a nondisjunction of the sex chromosomes.

A typical polymerase chain reaction is conducted using DNA polymerase derived from T. aquaticus, a bacterium. Which of these experimental conditions is LEAST likely to denature this species-specific polymerase?

Exposure to temperatures in excess of 90 °C

Which of the following bases is weakest?

H2O Choices A, B, and D are either strong or weak bases. Water is amphoteric, with the ability to act as an acid or a base. In other words, water is both a very weak acid and a very weak base

Which muscle type(s) is/are always uninucleated? I. Skeletal muscle II. Smooth muscle III. Cardiac muscle

II only Smooth muscle cells have one nucleus (II).

In a healthy patient, which area of the renal interstitium has the highest solute concentrations?

Inner medulla As you travel from the cortex towards the inside of the kidney, or the medulla, the solute concentration increases.

Which of the following correctly identifies the role of observation in ethnic studies?

It may be unobtrusive or participant. Observation is essential in science. Scientists use observation to collect and record data, which enables them to construct and then test hypotheses and theories. Scientists can observe in many ways: with their own senses as passive observers or as active participants, potentially aided by tools such as scanners or transmitters

With this in mind, how would the solubility of CuCO3 in a water / ammonia solution differ from its solubility in water alone? Assume that copper ions do not form complexes in pure water.

It would be more soluble in the water / ammonia solution.

Jamy recently turned fifteen. Lately, she started watching a television series that shows the teenage main character being rude to his parents; in many episodes, he is celebrated by his peers as independent. Jamy has recently become short-tempered with her own parents and often snaps at them. The most feasible explanation for this scenario is that:

Jamy has adopted and internalized the stereotyped attitudes and behaviors of the teenager in the TV series.

Ciliopathy refers to a group of conditions in which cilia fail to form or function properly. Ciliopathy of motile cilia is a possible cause of dysfunction in which of the following organ systems?

Reproductive The uterus relies on motile cilia in its epithelium to move the embryo from the ovary and Fallopian tubes. Dysfunction of these cilia can result in an ectopic pregnancy. Cilia are present on most cells of the body, but motile cilia that move under their own power are found only in the uterine lining and the trachea.

hich of the following choices accurately represent(s) the Henderson-Hasselbalch equation?

The Henderson-Hasselbalch equation can be written as either pH = pKa + log([A-]/[HA]) or pOH = pKb + log([HA]/[A-]) (I).

Which of these parts of the sarcomere does not change dimensions through the course of muscle contraction?

The Z line The Z line is the name given to the junction, or "border," at either end of the sarcomere. These structures are orthogonal to the direction of contraction and do not expand or become smaller during muscle movement.

After numerous unsuccessful attempts to breed a male rabbit, a researcher decides to examine its sperm under a microscope. He finds that the sperm cells have normal tails and healthy motility. What is the most likely issue?

The digestive enzymes in the acrosomes have been modified and are no longer functional. The digestive enzymes in the acrosome are used to dissolve the protective coating on the ovum, allowing the sperm cell to enter and fertilize. Of the options given, this is the only reasonable choice.

A scientist conducts an experiment in which two groups of subjects view a series of photos. She notices that the first group can easily discern the relative sizes of the objects in the photos, but have a harder time determining depth. The second group can easily perceive both depth and relative size. According to these findings, what might we deduce about the difference between the two groups in the experimental design?

The first group observed the photos with only one eye, while the second observed the photos with both eyes. D is correct. The only known difference between these groups is that participants in one could determine depth, while those in the other could not. The determination of relative size and depth are monocular and binocular cues, respectively. For this reason, we can assume that both groups had at least one eye open, but the first likely had the other closed while the second was allowed to use both eyes.

In which of the following locations can ketolysis NOT occur?

The liver Ketolysis, or the breakdown of ketone bodies, is a metabolic process that can provide energy. The liver lacks a particular enzyme required for this process.

A patient is suffering from decreased or inappropriate sensation of all sensory stimuli except for olfactory ones. How might one account for these symptoms?

The patient has slow progressive degeneration of the thalamus.

A soldier suffers an injury to his spinal cord from a piece of shrapnel. Specifically, a shard of metal enters his spine from the rear between two thoracic vertebrae and severs many of the rearmost nerves, though it stops before severing any nerves in the front or middle of the cord. What symptoms will this injury likely cause for him?

The soldier will likely lose feeling in parts of the body below the injury.

Which of the following molecules is NOT expected to drain to the liver prior to arriving at the vena cava?

Vitamin E Vitamin E is the only molecule listed that is lipid-soluble and is absorbed by the lacteals in the intestines.

Furan, a small organic compound used in many synthesis reactions, is:

a heteroaromatic species. D is correct. Furan is a five-membered aromatic ring that contains an oxygen. Because it is aromatic in nature, and because its ring structure contains oxygen as well as carbon, it can be considered heteroaromatic.

A researcher sequences the genome of an unknown prokaryotic cell. When looking at the proteins coded for by this organism, she finds a protein that would not normally appear in a prokaryote. The molecule in question could be:

a protein with sequential similarities to eukaryotic components of the spliceosome. Prokaryotes do not conduct splicing or other post-transcriptional modifications.

The most reactive carboxylic acid derivatives are:

acyl halides. Acyl halides are the most reactive because their halogens are very good leaving groups. In other words, they are typically stable as anions in solution.

Which of the following demographic characteristics of a population would be most useful in determining its risk for various diseases?

age

Palytoxin is a chemical isolated from certain species of coral; it effectively forces the Na+/K+ ATPase into an open conformation, allowing free diffusion of ions and rapid death. All of the listed effects would likely result from the administration of palytoxin EXCEPT:

cell membrane potentials would rapidly hyperpolarize. A is correct. If ion concentrations equalized on both sides of the plasma membrane, cell potentials would depolarize (become less negative), not hyperpolarize. In fact, it is the variation in ion concentration that establishes membrane potentials in the first place.

Which of the following refers to the fading of memory traces from short-term memory?

decay

happens when an individual focuses on developing specific social skills. This child is not actively seeking to establish certain skills; instead, he is absorbing values and expectations from his surroundings.

developmental socialization

This molecule does not travel from the nucleus to the cytoplasm.

hnRNA hnRNA, or pre-mRNA, refers only to mRNA that has not yet undergone post-transcriptional modification. In other words, it has not yet left the nucleus. mRNA and tRNA are synthesized in the nucleus and later move to the cytoplasm, while rRNA is made in the nucleolus (a region of the nucleus) and also leaves to enter the cytoplasmic space.

Intramuscular glucagon (IMG) can be administered via autoinjector pen in a similar fashion to the injection of epinephrine for allergic reactions. IMG is most likely used to treat:

hypoglycemia.

The difference between monocular and binocular cues is that:

monocular cues can be perceived with one eye, while binocular cues require both eyes.

Isooctane and n-octane are best described as:

structural isomers. Isooctane is also known by its IUPAC name, 2,2,4-trimethylpentane. While you need no familiarity with this compound for the MCAT, we can infer that it differs from n-octane only in the branching of its carbons. Structural isomers are compounds that share a molecular formula but have different bonding configurations.

In the process used to create cDNA, the ssDNA produced by reverse transcriptase can be made into dsDNA through treatment with DNA polymerase I. In this case, no primer is required because:

the 3' end of the DNA strand often loops back onto itself, thereby providing a primer for the enzyme. The 3' end of ssDNA often loops back onto itself due to complementary base pairing. This serves as a primer for DNA polymerase I, providing the free 3' OH group necessary for the enzyme to initiate synthesis.

A book rests horizontally on a table. The book experiences a gravitational force of mg due to the earth's gravity. According to Newton's third law:

the earth experiences a gravitational force of mg from the book.

All of the following statements describe the net result of oxidative phosphorylation EXCEPT:

the products of oxidative phosphorylation are retained for the next round of cyclic phosphorylation. D is correct. Ultimately, the products of oxidative phosphorylation are ATP, water, NAD+, and FAD. These products are not retained in the same way that oxaloacetate is during the citric acid cycle.

Consider a positively-charged particle that is experiencing a force due to an external electric field. Which of the following is/are conserved for the particle? I. Potential energy II. Kinetic energy III. Total energy IV. Momentum

total energy

A chemistry student is working with the organic molecule shown below. If she treats this species with 1-propyne in the presence of sodium hydride, the resulting reaction will proceed:

via an SN2 pathway and yield 2-butyne, an organic salt, and hydrogen gas. A is correct. Theoretically, if a carbocation were to form, it could only be on the methoxy carbon shown above (making the oxygen atom and its large organic substituent the leaving group). This carbocation would not be sufficiently substituted, which forbids an SN1 mechanism. Note also that, while protonation of the oxygen atom could facilitate substitution at the adjacent ring carbon, this is impossible due to the strongly basic environment promoted by sodium hydride. Instead, the methoxy carbon will undergo nucleophilic attack by the conjugate base of 1-propyne, resulting in the formation of a four-carbon molecule, 2-butyne.

All of the following statements regarding the visual pathway are true EXCEPT:

visual information does not travel through the parietal lobe. Although visual signals ultimately end in the occipital lobe, they do move through both the parietal and temporal lobes before reaching their final destination.

Weber's law can be applied to:

visual stimuli, as when a man attempts to distinguish between images at different brightness levels. Although Weber's law is used for a wide variety of stimulus types, it specifically relates to the proportionality of the just-noticeable difference perceived by an individual. Only this choice involves the same person attempting to notice differences between stimuli with distinct intensities.

Social interactionism, a sociological perspective related to the work of George Herbert Mead, holds that:

we assign meaning to concepts and objects based on our interactions with society, and act based on these systems of meaning. This statement sums up the defining principles of social interactionism, a perspective that is also closely tied to language.

Uncoupling agents, such as 2,4-dinitrophenol, operate by facilitating proton diffusion across the mitochondrial membrane, dissipating the proton gradient. After administration of a concentrated solution of 2,4-dinitrophenol to an actively respiring muscle cell, the overall rate of ATP synthesis:

will decrease by over 50% without immediately halting the electron transport chain. Uncoupling agents dissipate the proton gradient that is generated by the four complexes in the electron transport chain. The effect is analogous to poking holes in the inner membrane; as a result, protons cannot be sequestered outside the matrix. Without the proton gradient, no flow of protons can occur through ATP synthase to regenerate ATP. Since most ATP in typical cells is produced via the action of ATP synthase during oxidative phosphorylation, ATP production will decrease by over 50%. However, the electron transport chain will continue; it simply will no longer be "coupled" to ATP production.

destruction of dopaminergic neurons

parkinson's disease

Keq is not effected by

pressure changes

Keq varies with

temperature

A lipid is saponified to yield two fatty acid salts and one glycerol-based molecule. How many equivalents of NaOH were needed for this reaction?

2 In a saponification reaction, OH- acts as a nucleophile and attacks one of the carbonyl carbons to cleave the associated ester bond. Since one equivalent of NaOH is required to hydrolyze each ester, and since the question stem describes the freeing of two fatty acid chains, two equivalents are required.

A researcher obtains pure glyceraldehyde 3-phosphate and uses it to mimic glycolysis and gluconeogenesis in vitro. If the researcher uses 6 moles of G3P, what is the maximum number of moles of pyruvate or moles of glucose that can be generated?

6 moles of pyruvate or 3 moles of glucose For every molecule of glucose, two three-carbon G3P intermediates may be produced, which then result in two pyruvate molecules as products. Thus, following this ratio, six moles of G3P correlates to six moles of pyruvate. Traveling in the reverse direction, as would be expected during gluconeogenesis, would yield only three moles of glucose.

A scientist holds the personal belief that dreams are sequences of events randomly ordered by the brain while we sleep. This belief most closely coincides with which theory of dreams?

Cognitive dream theory C is correct. The cognitive dream theory posits that dreams are just stream-of-consciousness scenarios that are randomly ordered by the dreamer's brain.

A square, closed segment of copper wire is placed in an external magnetic field of strength B that points into the page. If the strength of the field is decreased to 0.5 B, what will happen to the wire?

It will experience a clockwise current. This scenario relates to electromagnetic induction and to Lenz's law. This law, which brings to mind conservation of energy and even Le Châtelier's principle, states that a change to a magnetic field will always generate a current that counteracts that change. Here, the external field is becoming weaker, or less heavily positioned into the page. To resist this change, it will induce a current that promotes a field pointing the same direction, into the page. The right-hand rule tells us that this current must travel clockwise.

Paclitaxel is a drug often used during chemotherapy to treat various cancers. This compound binds to the beta-tubulin subunits of microtubules, stabilizing the tubulin polymer and preventing its disassembly. Which phase of mitosis does this affect?

Metaphase; stabilizing tubulin structures would prevent chromosomes from achieving the required configuration on the spindle apparatus.

What structural elements make AT-rich sequences especially appropriate targets for initiation proteins during replication?

The interaction between adenine and thymine involves fewer hydrogen bonds than that between cytosine and guanine.

How does the radius of curvature of a convex lens relate to its power?

The two quantities are inversely proportional, and can be related using the equation r = 2/P. D is correct. The larger the radius of curvature, the thinner - and weaker - a lens will be. This relationship can be described mathematically using two known equations, r = 2f and P = 1/f. Rearrangement yields f = 1/P, and substitution into the first equation gives r = 2/P.

All Gestalt principles are governed by the "law of Prägnanz." To what does this refer?

The underlying principle that, in organizing perceived objects, we will always seek to make things as precise, simple, symmetrical, and meaningful as we can

It has recently been discovered that dogs have very similar attention patterns to humans. A dog may hear his name being called, but continue to focus on a hole he is digging. However, when he hears his name called more angrily, he might stop his task and decide to come inside. What model of attention best suits this scenario?

Treisman's attenuation model, because the dog can initially focus on digging, but perceives the stimulus of the human voice to intensify as it grows angrier

A spring system is shown below, with m representing the mass attached to the spring, k representing the spring constant, and x denoting the distance stretched or compressed from the spring's equilibrium position. What expression gives the minimum value for the magnitude of velocity of the mass after the system is stretched and released?

0 A is correct. Despite the variable-heavy answers, no calculations are needed! The minimum value for the magnitude of velocity of the mass is zero. In either a spring or a pendulum, a mass will reach a zero velocity at the extremes of its oscillation. In other words, velocity must be zero when the mass is changing directions. At these points, the mass has maximum potential and no kinetic energy.

Perchloric acid is a superacid used in the production of rocket fuel. If two moles of perchloric acid are mixed with four liters of water, the normality of the resulting solution will be closest to:

0.5 N. As a strong acid, perchloric acid is a compound that you should be familiar with for the MCAT. It has a formula of HClO4, making it a monoprotic acid. For this reason, its normality is the same as its molarity, or (2 moles HClO4) / (approximately 4 L total solution) = 0.5 M = 0.5 N.

If a noise machine requires 100 W to produce a continuous 10-dB sound, how much power is needed to produce a continuous 80-dB sound?

103 MW Each 10 dB represents a tenfold increase in sound intensity. Thus, from 10 dB to 80 dB (Δ70 dB), we need to raise intensity by a factor of 107. The original intensity is 102 W, so (102)(107) = 109 W, or 1000 MW.

What is the pH of a 0.005 M solution of KOH?

11.5 First, remember that pH + pOH = 14 at room temperature. The pH of a 0.001 M solution of strong acid would be 3, so we can assume that 0.005 M KOH (a strong base) would have a pOH around 2.5. Its pH, then, must be around 11.5.

If a molecule of palmitic acid (C16H32O2) fully undergoes beta-oxidation, how many molecules of acetyl-CoA will be generated?

8 molecules Acetyl-CoA is produced in the final step of beta-oxidation, when it forms alongside a new fatty acid that is reduced in length by two carbons. This process can then repeat. Thus, to find the number of acetyl-CoA molecules that will be produced from complete reaction of a fatty acid, simply divide the number of carbons by two. In this case, palmitic acid has 16 carbons, which will yield 8 molecules of acetyl-CoA.

Which disease or trauma would most likely result in damage, whether permanent or temporary, to a patient's vestibular sense?

An inner ear infection C is correct. The vestibular sense, effectively, is the sense of balance. This is controlled by the cochlea and the inner ear. Additionally, it does not relate to any of the other three choices.

In addition to the role depicted in the figure, which of the following is likely performed by PLC?

Conversion of phosphatidylcholine to arachidonic acid A is correct. The figure shows PLC bound to GTP, or guanosine triphosphate. Judging from its name, phosphatidylcholine contains a phosphate moiety, which would likely be recognized by PLC. Arachidonic acid is a lipid without a phosphate moiety. Overall, choice A best resembles the figure; in fact, PLC does perform this reaction in the course of mediating the immune response.

The treatment of patients with multiple neurological or psychiatric disorders is made more difficult by the fact that some disorders have opposing effects on the underlying brain chemistry. The treatment of a patient with both schizophrenia and Parkinson's disease would be complicated by the fact that these disorders are related to imbalances in production and sensitivity, but in opposite ways, of which neurotransmitter?

Dopamine C is correct. It is thought that hypersensitivity of dopaminergic receptors in multiple areas of the brain is responsible for the symptoms of schizophrenia. Conversely, in Parkinson's disease, neurons in the substantia nigra release insufficient amounts of dopamine, causing issues with movement. The administration of dopaminergic antagonists to treat schizophrenia can result in Parkinson's-like symptoms.

How does the behavior of the centromeres during meiosis differ from that during mitosis?

During meiosis, the centromeres do not split during the first incarnation of anaphase. A is correct. The centromeres are the attachment sites for the spindle fibers to pull apart the chromosomes during anaphase. This occurs during anaphase II in meiosis, as anaphase I only involved the segregation of homologous chromosomes (and not sister chromatids) into daughter cells.

After a molecule has been engulfed via endocytosis, in what order does it progress through membrane-bound compartments before it is degraded?

Early endosome, late endosome, lysosome During endocytosis, external molecules or pathogens are first engulfed in an invagination of the cell membrane known as a vesicle. These vesicles initially deliver their contents to early endosomes, which are also membrane-bound. These contents then progress to late endosomes, which fuse with lysosomes for degradation.

Which pair of gases is most likely to produce ideal gas-like behavior?

HCl and CH4 The most ideal gases are those that experience the weakest intermolecular forces. HCl and CH4 have the least polar bonds of the gases listed and thus will exert the weakest attractive forces on other molecules.

ACE inhibitors are a common class of drugs that prevent the conversion of angiotensin I to angiotensin II. What is a likely effect of prescribing this drug?

Increased potassium reabsorption Angiotensin II also increases aldosterone levels, and aldosterone excretes potassium. Lower aldosterone levels due to ACE inhibitors would then raise potassium levels.

The mixture of ethylamine and propionyl chloride should create:

N-ethylpropionamide. A is correct. N-ethylpropionamide is an amide. This type of molecule results from the combination of an alkyl amine and an acyl chloride, such as propionyl chloride.

If C-14 undergoes two sequential beta-minus decays, what will the daughter isotope be?

Oxygen-14 In beta-minus decay, an electron is emitted as a neutron converts to a proton. The atomic mass remains the same, and the atomic number increases.

Which mechanism of socialization is used by the military to change the behavior of new recruits?

Resocialization Resocialization is the process by which one's sense of social values, beliefs, and norms are reconditioned. This process is achieved through an intense social process that often takes place in a total institution.

Which of the structures below is NOT an exocrine gland?

The adrenal gland Of the options listed, only the adrenal gland has no exocrine function. The liver and pancreas secrete many hormones into the bloodstream, but also release digestive enzymes into the lumen of GI tract, an exocrine function. Salivary glands are also exocrine glands.

A certain astronaut has lived aboard the International Space Station for more than two years. If he begins to having trouble sensing rotational acceleration, which structure in his ear may be damaged?

The ampullae

Of the following statements, which gives the best description of the Bohr model of the atom?

The atom contains a positively-charged nucleus surrounded by electrons that orbit the nucleus in defined paths, similar to planetary bodies that circle the sun.

A number of scientists are attempting to study the effects of mutating the genes coding for cadherin proteins in cancer cells. After assessing several random mutations, they find a cell culture that does not form desmosomes correctly. However, the gene coding for cadherins appear to be normal. What other cellular function must have been affected by the mutation?

The cell has lost the ability to make intermediate filaments. D is correct. Desmosomes are cellular junctions that are composed of cadherins. Since the cadherin gene is intact here, something else must be wrong with the cell. Desmosomes are anchored to the cytoskeleton via the intermediate filaments. Therefore, dysfunctional intermediate filaments would result in the inability to form desmosomes and is likely involved here.

What will be the effect on the pH of 1 L of pure water at 298 K if heat is added to the system?

The pH will decrease. The dissociation of water into H+ and OH- is an endothermic process, so increasing the temperature will yield more H+ and OH- ions(products). While the water will still be neutral, both pH and pOH will be lowered. (Note that many students have the misconception that pH + pOH is always equal to 14! This is not true; instead, pH + pOH = pKw, which is equal to 14 only under standard conditions. Raising the temperature increases Kw, which decreases pKw. Since pKw is now lowered, both pH and pOH can decrease, and both do so to the same extent.)

At 75 °C, which molecule will be more abundant?

The molecule on the left The keto form is almost always more stable than the enol form, with the exception of special circumstances, such as cases in which formation of the enol confers aromaticity on the molecule.

Which reaction from the TCA cycle can be coupled with the conversion of NAD+ to NADH?

The transition from NAD+ to NADH is a reduction reaction, and thus must be coupled with an oxidation reaction. Choice D shows a reaction in which the hydroxyl group in the reactant is oxidized to a carbonyl group in the product.

In a particular relationship, doubling thing A is shown to halve thing B, while reducing thing A notably increases thing B. How can this interaction be described?

The two factors are negatively correlated, and thing A is the independent variable.

In any casino you enter, you will see people playing slot machines, hoping for a payout. The operation of these machines is based on which system of reinforcement?

Variable-ratio Variable-ratio reinforcement is a schedule of reinforcement wherein a reinforcer is provided following a pre-determined average number of responses.

refers to the body's specific targeted response against invading pathogens

adaptive immunity

Urea, a metabolic waste product, contains which of the following functional groups? I. Amide II. Imide III. Imine

amide

An unconditioned stimulus can accurately be described in all of the following ways EXCEPT:

as an event or object that later becomes conditioned. C is correct. An unconditioned stimulus is one that naturally produces a particular response (for example, food causing Pavlov's dogs to salivate). In contrast, a conditioned stimulus does not evoke an innate reaction, but was associated with the unconditioned stimulus until that reaction was "learned." A neutral stimulus, not an unconditioned stimulus, becomes conditioned.

Of the following, the only diamagnetic element is:

cadmium. D is correct. Diamagnetic materials are those that contain no unpaired electrons. Cadmium, which possesses five fully-filled d orbitals, is the only diamagnetic element of the choices given.

states that physiological arousal and emotion happen simultaneously and separately

cannon bard

Scurvy, a form of malnutrition caused by an absence of vitamin C, is infamous as an ailment of sailors and other people who lack access to fresh food for long periods. Scurvy is characterized by a weakening of connective tissues in the gums and other mucous membranes, which will frequently display bleeding under slight stress. Vitamin C must therefore be key to the synthesis of which component of most connective tissues?

collagen

A mirror with a focal length of +5 m could be described as:

concave and converging. All mirrors or lenses with positive focal lengths represent converging systems. Converging mirrors are concave, while converging lenses are convex.

focuses on health factors that affect a population in a defined geographic location

ecological study

During the Gabriel synthesis of primary amines, the primary carbon serves as the:

electrophile. During Gabriel synthesis, the nitrogen of a phthalimide molecule acts like a nucleophile, attacking the electrophilic primary carbon of an alkyl halide.

If you wished to convert octanoic acid into a significantly more reactive molecule, you would most likely:

expose it to SOCl2. Thionyl chloride (SOCl2) is a reagent commonly used to convert carboxylic acids into acyl halides. This particular reaction would produce octanoyl chloride, which (like other acyl halides) is highly reactive due to its halogen leaving group.

During human embryogenesis, limbs develop from limb buds. A scientist has discovered that he can specify which limb (arm or leg) that a bud differentiates into by:

expressing different limb-specific transcription factors. The scientist has exploited pattern formation to specify which limb arises from a particular bud. While cells in the limb buds receive the same signals, they are interpreted differently to specify the limb that actually forms. This requires limb-specific transcription factors, which activate the transcription of specific genes to initiate differentiation.

a molecule that regulates gene expression by increasing transcription rate

inducer

All cells in a developing organism are initially identical. However, during embryogenesis, they are able to differentiate into various cell types and localize using positional information. This phenomenon, known as pattern formation, requires all of the following EXCEPT:

maternal growth factors. In healthy developing organisms, maternal growth factors do not pass through the placenta. Thus, they play no role in pattern formation.

A pharmaceutical researcher must choose a compound with the worst possible leaving group to prevent unwanted side reactions in a certain synthetic procedure. Of the following, he should use:

methyl ethanoate

Of the choices below, the only one that is LESS reactive than isobutyric anhydride is:

methyl propanoate. Isobutyric anhydride is an acid anhydride, one of the more reactive of the carboxylic acid derivatives. Esters, in contrast, are moderately unreactive, since their leaving groups are extremely unstable in solution. Methyl propanoate is an ester, as is evident from its nomenclature

The most widely posited explanation for motion sickness is provided by:

sensory conflict theory. Motion sickness is thought to arise when sensory input from the semicircular canals (angular motion) and otoliths (linear motion) are in conflict with sensory input from the eyes and muscles.

A student combines 50 mL of 0.80 M HF with 40 mL of 0.75 M KOH. He predicts that, since he is mixing a weak acid with a strong base, the final pH of the solution will be greater than 7. However, a piece of blue litmus paper dipped into the beaker immediately turns red. Which choice accurately describes the student's mistake?

pH depends on the concentrations of H+ and OH-, regardless of acid and base strength, and the final solution contains more moles of H+ than OH-.

William recently purchased a new cell phone with an unfamiliar ringtone. At first, he jumps up and reaches for his phone at the sound of any musical tones, but later begins to react only to his phone's unique melody. William is experiencing:

stimulus discrimination, since a learned association becomes more specific. C is correct. This question actually relates to classical conditioning, not to memory (which the other choices focus upon). William has a conditioned response to his phone's ringtone that later becomes more specific; this is the definition of stimulus discrimination.

This figure depicts proline, one of the twenty standard amino acids. Its uniqueness gives it a specific role in the formation of protein secondary structure. The structural element of proline that enables this role is:

the cyclic structure of its side chain bound to its own amino group, which gives it rigidity. D is correct. Notice that proline's side chain is actually bound to its amino group instead of projecting outward from the alpha carbon. This cyclic side chain results in a strict 60° angle between proline and neighboring amino acids. Proline also plays a large role in beta turns.

According to George Herbert Mead, the "I" is best characterized as:

the free will that one uses to uniquely respond to a situation, generally within the constraints of societal norms. A is correct. Mead proposed the concepts of the "I" and the "me," which make up two parts of his "theory of the social self." With the mention of free will and individuality, this choice is the best description of Mead's "I." The "I" is a person's creative, independent response to social situations. Note, however, that the actions of the "I" are limited by the ideas of socially proper behavior that have been internalized by the "me."

The figure below shows a developing chordate. Image adapted from OpenStax College under CC BY 3.0 The name and function of the circular tube, indicated by the arrow in the figure above, is:

the notochord; it develops into the vertebral column. The notochord is located ventral to the neural tube (in other words, below the circular structure marked in the figure). The notochord provides the primitive axis of the developing embryo and, in vertebrates, develops into the vertebral column.

When the smallpox vaccine was created in 1798, the first person to receive the vaccine demonstrated an example of:

artificial active immunity.

Mental organization patterns that help us make decisions or perform complex actions include: I. scripts. II. schemas. III. working memory. IV. heuristics.

scripts, schemas, and heuristics

The function -cos(t) is how many degrees out of phase with sin(t)?

-270° A is correct. The function cos(t) is 90° "behind" sin(t) (try looking at a graph of both functions to confirm this). Additionally, -cos(t) is perfectly out of phase with cos(t), meaning that the two are separated by 180°. Adding these two phase lags yields a total of 270°. The answer is negative since it is a phase lag, meaning that -cos(t) is staggered 270° behind sin(t).

Which of the following membrane potentials is logical for a sensory neuron that is experiencing depolarization?

-45 mV Depolarization refers to any phase in which the cell becomes more positive (or less polarized) than its resting value. Unless told otherwise, we can generally estimate that the resting membrane potential for a typical neuron is -70 mV. Thus, -45 mV, which is less negative, is a value that could certainly be attained during depolarization.

Although social norms vary from community to community, many cultures administer similar consequences for disobeying their specific norms. Which of the statements below is / are true? I. Formal deviance generally results in legal sanctions. II. Informal deviance generally results in legal sanctions. III. Informal deviance generally results in social sanctions. IV. Both informal and formal deviance can result in stigma.

. I, III, and IV D is correct. Deviance is a word used for the violation of social norms. Specifically, formal deviance is the disobedience of formal norms, including laws; this behavior results in legal sanctions such as fines and imprisonment. Predictably, informal deviance involves the breaking of less explicit norms, such as societal standards and expected roles. This form of deviance results in social sanctions that include shame, criticism, and disapproval. Finally, either type of deviance may result in social stigma, making statement IV correct.

The moon's orbit around the Earth is approximately circular. How much work is done on the moon by the force of gravity when the moon completes half an orbit around the Earth? (Note: radius of moon's orbit: 3.85 × 105 m; mass of Earth: 6 × 1024 kg; mass of moon: 7.3 × 1022 kg; moon orbital time: 27 days)

0 J B is correct. For an object moving in a circle, the force is towards the center of the circle, but the velocity vector points at a tangent out from the circle. This makes the angle between the force and velocity vectors 90º. The equation for work is W = Fd cosθ. The cosine of 90º is zero, so the work done by a force on any object moving perpendicular to the force is always zero. Hence, the work done by a centripetal force on an object moving in a circle is zero.

Sulfur trioxide acts as an electrophile in sulfonation reactions. When the sulfur atom is attacked by a benzene ring, a single bond forms and one oxygen atom loses its double bond character. At that time, the formal charges on the sulfur atom and the single-bonded oxygen are, respectively:

0 and -1. D is correct. In the original state of sulfur trioxide, both sulfur and oxygen have formal charges of zero. If sulfur gains an extra bond from the benzene ring while losing one to oxygen, it retains the same formal charge. However, because the oxygen gains two nonbonding electrons while losing two bonding electrons, its formal charge changes. We can calculate it as 6 - (6 + 1) = -1.

When 100 mL of 5 M phosphoric acid is completely deprotonated by water at high temperature, what is the mole fraction of the phosphate anion in solution? Note that the solution contains 18 mol of water.

0.025 Since phosphoric acid is triprotic, the complete deprotonation of 0.5 moles of H3PO4 will require 1.5 moles of hydroxide, or the full amount present. Each of these OH- molecules will gain a proton, leaving us with 19.5 moles of water and 0.5 moles of PO43-. The total number of moles in solution is 19.5 + 0.5, or 20 moles. Thus, the mole fraction of PO43- is 0.5 / 20, or 0.025.

At 0.5 dB, a sound does not even stir the hair cells in an animal's inner ear. However, the sound intensifies, reaching 2 dB, at which point the associated signal is sent to the animal's central nervous system. Finally, the sound further loudens to 130 dB, causing the animal discomfort before gradually returning to 0.5 dB. At what point is the animal's absolute threshold reached?

0.5 dB B is correct. The absolute threshold is the point at which the sensory system is activated, but not necessarily when the sound is perceived.

Enzyme H, a macromolecule that plays a crucial role in metabolism, is under study by a team of researchers. They observe that the apparent Km of the enzyme-catalyzed reaction is 0.76 mM in the presence of a noncompetitive inhibitor, Compound P. In the absence of any inhibitor, the Km of the same system is closest to:

0.75 mM. Noncompetitive inhibitors do not change the apparent Km of their associated enzymes, though they do decrease the maximum rate of reaction. 0.75 mM is the closest choice to the Km given by the question stem.

The following equation depicts the net reaction for the Krebs cycle. Acetyl-CoA + NAD+ + FAD+ + GDP + Pi + H2O ! CO2 + NADH + FADH2 + GTP + H+ + CoA What are the correct stoichiometric ratios?

1 Acetyl-CoA, 3 NAD+, 1 FAD+, 2 CO2 , 3 NADH, 1 FADH2, 1 CoA D is correct. The net reaction below is written with correct stoichiometric ratios. Acetyl-CoA + 3 NAD+ + FAD+ + GDP + Pi + 2 H2O ! 2 CO2 + 3 NADH + FADH2 + GTP + 2H+ + CoA

If the energy of a photon is 2 × 10-20 J and Planck's constant is equal to 6.62 × 10-34 m2⋅kg/s, the photon's wavelength is:

1 x 10-3 cm The equations E = hf and c = λf can be combined into E = hc/λ, which can then be rearranged into λ= hc/E. Plugging the values into the equation yields: 𝝺 = (6.62 x 10-34 m2 kg s-1)(3 x 108 m s-1) / (2 x 10-20 J) = (6.62 x 10-34 m2 kg s-1)(3 x 108 m s-1) / (2 x 10-20 kg m2 s-2) = (6.62 x 10-34)(3 x 108 m) / (2 x 10-20) = 9.93 x 10-6 m = 1 x 10-5 m rounded to 1 significant figure = 1 x 10-3 cm unit conversion (mult by 102 cm/m)

A hypothetical neuron possesses a high number of voltage-gated potassium channels. Imagine that one of these ion channels is open for 10 ms, during which it lets 1 mmol of K+ ions out of the cell. What amount of current passes through the channel during this interval? (Note: an electron's charge is -1.60 × 10−19 C)

1 × 104 A The current is given by: The amount of charge passing through the channel / 𝚫t = (# of K+ ions)(charge per K+ ion) / 𝚫t = (1 x 10-3 mol)(6.02 x 1023 ions/mol)(1.60 x 10-19 C) / 𝚫t = (6.02)(1.60) x 10-3 + 23 - 19 C / 10 x 10-3 s = 10 x 101 C / 10-2 s = 1 x 104 A

An engineering student fashions a very large object with the intention of studying buoyancy. The object is cube-shaped with a side length of 5 m. The top half of the cube is composed of aluminum (ρ = 2712 kg/m3), while the lower half is made of dogwood (ρ = 750 kg/m3). The student places the object in a tank of hexanol (ρ = 811 kg/m3). The student decides to try placing the cube on an underwater scale on the floor of the hexanol tank. What reading will the scale give for the cube's weight in hexanol?

1.1 × 106 N Using Fb = ρVg, we know that the buoyant force on the cube is 1 × 106 N. However, we also need to know the weight of the cube in air, for which we need to calculate its mass. Since the volume of the cube is 125 m3, 62.5 m3 is composed of aluminum and 62.5 m3 is made of dogwood. For the aluminum, (62.5 m3)(2712 kg/m3) is approximately (6.0 × 101)(2.7 × 103), or 16.2 × 104 kg. For the dogwood, (62.5 m3)(750 kg/m3) is about (6.0 × 101)(7.5 × 102), or 4.5 × 104 kg. Finally, we can add these values to obtain a total mass of around 20.7 × 104, or 2.1 × 105, kg. This yields a weight of 2.1 × 106 N. Last of all, the apparent weight of the cube in hexanol is the difference between the actual weight and the buoyant force, or 1.1 × 106 N.

A 0.1 cm3 sample of nitroglycerin has a mass of 160 mg. What is the density of nitroglycerin?

1.6 × 103 kg / m3 Density = mass / volume. We are given both mass and volume, but neither is written in the same units as the answer choices. First, then, let's convert volume to cubic meters. Remember, even though 100 centimeters equals one meter, 100 cubic centimeters does not equal one cubic meter! Instead, one cubic meter equals (100 cm)3, or 106 cubic centimeters. In other words, one cubic centimeter equals 10-6 cubic meters. 0.1 cm3 = 0.1 × 10-6 m3 = × 10-7 m3. Next, 160 mg = 0.16 g = 0.00016 kg = 1.6 × 10-4 kg. Finally, 1.6 × 10-4 kg / 10-7 m3 gives our answer.

At 760 mmHg and 25°C, the solubility of CO2 gas in soda is 2.5 x 10-5 M. In Gnome, Alaska, the atmospheric pressure is 0.750 atm. What would be the expected solubility of CO2 gas in soda in Gnome at 25°C?

1.875 x 10-5 M The solubility of a gas is directly proportional to the partial pressure of that gas above the solution in which it dissolves. If the partial pressure in Gnome is 0.75 times the pressure in the original situation, the solubility in Gnome will be 75% of the original solubility.

DNA replication is understood to be semiconservative. To ascertain this, a geneticist radiolabeled a fragment of dsDNA and allowed it to replicate. After four successive replication cycles, what fraction of the total DNA consists of the original parent material?

1/16 C is correct. In semiconservative replication, the parent dsDNA is denatured so that each strand can be used as a template to form a new piece of DNA. As a result, the two dsDNA products of the first replication will each contain one strand of original DNA and one new strand. After the fourth replication, a total of 16 dsDNA molecules will exist, or 32 single strands. Of those molecules, only two of them come from the original parent DNA. Thus, our fraction is 2/32, or 1/16.

The instance of nondisjunction for the X chromosome in females over the age of 30 is about one out of every 130 live births. If a woman over 30 is pregnant, assuming that the risk of nondisjunction from the father is negligible, what is the approximate likelihood that the baby will be born with a trisomy in its sex chromosomes?

1/200 C is correct. The probability of nondisjunction is 1/130, or about 0.008. Assuming that this happens, the mother may have a baby with Turner syndrome (X0), triple X (XXX), or Klinefelter syndrome (XXY). If the child is Y0, it will not be viable and the pregnancy will be terminated far before term. We are asked for trisomies, which include only triple X and Klinefelter syndrome. So, if she has a viable baby after nondisjunction, it has a 2/3 chance of having a trisomy. Multiplying the probabilities as follows yields our answer: (1/130)(2/3) = 0.005 or a 0.5% chance.

Suppose an organism contains an enzyme that can insert one random string of nucleotides into the coding portion of a gene. If this enzyme performs its function once on a particular gene, what is the probability that the resulting protein will be translated in the same frame as it would be if unmodified?

1/3 If this enzyme inserts a truly random number of bases, it will only add a multiple of three bases one-third of the time (because only one-third of all whole numbers are multiples of 3). Whenever it inserts a different number (one, two, four, five, seven, etc.), the reading frame will change. This leaves a one-third chance that the frame will remain unmodified, and a two-thirds chance of a shifted frame.

A copper wire with resistance of 0.25 Ω is attached in series to a 2 V battery and a 5 Ω resistor. An ammeter (Rint = 1 mΩ) is attached to the circuit and the current is read. If the wire is replaced with another copper wire with half the diameter, what will be the new current reading?

1/3 A Remember that for a wire, R = ρL/A. When the diameter of the wire is halved, that also halves the radius (since d = 2r, diameter and radius are directly proportional). The area of a circular wire is equal to πr2, so halving the radius reduces the area to one-quarter of its previous value. According to the equation for resistance given above, this would quadruple the resistance to 1 Ω. The total resistance of the circuit is now 5 + 1 = 6 Ω. Using Ohm's law, we get I =V/R = 2/6 = 1/3 A.4/11

Robert is standing on a bathroom scale as he is riding in an elevator that is accelerating downwards at ¾g. His apparent weight, as measured by a bathroom scale, would be what fraction of his true weight?

1/4 We can set up the equation F = ma, with a representing the net acceleration of the elevator. This net force is a result of the upward and downward forces on the scale/elevator system. If we define up to be positive and down to be negative: ma = FN - Fg. FN is the normal force, which represents the force upwards on Robert from the scale, aka the weight read on the scale (normally this should be equal to mg). FN = Fg + ma; since Fg = mg, we can say that FN = m(a + g). Defining a as negative, the question tells us that a = -(3/4)g. We can then rewrite this expression as FN = m(g-3/4g) = (mg)/4, which means that his weight will appear to be 1/4 of the normal weight. Conceptually, we can also think of it this way: if the elevator were to be accelerating downward at -g, his apparent weight would be zero. The acceleration being -3/4g means that it is most of the way to being -g, meaning that most of his apparent weight will "disappear." Since this is a linear relationship, we can conclude that he will experience 1/4 of his normal weight. We do not have to know his specific weight to derive this proportion.

For carbonic acid, Ka1 = 4.3 × 10-7 and Ka2 = 5.6 × 10-11. What is the pH of a solution made with equimolar amounts of sodium bicarbonate and potassium carbonate?

10.25 If this solution contains equal concentrations of HCO3- and CO32-, we must be at one of the titration's half-equivalence points. Here, pH must equal pKa - but which one? Since the first proton on H2CO3 has already been removed, we are at the second half-equivalence point, and the relevant pKa is pKa2. pKa2 = -log(Ka2), which should fall between 10 and 11.

A small metal ball (m = 0.5 kg) is launched off a spring at a 30° angle to the ground, as pictured in the diagram below. Friction and air resistance are negligible in this case. If the ball has a horizontal velocity of 17.3 m/s after one second of flight, what was its initial kinetic energy? Note that sin(30°) = 0.5 and cos(30°) = 0.866.

100 J B is correct. Initial kinetic energy can be quickly calculated if we know the total initial velocity of the ball. Luckily, we are given horizontal velocity, which remains constant throughout a projectile arc as long as air resistance can be ignored. For that reason, we know that the initial vx is also equal to 17.3 m/s. In other words, vtotal(cos 30°) = 17.3; cos 30° = √3/2, or about 0.866. Finishing that calculation tells us that the total initial velocity of the ball was 20 m/s. KE = ½ (0.5 kg)(20 m/s)2 = 100 J.

Throughout both the Krebs cycle and oxidative phosphorylation, how many molecules of ATP are formed per molecule of acetyl-CoA?

12 Acetyl-CoA is fed into the Krebs cycle to produce three NADH, one FADH2, and one GTP molecule, the last of which is immediately converted to ATP. Each NADH is later used to form three ATP molecules, while the single FADH2 unit is utilized to synthesize two. In total, 12 molecules of ATP are created.

A sound wave with a frequency of 350 Hz is emitted from a speaker placed at x = 0 m. Another speaker, emitting a noise with the same frequency and decibel level, is positioned at x = d m as shown in the figure. What must the value of d be, if an observer standing at x > d hears a sound that has twice the amplitude of either individual sound? Assume that sound travels at 300 m/s.

12/7 m B is correct. This problem requires several steps. First, the question stem indicates that these waves display maximum constructive interference, meaning that they must be either 0 or 360 degrees out of phase. Additionally, we know from given information that the period of each wave is 1 / 350 s. Assume that the speakers are spaced a full wavelength apart. To determine how far this is in space, multiply the period by the velocity of the wave to get (1 / 350 s )(300 m/s), or 6/7 m. This is the distance that represents a 360° phase gap. In actuality, the speakers could be separated by any multiple of this distance. The only logical answer is 12/7 m, which denotes two full wavelengths.

Which of the following is closest to the bond angle between the carbons in a molecule of acetone?

120º Acetone (shown below) has a central carbon atom that is double-bonded to an oxygen and single-bonded to two carbon atoms. Since it is bound to three substituents and has no lone pairs, the hybridization around this carbon is sp2, creating a trigonal planar geometry and a bond angle of 120º.

Monochromatic light with a wavelength of 580 nm is incident on a screen with two thin slits. Light leaves these slits and shines on an optical screen, creating the effect seen below. A scientist measures the distance from one slit to a dark area, then measures the distance from the other slit to the same dark region. Which of the following could be the calculated difference between these two values?

1450 nm Since the area described is a dark region on the optical screen, it must result from destructive interference. This tells you that the two waves involved must be completely out of phase, or differ by a half-wavelength. Given that the wavelength is 580 nm, the difference between the distances traveled must be d = 290 + 580n where n= 1, 2, 3, etc.

An enzyme which catalyzes the reaction E + S ↔ ES → E + P is mixed with 5 mM of substrate. The initial rate of product formation is 25% of the reaction's Vmax. The Km for the enzyme is:

15 mM. To solve this problem, you must use the Michaelis-Menten equation: v0 = Vmax[S]/ (Km +[S]). Plug in Vmax/4 for v0 and 5 mM for [S]. The Vmax terms will cancel, and solving will yield 15 mM.

Ba(NO3)2 (aq) + H2SO4 (aq) 🡪 BaSO4 (s) + HNO3 (aq) How many molecules of product form when 5 molecules of barium nitrate completely react with 5 molecules of sulfuric acid?

15 molecules C is correct. This equation is not given in its balanced form. When we do balance it, we find that Ba(NO3)2 + H2SO4 🡪 BaSO4 + 2 HNO3. In other words, for every mole of barium nitrate, three total moles of product will form (two moles of HNO3 and one mole of BaSO4). Thus, for every 5 moles of barium nitrate, 15 moles of product will be created.

Monochromatic light with a wavelength of 640 nm is directed onto a screen with two slits. Light leaves these slits and shines on an optical screen, creating the effect seen below. A lab assistant measures the distance from one slit to a bright band, then measures the distance from the other slit to the same bright region. Which of the following could be the calculated difference between these two values?

1920 nm Since the area described is a bright region on the optical screen, it must result from constructive interference. This tells you that the two waves involved must be completely in phase, or differ by exact multiples of a wavelength. Given that the wavelength is 640 nm, the only logical differences between the distances traveled are 640 nm, 1280 nm, 1920 nm, etc.

The reduction half-reaction for the silver cation is Ag+ + e- → Ag(s). This process is used to electroplate an object using silver from solution. How long will it take to deposit 21.6 g of silver, given a current of 100 A?

193 seconds 21.6 g of silver is 0.2 moles. Since one electron can reduce one silver cation to Ag, we need 0.2 moles of electrons. Using Faraday's constant, calculate (0.2 mol)(9.65 x 104C/mol) = 1.9 x 104 C. Remember that one ampere is equal to one coulomb/s, so (1.9 x 104 C)/(100 C/s) = 190 seconds.

A 1-L beaker is filled with 2 moles of liquid methanol and 0.25 moles of NaCl. The ratio of ions to atoms in this solution is:

1:25. C is correct. Atoms are the tiny units that compose matter; they consist of electron(s) and a central nucleus. The term "ion" refers specifically to charged atoms. Luckily, the number of moles of a substance directly relates to the number of particles by way of Avogadro's number, so a direct comparison of moles is all we need to find our answer. However, two moles of a molecule is not equivalent to two moles of atoms. For example, since methanol has a formula of CH3OH, it contains a total of six atoms per molecule. In contrast, NaCl will dissociate into two ions: Na+ and Cl-. In this solution, we have 0.25 moles of NaCl, which can be thought of as (0.25 moles * 2) or 0.5 ions. For atoms, we have (2 moles * 6) + (0.25 * 2) or 12.5; remember that ions are technically atoms as well! This gives us a ratio of 0.5:12.5 or 1:25.

Which of the following represents the strongest oxidizing agent?

1s22s22p63s23p64s23d8 An oxidizing agent is a species that is reduced, meaning that it must "want" to gain electrons. Choice A is the only option in which the atom has a d valence subshell that is not half-or fully-filled. This represents a relatively unstable state that would tend to promote the gain of electrons. Note that choice D does have a half-filled 4s valence subshell, but this implies that it is an alkali metal, which would tend to lose that lone 4s electron rather than gaining any electrons.

A sample of an unknown gas produced by gut bacteria is analyzed in a laboratory. If the gas has a mass of 50 mg and occupies a volume of 2.5 × 104 cm3, what is its density?

2 x 10-3 kg / m3 C is correct. First, we need to convert the volume to cubic meters. 2.5 × 104 cm3 × (1 m3 / 106 cm3) = 2.5 × 10-2 m3. Then, 50 mg / 2.5 × 10-2 m3 yields 20 × 102 mg / m3, or 2000 mg / m3. Then to convert mg into kg, 2000 mg / m3 x (1 kg / 106 mg) = 2000 x 10-6 kg / m3 or 2 x 10-3 kg / m3.

The index of refraction of pure water at a temperature A is 1.5. What is the speed of light in pure water at this temperature?

2 x 108 m/s Remember that for light, n = c/v,where n is the index of refraction, c is the speed of light in a vacuum, and v is the speed of light in some medium, like water. 1.5 = 3 x 108/v so v = 2 x 108

17.5 g of ammonium hydroxide (NH4OH) is mixed with 250 mL of an unknown solvent with a density of 1.5 g/mL. The molarity of this ammonium hydroxide solution is:

2.0 M. Molarity is measured in moles of solute per liter of solution, so we first must find the moles of ammonium hydroxide present. 17.5 g NH4OH × (1 mol NH4OH / 35 g) = 0.5 mol NH4OH. Now, we can easily find the solution's molarity by taking 0.5 mol NH4OH / 0.25 L solution, or 2.0 M. Note that we can ignore the density of the solvent, as it is irrelevant to these calculations.

72 g of NH4Cl is dissolved in 1 kg of water under standard conditions. What is the molality of ions found in the resulting solution?

2.7 m Molality is defined as moles of solute divided by kilograms of solvent. We originally have approximately 1.35 moles of NH4Cl, but note that this question asks for the molality of ions, not of NH4Cl as a whole. NH4Cl fully dissociates to give 1.35 moles of both NH4+ and Cl- in the 1-kg solution. (1.35 mol + 1.35 mol) / (1 kg) = 2.7 moles/kg.

The masses of a proton and a neutron are 1.0072 amu and 1.0087 amu, respectively. If the measured mass of a deuterium nucleus (2H) is 2.0140 amu, how much energy must be released in the synthesis of one such nucleus? (1 amu = 1.66 × 10-27 kg)

2.8 × 10-13 J First, calculate the mass defect. A deuterium nucleus has one proton and one neutron, meaning that the total predicted mass of the nucleus is 2.0159 amu. Subtracting the actual mass of deuterium yields a mass defect of roughly 0.002 amu. The next step is to convert the mass to kilograms, the units required for the equation E = mc2. 0.002 amu × (1.66 × 10-27 kg/amu) is equal to about 3.3 × 10-30 kg. Finally, plug in to E = mc2. E = (3.3 × 10-30) (3 × 108)2 = (3.3 × 10-30) (9 × 1016) = 30 × 10-14, or 3 × 10-13, J, which is closest to choice B.

A Doppler ultrasound device uses the Doppler effect to determine the speed of the blood traveling through a vessel. The tech is holding the probe at a right angle to the direction of blood flow. Assuming the actual flow is 10 cm/s, what frequency would the ultrasound device measure for sound waves bouncing off the blood? (Note: frequency of ultrasound wave emitted: 20 MHz; assume human tissue is equivalent to water; speed of ultrasound in tissue: 1500 m/s)

20 MHz B is correct. The Doppler effect relies on the notion that one object is moving relative to another object. In the classic example of a car honking its horn, the pitch of the horn is higher (high frequency) when the car is approaching the listener and lower when the car is moving away from the listener (lower frequency). For an ultrasound device to detect a Doppler shift, the blood being measured must be moving towards the sensor or away from it. By holding the device at a right angle to the direction of blood flow, the technician is preventing the machine from detecting any Doppler shift. Thus the best answer is B.

A two-pulley system allows a dockworker to apply a 1000-N force over a horizontal distance of 2 m, while lifting a crate by 1 m vertically. How much gravitational potential energy does the crate gain? (Ignore the effects of friction.)

2000 J With simple machines, energy is conserved. The man puts in 1000*2 J of work, so the crate must gain that much potential energy.

An artificial lung is filled with various gases to reflect those present in the atmosphere on another planet. The lung contains equimolar amounts of nitrogen and oxygen, argon with a mole fraction of 0.20, and carbon dioxide with a mole fraction of 0.46. In this system, the percent composition by mass of argon is:

21%. To find the percent composition of argon in the balloon, the total mass of all other elements present must be calculated. For convenience, we can assume that there are 100 mol of gas in the balloon. Let's begin with argon itself, which has a mole fraction of 0.20. This is equivalent to 20 mol (0.20 × 100 mol), and since Ar has a molecular weight of approximately 40 g/mol, we can conclude that 800 g of Ar are present. For carbon dioxide, which has a molecular weight of 44 g/mol, 46 mol is equivalent to 2024 g. Finally, though we do not know the mole fractions of nitrogen and oxygen, we know that the remaining mole fraction of 0.34 is divided evenly between them, giving us 17 mol (or 544 g) of O2 and 17 mol (or 476 g) of N2. The percent composition of argon is thus (800 g Ar / 3844 g total mass) × 100, or about 21%.

Given an angle of incidence of 30°, what will be the angle of refraction of a photon striking a glass surface (n = √2) in air (n = 1)?

21° Using Snell's law, you should get 1*sin(30°) = √2*sin(θ). Since sin(30°) = 1/2, this simplifies to 1/(2√2) = sin(θ). At this point, don't panic! Since light is moving from a lower into a higher index of refraction, it must bend toward the normal, meaning that our answer must be less than 30°. However, our answer cannot be as low as 0, since sin(0) = 0, not 1/(2√2). Thus, we are left with 21° as the most reasonable choice.

What is the percent composition by mass of copper in a covered beaker containing equimolar amounts of copper (II) nitrate (molar mass 187.6 g/mol), sulfur (molar mass 32.06 g/mol), water, and nitrogen monoxide? Assume none of the species react in the beaker.

24% B is correct. Since the question states that equimolar amounts of all four substances are present, we can simply take the mass of copper in one mole of copper nitrate and divide by the sum of the molar masses of these species, including copper nitrate itself. The total of the four molar masses is 187.6 + 32.06 + 18 + 30, or about 267.7. Thus, the percent composition of Cu is equal to (63.5 / 267.7) × 100% = 23.7%. Note that rounding to 60 / 270 makes the math significantly easier and simplifies to 2 / 9, or 22%.

What is the effective nuclear charge of an electron in the n = 4 shell of iodine?

25 B is correct. The effective nuclear charge can be found by subtracting the number of electrons in all shells preceding the one in question from the nuclear charge (the number of protons). Shells 1-3 contain 28 electrons, while an iodine nucleus possesses 53 protons. Thus, the effective nuclear charge on the n = 4 subshell is (53 - 28) or 25.

A woman (but not her husband) suffers from Von Willebrand disease, an autosomal dominant condition that shows 50% penetrance. This condition is expected to be phenotypically expressed in what percent of her offspring?

25% A is correct. Individuals with autosomal dominant diseases are almost always heterozygotes, as the possession of two mutated alleles is nearly always lethal. Therefore, this woman has a 50% chance of passing the Von Willebrand allele to each of her children. Due to the mentioned penetrance value, such a child then has a 50% chance of expressing the trait. (0.5)(0.5) = 0.25, or 25%.

758 of 1000 individuals have a dominant trait in which they cannot differentiate between the colors periwinkle and lavender. Assuming that the population is in Hardy-Weinberg equilibrium, what fraction of the population consists of people homozygous for this allele?

25% The 758 individuals described are those who express the dominant allele, meaning that they can be either homozygous or heterozygous for the trait. If we round 758 to 750, we can estimate that 75% of the population has the inability to differentiate these colors; in other words, p2 + 2pq = 0.75. While this may be difficult to solve for p, we can easily use it to find q2 using the equation p2 + 2pq + q2 = 1. If 0.75 + q2 = 1, then q2 = 0.25 and q = 0.5. Thus, p + 0.5 = 1, and p = 0.5. Finally, we can use this value to find that p2 (the proportion of homozygous dominant individuals) is 0.25, or 25% of the population.

A weight is suspended from the ceiling by two massless ropes. If the force exerted by tension on Rope 1 is 15 N, what force is felt by Rope 2?

25.5 N Since the object is not moving (static equilibrium), the horizontal components of the tension force must be equal and opposite. Rope 1 has a tension of 15 N, giving it a horizontal component of (15 N)(cos 30) or around 13 N. Next, we can solve for the force along the second rope by setting 13 N equal to F2cos60. Solving for force gives us F2 = 13 / (cos 60), or 26 N. This value is closest to choice D.

A toy boat constructed of balsa wood (c = 0.7 J/g °C) is set in 1 L of hot water (c = 4.2 J/g° C). If all the water is brought into thermal equilibrium with 1 kg of balsa wood that is 30 degrees lower in temperature, how much will the temperature of the wood increase (°C)?

26 1 L of water has a mass of 1 kg, so the masses of the two objects are equal. When the water and the wood reach thermal equilibrium, their temperatures will be equal as well. If the water had the same specific heat value as the balsa wood, each would change an equal number of degrees; in other words, the temperature of the water would decrease by 15°, while the temperature of the wood would increase by 15°. However, the specific heat of the wood is much lower than that of the water, meaning the wood should experience a greater increase in temperature. Choice C (26°) is the only logical choice, as it is greater than 15° but less than the full 30°. Alternatively, remember that q = mcΔT, and note that the magnitude of the heat change (q) experienced by the water must equal that experienced by the wood. Thus, (mcΔT)wood = (mcΔT)water. The masses are the same, so m cancels, leaving us with: cwood ΔTwood = cwater ΔTwater (0.7 J/g °C)(ΔTwood) = (4.2 J/g° C)(ΔTwater) (ΔTwood) / (ΔTwater) = (4.2 J/g° C) / (0.7 J/g °C) = 6 Thus, the wood changes by 6 times as many degrees as the water does. The closest answer is C, which states that the wood changed by 26°, meaning that the water must have changed by 4°. 26 is roughly 6 x 4.

A molecule of particular interest to a chemical engineer is depicted below. A sample of this species is dissolved in deuterated chloroform. When an 1H NMR spectrum for this sample is obtained, it would show peaks with an integration ratio of:

2:2:1. Labile hydrogen atoms do not appear in NMR spectra due to a rapid exchange with protons in the solvent, so the H atoms bound to the nitrogens of the amide can be ignored. The two phenyl rings reside in identical chemical environments, so all protons of equivalent position on these opposing rings may be considered equivalent as well. We then see that three sets of protons exist: four identical atoms on the phenyl carbons nearest the amide structure, four more on the adjacent phenyl carbons, and two on the carbons that are para to the tertiary ring carbons. The ratio of 4:4:2 reduces to 2:2:1, making choice A correct.

Write a balanced chemical equation for the single-displacement reaction of chromium (III) nitrate with solid zinc.

2Cr(NO3)3 (aq) + 3Zn (s) → 3Zn(NO3)2 (aq) + 2Cr (s) C is correct. This choice correctly assesses the charges of chromium as +3, nitrate as -1, and zinc as +2. Additionally, it is properly balanced.

A one-step reaction between two alkenes can produce a cyclic structure like that shown below. Given the information above, determine the overall reaction order and the units of the rate constant.

2nd order; M-1s-1 D is correct. The question stem states that this is a one-step reaction, so we can use the equation to determine the rate law. This gives us rate = k[1,3-dibutene]2 - in other words, a bimolecular, or second-order, reaction. Since rate must be given in units of M/s while concentrations are given in M, the rate constant, k, must have units of M-1s-1.

The Krebs cycle involves the reaction of NAD+ and FAD with substrates to form NADH and FADH2. During an interval of time when 9 molecules of NADH are produced, how many molecules of FADH2 will be synthesized?

3 molecules During the Krebs cycle, each unit of acetyl-CoA forms three molecules of NADH and one molecule of FADH2. In other words, the NADH-to-FADH2 ratio is 3:1. During a time period when nine molecules of NADH are produced, then, three units of FADH2 must also form.

Sucrose is a disaccharide that can be cleaved into D-glucose and D-fructose. Using ATP as a phosphate source, hexokinase can phosphorylate D-fructose, forming a product that may then enter glycolysis. Beginning with the entry of fructose 6-phosphate, what will be the net ATP production of one round of glycolysis?

3 molecules Since fructose 6-phosphate has already been phosphorylated once, it will enter into glycolysis after Step 2. This will save one ATP molecule that was previously required to phosphorylate glucose, increasing the net production from two molecules to three.

On the H1 NMR spectrum for this species, how many distinct peaks should be visible?

3 peaks B is correct. To answer this question, we must determine the number of sets of chemically equivalent hydrogen nuclei on this molecule. The three protons on the leftmost carbon are chemically identical to each other and to those on the rightmost carbon, so they constitute one set of H nuclei. The tertiary carbons each possess a single proton, both of which are identical; finally, the protons on the -OH groups are indistinguishable as well. In total, three different "types" of distinct protons exist here.

A charged particle is accelerated from rest to 100 m/s by an electric field. This particle has a charge-to-mass ratio of 0.17 C/kg. If this acceleration occurs in 20 µs, how strong must the electric field be?

3 × 107 N/C First, set F = ma equal to F = qE and solve for E. This setup yields the equation E = ma/q. To calculate acceleration, simply divide the change in velocity (100 m/s) by the time (20 µs) to get a = 5 × 106 m/s2. Now, we need to decide what to do with the quantity m/q. However, note that this is the reciprocal of the charge-to-mass ratio, meaning that it must have a value of 1/(0.17) or 6! Multiplying these terms gives us 30 × 106, or 3 × 107 N/C.

What is a possible set of quantum numbers for one of copper's d-block electrons?

3, 2, -1, -1/2 Copper is located in the 3d block, giving it a principal quantum number of 3. Since the question stem refers specifically to the d-block electrons, the relevant azimuthal quantum number must be 2. Potential magnetic quantum numbers range from -2 to 2, while the spin number of any electron can only be +1/2 or -1/2.

A chemist uses mass spectrometry to analyze an unknown sample. Along with an M+ peak at approximately 102, he notes several peaks, one of which corresponds to an m/z value of 17. Of the listed choices, this sample is likely:

3-hexanol, and the peak at m/z = 17 corresponds to a hydroxyl group. D is correct. In mass spectrometry, the M+ peak correlates to the molecule as a whole after it has lost a single electron. For this reason, we can deduce that the molecular weight of the sample is 102 amu, identical to that of 3-hexanol. Additionally, the m/z ratio is equivalent to the mass of a particle divided by its charge, which is typically +1. As -OH has a mass of 17 amu, D is the best answer.

Four blocks of mass M are connected by a massless cord. If the cord is pulled from the left with a force of 15 N, causing all of the blocks to accelerate, what net force will be experienced by the block that is second from the right?

3.75 N A is correct. A 15-N force is being exerted on a system with a total mass of 4M. We can therefore plug in to Newton's equation for net force to yield 15 N = 4Ma. Next, solving for acceleration gives us a = (15 N) / (4M). Since all of the blocks are connected, they will accelerate at the same rate. Substituting an acceleration of (15 N) / (4M) and a mass of M (since we are now considering only one of the blocks) gives F = M[(15 N) / 4M] = 15/4. 15/4 N is equal to 3.75 N.

A student must memorize the following list of numbers: 303, 197, 216, 42, 304, 76, 112. Which of the numbers will he probably retrieve the most easily?

303 (due to the primacy effect) and 112 (due to the recency effect) B is correct. When presented with items in list format, people tend to remember the first and last entries better than any of those in the middle. This phenomenon is called the serial position effect. Specifically, the primacy effect relates to the first term while the recency effect relates to the last (or most recently heard).

What is the approximate gram equivalent weight of phosphoric acid?

33 g For an acid, the gram equivalent weight is the mass per acidic proton. H3PO4 has a molar mass of 3 + 31 + (16)(4) = 98 g/mol, or approximately 100 g/mol. However, H3PO4 has three acidic protons. (100 g/mol) / (3 protons) = about 33 g.

A combustion engine is 60% efficient. If the engine is used to lift a 100 kg crate 20 m, how much chemical energy, in the form of fuel, must have been supplied to the engine?

33,000 J Eoutput = W = mgh = (100 kg) (10 m/s2) (20 m) = 20,000 J But also, Eoutput = efficiency · Einput. Hence, where "Ɛ" stands for efficiency: Ɛ · Einput = 20,000 J Einput = (20,000 J) / Ɛ = (20,000 J) / 0.6 The following manipulations are for ease of hand-calculation: = (10,000 J) / 0.3 = (10,000 J) (1/ 0.3) = (10,000 J) (3.33) = 33,000 J

A circuit is constructed with a 12-V battery and four identical resistors, each with a resistance of 16 Ω, hooked up in parallel. What is the total power dissipated by the circuit?

36 W The question asks us to solve for power, which is measured in watts; choices B and C use the wrong units and can be immediately eliminated. To choose between the remaining answers, we must use one of the three simple power equations: P = IV, P = I2R, or P = V2/R. (These equations are all derived from Ohm's law, so you can use whichever one is most convenient for the situation at hand.) We already have voltage, and we can solve for resistance: 1/Rtot = 1/R1 + 1/R2 + 1/R3 + 1/R4 1/Rtot = 1/16 + 1/16 + 1/16 + 1/16 1/Rtot = 4/16 = 1/4 Rtot = 4 Ω Next, we must solve for power: P = V2/R = (12)2/4 = 144/4 = 36 W

845 of 1000 individuals have a specific olfactory receptor for sulfur, which is expressed by a dominant allele. Approximately what fraction of the population consists of people with two copies of this allele?

36% C is correct. The 845 individuals described are those who express the dominant allele, meaning that they can be either homozygous or heterozygous for the trait. If we round 845 to 840, we can estimate that 84% of the population has the sulfur receptor; in other words, p2 + 2pq = 0.84. While this may be difficult to solve for p, we can easily use it to find q2 using the equation p2 + 2pq + q2 = 1. Finally, we can calculate the percentage of homozygous dominant individuals by solving for p2.

Write a balanced chemical equation for the single-displacement reaction of iron (II) nitrate and aluminum, which forms aluminum nitrate and solid iron.

3Fe(NO3)2 (aq) + 2Al (s) 🡪 2Al(NO3)3 (aq) + 3Fe (s) D is correct. Since nitrate has a charge of -1, iron (II) nitrate must include two nitrate ions for each iron atom. On the product side, aluminum has a +3 charge, meaning that aluminum nitrate must include three nitrate anions. The pure metals (aluminum and iron) should both be neutral and unimolecular.

Chymotrypsin is an enzyme that selectively cleaves peptide bonds at the carbonyl end of aromatic amino acids. A peptide sequence is given below. LYPPGLWNKYATSYF How many potential chymotrypsin cleavage sites are located in the structure of this molecule?

4 First, recall that the three aromatic amino acids are tryptophan (W), tyrosine (Y), and phenylalanine (F). Next, remember that peptide sequences are always written in the N-to-C direction. In other words, the N-terminal residue is an L (leucine), while the C-terminal residue is phenylalanine. If we ignore that aromatic residue (since it is not attached to another amino acid at its carbonyl end), there are four sites where chymotrypsin can act.

How many resonance structures are exhibited by phenoxide, the conjugate base of phenol?

4 Phenoxide, which can also be described as a phenolate ion, has a number of distinct resonance structures. The five resonance forms of this anion are shown below.

Succinic acid (C4H6O4) is a dicarboxylic acid with a variety of uses. When 472 g of succinic acid is mixed with water to fill a 2 L container, what is the normality of the solution?

4 N Since succinic acid is a dicarboxylic acid, and since carboxylic acids readily become deprotonated, succinic acid is diprotic. Normality is a measurement that is very similar to molarity, except that it measures moles of equivalents per liter of solution. Here, 472 g C4H6O4 × (1 mol / 118 g C4H6O4) = 4 moles of C4H6O4, which possess 8 equivalents of protons. The normality is thus (8 equivalents of H+) / 2 L = 4 N.

What is the expected [H+] of human blood at physiological pH?

4 x 10-8 M A pH of 7 corresponds to an [H+] of 10-7 M, while a pH of 8 denotes an [H+] of 10-8 M. Physiological pH (7.4) falls roughly in the middle of 7 and 8, so our [H+] should fall roughly in the middle of 10-7 M and 10-8 M. The value that falls exactly between these numbers is 5 x 10-8. The closest answer choice to this value is 4 x 10-8.

What is the approximate charge carried by 4 moles of electrons?

4 x 105 C For this question, it is necessary to know Faraday's constant, which is the charge of one mole of electrons (9.65 x 104 coulombs). When this value is multiplied by four, it gives choice C.

A dentist wishes to observe the oxygen consumption of several bacteria found in the oral cavity in order to develop a more comprehensive dental plan for his patients. The following represent the the results of the study, where dots indicate bacterial growth in dense agar media. Two types of bacteria are common on tooth enamel: one that lives in areas open to the environment and another that uses the former bacteria as an air shield and lives between teeth under layers of secreted plaque for protection. In which test tube would you find each respective bacteria?

4, 2 The first bacterium is exposed to the environment; therefore, it may be either an obligate aerobe, a facultative aerobe, or an aerotolerant anaerobe. However, the second bacterium must be shielded from the environment to survive and must therefore be an obligate aerobe. Tube 1 indicates an obligate aerobe, Tube 2 an obligate anaerobe, Tube 3 a facultative anaerobe, and Tube 4 an aerotolerant anaerobe.

A researcher hypothesizes that the carboxylic acid below may have potential applications in the fuel cells of electric cars. According to IUPAC rules, the name of this molecule is:

4-hydroxy-3-methylbutanoic acid. C is correct. If we follow IUPAC naming conventions, the highest-priority group on this molecule is the carboxylic acid. We thus know that the rightmost carbon should be given the number "1" and that the suffix should be "-oic acid." The other functional groups ("hydroxy" and "methyl") are then listed in alphabetical order with the numbered carbon to which they are attached.

What is the kinetic energy of 60 cm3 of pure water if it is flowing at a velocity of 12 m/s? Assume that this fluid is mimicking ideal behavior.

4.3 J To solve this problem, simply remember that the kinetic energy per unit volume for a fluid is given by ½ ρv2. For this reason, its total kinetic energy is simply ½ ρv2 * volume. Make sure to memorize the density of pure water, which is 1000 kg/m3 (or 1 g/cm3, or 1 kg/L; you should know this value in multiple different units). Finally, ensure that your units align by converting volume to cubic meters. 1 cm3 = 1 × 10-6 m3, so 60 cm3 = 60 × 10-6, or 6 × 10-5, m3. [½ (1000 kg/m3)(12 m/s)2](6 × 10-5 m3) = (7.2 × 104)(6 × 10-5), or 4.32 J.

A 90-g cell phone is unfortunately dropped directly off the roof of a one-story building. If the phone takes 1 s to hit the ground, what was its gravitational potential energy while still on the roof?

4.5 J B is correct. First, we must find the height from which the phone dropped. To do this, we can use the kinematics equation Δx = vit + ½ at2. Initially, the phone had a vertical velocity of 0 m/s, so this equation simplifies to Δx = ½ at2. When we plug in 1 second as "t" and 10 m/s2 as "a," we see that the phone's initial height must have been 5 m. Since it has a mass of 90 g (or 0.09 kg), its initial gravitational potential energy is mgh, or (0.09 kg)(10 m/s2)(5 m) or 4.5 J.

The mass of a proton is 1.0072 amu, while the mass of a neutron is 1.0086 amu. Given that the measured mass of a helium nucleus is 4.0015 amu, how much energy is converted from mass during the formation of one such nucleus? (1 amu= 1.66 × 10-27 kg)

4.5 × 10-12 J First, calculate the mass defect. A helium nucleus has two protons and two neutrons, meaning that the total mass of the protons is 2.0144 amu while that of the neutrons is 2.0172 amu. Combining these values gives a predicted mass of 4.0316 amu. Because the actual mass is 4.0015 amu, the mass defect is roughly .030 amu. The next step is to convert the mass to kilograms, the units required for the equation E = mc2. 0.03 amu × (1.66 × 10-27 kg / amu) = 4.98 × 10-29, or about 5 × 10-29, kg. Finally, plug in to E = mc2. E = (5 × 10-29) (3 × 108)2 = (5 × 10-29) (9 × 1016) = 4.5 × 10-12 J.

What is the pH of a 1 M aqueous HCN solution?

4.6 HCN is a very weak acid, with a pKa of 9.2. To find our answer, we must discern how much H+ is formed from the dissociation of this species in water. Remember, for every H+ ion that forms, one CN- ion will form as well; for this reason, we can set x2 equal to the Ka, 10-9.2. Rounding this to 10-9 and taking its square root is roughly equal to 10-4.5, which correlates to a pH between 4 and 5.

In the system shown above, which expression represents the pressure exerted on the right piston? (input is 400N)

400/A Pressure is defined as P = F/A and the pressure will be transmitted throughout the system undiminished.

A marathon runner is jogging in a loop around the city with a velocity of 2.5 m/s. If the loop is perfectly circular and has a diameter of 3 km, after how many seconds will the runner reach his starting position?

4000 s C is correct. If the loop's diameter is 3 km, its circumference is 3π, or around 10, km. Since the runner's velocity is given in m/s, we'll need to convert the other value to either km or m. Let's convert 10 km to 10 × 103 m. We can thus see that traveling this distance will take 10 × 103 m ÷ 2.5 m/s = 4 × 103 s.

Red hair is recessive to all other hair colors. If the incidence of red-headed individuals within a certain population is only 9%, what percent of that population are heterozygous for the "red hair" allele?

42% D is correct. The people who express the trait for red hair must possess two copies of the allele. We can therefore solve for q, knowing that 0.09 represents q2. Once we have found that q = 0.3, we can use p + q = 1 to determine that p = 0.7. Finally, with these values in mind, the carrier rate must be expressed by 2pq = 2(0.7)(0.3) = 0.42.

The ratio of protons to neutrons in radon-222 is:

43:68. C is correct. Radon-222, like all isotopes of radon, has 86 protons. It therefore must have (222 - 86) or 136 neutrons, yielding a ratio of 86:136. This relationship can be simplified to 43:68.

The combustion of sodium in oxygen, a redox reaction, can be written as:

4Na + O2 + 4e- 🡪 2Na2O + 4e- C is correct. Like other redox reactions, this can be written as two separate half-reactions. For the oxidation half-reaction, Na 🡪 Na+ + e-. For the reduction component, O2 + 4e- 🡪 2O2-. However, the sodium half-reaction must be multiplied by four to give the same number of electrons on both sides. When we combine the two, we find that 4Na + O2 + 4e- 🡪 2Na2O + 4e-.

A particular biomolecule is known to have 30 diastereomers and does not exist in any meso forms. How many chiral centers must this molecule have?

5 A molecule with "n" stereocenters will have a maximum of 2n stereoisomers. (The reference to a "maximum" value stems from the possibility of meso structures; when these molecules exist, they do not count as stereoisomers because they lack optical activity. However, the question stem eliminated this possibility.) Here, we have the original compound as well as its own enantiomer, neither of which count as diastereomers. Adding the remaining 30 yields a total of 32 stereoisomeric forms. Since 2n = 32, n must equal 5.

The following peptide sequence is phosphorylated. TSGLVDSRTPEAGHAY If all of the threonine R-groups have tert-butyl protecting groups attached, how many available phosphorylation sites are present?

5 B is correct. The one-letter codes for serine, threonine, tyrosine, histidine, and arginine are S, T, Y, H, and R, respectively. These amino acids occupy 7 residues in this peptide, 2 of which are threonines. Thus, there are five potential phosphorylation sites.

The organic molecule shown below is currently under study by a team of research chemists. On the 1H NMR spectrum for this species, how many distinct signals should be visible?

5 signals To answer this question, we must determine the number of sets of chemically equivalent hydrogen nuclei on this molecule. Starting from the right, the -COOH proton is certainly distinct from the others, as it is very close to multiple electronegative atoms and will be shifted downfield. Carbon 2 (the alpha carbon) also possesses a unique proton, as do each of the two carbons adjacent to that atom; finally, the leftmost atom (carbon 4) has three equivalent protons of its own. In total, five different "types" of distinct protons exist here.

H3PO4 + Mg(OH)2 🡪 H2O + Mg3(PO4)2 A student reacts exactly 2 moles of phosphoric acid with 6 moles of magnesium hydroxide. This time, he measures 13.15 grams of magnesium phosphate (molar mass 263 g/mol) produced. What is the percent yield?

5% A is correct. Using the reasoning for #1269, we know that the balanced reaction is 2 H3PO4 + 3 Mg(OH)2 🡪 6 H2O + Mg3(PO4)2 and that phosphoric acid is limiting. Theoretically, a complete reaction should yield 1 mol, or 263 g, magnesium phosphate. Percent yield can be calculated by taking (actual yield / theoretical yield) × 100% = (13.15 g / 263 g) × 100% = 5.0%.

with an envious stereotype. The stereotype content model sorts relationships along two scales: competence and warmth. When two individuals are part of different in-groups, they tend to perceive each other with either low or high competence, meaning that they either consider them incapable and worthy of pity or capable and competitive. As Jackson is actively fighting with Jay on the tennis court, he certainly would view him as highly competent. Additionally, since the two boys are no longer close friends, we can assume that Jackson sees Jay with low, not high, warmth. These factors combine to produce an envious stereotype.

5' AAAACCTTAACGCGAAC 3'. This sequence is both complementary and antiparallel to the gene of interest and contains a 5' poly-A tail.

A wild-type mRNA transcript is shown below. 5' AUG GAU GAA CAU UGU GUU UUU AGU UGA 3' In a mutant polypeptide transcribed from this sequence, a point mutation results in a premature stop codon. As a result, the polypeptide is truncated to four residues instead of eight. Which of these mRNA sequences is most likely to code for this mutant protein?

5' AUG GAU GAA CAU UGA GUU UUU AGU UGA 3' Note that the sequence of the wild-type mRNA begins with AUG, which is the start codon and correlates to methionine. This codon must represent the first amino acid in our polypeptide. Additionally, the question stem indicates that the mutant has a nonsense mutation that results in a polypeptide with only four amino acids. This means that the fifth codon must be a stop codon, as that codon itself does not code for a residue. When glancing through the solutions, we see that only option A shows a stop codon (UGA) in the fifth position.

The sequence below is found in the sense strand of a human gene, upstream from the region that codes for the bulk of the associated protein's active site. 5' CCCGTATAC 3' A researcher desires to render this protein entirely nonfunctional. A mutation into which of the following sequences is most likely to accomplish this goal?

5' CCCGTATAA 3' In order to most definitively knock out the protein's functionality, the associated mRNA should be truncated with a premature stop codon. In eukaryotes, the three stop codons are UGA, UAA, and UAG. Since the question stem gives the sequence of the sense DNA strand, we simply need to replace each thymine with a uracil base to find the corresponding mRNA. In choice D, TAC has been mutated into TAA, a segment that will correspond to the UAA stop codon in mRNA. (Again, note that the question specified that this DNA is the sense strand, not the antisense strand. The sense strand has the same sequence as the product mRNA that is translated, except sense DNA has thymine instead of uracil. If you incorrectly thought we were given the antisense strand of DNA, you may have tried to find the complementary strand of that which is given, which will not give you the correct answer here.)

The gene encoding a protein involved in cell cycle regulation is analyzed, and researchers conclude that its sense strand contains the following sequence: 5'-AATTGCGCATTGC-3'. The sequence of the corresponding mRNA following transcription is:

5'-AAUUGCGCAUUGC-3'. A is correct. The sequence given represents part of the sense strand, which is not directly transcribed into mRNA. Instead, it is its complementary partner, the antisense strand, that is transcribed. This means that the resulting mRNA will have the same sequence and directionality as the sense strand (but with uracil instead of thymine).

A certain protease has a catalytic pocket that contains three consecutive glutamic acid residues. This region is coded for by the mRNA sequence 5'-GAAGAAGAA-3'. If mutated, which new sequence is most likely to yield the same protein product as the wild-type?

5'-GAGGAGGAA-3' B is correct. While there is no need to memorize the entire genetic code, remember that the most likely place for a silent mutation to be found is in the third base of a codon. The sequence in choice B has two mutations, but both are found in this third, or "wobble," base. In fact, the two point mutations are identical, meaning that if one is silent, the other must be too.

If Protein X has a pI of 3.5 and Protein Y has a pI of 5, at which of these pH values will electrophoretic isolation of Protein X occur most easily?

5.0 At a pH of 5, Protein Y will be neutral and stationary (since its isoelectric point is 5). Protein X, however, will be negatively charged and will move toward the anode in an isoelectric focusing procedure. At all other values, both proteins will be positively charged and will move toward the cathode to some extent.

A small metal racecar (pictured as the square below) is positioned at the top of a curved ramp and released. The mass of the car (m) is 2 kg, the length of the track (L) is 15 m, and the track's height (h) is 3 m, while µk = 0.1. (Note: assume the average frictional force during the drop is µmg.) With what velocity will the racecar leave the track? Assume that the car's wheels slide instead of rotating.

5.5 m/s A is correct. The kinetic energy at the end of the track is equal to the initial potential energy (mgh) minus the energy lost to friction. Since that dissipated energy is the same as the work performed by the frictional force, it can be written as µkmgL. In other words, ½ mv2 = mgh - µkmgL. Inserting given values shows us that ½ mv2 = (2 kg)(10 m/s2)(3 m) - (0.1)(2 kg)(10 m/s2)(15), so KE = 30 J. The final step is to recognize that velocity is thus equal to √30, which is closest to 5.5 m/s.

The association constant, Ka, of Epithelial Growth Factor Receptor (EGFR) binding to Epithelial Growth Factor (EGF) is 5.61 x 106. What is the Keq of EGFR + EGF → EGFR-EGF?

5.61 x 106 Ka, like many other K values, is an equilibrium constant. Hence, the association constant for the given equation is exactly the same as the equilibrium constant of the equation as written. Ka = Keq = [EGFR-EGF]/[EGFR][EGF].

Phenylketonuria is a rare autosomal recessive disorder with a carrier rate of 1/50 within Caucasian populations. A family enters a clinic with a sick child. Although neither parent is experiencing symptoms, you suspect PKU after a family history reveals that the child's paternal great-grandmother had this condition. What are the chances, respectively, that the father is a carrier and that the child expresses this trait?

50%, <1% Since the child's great-grandmother had PKU, she must have possessed two copies of the recessive allele. She passed one of these alleles to her offspring (the father's mother or father), making him or her an obligate carrier. This individual then had a 50% chance of passing it to the father of the sick child. (Note that we do not need to consider the father's other parent in this calculation, as the carrier rate is extremely small.) Now, we know that the father had a 50% chance of inheriting one allele and thus has a 50% chance of giving it to his child. As we lack any specific information about the child's mother, we can assume that she, like the population in general, runs a 1/50 risk of being a carrier. If so, she, too, has a 50% chance of giving her disease allele to the child. Multiplying these proportions yields (1/2)(1/2)(1/50)(1/2), or a 1/400 chance that the child is affected by PKU. A one-in-four-hundred risk is less than 1%.

The diagram below represents the electronic configuration of a selenium atom. It is implied that all shells other than the outermost shell, which contains 4 electrons, are full. If the above atom is oxidized twice, what is the new effective nuclear charge when n = 4?

6 B is correct. Even if this atom is oxidized twice, the shells interior to n = 4 will still contain same number of electrons. Similarly, selenium will still include 34 protons. Thus, the effective nuclear charge (as calculated for #1238) will not change.

How many unique tripeptides can be formed from 1 serine residue, 1 threonine residue, and 1 isoleucine residue?

6 Each unique order of amino acids represents a distinct tripeptide. Here, the possible combinations are STI, SIT, TIS, TSI, ITS, and IST.

A maglev train is following you at 140 m/s while emitting a sound with a frequency of 400 Hz. If you are driving away from the train at a velocity of 40 m/s, what frequency do you hear? Note that the speed of sound is 340 m/s.

600 Hz This is a Doppler effect question, for which you should use the equation f' = f (vsound ∓ vreceiver / vsound ∓ vsource). Next, we can plug given values into the equation to get: f' = (400 Hz)(340 m/s - 40 m/s)/(340 m/s - 140 m/s) = 600 Hz.

There are 100 cats in a population that follows Hardy-Weinberg conditions. 4 cats display a recessive white coat color. How many of the cats are homozygous for the dominant brown coat?

64 4/100 cats are recessive, which means that q2 = .04 and q = 0.2. P must then be 0.8. The number of homozygous dominant is p2 x 100.

Twenty standard amino acids are found in typical human proteins. How many unique mRNA codons exist?

64 D is correct. Each codon consists of three consecutive nucleotides. At each position, one of four nitrogenous bases may be present. For this reason, the total number of distinct codons is 43 or 64.

Brown mice are homozygous (BB), while yellow mice are heterozygous (Bb). If the homozygous recessive genotype (bb) is not viable, what percentage of the offspring will be yellow when two heterozygotes mate?

66% Of the four possible offspring that can form, two of them would be yellow. Since the recessive genotype is non-viable, then 2/3 are yellow, or 66%.

A small, flat plane cut entirely from diamond is submerged in a tank of fresh water. A ray of visible light travels through this water to contact the water-diamond interface, as shown. If z = 21°, what will be the angle of reflection with respect to the normal? Assume no total internal reflection takes place.

69° For a ray that contacts a flat surface, the angle of incidence is equal to the angle of reflection. However, note that this question asks for the angle with respect to the normal! The diagram shows the angle "z" with respect to the surface itself. In contrast, the normal is the line that runs perpendicular, or at a 90° angle, to the diamond surface. For this reason, both the angle of incidence and that of reflection can be measured as (90 - 21) or 69° when compared to the normal.

A student combines 364.6 g of HCl with 80 g of NaOH in 5 L of water. What additional volume of H2O must be added to this mixture to yield a solution with a pH of 1? Note that the molar mass of HCl is 35.46 g/mole, while that of NaOH is 40 g/mole.

75 L 364.6 g of HCl represents 10 moles of this strong acid. In contrast, 80 g of NaOH is only 2 moles of this strong base. When mixed in solution, the base will fully neutralize 2 moles of HCl according to the 1:1 stoichiometric ratio between these reagents. 8 moles of HCl will remain in 5 L of water; however, we need a total H+ concentration of 0.1 M to yield a pH of 1. This concentration can best be obtained by adding enough water to establish a total volume of 80 L.

Sodium bicarbonate, a commonly used antacid, produces carbon dioxide and salt water when mixed with hydrochloric acid in the gut. The reaction for this process is as follows: HCl (aq) + NaHCO3 (aq) 🡪 H2O (l) + CO2 (g) + NaCl (aq) The reaction is run to completion in an open container, after which all excess sodium bicarbonate is filtered and removed. What is the percent by mass of salt in the remaining solution?

75% NaCl D is correct. Mass percent can be calculated by dividing the mass of NaCl by the mass of the entire solution. Because the reaction was run in an open container, CO2 should not be included; as a gas, it easily leaves the solution. To find the number of grams of NaCl, take 250 mL HCl × (2 mol HCl / 1000 mL) × (1 mol NaCl / 1 mol HCl) = 0.5 mol NaCl × 58 g/mol = 29 g NaCl. For H2O, 0.5 mol H2O × 18 g/mol = 9 g H2O. The total mass of the solution is thus 38 g, and 29 g / 38 g × 100% = 75%.

Ammonium dichromate decomposes in a violent reaction that is often used in science demonstrations due to its "volcano-like" appearance. The equation for this process is as follows: (NH4)2Cr2O7 (s) 🡪 Cr2O3 (aq) + H2O (l) + N2 (g) A student attempts to decompose 500 grams of ammonium dichromate but only measures 12 grams of N2 gas produced. The absolute value of the percent error for this student's experiment is:

80% Converting the given 500g of reactant to moles, we get around 2 moles of initial amount of ammonium dichromate. Using stoichiometry, we know that N2 must also consist of around 2 moles, and 2 moles of is N2 is around 55 grams (since it has a molar mass of 28g/mol). The question states that the resulting amount is 12 grams - using the percent error formula, PE = (|accepted value - experimental value| / accepted value) x 100%, We know that the percent error is estimated to be about 80%.

A balloon is filled with 100 mg of various gases to mimic those present in the Martian atmosphere. The balloon contains equimolar amounts of nitrogen and oxygen, argon with a mole fraction of 0.10, and carbon dioxide with a mole fraction of 0.26. In this system, the percent composition by mass of carbon is:

9% A is correct. To find the percent composition of carbon in the balloon, the total mass of all other elements present must be calculated. For convenience, we can assume that there are 100 mol of gas in the balloon. Let's begin with argon, which has a mole fraction of 0.10. This is equivalent to 10 mol (0.10 × 100 mol), and since Ar has a molecular weight of approximately 40 g/mol, we can conclude that 400 g of Ar are present. For carbon dioxide, which has a molecular weight of 44 g/mol, 26 mol is equivalent to 1144 g; note, however, that only 312 g of this amount is carbon. Finally, though we do not know the mole fractions of nitrogen and oxygen, we know that the remaining mole fraction of 0.64 is divided evenly between them, giving us 32 mol (or 1024 g) of O2 and 32 mol (or 896 g) of N2. The percent composition of carbon is (312 g C / 3464 g total mass) x 100, or about 9%.

What is the molar solubility of an insoluble solid, AB, if the Ksp for AB is 8.3 x 10-11? Note that this solid is ionic and is composed of the ions A+ and B-.

9.1 x 10-6 mol/L Even insoluble ionic compounds dissociate into their component ions to a small extent in water. To find molar solubility (s) for a compound that dissociates into only two ions per molecule, use the equation s2 = Ksp. Here, s2 = 8.3 x 10-11, which is equivalent to 83 x 10-12. To find the square root of this value, take the square root of the coefficient and cut the exponent in half. 83 is approximately 81, and the square root of 81 is 9. This gives us a final answer of s = 9 x 10-6 mol/L, which is closest to choice C.

A 24-kg lawnmower is sitting in a shed. It is then dragged along the ground with a constant force of 120 N. Assuming that no friction is present, how far has it traveled when its velocity reaches 30 m/s?

90 m D is correct. First, use F = ma to find acceleration. 120 N = (24 kg)(a), giving an acceleration of 5 m/s2. Now, this acceleration value is plugged into the formula vf2 = vi2 + 2aΔx to calculate distance. (30 m/s)2 = (0 m/s)2 + 2(5 m/s2)(x) shows that x = 90 m.

A cylinder fitted with a piston exists in a high-pressure chamber (3 atm) with an initial volume of 1 L. If a sufficient quantity of a hydrocarbon material is combusted inside the cylinder to produce 1 kJ of energy, and if the volume of the chamber then increases to 1.5 L, what percent of the fuel's energy was lost to friction and heat? (Note: 1 atm = 101,325 Pa)

90% C is correct. The energy used for expansion must come from the energy of combustion minus the amount lost to friction. Einput = 1.5 x 103 J Woutput = P · 𝚫V Woutput = 3 atm · 0.5 L Woutput = 1.5 atm · L Woutput = 1.5 atm · L (101325 Nm-2/atm)(103 cm3/L)(m/102 cm)3 Woutput = (1.5)(1 x 105 Nm-2)(103)(m3/106) Woutput = 1.5 x 102 J Elost = Einput - Woutput Elost = 1.5 x 103 J - 1.5 x 102 J Elost = 13.5 x 102 J or 9 (1.5 x 102 J) %Elost = Elost / Einput x 100% %Elost = [9 (1.5 x 102 J) / 10 (1.5 x 102 J)] x 100% %Elost = 90%

A billiard ball is submerged in a small tank of water at a depth of exactly 60 cm. However, the tank is located in Denver, Colorado, where the ambient pressure is roughly 0.83 atm. What is the absolute pressure experienced by the ball?

90,245 Pa D is correct. The question asks for absolute pressure, which includes both gauge and atmospheric pressure. First, let's find the gauge pressure using the equation P = ρgy, where ρ denotes the density of the fluid and y refers to the depth of the object. Since all answer choices are in pascals, we need to use SI units to find our answer, meaning that we should use 0.60 m, not 60 cm, for y. The density of water is 1000 kg/m3 and gravity can be approximated as 10 m/s2, yielding a gauge pressure of (1000 kg/m3)(10 m/s2)(0.60 m) or 6000 Pa. Finally, we must add atmospheric pressure, which is 0.83 atm. Since 1 atm is equal to 101,500 Pa, 0.83 atm is equivalent to 84,245 Pa and our final answer is 90,245 Pa. Note that rounding is perfectly fine in a problem such as this one.

What is the approximate angle between adjacent bonding electrons in XeF4?

90° XeF4 has four bonds and two lone pairs, giving it an octahedral shape. The lone pairs will be situated 180° from each other, with the bonds arranged in a square around the central atom. The angle between any two adjacent bonds will thus be 90°.

A prosthetics researcher is conducting tests on a 40-cm-long prosthetic forearm, which is positioned according to the simplified diagram below. He exerts an upward force of 5 N on the forearm 30 cm to the right of the point of rotation (the elbow joint). If the forearm has a weight of 15 N that is uniformly distributed, which of the forces listed below will allow the system to exist in rotational equilibrium?

A 20-N upward force exerted 7.5 cm to the right of the point of rotation B is correct. When dealing with multiple torques, it's helpful to separate them based on whether they rotate the system clockwise or counterclockwise. To exist in rotational equilibrium, the system must have no net torque, meaning that the counterclockwise and clockwise torques must be equal. The only counterclockwise torque shown is due to the upward 5-N force. Remember, torque = F r sin ?, where r represents the distance between the point of application of force and the fulcrum, or rotation point; ? represents the angle at which the force is applied. Here, then, torquecounterclockwise = (5 N)(0.3 m)(sin 90°) = (1.5 Nm)(1) = 1.5 Nm. It may not immediately appear that any clockwise torques are present, but remember, this prosthetic forearm has a weight of 15 N! When positioned as shown in the diagram, this weight will "want" to pull it downward, making it a clockwise torque. Since the forearm has a uniformly-distributed mass, we can position this torque at its center of mass, exactly in the center of the forearm (20 cm to the right of the rotation point). Torqueclockwise = (15 N)(0.2 m)(sin 90°) = (3 Nm)(1) = 3 Nm. Now, we can see that we have a net torque of 3 Nm clockwise - 1.5 Nm counterclockwise = 1.5 Nm clockwise, or "down." We thus need a counterclockwise, or "upward," force of 1.5 Nm to bring the system to equilibrium. This eliminates options C and D immediately. For choice B, (20 N)(0.075 m)(sin 90°) = 1.5 Nm, giving us the proper magnitude of force required to bring our net force to zero.

Fluid X has a negligible viscosity, but is moderately compressible. A student, tasked with making several hypothetical calculations, wrongly assumes that this substance is perfectly ideal. Which is likely to yield a calculated result that deviates more from its real-life value?

A calculation involving the continuity equation Both Bernoulli's and the continuity equation assume that the relevant fluid is ideal. However, the concept of an ideal fluid involves multiple assumptions. Ideal fluids are non-viscous (like Fluid X), but differ from Fluid X in that they are incompressible. The continuity equation, often written as A1v1 = A2v2, relates much more to compressibility than to viscosity. In short, as long as a fluid cannot be compressed or expanded, its volume flow rate through a series of tubes should be constant at any moment in time. As Fluid X is compressible, this same concept will not hold true and the student will miscalculate his answer.

A certain genetic neurological syndrome is currently being studied to determine its effects on and interactions with other aspects of health. However, this syndrome is so rare that only approximately 1 in 500,000 neonates are born with it. Which type of study would a researcher of this condition benefit most from planning and executing?

A case study, as the best approach here is to make a detailed study of a small number of individuals B is correct. Case studies tend to have a very narrow, detailed focus; they aim to thoroughly elucidate the contextual factors that affect a small number of subjects, as opposed to establishing broad trends for a large sample. Here, the most remarkable information given is the incredibly small number of individuals with this disease. This rules out the use of other methods that rely on larger samples. Additionally, the emphasis on contextual interactions between this syndrome and the health of the subjects would also benefit from a case study methodology.

How might environment influence the genetic expression of a particular personality trait?

A girl whose father has an anxiety disorder has a very tumultuous childhood; she later develops the same condition as a teenager. B is correct. This girl already has a genetic susceptibility to an anxiety disorder, as her father has the condition. Her tumultuous childhood could then have secured the probability that she would then develop the disorder herself. This occurs often with psychiatric disorders, where environmental factors seem to "bring out" the expression of certain genes.

The opponent-process theory has often been cited in cases similar to those of which individual?

A heroin addict who cannot seem to quit The opponent-process theory is often used as a model to explain drug addiction. This theory states that many human behaviors and emotional responses have two antagonistic components. For example, a drug addict feels pleasure (the euphoria associated with certain neurotransmitter) when he takes the drug, but withdrawal (the lack of these neurotransmitters, causing unhappiness) when he stops. Even more interestingly, the opponent-process theory also relates to physical symptoms. Since heroin is a depressant, the drug addict's body compensated by "speeding itself up" after many instances of taking the drug, with an increased heart rate, shaky muscles, etc. While the addict did not notice this while he was still on the substance, once he stopped, this "opponent process" became the only thing he experienced and caused withdrawal symptoms.

A student is analyzing two images of distinct muscle types, observed through a microscope at his physiology lab. Image adapted from OpenStax College under CC BY 3.0. The muscle type in A differs from B in that: I. A is smooth muscle while B is cardiac muscle. II. they possess different numbers and types of cell-cell junctions. III. A contains structured sarcomeres, while B does not.

A is smooth muscle while B is cardiac muscle and they possess different numbers and types of cell-cell junctions.

Tyrosine kinase inhibitors (TKIs) were developed near the end of the twentieth century and proved highly effective in binding the receptor tyrosine kinase (RTK) and disrupting its interaction with its immediate partner, Grb2. Most forms of myelogenous leukemia involve a unique chromosomal abnormality, resulting in the formation of a constitutively-acting kinase variant called Bcr-Abl. The resulting uncontrolled growth signal leads to a rare form of cancer. Recently, even third-generation TKIs have become ineffective in the management of this previously treatable condition. Which of these genetic changes could account for this resistance?

A mutation in Ras, a downstream effector of Bcr-Abl, that renders it constitutively active B is correct. Since Ras is located downstream of Bcr-Abl, TKIs that previously facilitated the inactivation of Bcr-Abl will now be ineffective in halting the progression of this signaling pathway. From this question alone, we do not know the exact role of Ras in the pathway, but this is the only option that describes a downstream mutation that could feasibly cause increased activation.

Which of these functional groups or bonds is NOT found in all nucleosides?

A phosphodiester bond C is correct. A nucleoside consists of a five-carbon pentose sugar and a nitrogenous base, either a purine or a pyrimidine. Purine bases consist of a pyrimidine ring attached to an imidazole group; as a result, both B and D are correct statements. Finally, an N-glycosidic linkage attaches the pentose sugar to the nitrogenous base. This makes A accurate as well.

Which of the following situations relates best to stimulus discrimination?

A rat is first rewarded whenever any light is flashed, causing him to run to a food dispenser. The experimenter starts reinforcing only the brightest flashes, and weeks later, the rat does not even respond to dim flashes. C is correct. Stimulus discrimination occurs whenever the conditioned stimulus required to generate a response is made more specific. In other words, any stimulus within a broad category (all flashing lights) originally was able to elicit a conditioned response. Through selective reinforcement of some stimuli over others, this category was narrowed to certain light flashes only.

The structure of sucrose, a physiologically relevant disaccharide, is shown here. A sample of sucrose is treated with hydrochloric acid. The solution is then neutralized with a weak base, and Tollens' reagent is added to the reaction vessel. Which of the following is likely to occur next?

A silver mirror will form on the surface of the vessel, since glucose is a reducing sugar. Acid hydrolysis of sucrose will yield glucose and fructose, the two monomeric components of this disaccharide. Since all monosaccharides are reducing sugars, it would thus enable formation of a silver mirror, a positive result.

A monobactam structure is characterized by a β-lactam ring that is not fused to a neighboring cyclic group. The general structure of a monobactam is shown here. Which of the following statements is true regarding the C-N bonds in the lactam ring?

A substantial amount of ring strain is caused by the hybridization of the C-N bond. Due to the double bond character of the amide group, no rotation can exist around the ring's sigma bonds to facilitate the formation of the most stable potential structure. In addition, four-membered rings are generally subject to heavy strain.

Which substituent can be attached to ethanol to increase the pKa of the hydroxyl group?

A tert-butyl group The question is asking for a substituent that will increase the pKa, or make the proton less acidic. In other words, we are looking for an electron-donating group. Attachment of such a substituent will add electron density to the hydroxyl oxygen, making it more negatively charged and less likely to lose its proton. (Remember, the strength of an acid is directly correlated to the stability of its conjugate base.) Tert-butyl, like other alkyl substituents, is electron-donating.

A researcher has a fruit fly that demonstrates the dominant characteristics for eye color (E) and wing size (W) but does not know its genotype. To determine the fly's genotype, she could carry out which of the following procedures?

A test cross with an organism that has the genotype wwee B is correct. Since an organism with the dominant gene will always display it, a researcher cannot tell simply through observation whether the fruit fly is WWEE, WwEE, WWEe, or WwEe. The typical way to make this determination is to carry out a test cross: a mating between the unknown individual and an individual known to be homozygous recessive (wwee) for the traits in question. Then, if any of the offspring demonstrate the recessive phenotype (ww or ee), we can conclude that the unknown organism was heterozygous for that trait.

The hydrogen labeled A is expected to have which of the following 1H-NMR signals?

A triplet The number of peaks into which an NMR signal splits follows the rule n + 1, where n is the number of hydrogen atoms attached to the neighboring carbon. Note that the neighboring carbon is the carbon next to the carbon on which the hydrogen of interest is located! Here, since our hydrogen of interest is bound directly to the carbon that is also bound to two bromine atoms, we want to look at the carbon next to that position (the carbon bound to two hydrogens and a chlorine atom). Since we have two hydrogens on that position, 2 + 1 = 3.

Dr. Ozzy, a psychologist, firmly agrees with the correspondent inference theory. Dr. Ozzy would most likely expect us to attribute which of these behaviors to dispositional factors?

A woman in front of you in a drive-thru chooses to pay for both her bill and yours. B is correct. The correspondent inference theory focuses on the perception of a person's behavior as corresponding to his or her personality. In other words, it attempts to dictate when we will make dispositional attributions. According to this theory, we tend to make such attributions when the action is seen as intentional, when it directly harms or benefits us, and when it differs from what is typically expected by society. In choice B, the woman does something unusual to purposely and directly benefit you. As a result, you will likely perceive her to be a good or generous person.

A student is asked to draw a nucleoside on the board. Which of these descriptions constitutes an accurate response?

A xylose sugar attached to a pyrimidine ring with an imidazole substituent A nucleoside includes a five-carbon sugar attached to a nitrogenous base. Purines, including adenine and guanine, contain pyrimidine rings with fused imidazole groups. Additionally, while ribose and deoxyribose are the most common sugars found in nucleosides, the definition of such a structure technically includes any five-carbon sugar. Xylose thus fits this criterion.

Which of these situations involves a person whose view of morality falls within the conventional stage of Kohlberg's stages of moral development?

A young man never speeds when driving his car because he thinks that people should abide by the rules of the law, and that if everyone drove too fast, the roads would not be safe.

Given the following gene recombination frequencies: AB: 12% AC: 36% AD: 14% BC: 24% BD: 2% CD: 22% What is the expected order of the linked genes A, B, C and D?

ABDC AC has the highest recombination frequency, which indicates that genes A and C are the furthest apart. AB's recombination frequency is smaller than AD's recombination frequency, which means that gene B must be closer to gene A than gene D is.

Participants in a reality show who are asked to live in a dark room for several consecutive days probably release less of which hormone?

ACTH Adrenocorticotropic hormone (ACTH) is released by the anterior pituitary gland when stimulated by corticotropin-releasing factor (CRH) from the hypothalamus. This chain of events is triggered by the increase in light incident on the retinas during waking. In a dark environment, this cascade would likely be disrupted, since the hypothalamus would receive fewer stimuli to promote the release of CRH.

Of the pairings below, which accurately matches the hormone with the organ that synthesizes it?

ADH - hypothalamus Remember that the posterior pituitary does not produce the hormones it secretes! ADH is produced in the hypothalamus, then stored and released from the posterior pituitary.

A phosphotransferase enzyme with a cysteine residue in its active site acts on ATP to form the below intermediate. What molecule acted as a leaving group in the first step of this interaction?

ADP The question stem describes ATP as a starting reagent. However, the diagram shows only a single phosphate group bound to the sulfur atom of the mentioned cysteine residue. For this reason, ADP must have left as a leaving group.

The enzyme adenylate kinase catalyzes a particular reaction: ADP + ADP <--> ATP + AMP The forward reaction predominates when the cell is using ATP at a high rate. Which of these species likely upregulate(s) the activity of a glycolytic enzyme? I. AMP II. ADP III. ATP

AMP Since we know that high use of ATP causes the forward reaction to predominate, we can expect more of its products to be present when the cell is in an energetically demanding state. However, ATP will be used rapidly, leaving high concentrations of intracellular AMP. Simultaneously, the cell will be performing glycolysis at a high rate in an effort to feed the citric acid cycle and the electron transport chain. Thus, high [AMP] coincides with rapid glycolytic behavior, and we can infer that AMP upregulates one or more glycolytic enzymes. In reality, AMP is a main effector of phosphofructokinase.

A biologist views the myosin head of a single thick filament, which is protruding in the form of a cross-bridge that is near, but not attached to, a thin filament. The cross-bridge is currently positioned at a 90° angle to the thin filament. This moment in time occurs immediately after:

ATP is hydrolyzed to form ADP and inorganic phosphate. The question stem mentions that actin and myosin are not attached, but that they are positioned perpendicularly, implying that they are preparing for future binding. Hydrolysis of ATP into ADP and inorganic phosphate restores the myosin head to a 90° angle, "cocking" it for new attachment and the ensuing power stroke.

If a mutation in the gene coding for NAD+ resulted in a nonfunctional coenzyme, what change would occur to the rate of ATP synthesis?

ATP production would be reduced, since FADH2 would still be capable of providing electrons through Complex II.

If a mixture of acetic acid and ethane is subjected to a thin-layer chromatography procedure, which compound will have the smaller Rf value?

Acetic acid, due to its carboxylic acid group A typical TLC procedure includes a polar stationary phase and a nonpolar mobile phase. Thus, more polar compounds will interact with the plate and travel a shorter distance than nonpolar compounds. Since Rf is a measure of distance traveled by the sample in comparison to the solvent front, a shorter distance correlates to a lower Rf. Acetic acid is polar due to its COOH functional group and ability to form hydrogen bonds.

With regard to leaving group alone, which of these species is the most unstable?

Acetyl iodide Of the four halogens included in these molecules, iodine serves as the best leaving group. To understand this concept, consider the periodic table. As a member of a much lower period than (for example) fluorine, iodine is a very large atom. As such, it can easily delocalize the negative charge gained when it exits as a leaving group. The better the leaving group, the more reactive the compound, and a more reactive molecule is by definition less stable.

A researcher is studying the effects of molecular structure on a variety of properties, including boiling point and acidity. Of the following compounds, which has the most acidic alpha proton?

Acetylacetone C is correct. Each of these compounds contains at least one alpha proton, which are especially acidic due to the potential resonance stabilization of the conjugate base. However, acetylacetone is a dicarbonyl compound. Specifically, it has two carbonyl groups that flank a single, middle carbon, which is alpha to both of them. If you draw the resonance states for this compound after the central carbon has been deprotonated, you'll see that both carbonyls contribute a stabilizing effect, making these hydrogens highly acidic.

An elderly male patient arrives at a clinic complaining of erectile dysfunction. Due to a family history of heart disease, he is hesitant to start taking Viagra. An alternate treatment regimen might prescribe supplements to increase the levels of which neurotransmitter?

Acetylcholine The parasympathetic nervous system is responsible for delivering impulses to the male genitals. These signals relax certain smooth muscles, allowing blood to flow into the penis during an erection. The parasympathetic system also delivers acetylcholine to all its effectors. Therefore, increasing the body's acetylcholine levels could help this patient with erectile dysfunction.

The following diagram shows several reactants and intermediates involved in a multi-step reaction. After successfully completing the reaction, the researcher decides to regenerate the original reactants from the product molecules. What method will give him the best results?

Acid-catalyzed hydrolysis A is correct. Under acidic conditions, one of the electronegative oxygen atoms in the acetal can be protonated, creating a more favorable leaving group. Such a group is necessary if we wish to lose the methanol groups and regenerate the carbon-oxygen double bond.

A patient with persistent difficulties fighting off bacterial infections is found to have a genetic mutation that adversely affects the speed of phagocytosis in macrophages. This mutation most likely impairs which type of cytoskeletal polymer?

Actin filaments Macrophages must undergo rapid actin reorganization during phagocytosis. If a macrophage cannot engulf bacteria in this manner, its overall function would be considered very impaired.

In the complex process of "bionic eye" design, scientists are attempting to eliminate the natural blind spot exhibited by the human eye. Which solution, if possible, would best address this problem?

Add more cones and rods to the optic disc C is correct. The human blind spot is present because the optic disc, which leads to the optic nerve, contains no photoreceptors. In humans, it makes sense that this region does not include rods or cones, especially since the blind spot is small and not enormously evolutionarily disadvantageous. However, we can assume that the scientists have the capacity to add photoreceptors without impairing the normal functions of their synthetic visual system.

A certain tribal language has a word for "one" and for "two" but any more than this is described as "many". An anthropologist attempts to teach members of the tribe to count, using Spanish words. According to the Sapir-Whorf hypothesis, which of the following would most likely be true?

Adult members of the tribe will have an exceptionally difficult time developing numeracy. B is correct. The Sapir-Whorf hypothesis holds that the language a person speaks influences a person's thoughts and decisions. For example, if a person's language has no word for something, then the person will not be able to easily entertain thoughts about that thing. Here, the language has no words for numbers. Thus adult speakers of the language would have difficulty understanding numbers and developing basic numeracy.

Al3+ has a reduction potential of -1.66, while Cd2+ has a reduction potential of -0.40. In an electrolytic cell at standard conditions:

Al3+ will reduce at the cathode and Cd (s) will oxidize at the anode. Reactions in electrolytic cells occur nonspontaneously. Since Al3+ has a more negative reduction potential than Cd2+, it is Al3+ that will reduce and Cd (s) that will oxidize. Oxidation and reduction always occur according to the mnemonic "REDCAT": reduction happens at the cathode and oxidation takes place at the anode.

The following diagram shows several reactants and intermediates involved in a multi-step reaction. Using the diagram above, correctly label the types of compound from left to right.

Aldehyde, alcohol, hemiacetal, alcohol, acetal A is correct. The first reactant is an aldehyde, or a carbonyl bound to both an R group and a hydrogen. Reaction with one equivalent of alcohol produces a hemiacetal, or "half acetal," while reaction with a second equivalent forms a full acetal. These compounds are interesting in that they include two oxygen atoms bound to the same carbon. For a hemiacetal, one of those is part of an -OH group, while the other is part of an ether (-OR). For an acetal, both oxygen atoms exist within -OR groups.

Alkali metals, when placed in water, are highly reactive and sometimes capable of causing large explosions. Which of the following properties of these metals serves as a cause of this reactivity?

Alkali metals have very low ionization energies. D is correct. Members of the alkali metals have one valence electron. When this electron is lost, these elements gain a noble gas configuration, making them very stable. For this reason, alkali metals have very low ionization energies and lose an electron extremely easily. In this reaction, hydrogen gas is produced and the metal forms its corresponding oxide.

Which isomeric form of 2-chloro-3-bromobutane is most thermodynamically stable?

Alkanes do not have cis and trans isomers. The compound named in the question stem contains only single bonds. Substituents are able to freely rotate around such bonds, an action that is prevented in alkenes. In other words, bound atoms or groups are never "locked" in place when positioned around a single bond. For this reason, cis and trans isomers do not exist in alkanes.

Of these statements, choose the one that is NOT true of chiral compounds.

All amino acids that are L in configuration are also S. B is correct. While all common chiral amino acids exist in their L forms in humans, not all are designated as S. Specifically, cysteine is the lone R residue. The D/L naming convention is determined by the placement of the amino group and is thus independent of the R-group, while the R/S convention assigns priority to the atoms surrounding the chiral center and is affected by the side chain.

The figure below depicts a model of enzyme regulation. Which term accurately characterizes the function of the component marked "D"?

Allosteric regulator B is correct. The figure depicts a molecule labeled "D" that binds to the enzyme. When the molecule attaches, the enzyme changes conformation and the original substrate is prevented from binding. Since the molecule marked "D" does not bind in the active site, this is consistent with allosteric regulation.

Consider the conjugate bases of these four compounds: an alcohol, a phenol, a carboxylic acid, and an amine. Which of these rankings correctly orders these species in terms of most to least basic?

Amine > alcohol > phenol > carboxylic acid A is correct. As common bases themselves, amines should clearly be listed first, while carboxylic acids should be last. To compare a phenol with a regular alcohol, remember that a stronger acid must have a weaker conjugate base. As resonance-stabilized species, phenols are more acidic than alcohols, meaning that alcohols have the more basic conjugates of the two.

A researcher wants to mix two distinct prokaryotic strains to observe conjugation between them. Which of these pairings could result in the successful transfer of chromosomal genetic material in this fashion?

An Hfr bacterium with a F- bacterium The F factor is a plasmid that encodes for a sex pilus and allows for genetic exchange. Those positive for the factor can create such a pilus and transfer genetic information to individuals lacking the gene. When the F factor plasmid becomes spliced within the bacterium's chromosome, this results in an Hfr bacterium. When Hfr cells attempt to transfer the F factor via conjugation, genomic material can be transferred as well.

Of these combinations, which could result in a successful transfer of genetic material through conjugation?

An Hfr bacterium with an F- cell The F, or fertility, factor is a piece of genetic material that codes for the construction of a sex pilus. This bridge allows a bacterium to transfer the factor to another cell, promoting genetic diversity. However, only bacteria with the factor can initiate this transfer, and only to cells that lack it. Typically, the F factor is held outside the main genome in the form of a small plasmid. However, Hfr bacteria represent an interesting phenomenon; in these cells, the F plasmid has become incorporated into the organism's circular chromosome. As these cells do contain the F factor, they can initiate conjugation to F- cells, but they often transfer genomic material along with part or all of the factor.

Hemolytic disease of the newborn results when a pregnant female produces IgG antibodies specific for a particular erythrocyte antigen, known as "Rh." Although the placenta normally prevents mixing of fetal and maternal blood, the act of childbirth often involves bleeding that can permit sensitization of maternal immune cells to Rh. Which of the following scenarios carries the highest risk for development of hemolytic disease of the newborn?

An Rh-negative mother on her second pregnancy, carrying an Rh-positive fetus D is correct. The mother will only mount an immune response to an antigen that her immune system recognizes as foreign. For this reason, she must be negative for the Rh antigen. Furthermore, the formation of antibodies and the generation of a memory humoral response take between 48 and 96 hours. This means that the first pregnancy is at minimal risk for hemolytic disease of the newborn. After the first delivery, however, the maternal immune system has been exposed to fetal Rh. Thus subsequent exposure to Rh will elicit a much stronger response.

A chemist performs a two-step synthesis using the enamine shown below. In Step (1), the enamine behaves like an enol and reacts with an alpha,beta-unsaturated aldehyde. In Step (2), the intermediate from the first step is converted to the final product. What is the expected structure of the final product?

An enol would act as an nucleophile and attack an electrophile using its alpha carbon; therefore, the enamine will do the same. Recall that the reactive electrophilic site on an alpha,beta-unsaturated carbonyl compound is the beta carbon, so the enamine will attack here, as shown below. Adding water and catalytic acid to the intermediate will protonate the enolate and convert the imininium to a carbonyl group (remember, adding water to an imine or enamine will convert these groups back to the carbonyl).

Which of the following demonstrates the use of crystallized intelligence?

An experienced accountant fills out several financial statements over the course of an hour.

In a negative inducible operon, a repressor protein binds to the operon and the genes are not actively transcribed. In such an operon, which of the following could be added to restore the operon to a transcriptionally active state?

An inducer In a negative inducible operon, transcription is inhibited by a repressor; this results in a basal transcription rate near zero. However, transcription can be "switched on" by the addition of an inducer protein, which blocks the repressor from binding to the operon.

Which of the following is an example of acute stress?

Andrew can't find his car keys and frantically searches the house before leaving for work. A is correct. Acute stress arises from daily situations that are unpredictable, represent a threat to the individual's ego, or are difficult to control. Andrew's trouble finding his car keys before work represents this kind of short-term stress. Note that "acute" refers to duration rather than intensity of symptoms, although acute symptoms are very often intense.

Fischer esterification is easily achievable in acidic solution. An organic chemistry student plans to conduct such a reaction with the species shown below. Which conditions should be used to obtain the highest possible yield of the desired cyclic ester product?

Anhydrous sulfuric acid B is correct. Ideally, acidic conditions serve to effectively protonate the carbonyl of the carboxylic acid and further polarize the bond. However, some species react in alternative ways. Since sulfuric acid is a poor nucleophile, it is unlikely to cause any undesirable side reactions. In reality, note that side reactions between two different organic acid molecules are unavoidable. That is, a second molecule of the organic acid could be induced to add across the double bond. With this in mind, we are asked only to identify the catalyst that offers the highest possible degree of success.

Three amino acids are subjected to reversed-phase chromatography: arginine, phenylalanine, and serine. From first to last, in what order will these compounds be eluted?

Arginine, serine, phenylalanine A is correct. In reversed-phase chromatography, the stationary phase is hydrophobic, while the mobile phase is hydrophilic. Arginine, as a charged amino acid, will elute out first, as it will not interact with the hydrophobic stationary phase at all. Serine has a polar side chain and will interact minimally with the hydrophobic environment, while phenylalanine has a hydrophobic side chain and will interact most strongly to the stationary phase.

Aminotransferase is used to reversibly replace a ketone oxygen with an amino group. This enzyme is utilized during amino acid metabolism to convert certain amino acids into oxaloacetate, which is then incorporated into the TCA cycle. If an aminotransferase enzyme acts on oxaloacetate, pictured above, which amino acid will be formed?

Aspartic acid

Which of these statements explains the adaptive role of emotion in relation to Darwin's theory of evolution?

At some point in human history, individuals who experienced emotion had an advantage over those who did not.

As one moves across the periodic table from right to left, atomic radius ______ and electronegativity ______.

Atomic radius increases as you move down and to the left along the table. Electronegativity increases as you move up and to the right, so since this question asks about moving the opposite direction (from right to left), electronegativity must decrease.

A group of patients have difficulty focusing on fixed objects when moving across the room or even rotating their heads. What structure is LEAST likely to be damaged in these individuals, based on their symptoms?

Auditory cortex D is correct. The auditory cortex is responsible for processing sound information in the temporal lobe after it has passed through the medial geniculate nucleus. It is not mentioned that these patients have trouble perceiving sound. Note that some information is also sent to the inferior colliculus, which helps keep the eyes focused on singular points even when the head is rotating. This key part of the vestibulo-ocular reflex could easily be damaged in these people.

The following diagram depicts a model of the structure of transfer RNA. At which position do aminoacyl tRNA synthetases attach amino acid residues to the tRNA molecule?

B B is correct. tRNAs are "charged," or bound to an amino acid, by enzymes known as aminoacyl tRNA synthetases. The amino acid corresponding to the tRNA molecule is attached to the 3' end of the RNA. Attachment takes place via covalent bonding to the CCA tail, which is indicated by the label B.

A biologist is interested in the composition of a certain type of cell membrane. To study this concept, he saponifies various membrane lipids to remove their glycerol backbones. The resulting fatty acids are labeled in the diagram below. The researcher isolates a segment of membrane thought to contain a number of lipid rafts. Which fatty acid from the diagram is most likely a component of this sample?

B Lipid rafts surround transmembrane proteins and serve signaling purposes. These structures tend to be comprised of long, saturated lipids that strongly adhere to each other and to their associated protein. Of the two saturated fatty acids shown (A and B), molecule B is longer; thus, it is more likely to be found within a lipid raft.

Which of the following is a Lewis acid?

BCl3 A Lewis acid is a compound that can accept an electron pair. BCl3, which lacks a full octet, is a classic example.

Which of the following elements has the lowest electron affinity?

Be Electron affinity can conceptualized as an element's "desire" to gain an electron. Beryllium actually prefers to be a positively charged species, Be2+. As such, it needs to lose only two electrons to reach noble gas configuration, and its electron affinity is virtually negligible.

Due to a point mutation during replication, one guanine base is now paired with adenine instead of cytosine. How might the cell's proofreading machinery be able to recognize this mistake?

Because both adenine and guanine are purines, the DNA strand will be wider at the site of the mutation. Both adenine and guanine are purine bases, which have structures composed of two fused rings. In contrast, pyrimidine structures contain only one ring. A purine-purine base pairing would thus be wider than the typical purine-pyrimidine pairing.

A neurogeneticist is interested in the composition of a certain type of cell membrane. To study this concept, he saponifies various membrane lipids to remove their glycerol backbones. The resulting fatty acids are characterized below. The geneticist then compares four different cell membranes that vary in their fatty acid composition and in their relative permeabilities. Which of the labeled fatty acids would likely dominate in the most permeable of the membranes?

Because the double bonds present in unsaturated fatty acids create "kinks" in the molecules' tails, lipids containing these acids have trouble adhering to each other as strongly as lipids with mainly saturated tails. The higher the unsaturated lipid concentration, the more fluid and permeable a membrane will be. Of the fatty acids in the diagram, D is the most unsaturated; thus, it is most likely to exist in a more permeable membrane.

The crocodile death roll is an impressive hunting method in which the reptile drags its prey underwater and rolls at a high speed. Crocodiles also possess hemoglobin that can bind bicarbonate at the a1b2 interface. How might this feature enable crocodiles to remain underwater for extended periods of time?

Binding HCO3- stabilizes the T state of hemoglobin, allowing for more oxygen to be released into the tissue. By holding its breath, a crocodile can drastically raise its concentration of plasma bicarbonate. This increase in [HCO3-] also occurs in humans who do not exhale enough CO2; the difference, however, is that crocodile hemoglobin can bind bicarbonate. If this action stabilizes the T, or "tense," state of hemoglobin, it can decrease the affinity of that molecule for oxygen. O2 may then be quickly released.

Post-transcriptional silencing is theorized to prevent some gene expression in response to stress. Which of these modifications could prevent functional mRNA from being translated?

Binding to miRNA C is correct. MicroRNAs are responsible for post-transcriptional silencing. These short ncRNA molecules bind to mRNA, signaling its destruction or promoting degradation of its protective post-transcriptional modifications.

Marvin is painting the "rainbow effect" of white light separation as it passes through glass. Which of the following separated colors would he observe to have the greatest deviation from the normal?

Blue For visible light, the greater the frequency or shorter the wavelength, the more it refracts. Violet light, being slowed down to a greater extent by the absorption and re-emission process, refracts more than red light. Colors of the visible light spectrum that have shorter wavelengths (BIV) will deviate more from their original path than the colors with longer wavelengths (ROY).

How does the Bohr model account for the emittance of electromagnetic radiation at specific wavelengths and frequencies?

Bohr proposed that electrons could only stably orbit the nucleus at specific distances, and that jumping between these distances was responsible for the emittance of electromagnetic waves. A is correct. Bohr proposed "stationary orbits," or orbits at the specific distances at which an electron is stable. These orbits are associated with energy levels. Jumping between orbits must then require either the absorption or the emittance of a photon with a frequency that is related to the difference in energy levels between the orbits and to Planck's constant.

Generally, women and men experience the feeling of empathy differently. Describe this difference.

Both men and women feel empathy, but women are more likely to express it through overt communication. C is correct. Empathy refers to the ability to identify with another person's emotions or "walk a mile in their shoes." Research suggests that both men and women feel empathy relatively equally, but women are more likely to express it outright (by crying, verbally expressing genuine concern, etc.)

One popular analytical procedure is thin-layer chromatography (TLC) involving a hexane mobile phase and a plate coated in silica gel. Which of the molecules below will adhere best to the plate?

C is correct. The key to this question is understanding that silica (SiO2) is a highly polar molecule. Therefore, this TLC procedure involves a polar stationary phase and a nonpolar mobile phase (hexane, or C6H14). Compounds prefer to associate with other molecules of similar polarities, so the most polar molecule listed will interact best with the silica plate. Here, that molecule is 2,3-butanediol, as its hydroxyl groups allow it to form multiple hydrogen bonds.

Curcumin is the active ingredient in turmeric (a ginger-like plant) and has a molar mass of 368.4 g/mol. If curcumin contains 68.5% carbon, 5.5% hydrogen, and 26% oxygen by mass, what is its molecular formula?

C21H20O6 First, we must find the molar ratio of the elements involved. Beginning with carbon, we find that 68.5 g C × (1 mol C / 12 g C) = approximately 6 mol. For hydrogen, 5.5 g H × (1 mol H / 1 g H) = 5.5 mol. Finally, for oxygen, 26 g O × (1 mol O / 16 g O) = around 1.6 mol O. Since oxygen has the smallest molar value, we divide the other numbers by 1.6 to find the overall ratio. Starting with carbon, 6 mol / 1.6 mol = around 4 moles. For hydrogen, 5.5 mol / 1.6 mol = around 3.5 moles, and oxygen, of course, is given a molar value of 1. Unfortunately, this yields a formula of C4H3.5O, which contains a non-whole number and does not match the given molar mass of the compound. To solve, we can simply see that C4H3.5O has a molar mass of 67.5 g / mol, slightly less than one-fifth of the molar mass. Multiplying the subscripts by 5 gives an answer closest to A.

Fas, which is expressed by most healthy somatic cells, is a membrane receptor that triggers apoptosis when bound by Fas ligand. Both Fas ligand and Fas itself are expressed on the surface of CD8+ T cells, the former of which enables, at least in part, their cytotoxic function. Cells that are LEAST likely to express Fas ligand include:

CD4+ T cells. C is correct. The question stem tells us that Fas ligand serves a cytotoxic role; in other words, it induces the death of cells that it contacts and binds. Unlike CD8+ cells, CD4+ T cells do not have substantial cytotoxic activity. Although they do express Fas, they have no reason to express Fas ligand.

Hashimoto's disease is marked by decreased thyroid function, thyroid gland destruction, and symptomatic reduction in circulating thyroid hormone. Substantial evidence exists to suggest that this condition develops, at least in part, as a result of autoimmunity. Which of the following, if true, would provide the strongest evidence against this autoimmunity hypothesis?

CD8+ T cells isolated directly from the thyroid gland of a Hashimoto's patient display virtually no affinity for any thyrocyte surface antigens. C is correct. This suggests that the patient's own immune cells are not displaying a reaction to the cells of the thyroid. Since the question asks for a fact that provides evidence against the auto-immunity hypothesis, this fits the bill.

An unknown organic compound weighing 1980 mg is fully combusted at high pressure, yielding 900 mg of carbon dioxide and 360 mg of water. What is its empirical formula?

CH2O5 B is correct. First, remember that all of the carbon in the original compound ends up in the form of carbon dioxide, while all of the hydrogen will be present as part of the water. Thus, we need to determine how many moles of the measured CO2 are carbon and how many moles of the measured H2O are hydrogen. 900 mg of CO2 contains 0.270 grams of carbon, which is 0.02 moles. 360 mg of H2O contains 0.04 grams of hydrogen, which is 0.04 moles. Finally, the remaining mass left unaccounted for in the original compound must represent oxygen. So, 1980 mg - 270mg - 40 mg = 1670 mg oxygen, or .10 moles. Since carbon is present in the smallest molar value, our ratio (and empirical formula) must be CH2O5.

Which of the following atoms or ions is isoelectronic to Cl-?

Ca2+ Isoelectronic species have the same number of electrons. From the periodic table, we can see that both Cl- and Ca2+ possess 18 electrons.

Which of the following accurately lists a second messenger and its role in one or more signaling cascades?

Ca2+ serves to amplify the signal(s) in which it is involved. Calcium ion is a classic example of a second messenger. In a typical signaling cascade, a ligand serves as the "first messenger" by binding to a membrane receptor outside the cell This binding triggers the activation of a second messenger, which typically amplifies the relevant signal by activating kinase molecules.

Unrestricted cell division is a hallmark of cancer cells. What is a reasonable explanation for this characteristic?

Cancer cells lack cell cycle checkpoints. Cell cycle checkpoints are incredibly important in the regulation of cell growth. Typically, if its environment lacks nutrients, a cell may enter the G0 phase, while if DNA replication was inaccurate, it will remain in the G2 phase. Cancer cells, on the contrary, often lack such checkpoints, making them prone to unrestricted proliferation.

The urea cycle is a cyclic process similar to the citric acid cycle. Ornithine is regenerated during one turn of the cycle and used for the next. Which pairing correctly matches the reactants and products of this cycle, respectively?

Carbon dioxide and ammonia, urea B is correct. The urea cycle produces urea from ammonia; it is necessary to prevent the buildup of ammonia, a toxic nitrogenous product. The process is cyclic, and the first set of reactants include ATP, ammonia, and carbonate, which is equivalent to water and carbon dioxide. The end products of the cycle are urea and ornithine, the latter of which is reused during the next series of reactions.

A reasearcher places a large number of ammonia molecules into a magnetic field and notices that they align with the field. He then attempts the same process with carbon dioxide. Despite the fact that both of these compounds contain polar covalent bonds, CO2 does not align with the magnetic field. How can this observation be explained?

Carbon dioxide is a linear molecule, so although its bonds are polar covalent, it has no net dipole. Carbon dioxide is a linear molecule. Even though each C-O bond is polar, the overall symmetry causes the individual dipoles to cancel. Since no molecular dipole is present, CO2 would not align with the magnetic field.

In peripheral blood vessels, carbon dioxide from adjacent tissues diffuses into red blood cells. Here, it is converted into carbonic acid, which in turn dissociates into protons and bicarbonate; this process is catalyzed by the enzyme carbonic anhydrase. In the lungs, bicarbonate reenters the red blood cells, where it is turned back into carbon dioxide. What enzyme catalyzes the reverse reaction?

Carbonic anhydrase C is correct. An enzyme reduces the activation energy for both the forward and reverse reactions. The direction of reaction in this case is governed by the relative amounts of reactants and products, i.e. the difference between the reaction quotient and equilibrium. In tissue, high amounts of carbon dioxide exist outside of the red blood cell, driving diffusion into the cell and consumption of the carbon dioxide by carbonic anhydrase. In the lungs, there is low carbon dioxide tension outside of the cell, driving production of the gas by carbonic anhydrase and subsequent diffusion out of the cell into the blood.

Previous studies have determined that black patients with cardiac symptoms receive an inferior quality of care in comparison to white patients. An administrator at a neurology clinic wishes to determine whether a similar disparity exists at his facility. To do so, he recruits a group of elderly black men to feign stroke symptoms, observe their own treatment, and fill out a survey rating several aspects of their care on a scale of 1 to 5. Their average rating for most aspects was low, particularly on the scale for "The physician listens to my concerns," which averaged 1.5. Which of the following statements could be accurately made about this study?

Care quality was operationalized as a quantitative variable.

A young girl is experiencing a crush for the first time. When she tells her friends, she says "I hope he likes me the way I like him." Which type of self-concept development is this, and why?

Categorical self; the young girl is realizing that she is an object that other people may also perceive. The development of self-concept, which occurs when children are very young, is divided into two steps. The aspects involved are the existential and categorical selves. In the development of the categorical self, a child comes to understand that he or she is subject to the perceptions of other individuals. Here, the young girl is considering how she might be perceived by another entity, the young boy on whom she has a crush. This girl is demonstrating her ability to possess perspective, or to view the world through the eyes of another. All of this information indicates that she is developing her categorical self, and thus understands that she is an "object" that can be experienced by her crush and others.

Lipids are an important component of cell membranes. Which pair of terms lists a class of lipids that is necessary to maintain membrane fluidity and one that is used as markers for cell signaling, respectively?

Cholesterols, glycosphingolipids C is correct. Cholesterols are cyclic molecules that exist in the phospholipid bilayer to maintain fluidity and shape; without them, the bilayer would be too rigid. In contrast, glycosphingolipids are sphingolipids with a sugar attached to the head group. These molecules protrude from the membrane to act as tags for cell signaling.

Development of the mesoderm leads to the growth of a coelom, inside of which growing organs are protected. The mesoderm can develop into multiple sub-mesodermal tissues. Which of the following pairings correctly matches a sub-mesodermal tissue layer with its corresponding tissue?

Chorda-mesoderm - notochord The chorda-mesoderm, as its name implies, develops into the notochord. The intermediate mesoderm develops into gonads and kidneys, the lateral plate mesoderm develops into the gut wall and circulatory system, and the paraxial mesoderm develops into skeletal muscle and cartilage.

Within the nervous system, neurons compose only a fraction of the total cellular population. The remaining cells, broadly termed neuroglia, perform a variety of supporting roles. Which of the following is LEAST likely to be a function of peripheral glia?

Circulation of CSF via the action of cilia present on the cell surface The peripheral nervous system is not bathed in CSF, so this cannot possibly be a function of peripheral glia. Note that in the CNS, ependymal cells do use cilia to circulate CSF throughout the brain and spinal cord, a function that is essential for the effective clearing of extracellular waste.

High levels of acetyl-CoA stimulate which of the following enzymes? I. Citrate synthase II. Pyruvate carboxylase III. Pyruvate dehydrogenase

Citrate synthase and Pyruvate carboxylase

Which sequence best describes the pathway used to transmit auditory information in humans?

Cochlea → organ of Corti → medial geniculate nucleus → auditory cortex Sound travels from the cochlea in the ear to the organ of Corti, which translates physical vibrations into electrical signals. From here, these signals leave the ear and move via the auditory nerve to the brain stem, eventually reaching the medial geniculate nucleus. This information is then processed in the auditory cortex.

bind to the free enzyme at the active site

Competitive inhibitors

Transition shock, including culture shock, can be experienced by an individual who is in a state of disorientation created by an unfamiliar environment. Which of the following are symptoms of transition shock?

Concern over cleanliness a glazed stare compulsive eating

Which of the following is NOT one of Goldberg's five factors or scales along which personality can vary?

Conformity Goldberg's five dimensions are agreeableness (the degree to which one values getting along with others), extraversion (the degree to which one engages with people and the external world) conscientiousness (a person's level of self-discipline and focus on achievement) neuroticism (the degree to which one experiences negative emotions like anxiety, depression, or anger) and openness to experience (the amount of value placed on novelty, creativity, and variety of experiences). Conformity is not a factor on this list.

Mike, an American service member, prevents a mugging on a train one day and is lauded as a hero. Which type of cue likely aided the public in making this dispositional attribution?

Consensus - Mike performed an action that most individuals did not have the foresight or quick thinking to do, so his actions can be attributed to his personality. B is correct. A consensus cue focuses on the closeness of the behavior to that which is typically expected by society. In other words, when people act just like everyone around them, we tend to attribute their behavior to situational factors. However, when individuals deviate from common social behavior, we use that deviation to assess their personality. Here, most individuals on the train clearly were either unwilling or incapable of stopping the mugging. Since Mike did so, the motivation behind his actions will be cited as dispositional and the public will gain a favorable impression of his personality.

Which of these behaviors or physiological processes would NOT stimulate glycogenolysis?

Consumption of a carbohydrate-rich meal C is correct. Glycogenolysis denotes the breakdown of glycogen, a polymer of glucose that is stored by the body for future energy needs. The body utilizes glycogen when food consumption does not provide sufficient energy; in other words, glycogenolysis occurs when one has not eaten recently or exists in a state of increased ATP demand. Eating a carbohydrate-rich meal would significantly reduce the need for such a process.

Bats, birds, and butterflies all developed wings in order to utilize the sky and all of its associated advantages. What type of evolution is involved here?

Convergent evolution Convergent evolution occurs when entirely separate lineages gradually appear more similar over time. Here, bats, birds, and butterflies are very distantly related, but all evolve to possess wings through different mechanisms. In the end, these species resemble each other despite their genetic differences.

Far-western blotting is a variation on the more commonly used western blotting. In this technique, a tissue extract containing many proteins is transferred to a nitrocellulose membrane; proteins of interest are then detected using radiolabeled "bait" proteins. Generally, these bait molecules are recombinant products that are selected according to known protein-protein interactions with the "prey" species on the membrane. Considering that the far-western blotting protocol involves denaturation of the prey proteins before application of the bait molecules, it most likely detects the presence of "prey" using which type of interaction?

Covalent bonding between two amino acid residues A is correct. Since the target proteins are denatured, any interaction that depends on secondary or higher levels of structure is unlikely to occur. This eliminates all answer choices except for A. Since denaturation does not involve the destruction of individual amino acid residues, it is perfectly reasonable to assume that such cross-linking could still occur and lead to identification of the prey protein.

Creutzfeld-Jakob Disease (CJD) has been directly attributed to the consumption of misfolded, highly stable protein fragments called prions. However, alternative hypotheses have suggested a viral origin for this disease. Which of the following would most strongly support the prion hypothesis over the viral theory?

Crude brain lysates subjected to toxic levels of ionizing radiation still reliably induce pathology when fed to healthy mice. Ionizing radiation damages nucleic acids and leads to mutations. Since viruses depend on intact nucleic acids to transfer genetic information, it is unlikely that CDJ has a viral cause. Peptide bonds are less affected by ionizing radiation, and a hallmark of prion proteins is their thermodynamic stability.

Which of these options correctly lists five elements in order of decreasing reactivity with water?

Cs, K, Ca, Mg, Be Reactivity with water is heavily based on ionization energy. Specifically, elements with low IE values tend to lose electrons easily, allowing them to rapidly form oxides in water. In general, Group 1 elements are more reactive than Group 2 ones, as they lose single electrons very readily. (In fact, sodium and potassium metals are known for exploding when placed in contact with water.) Only choice C lists the five elements in question from lowest to highest ionization energy.

Which of the molecules below, if present in a 4 M aqueous solution, would not rotate plane-polarized light?

D is correct. Imagine that you rotate the substituents on the right chiral carbon upwards. It can then be seen that this molecule has a line of symmetry. Symmetrical molecules with multiple chiral centers are known as meso compounds and do not rotate polarized light.

Which of the following choices accurately identifies a property of deoxyribonucleic acid (DNA)? I. DNA carries the instructions necessary for all cellular functions. II. DNA forms a double helix that is exclusively stabilized by hydrogen-bonding interactions between the nitrogenous bases. III. DNA can be found in a supercoiled structure when it is not being actively transcribed or replicated.

DNA carries the instructions necessary for all cellular functions and DNA can be found in a supercoiled structure when it is not being actively transcribed or replicated.

While DNA and RNA are strikingly similar macromolecules, they also differ in a variety of ways. Which of the following statements regarding DNA and RNA is accurate? I. DNA contains a more stable sugar than RNA. II. RNA can catalyze biochemical reactions, but DNA cannot. III. DNA and RNA cannot base pair with each other. IV. DNA and RNA are found throughout the cell.

DNA contains a more stable sugar than RNA and RNA can catalyze biochemical reactions, but DNA cannot.

A common phenomenon involving changes in protein structure can be observed when frying eggs. The transparent gel surrounding the yolk becomes opaque and hardens when exposed to high levels of heat. What specific mechanism is responsible for this change?

Disruption of the hydrophobic interactions that form the protein's tertiary structure B is correct. The heat used in the frying process denatures the tertiary structure of the protein. The protein unfolds, exposing its hydrophobic interior to the environment and facilitating bond formation with other protein molecules. This results in a more solid, hard egg white.

Under certain conditions, the plasma pH (normally about 7.35) can drop, a phenomenon known as acidosis. How do amino acids play a role in buffering to mitigate this condition? I. Due to increased hydrogen ion concentrations, the side chain of arginine can gain a proton. II. Due to increased hydrogen ion concentrations, the side chain of glutamic acid can gain a proton. III. Due to decreased hydrogen ion concentrations, the side chain of histidine can lose a proton.

Due to increased hydrogen ion concentrations, the side chain of glutamic acid can gain a proton.

The following diagram shows several reactants and intermediates involved in a multi-step reaction. If a laboratory researcher is attempting to conduct this reaction, which conditions should he use to facilitate the first step?

Either acidic or basic conditions C is correct. The first step in this reaction is the formation of the hemiacetal. Since the oxygen atom in methanol is a fairly good nucleophile, and since nucleophiles readily attack the carbons of carbonyl groups, this step can occur under either acidic or basic conditions. Note that the formation of the acetal, the second step, is only possible under acidic conditions.

During an action potential, voltage-gated channels open and result in a flow of sodium into the cell. This influx stabilizes at a membrane potential of +35 mV, even though the extracellular concentration of sodium is still higher than the Na+ concentration inside the cell. How can this occurrence be explained?

Electrochemical equilibrium has been reached due to the high intracellular concentration of K+. D is correct. Though sodium is not at equilibrium with regard to concentration alone, its influx has caused the cell to become more positive than the extracellular fluid. Electrostatic forces repel cations and prevent more sodium from entering the cell.

Borderline personality disorder may have the least informative name of all major personality disorders; in fact, the APA has considered changing its official name in future editions of the DSM. Based on the symptoms of BPD, what might be a more fitting name for the disorder?

Emotionally unstable personality disorder B is correct. Borderline personality disorder is characterized by an unusual intensity of emotions. Individuals with BPD often have trouble seeing people in anything other than black-or-white terms (as wholly good or completely evil). In addition, these views and associated emotions are prone to sudden flips to the opposite extreme based on minor provocations.

Among other symptoms, withdrawal from opiates such as morphine and heroin is associated with physical hallmarks including cramps, aches and pains. This occurs because the brain has become habituated to opiate stimulation and, in response, has reduced its sensitivity to which naturally-occurring neurotransmitters?

Endorphins C is correct. Endorphins are known as the "natural painkillers" of the brain, and are released en masse after activities like vigorous exercise and sex. However, they are also continuously secreted at low levels during normal activity. As opiates like heroin and morphine act on the same receptors as endorphins, these receptors are downregulated when an addict becomes habituated to such substances. This renders him (or her) less sensitive to his own natural endorphins, so he will experience understimulation of these pathways when drug use is ceased, resulting in generalized pain.

A particular protein largely lacks both secondary and tertiary structure. Which factor, if any, is mainly responsible for the resting state of this protein?

Entropy Hydrogen bonds, disulfide bonds, and dipole-dipole interactions produce the complex folding patterns seen in secondary and tertiary structure. However, if a protein does not have these elements of structure, it will adopt a state in which its entropy is maximized. This will result in a shape somewhere between a state of tight, complex folding and one of a completely linear arrangement.

The diagram below depicts a simplified gastrula. What tissues will arise from the cells marked by the dark arrow?

Epidermis, nervous system, lens of the eye, hair The arrow points to ectodermal cells. This germ layer differentiates into "external" features including the epidermis, lens of the eye, fingernails, and hair. It also gives rise to the nervous system via neurulation.

Transmembrane proteins are able to freely diffuse throughout the cellular membrane, typically leading to a random distribution. However, in epithelial cells that line the small intestine, the membrane possesses polarity. As a result, the transmembrane proteins found on one side of the cell have a different distribution from those on the other. How is this possible?

Epithelial cells are connected by tight junctions, which prevent movement of transmembrane proteins from one side of the cell to the other. A is correct. Epithelial cells separate the lumen of the small intestine from the rest of the body and mediate the absorption of nutrients. Thus, these cells must form a barrier that does not allow fluid to pass. This is accomplished through tight junctions. A secondary effect of tight junctions is that they allow for the creation of polarity, since transmembrane proteins are unable to cross to the opposite side of the cell.

Protein metabolism produces a toxic nitrogenous waste. Together, the liver and kidneys work to chemically alter this product into one that can be excreted by the body. Which toxic product is initially formed, and into which metabolite is it changed for excretion?

NH3 is converted to CH4N2O using ATP, which can be excreted.

If a mutation in a eukaryotic cell eliminated all functioning protein kinase A, how would fatty acid metabolism be affected?

Fatty acid metabolism would decrease dramatically. B is correct. Protein kinase A is an important enzyme for the regulation and control of fatty acid metabolism. Signaling molecules such as glucagon, epinephrine, and norepinephrine activate this kinase through a G-protein coupled receptor pathway. Once they bind the GPCR, this activates a G protein that then activates adenylyl cyclase to produce cyclic AMP. Cyclic AMP then activates protein kinase A, which phosphorylates and activates hormone-sensitive lipase, the enzyme responsible for hydrolyzing stores of fat.

A pharmaceutical compound targets and inhibits the carnitine shuttles used by cells. A human cell treated with this medication currently displays excess acetyl-CoA inside the mitochondria and insufficient cytosolic acetyl-CoA. What will be the most direct result of this circumstance?

Fatty acid synthesis will be inhibited. While an inhibited carnitine shuttle will cause many long-term issues, the question stem asks us to consider only the most immediate effects. Fatty acid synthesis takes place in the cytosol and requires acetyl-CoA, so this process will be inhibited if cytosolic concentrations of this molecule are too low.

What is the difference between filtration and secretion?

Filtration refers to the original movement of fluid and solute into the Bowman's capsule, while secretion relates to the transport of specific toxins and protons into later regions of the nephron. Filtration, secretion, and reabsorption are extremely easy terms to confuse. Remember, filtration refers to the general movement of plasma from the glomerulus (a tight network of capillaries) into the entrance of the nephron at the Bowman's capsule. This process is extremely general, with virtually any molecule smaller than protein allowed passage into the filtrate.

A missense mutation replaces an alanine residue located in the interior of a soluble protein with a serine residue. What will be the most likely effect of this change on the protein?

Folding will be disrupted due to the mistaken inclusion of a polar amino acid. A is correct. Serine is a polar amino acid, while alanine is nonpolar. This will disrupt the hydrophobic effects related to protein folding.

When conducting research on medicalization, what is the next step in the scientific method after a review of the literature?

Formulating a hypothesis The steps of the scientific method are: Ask a question, do background research, construct a hypothesis, test the hypothesis via experimentation, analyze the data/draw conclusions, and finally, communicate results.

Interphase, which comprises the bulk of the cell cycle, is itself split into multiple phases. Which segment of interphase generally spans the longest period of time?

G1 During the G1 phase, the cell conducts protein and organelle synthesis at a high rate while the cell grows in size. The transition from the G1 to the S phase is termed the "restriction point" and constitutes the rate-limiting step in the cell cycle.

If a cell has not yet reached an adequate size to progress further, in which phase will it remain until the issue is rectified?

G1 The G1 phase is responsible for protein and organelle production in addition to cell growth. Two major checkpoints mark the cell cycle: one during G1 and the other in G2. In order to pass the G1 checkpoint, the cell must have reached a sufficient size and synthesized enough organelles to move into the S phase.

If the anterior pituitary were suddenly rendered incapable of producing hormones, which of the following compounds would be directly affected? I. ADH II. GH III. LH IV. PTH

GH and LH

Insulin acts to lower blood glucose levels via cellular uptake that utilizes which transporter?

GLUT4 Insulin causes GLUT4 transporters to translocate to the cell membrane, allowing the cell to take up more glucose.

Which of the following DNA segments contains a restriction enzyme recognition sequence?

GTAAGCTTCACG B is correct. Only AAGCTT, found in choice B, would result in a palindromic complement. Restriction enzyme recognition sites are palindromic double-stranded sequences.

A sociologist uses adoption studies to examine the prevalence of criminal behavior in young adults. She finds that children born to incarcerated offenders but raised in a different household have a higher chance of committing a felony than members of a control group. What can we conclude from this information, if the researchers find that p < 0.05?

Genetics probably plays a role in the determination of future criminal behavior. A p value that is smaller than 0.05 generally denotes a statistically significant relationship. In other words, the sociologist has established an apparent link between criminally active relatives and a child's future tendency to be a criminal himself. However, this is simply a single study and appears to be very limited; for example, it assessed only felonies, not criminal actions in general, and by definition missed individuals who were not caught. For this reason, choice B represents the least extreme conclusion.

A second-grade boy loves playing soccer outside, even when alone. He originally found a soccer ball in his family's garage and began kicking it around the yard entirely of his own accord. If the boy's parents want to reinforce this behavior, which action should they NOT take?

Give the boy $5 whenever they see him take the ball outside. This child is already intrinsically motivated to play soccer, as evidenced by the fact that he does so for long stretches on his own with no outside rewards. Research has shown that offering rewards (extrinsic motivation) to people who are already intrinsically motivated actually makes them less willing to engage in the task. For this reason, leaving the child alone is preferable to rewarding him in any predictable way. In contrast, the parents should not offer to pay the boy money (an extrinsic motivator). Choice C, then, is the answer to this NOT question.

Over the past few decades, industry has spread worldwide. While this transition has somewhat "flattened" the disparities in standards of living between regions of the world, it has also increased the inequality within countries. What term relates best to this major change?

Globalization C is correct. Globalization is the process by which the world becomes more interconnected. This worldwide change is especially relevant to economic interactions.

During an early step of glycolysis, glucose is phosphorylated to form glucose 6-phosphate. This reaction is catalyzed by which enzyme?

Glucokinase in liver cells and pancreatic beta cells, hexokinase in all other cells D is correct. Hexokinase mediates the first step in glycolysis by adding a phosphate group to the sixth carbon in order to trap glucose within the cell. Glucokinase is an isozyme of this molecule that is used in the liver and pancreatic beta cells. Glucokinase has a much lower affinity for glucose and is used to maintain blood sugar levels.

Which statement(s) regarding the regulation of glycolysis is/are true? I. Each step of glycolysis can be directly up- or downregulated by the presence of certain coenzymes or products. II. Glycolysis and gluconeogenesis share all of the same enzymes, since one process is the exact reverse of the other. III. Glucose 6-phosphate allosterically inhibits Step 1 of glycolysis. IV. AMP allosterically activates Step 3 of glycolysis, but allosterically inhibits the final step of the glycolytic pathway.

Glucose 6-phosphate allosterically inhibits Step 1 of glycolysis.

A synthetic "cell" contains 2.5 M of glucose and 3.5 M of sucrose, along with plenty of glucose and sucrose channels in its membrane. If it is placed in a solution of 5.0 M glucose and 1.0 M sucrose, in which direction will these solutes initially diffuse?

Glucose will diffuse into the cell, while sucrose will diffuse outward. Although the total concentration of solutes is identical inside and outside the cell, glucose and sucrose each possess their own concentration gradients. Glucose will move down its gradient from the extracellular solution into the cell, and sucrose will travel in the opposite direction.

Of the pathways fed into by the pentose phosphate pathway, which would be most affected if this enzyme were to lose functionality?

Glycolysis B is correct. Transketolase is responsible for catalyzing the reaction to form glyceraldehyde 3-phosphate and fructose 6-phosphate. Both of these products are part of glycolysis.

Which of these groupings accurately connects the Gram status of a bacterial species with its structural hallmarks?

Gram-positive - thick cell wall - outer membrane absent Gram-positive bacteria have a thick peptidoglycan wall. This allows them to stain purple, since crystal violet stain, once fixed with iodine, is unable to exit through this thick layer. Additionally, such a bacterium lacks a second membrane on the outside of its cell wall. This quality makes Gram-positive cells especially sensitive to antibiotics that target peptidoglycan.

Which reagent is missing from the reaction above, which depicts the activation of a zymogen?

H2O C is correct. The reaction depicts a hydrolysis mechanism, in which a molecule of water is used to split the substrate. Hydrolysis is a very common method of activating zymogens. By examining the reaction, you can see where the water molecule donates an H+ and an OH-.

A scientist radioactively labels the oxygen atoms in O2 before tracing their journey through the citric acid cycle and mitochondrial respiration. At the end of the study, the radioactive atoms will be found in:

H2O D is correct. As the final electron acceptor in the electron transport chain (ETC), oxygen is reduced to form H2O. Thus, the radioactively-labeled O atoms would be found in water.

A phosphotransferase enzyme with a cysteine residue in its active site acts on ATP to form the below intermediate. If the final product of this process is a free phosphate group, which reagent must be added to complete the reaction and free the substrate from the enzyme?

H2O H2O must be added to hydrolyze the substrate from the enzyme's active site. The water molecule will donate a proton to the cysteine residue and a hydroxyl group to the phosphate ion. Note that water is often used to promote hydrolysis reactions, which break bonds.

Methane bubbling from the ocean floor mixes with water, as represented by the following chemical equation: CH4 (g) + H2O (l) → CO (g) + 3H2 (g) In a biology experiment, 30 mg of methane and 18 mg of water are mixed in a closed container. What is the limiting reagent, and how much excess reagent remains when that species has been exhausted?

H2O; 14 mg CH4 To calculate the limiting reagent, we must determine how much methane is required to react with 18 mg of water. First, we need to convert grams to moles. 0.018 g H2O × 1 mol/18 g = 0.001 mol water, which requires 0.001 mol CH4, or 0.016 g, to fully react. Since we actually have 0.03 g of CH4, water must be the limiting reagent. 0.03 g CH4 - 0.016 g reacted = 0.014 g CH4 remaining.

List these four species in order of increasing pH, assuming that each is present in 0.10 M aqueous solution: H3PO4, H2PO4-, HClO4, PO43-.

HClO4 < H3PO4 < H2PO4- < PO43- Don't miss the inclusion of perchloric acid in the midst of three phosphorus-containing examples! HClO4 is a strong acid and thus has the lowest pH in solution of all four choices (assuming, of course, that starting concentrations are equal). Next, H3PO4 is a triprotic acid with a fairly stable conjugate base, although phosphoric acid certainly is not strong. Subsequent deprotonation yields stronger and stronger conjugate bases, making PO43- the most basic of the listed species. This tendency is very normal; intact polyprotic acids are always more acidic than their deprotonated conjugates.

The oxidation potentials of Na (s), Cd (s), and Ni (s) are 2.71, 0.40, and 0.28, respectively. A student sets up a galvanic cell with a cadmium anode and a nickel cathode under standard conditions. If he wished to make the cell potential of this apparatus more positive, which change should he implement?

He could increase the concentration of Ni2+. Since the cathode is composed of solid nickel metal, the relevant half-reaction must be Ni2+ + 2 e- 🡪 Ni (s). For this reason, raising [Ni2+] would increase the favorability of this reaction. According to Le Châtelier's principle, such a change would increase the reactant concentration and drive the associated redox reaction toward the products.

A novelist is attempting to create a character who believes in the just-world hypothesis. Which of these traits or behaviors should this character exhibit?

He firmly believes in karma and often says "what goes around comes around." A is correct. The just-world hypothesis is a bias toward believing that the world is fair - that good actions are rewarded and bad actions are punished. This is very similar to a belief in karma, or that people eventually get what they deserve.

A middle-aged man's home behavior differs greatly from his actions when he is at the office, running his company. According to the dramaturgical approach, what is this man doing?

He is expressing his back-stage self at home and his front-stage self at the office.

A researcher finishes synthesizing a peptide in a 3 mL reaction flask and wants to isolate it. The peptide is rich in threonine, glutamine, lysine, and glutamate resides. The researcher reduces the volume of the peptide solution to 0.5 mL, then plans to add 2.5 mL of a solvent that will cause the peptide to precipitate out, yielding a sample with higher purity. Which solvent should the researcher use?

Hexane C is correct. The four amino acids listed are polar and would thus be soluble in water and other polar solvents. However, we wish to form a precipitate, which would occur in a nonpolar solvent such as hexane.

The transcription factor AP-1 is a heterodimer consisting of c-jun and c-fos. C-jun and c-fos are soluble proteins that can be localized to either the cytosol or nucleus of a cell. C-jun and c-fos dimerize through a leucine zipper motif. In a leucine zipper motif, every 7 amino acid residues, or 2 full turns of an alpha helix, are leucine resides. Leucine and other amino acids on one face of the helix come together to form an opposite alpha helix that has a similar arrangement of leucine and other amino acids. Which solvent would be LEAST favorable for c-fos/c-jun dimerization?

Hexane For long questions like this one, begin by summarizing exactly what the question is asking. In short, leucine residues on different parts of a molecule are coming together to form a dimer. Leucine is hydrophobic, since its side chain contains only carbon and hydrogen. If the solvent were also hydrophobic, the face of a leucine zipper could interact just as favorably with the solvent as with the opposite alpha helix. Some leucine residues would likely interact only with the solvent, preventing formation of the dimer entirely. Hexane, with its nonpolar hydrocarbon structure (shown below), is the least polar solvent listed.

A lab director emphatically tells his organic chemistry students to conduct a certain separation procedure using fractional distillation. Which two compounds might these students be separating?

Hexanol and 2-methylpentanol C is correct. Fractional distillation is used to separate compounds with very close boiling points (generally less than 25 °C apart). The apparatus is very similar to that used for simple distillation, except a long fractionating column is placed directly above the distillation flask. As compounds must travel up the entire column in the vapor phase to move into the flask used to collect the products, enhanced separation is achieved. Of the pairs mentioned, hexanol and 2-methylpentanol are likely to have the most similar boiling points. Both are six-carbon alcohols; 2-methylpentanol just demonstrates slightly more branching, lowering its BP a few degrees.

During art class, a teacher asks her third-grade students to choose three of their favorite colored pencils to draw with. She scolds a girl who has picked blue, red, and green, saying that these colors are too "boyish." To what aspect of education does this teacher's response best relate?

Hidden curriculum Hidden curriculum refers to implicit or unspoken values that are taught in school. Often, parts of the "hidden curriculum" are not even intentional, but are conveyed unconsciously by teachers or administrators. This art teacher seems to believe that boys should act like boys and girls should act like girls. However, this idea clearly is not part of a written syllabus for the class. By scolding her student for picking boyish colors, she is enforcing a hidden curriculum.

A recently-identified species of eukaryote is able to survive at extremely high temperatures. If a group of scientists were able to analyze the composition of the cell membrane of such a species, what would they expect to observe?

High levels of cholesterol-like molecules and low levels of unsaturated fatty acids B is correct. In order for this species to thrive at high temperatures, its cell membranes must be very stable to avoid become overly fluid. Since unsaturated fatty acids increase the fluidity of plasma membranes, a low level of these would be expected in this organism. In contrast, cholesterol and other sterol-like molecules function as a type of buffer. These compounds prevent high or low temperatures from adversely affecting the fluidity of cell membranes; therefore, high levels of these would be expected.

A researcher is attempting to create an artificial cell membrane that retains its fluidity at extremely low temperatures. Which features should he incorporate in this membrane?

High levels of unsaturated fatty acids High levels of sterols

During the Krebs cycle, fumarate and water unite to form malate. How can this type of chemical reaction best be described?

Hydration During the synthesis of malate, fumarate experiences the addition of water across its double bond. This results in the replacement of this bond with an -OH group on one carbon and a hydrogen atom on the other. Such a process represents a classic hydration reaction.

A biologist exposes yeast cells to factors that stimulate the production of a certain lipid. After synthesis, this lipid typically inserts into the cytosolic leaflet of the endoplasmic reticulum's plasma membrane. The scientist simultaneously exposes the yeast cell to a chemical that disrupts vesicular transport. In which locations might this lipid be found shortly after these steps? I. The inner leaflet of the endoplasmic reticulum II. The nuclear membrane III. The Golgi apparatus

I and II B is correct. Lipids are typically inserted in the cytosolic leaflet of the ER after synthesis. Proteins positioned between the two leaflets, known as flippases, move lipids into the inner leaflet; the activity of these transporters would not be affected by a halt in vesicular transport. Choice III also makes sense; remember, the nuclear membrane is continuous with the ER. Thus, as lipids move freely according to the fluid mosaic model, they should be able to migrate from the ER to the surface of the nuclear membrane.

A scientist contemplates the reaction of benzoic acid with ammonia, with the intention of synthesizing the corresponding amide via a nucleophilic substitution reaction. As one might expect, this reaction is rather slow. However, the formation of the amide product is thermodynamically favorable. Which of the following reagents, if added to the reaction vessel, is likely to increase the speed of this reaction? I. Titanium tetrachloride, a Lewis acid II. Acetic acid, a weak Brønsted acid III. Palladium on carbon, a metallic catalyst

I and II C is correct. A Lewis acid is likely to complex with the carbonyl group, further polarizing the C=O bond and increasing the electrophilicity of the carbon atom. A similar effect can occur upon addition of a weak acid, where the proton coordinates with the carbonyl oxygen atom to further polarize the bond. (Critically, it should be noted that ammonia would probably also form a coordination complex with TiCl4. In addition, NH3 has the potential to accept a proton from a weak acid. However, we are told that the amide formation is thermodynamically favorable. Since neither of these two side reactions is productive in any way, they will simply reverse in solution, allowing the favorable overall reaction to proceed instead.)

Neuroplasticity has been a topic of considerable interest for several decades. Consistent stimulation at a particular synapse has been shown to induce a number of structural and metabolic changes in both pre- and postsynaptic neurons. These changes result in the "strengthening" of the synapse, making propagation of an action potential more efficient and sensitizing the synapse to further input. Considering that AMPA is a glutamate receptor, which of these actions would most likely contribute to the strengthening of a glutamatergic synapse? I. CaMKII-mediated phosphorylation of AMPAR, which improves channel conduction II. Calcium-mediated activation of intracellular scaffolding proteins with multiple AMPAR binding domains III. Endocytosis of perisynaptic AMPA receptors without subsequent proteolysis

I and II Increased conduction through AMPA receptors would permit the more efficient passage of extracellular cations into the neuron, facilitating a more rapid postsynaptic response. With regard to statement II, the activation of scaffolding proteins would permit tight co-localization of AMPA receptors on the postsynaptic membrane, effectively increasing the likelihood of glutamate binding. Scaffolding proteins prevent diffusion of transmembrane AMPA receptors out of the postsynaptic density, which allows the postsynaptic neuron to remain sensitive to presynaptic inputs.

Which of the following accurately describe(s) a function or characteristic of chromatography? I. Size-exclusion chromatography can be used to purify proteins. II. Thin-layer chromatography is effective at separating nonpolar compounds. III. Affinity chromatography sometimes contains a stationary phase coated in immunoglobulins.

I and III only Size-exclusion chromatography is commonly used to purify proteins based on size, making I correct. Affinity chromatography is a highly specific form of purification in which the stationary phase is known to bind to the protein of interest. A specific type of this procedure is immunoaffinity chromatography, which utilizes antibodies (or immunoglobulins) within the column.

Maxine is researching ways to reduce the activity of an enzyme implicated in a number of diseases. She is attempting to engineer an antagonist molecule that will competitively inhibit this disease-causing catalyst. To do so, Maxine could create a molecule that most closely resembles which of the following structures? I. The substrate of the enzyme-catalyzed reaction II. The transition state of the enzyme-catalyzed reaction III. The product of the enzyme-catalyzed reaction IV. The enzyme itself

I and II only Competitive inhibitors block the substrate from binding at the enzyme's active site. This can be done by either simulating the substrate itself or by simulating the transition state, because enzymes have a particularly strong affinity for the transition states of their associated reaction. This allows them to increase the rate of reaction by stabilizing the transition state and reducing the activation energy.

A scientist exposes a yeast cell to a chemical that inhibits the synthesis of flippase proteins. He then uses 14C to track the eventual locations of newly synthesized lipids via radiolabeling. After the administration of the aforementioned inhibitor, where would the new, radioactive lipids be found? I. In the nuclear membrane II. In the cytosolic side of the cell membrane III. In the extracellular side of the cell membrane

I and II only Newly synthesized lipids are originally inserted into the cytosolic leaflet of the endoplasmic reticular membrane. The membrane of the ER is continuous with the nuclear membrane; therefore, according to the fluid mosaic model, lipid molecules are able to freely move between structures, (I). Normally, synthesized lipids are transported from the ER to the cell membrane via vesicular transport. Here, lipids in the cytosolic leaflet of the ER membrane are made to comprise the cytosolic layer of the membrane of a vesicle, and later join the cytosol-facing side of the cell membrane when the vesicle merges with this structure (II).

Which of these listed properties of SDS make(s) it a useful reagent for gel electrophoresis? I. SDS confers a uniform charge density to all proteins in the sample. II. SDS is amphipathic, allowing it to denature most globular proteins. III. SDS contains a reductive thiol group, allowing it to break disulfide bonds.

I and II only SDS (sodium dodecyl sulfate) is a charged molecule with a long hydrophobic tail, which allows it to coat proteins with a uniform charge density. This prevents deviations that would otherwise result from the presence of charged amino acid residues, thus allowing proteins to be separated according to size alone (I). SDS is also a denaturant, which prevents proteins from running through the gel according to their cross-sectional profiles rather than their masses (II).

Tert-butanol is a common laboratory reagent that spontaneously degrades in solution to yield 2-methylpropene. To prevent this reaction and prolong the shelf life of tert-butanol, which of the following measures would be effective? I. Adding a small amount of pyridine to the storage vessel II. Storing the tert-butanol in a cold place III. Diluting the tert-butanol with a small amount of ethanol

I and II only The degradation described in the question stem is an elimination reaction. Since the relevant carbon is tertiary, this reaction must proceed via a carbocation intermediate. However, for this to occur, the -OH substituent must act as a leaving group, a process best facilitated by protonation to form water. The addition of pyridine (a weak base) will decrease the chance of this protonation, inhibiting carbocation formation (I). Note also that elimination reactions are favored by heat, so chilling the solution will also slow the reaction (II).

Of these pairings, which accurately group(s) an organelle with one of its functions? I. Rough ER - synthesis of transmembrane proteins II. Nucleolus - rRNA production and ribosome assembly III. Peroxisome - breakdown of proteins

I and II only The rough ER is dotted with bound ribosomes that serve to produce a wide variety of proteins. These products include transmembrane proteins (I). Additionally, the nucleolus is vital for the synthesis of ribosomes from protein and rRNA (II).

Which of the following statements is true? I. Acetals cannot form under basic conditions. II. Hemiacetals and hemiketals exist in equilibrium with aldehydes and ketones. III. In multistep reactions, acetals and ketals often serve as protecting groups for vulnerable esters. IV. Ketals contain two -OR groups, while acetals contain only one.

I and II only Under basic conditions, only hemiacetals can be formed. Acetals, which result from the reaction of a hemiacetal with an alcohol, require acidic conditions (I). Hemiacetals and hemiketals rapidly interconvert into aldehydes and ketones respectively (II).

A researcher trains rats to exhibit a variety of behaviors. In one experiment, the researcher plays a high-pitched sound, then presses a button to make the floor of the cage uncomfortably cold. If the rat pulls a lever within three seconds of hearing the sound, the researcher does not press the button and the cage remains room temperature. After just a few trials, all of the rats learn to pull the lever. Which of the following concepts are involved here? I. Negative reinforcement II. Positive punishment III. Avoidance learning IV. Escape learning

I and III A is correct. The researcher wants to increase the frequency of lever-pulling, meaning that he must use reinforcement, not punishment. This scenario involves negative reinforcement because something aversive is being removed. Finally, while both avoidance and escape learning are types of negative reinforcement, avoidance is relevant in this case because the rats have the potential to avoid the cold temperature entirely. Escape learning, in contrast, would not involve a warning sound, requiring the aversive condition to be present at least briefly before the rats act to end it.

Helen is a fifteen-year-old girl. Recently, Helen has been experimenting with alternative clothes and hairstyles to express her unique personality - she even dyed her hair pink! Although her parents are not happy with all of her changes, they understand that she is experimenting with her self-presentation. What are potential motives for Helen's behavior? I. An instrumental motive: Helen desires to increase her self-esteem by feeling more confident about her appearance. II. An expressive motive: Helen hopes to gain acceptance and respect from her friends with her new edgy look. III. An expressive motive: Helen wants to redefine herself and express her unique personality.

I and III only Helen's changes exemplify impression management, or an effort to change or influence other people's perceptions of her. Impression management can be explained by two main sets of motives: instrumental and expressive. Instrumental motives are drives or strong desires for increased self-esteem, which may come in the form of relationships or rewards. Helen wishes to feel more confident after adopting her new look (I). In contrast, expressive motives are displayed when an individual wants to take charge of his or her identity. In this case, Helen certainly seems like she desires to express her individualism and uniqueness (III).

Properties of transition metals include: I. the presence of multiple stable oxidation states. II. the tendency to form brightly-colored gases. III. the ability to exist in many paramagnetic compounds due to unpaired d-shell electrons.

I and III only Transition metals generally possess multiple distinct oxidation states. (As an example, consider manganese, which has oxidation numbers that range from -3 to +7) (I). Additionally, such metals are present in the d block of the periodic table, meaning that many of them do have unpaired d-shell electrons. As such, they can exist in paramagnetic states, thus III is correct (III).

Which of these statements accurately describe(s) the behavior of photoelectrons? I. A photoelectron's energy cannot exceed that of the incident photon. II. A photoelectron's energy may equal the energy of the incident photon. III. The threshold frequency is proportional to the likelihood that an electron will be ejected. IV. The intensity of incident light is proportional to the energy of the ejected photoelectrons.

I only A is correct. Due to conservation of energy, a photoelectron's energy cannot be greater than that of the incident photon.

Oxytocin (Pitocin) is routinely administered to pregnant women during difficult labor. However, it can only be given when contractions have progressed to a certain point, or it will be ineffective. Why is this true? I. Oxytocin is normally not the initial stimulus for uterine contractions. II. Exogenous oxytocin reduces production of oxytocin by the pituitary due to negative feedback. III. Oxytocin and progesterone compete for the same receptor binding site.

I only Oxytocin perpetuates labor by increasing uterine elasticity, which leads to stimulation of uterine stretch receptors. These positively feed back to the pituitary to increase oxytocin production. However, increasing uterine elasticity has no effect if the uterus is not already in the process of contracting. Interestingly, the initial signal for the induction of labor is still unknown.

Vaccines routinely fail to provide lasting immunity because the antigen is rapidly cleared by local macrophages and neutrophils long before antigen-presenting cells can display it to the adaptive immune system. The effectiveness of a novel vaccine could be enhanced by: I. co-administration of alum, a substance that triggers production of inflammatory cytokines. II. co-administration of a live flu virus. III. co-administration of soluble immunoglobulin specific for the vaccine antigen.

I only The inflammatory process involves the recruitment of immune cells, including antigen-presenting cells, to the affected area. This maximizes the chance of the antigen being picked up and taken to a nearby lymph node (I).

Which of these choices correctly identifies the function of a specific antibody? I. IgA - present in mucosal areas to prevent colonization by pathogens II. IgE - binds to allergens, causing the release of histamine III. IgM - part of the early humoral response, secreted as a pentamer IV. IgD - responsible for the majority of the humoral response

I, II, and III C is correct. IgA is an antibody found in the mucous membranes that helps prevent initial colonization by pathogens. IgE is primarily involved in the response to allergens, while IgM is responsible for the initial humoral response.

A researcher is investigating Compound X. She hypothesizes that it is an acetylcholinesterase inhibitor (AChEI). What possible effects can she reasonably expect Compound X to promote, if administered to human patients in a future clinical trial? I. Decreased heart rate II. Increased sympathetic nervous system activity at preganglionic nicotinic receptors III. Hypotension

I, II, and III D is correct. Acetylcholinesterase is an enzyme that degrades ACh, so its inhibition would magnify the effects of the neurotransmitter.

Of the following designations for the organization of the developing brain, which are labeled appropriately? I. Prosencephalon - limbic system, cerebral cortex, hypothalamus, thalamus II. Mesencephalon - inferior and superior colliculus III. Rhombencephalon - medulla oblongata, reticular formation, cerebellum

I, II, and III D is correct. The three major subdivisions of the embryonic brain are the prosencephalon (forebrain), mesencephalon (midbrain), and rhombencephalon (hindbrain). They give rise to the structures that are listed in the Roman numerals above.

HIV infects and kills helper T cells. Which of the following cell types will function less effectively in a patient in the late stages of AIDS? I. Macrophages II. B cells III. Cytotoxic T cells

I, II, and III Helper T cells are critical for activation of the humoral immune response (B cells) and the cellular immune response (T cells). Helper T cells also produce cytokines that fully activate macrophages, allowing them to establish a more acidic phagosomal pH and kill pathogens more effectively. In other words, loss of helper T cells compromises the entire immune response.

The following figure shows the pattern of hydrogen bonding found in the major and minor grooves of DNA. Restriction enzymes interact with the intact DNA double helix. Which of the following choices accurately explain(s) why these proteins bind to the major groove? I. The major groove offers more space for the enzymes to access the sides of the bases. II. In the minor groove, the enzymes would be unable to tell the difference between an A-T and a T-A pair. III. The enzymes cannot bind to phosphate groups in the DNA backbone in a sequence-specific way.

I, II, and III Proteins that interact with the intact DNA structure bind to the major groove for two main reasons: first, it offers more space for access to the bases (I), and second, it allow them to establish a more unambiguous pattern of H-bonding for proper sequence recognition (II and III).

A researcher attempts to maximize the yield of the following reaction: A (g) + B (g) → C (s) + D (g) (ΔH = -92 kJ/mol) Which of these changes will help accomplish his goal? I. Increasing the pressure II. Decreasing the temperature III. Removing product D as it forms IV. Lowering the concentration of reactant A

I, II, and III only An increase in pressure will shift the reaction to the right; this will decrease the number of moles of gas, counteracting the higher pressure (I). Note that the reaction is exothermic, meaning that heat can be written as a product. A reduction in temperature will provoke the reaction to counteract this change by shifting to the product side (II). Finally, removing product as it forms is a classic way to increase yield, as it will disrupt equilibrium and shift it to re-form the product molecules (III).

The activity of a Class I transposon, also known as a retrotransposon, involves: I. a conversion of DNA to RNA. II. a conversion of RNA to DNA. III. the production of an additional copy of the transposon. IV. an end result of the same total number of transposons.

I, II, and III only Class I transposons are known as "copy-and-paste" transposons, as they involve the creation of a new copy of the transposable element. These elements first undergo transcription into RNA using RNA polymerase (I); as their name implies, they are then reverse transcribed back into DNA and placed in a distinct location elsewhere in the genome (II and III).

You desire to measure the amount of antigen in a blood sample using a western blot. Which of these reagents or molecules will be necessary? I. Sodium dodecyl sulfate II. A reducing agent III. A detection antibody IV. An agarose gel

I, II, and III only In a western blotting procedure, the proteins in the original mixture are first separated on an SDS-PAGE gel. Typically, in addition to SDS, a reducing agent is used to break disulfide bonds that may be present between cysteine residues. Next, antibodies are used to detect particular species.

Tumor necrosis factor alpha (TNFα) is a cytokine produced by activated macrophages, the receptor for which is normally expressed by nearly all somatic cells. Late-stage cancers frequently present with cachexia, a wasting disorder resulting from chronic elevation of TNFα. Which of the following mutations would one expect to find in metastatic cancer cells? I. Decreased expression of TNFα receptors II. A gain-of-function that enables production of NF-κB, a protein shown to inhibit neuronal TNFα-mediated apoptosis III. A gain-of-function that enables production of TNFα IV. Increased expression of VEGF, a transcription factor that is vital for angiogenesis

I, II, and IV C is correct. We are told that TNFα is produced by macrophages; these cells become activated during robust immune activity, which enhances their phagocytic functions. This information suggests that TNFα participates in such a heightened immune response to fight cancerous cells. However, the malignant cells are likely to adapt to circumvent this attack. Reduction in the number of available TNFα binding sites by elimination of receptors would reduce the cytokine's effect and minimize the damage the immune response has on the cancer. If TNFα normally induces apoptosis, production of an apoptotic inhibitor would also decrease its effects and permit cancer survival. Finally, malignancies require increased blood supply to support their uncontrolled growth, and the question asks specifically about metastatic cancer, which grows rapidly.

Latrotoxin (LTX), a large globular protein produced by Latrodectus spiders, acts to perforate the presynaptic membrane at the axon terminal. The resulting channel is large enough to permit the free influx of calcium, as well as the passage of water and other small molecules. A Latrodectus bite would most likely result in: I. swelling of the axon terminal. II. fusion of docked vesicles with the plasma membrane. III. widespread release of glutamate within the CNS. IV. widespread release of acetylcholine within the PNS.

I, II, and IV only Swelling will result from the movement of water down its concentration gradient and into the neuron, which contains many large proteins and other solutes that contribute to its hypertonicity (I). Fusion of presynaptic vesicles is mediated by an increase in intracellular calcium, which the question mentions as a consequence of LTX action (II). Finally, in the peripheral nervous system, LTX will cause the widespread release of the principal motor neurotransmitter, acetylcholine (IV).

Timothy, the kicker on a local football team, wants the ball to stay in the air for a longer time after he kicks it. Right now, he tends to kick at a 30° angle from the ground and an 8 m/s total velocity. What change(s) can Timothy make to increase the "time of flight" of his kicks? I. Increasing the total velocity to 10 m/s while keeping all other factors constant II. Increasing the horizontal velocity of the ball while keeping all other factors constant III. Changing the angle to 45° with respect to the ground IV. Changing the angle to 90° with respect to the ground and lowering the velocity to 6 m/s

I, III, IV Time in flight is determined by vertical, not horizontal, components. Increasing the total velocity while leaving the angle unchanged must also increase the vertical velocity (and thus the time). For III and IV, let's do some math. Originally, Timothy was kicking at a 30° angle, meaning that the vertical component of that velocity was (8 m/s)(sin 30), or 4 m/s. With this information, we can use vf = vi + at to find the time of flight from the initial kick to the peak of the projectile arc. 0 m/s = 4 m/s + (10 m/s2)(t) yields a time of 0.4 s. Remember, though, that the total time of flight is twice this value, or 0.8 s. When Timothy changes his angle to 45°, the vertical component of that value becomes (8 m/s)(sin 45), or about 5.6 m/s; this corresponds to a total flight time of 1.1 s. When he makes the angle 90°, the new velocity of 6 m/s is entirely vertical. 0 m/s = 6 m/s + (10 m/s2)(t) yields a initial-to-peak time of 0.6 s, or a total time in flight of 1.2 s. D.

In a solution of DMSO, which of the following would act as the best leaving group?

I- DMSO is a polar, aprotic solvent. In such a solvent, nucleophilicity decreases as one moves down the periodic table. I- would thus be the least nucleophilic (least reactive) and the best leaving group.

Dyneins, motor proteins that are associated with microtubules, play a vital role in the transport of cellular components. How can the intracellular movement of dyneins be described? I. Dyneins travel toward the center of the cell. II. Dyneins move toward the minus ends of their associated microtubules. III. Dyneins are involved in retrograde transport.

I. Dyneins travel toward the center of the cell. II. Dyneins move toward the minus ends of their associated microtubules. III. Dyneins are involved in retrograde transport.

A student is studying the following chemical equation: MgF2 (aq) + Sr(NO3)2 (aq) → SrF2 (aq) + Mg(NO3)2 (aq) Which statements accurately describe this process? I. For every molecule of magnesium fluoride that reacts, one molecule of magnesium nitrate forms. II. For every atom of strontium that reacts, one molecule of magnesium nitrate forms. III. For every two moles of magnesium that react, one mole of magnesium nitrate forms. IV. For every two moles of fluorine that react, six moles of oxygen must be consumed as well.

I. For every molecule of magnesium fluoride that reacts, one molecule of magnesium nitrate forms. II. For every atom of strontium that reacts, one molecule of magnesium nitrate forms. IV. For every two moles of fluorine that react, six moles of oxygen must be consumed as well.

Which of these are domains of intelligence as defined by Gardner's theory of multiple intelligences? I. Musical intelligence II. Interpersonal intelligence III. Logical-mathematical intelligence IV. Fluid intelligence

I. Musical intelligence II. Interpersonal intelligence III. Logical-mathematical intelligence

Which of these statements accurately identify a function of transmembrane proteins? I. They act as receptors for hormones and initiate signal transduction pathways. II. They allow for transport of charged molecules across the cell membrane. III. They are responsible for the production of the majority of the ATP synthesized in eukaryotic cells.

I. They act as receptors for hormones and initiate signal transduction pathways. II. They allow for transport of charged molecules across the cell membrane. III. They are responsible for the production of the majority of the ATP synthesized in eukaryotic cells.

Which of the following functional groups is NOT a carboxylic acid derivative? I. An amide II. An ether III. A ketone IV. An acyl halide

II and III only A carboxylic acid derivative is a molecule that can be synthesized from a carboxylic acid via nucleophilic acyl substitution. Alternately, such a compound is one that contains a carbonyl group immediately adjacent to a heteroatom, generally one that is electronegative. Neither ketones nor ethers fit these requirements (II and III). (Note: do not confuse ethers with esters, which are classic examples of such derivatives!)

The enzyme adenylate kinase catalyzes the following reaction: ADP + ADP ↔ ATP + AMP The reverse process predominates when the cell is using ATP at a low rate. Which of these species likely downregulate(s) the activity of one or more glycolytic enzymes? I. AMP II. ATP III. ADP

II and III only Since we know that low use of ATP causes the reverse reaction to predominate, we can expect high concentrations of ADP to be found under these low-energy-use conditions. In contrast, more ATP and AMP should be present when the cell is in an energetically demanding state. However, ATP will then be used rapidly, leaving high concentrations of intracellular AMP and low concentrations of ADP. Since the question asks about the downregulation of glycolysis, and since this information alone implies that high [ADP] correlates with low energy demand, we can assume that the presence of ADP inhibits at least one glycolytic enzyme. Additionally, we know (or can easily assume) that ATP limits its own production in a classic negative feedback mechanism. After all, when excess energy is already present, there is no need to synthesize more.

Which types of muscle can exhibit myogenic activity? I. Skeletal muscle II. Smooth muscle III. Cardiac muscle

II and III only Smooth and cardiac muscle are myogenic and can be modulated by the autonomic nervous system (II and III).

Which of these statement(s) is / are correct? I. Prokaryotes, such as S. aureus, translate DNA into RNA in the cytosol. II. In certain viruses with RNA genomes, RNA may be converted to DNA via reverse transcription. III. In human cells, a large amount of DNA is never converted into protein form.

II and III only The central dogma states that DNA is used to form RNA via transcription, and that RNA is then converted into protein via translation. However, retroviruses (described in statement II) form an exception to this rule. These viruses are able to convert their RNA genomes into DNA through use of a particular enzyme, reverse transcriptase. Finally, statement III is accurate as well, as much of human DNA is spliced out after transcription in the form of introns.

A ray of white light moves through the air and strikes the surface of water in a beaker. The index of refraction of the water is 1.33 and the angle of incidence is 30º. All of the following are true EXCEPT: I. the angle of reflection is 30º. II. the angle of refraction is 30º. III. total internal reflection will result, depending on the critical angle.

II and III only The question asks "all of the following are true EXCEPT," indicating that we need to find the false statements - or, alternatively, that we need to eliminate the true ones. The question ends up being much easier than we expect. The angle of incidence always equals the angle of reflection (I). This immediately lets us narrow it down to choice C. Light entering a more dense medium will bend towards the normal. With an angle of incidence of 30º, the angle of refraction must be less than 30º (II). Total internal reflection can only result when a ray of light begins in a higher-index material and reaches a boundary with a lower-index one (e.g. starting in water and moving towards air). Here, the light ray started in air (n = 1) and moved into water (n ∼ 1.3), making total internal reflection impossible (III).

An individual who is a member of a higher socioeconomic class is more likely to: I. have weak ties and a smaller network of connections. II. have strong ties and a smaller network of connections. III. have less social capital, since much has been spent on attaining his or her current high status. IV. have more social capital, since more connected, wealthier people tend to wield more influence.

II and IV D is correct. People of higher socioeconomic status are more likely to form stronger ties with a smaller total number of individuals. The stronger the bonds and the smaller the group, the more personal influence they can exert over other people's opinions and behavior; consequently, this makes them better able to use that influence to benefit others as well as themselves. In addition, they will be likely to have more social capital, or networking capacity that can be used to obtain benefits.

Of these statements regarding demographic shifts, which are most likely true of a poor, undeveloped country that is very racially and culturally homogenous? I. It will have a higher rate of immigration than emigration. II. It will have a higher rate of emigration than immigration. III. It will exhibit a high fertility rate.IV. The approximate magnitude of its fertility rate cannot be determined without more data.

II and IV only A poor, undeveloped country is likely to display a high emigration rate, as many of its citizens seek employment and safety elsewhere (II). Fertility rate cannot be discerned simply from the economic success of a country; we need at least some population data (IV).

Which of these statements accurately describe the behavior of photoelectrons? I. A photoelectron's energy can exceed that of the incident photon. II. A photoelectron's energy cannot equal the energy of the incident photon. III. The threshold frequency and the likelihood of electron ejection are proportional. IV. The intensity of incident light is unrelated to the energy of an ejected photoelectron.

II and IV only Due to conservation of energy, the energy of a photoelectron may only be less than that of the incident photon. This concept is due to the existence of the work function, a quantity of energy required for an electron to be released at all. Since some of the incident photon's energy is devoted to overcoming this threshold, not all of it can be converted to kinetic energy for the ejected electron (II). Remember that the intensity of the incident ray determines the number, not the individual energies, of the ejected photoelectrons (IV).

Which of the following statements are true regarding spermatogenesis? I. Meiosis I marks the transition of a spermatogonium into a primary spermatocyte. II. In a secondary spermatocyte, sister chromatids are still paired in the same cell. III. Immediately before the second meiotic division, cells are diploid but chromosomes lack replicated copies.

II only Spermatogonia give rise to primary spermatocytes before meiosis even begins. Meiosis I, then, marks the division of a primary spermatocyte into two secondary daughter cells. At the end of meiosis I, which is also known as reductional division, cells are already haploid; however, chromosomes retain their identical copies in the form of attached sister chromatids (II).

Which of the following statements accurately characterize glycogen? I. It represents a glucose polymer that is stored in order to increase intracellular osmotic pressure. II. Entropy decreases when glucose is stored as glycogen, a process that is entropically unfavorable. III. Glycogen is highly branched to maximize the number of exposed non-reducing sugars. IV. The branching of glycogen increases its solubility in water.

II, III, and IV C is correct. These three statements are true. The branching of glycogen increases its solubility because it arranges its many hydroxyl groups in such a way that surrounding water molecules can easily hydrate them. Additionally, the enzymes used in glycogen degradation can only work on the non-reducing end of glucose, so by creating a branched structure, more of these active ends are exposed for reaction.

A student mixes 10 mL of 0.2 M potassium bromide with 30 mL of 0.15 M lead nitrate. The equation involved in this process is as follows: KBr (aq) + Pb(NO3)2 (aq) 🡪 PbBr2 (s) + KNO3 (aq) Which statements describing the process are true? I. For every molecule of potassium bromide consumed, two molecules of potassium nitrate are formed. II. For every molecule of lead nitrate consumed, two molecules of potassium nitrate are formed. III. For every two molecules of potassium bromide consumed, one molecule of lead bromide is formed. IV. For every atom of bromine that reacts, three oxygen atoms must be consumed as well.

II, III, and IV D is correct. This reaction must be balanced before any assessments can be made, giving us 2KBr (aq) + Pb(NO3)2 (aq) 🡪 PbBr2 (s) + 2KNO3 (aq). Since the molar ratio of lead nitrate to potassium nitrate is 1:2, statement II is accurate. There also is a 2:1 molar ratio of potassium bromide to lead bromide, making III true as well. Finally, statement IV is true because the reactants contain two bromine atoms for every six oxygen atoms.

Sphingosine is an important signaling lipid that can induce apoptosis in human monoblastic leukemia cells. While the mechanism has yet to be fully elucidated, elevated sphingosine has been shown to alter localization and expression of a number of intracellular proteins. Which of these statements, if true, would support the hypothesis that sphingosine acts within the cell as a proapoptotic signal? I. Ceramide synthase inhibitors, which reduce the synthesis of a sphingosine precursor, do not rescue T cells from apoptosis. II. Sphingosine reduces the cellular activity of Akt, a kinase activated by extracellular insulin binding. III. Sphingosine inhibits the expression of Bcl-2, a protein that is overexpressed in B-cell lymphoma cells. IV. Cytosolic 14-3-3 proteins favor a phosphorylated state in the presence of sphingosine, which prevents them from binding proapoptotic transcription factors.

II, III, and IV only Insulin is a known growth signal and activator of cellular metabolism. Thus, downstream inhibition of an insulin-mediated pathway would be consistent with promotion of cellular metabolic arrest and apoptosis (II). Bcl-2 is an oncoprotein, which usually means that it is functions in the progression of the cell cycle or the inhibition of apoptosis. If sphingosine inhibits the activity of such a protein, it is likely to be proapoptotic (III). Finally, since transcription factors must enter the nucleus to exert their effects, sequestration in the cytosol constitutes inhibition of such proteins. Therefore, if sphingosine promotes the phosphorylation of 14-3-3 proteins and prevents them from binding proapoptotic transcription factors in the cytosol, it is thereby permitting the translocation of these factors into the nucleus. This is a proapoptotic function (IV).

Esmeralda G., a patient in a case study, has significant trouble locating the sources of sounds. Additionally, she cannot focus her vision on a specific point in space while rotating her head. Which of these statements is likely true of this patient? I. Her medial geniculate nucleus is damaged, promoting her difficulty with localizing sounds. II. Her superior colliculus is damaged, since she cannot fixate on a point when rotating her head. III. Her superior olive is damaged, leading to her inability to detect the locations of sounds. IV. Her semicircular canals are damaged, creating the issue with sound localization.

II. Her superior colliculus is damaged, since she cannot fixate on a point when rotating her head. III. Her superior olive is damaged, leading to her inability to detect the locations of sounds.

Which of these pairings do(es) NOT correctly match a disorder with its typical symptoms? I. Bipolar I disorder - manic episodes that cycle with periods of major depression II. Panic disorder - sudden episodes of disabling anxiety that can cause shortness of breath, rapid heartbeat, and sweating III. Acute stress disorder - generalized anxiety or distress in response to daily life activities

III only Acute stress disorder can be conceptualized as a mild form of PTSD. As such, it typically stems from a specific mentally traumatic or fear-inducing event. This condition is characterized by flashbacks or emotional numbness with an onset within four weeks of the event in question. The description given in option III is a better fit for generalized anxiety disorder.

Which of these particles will experience a force when subjected to a 6.5 T magnetic field? I. A neutron traveling at 6 × 102 m/s II. A stationary proton III. An electron moving at 4 × 104 m/s perpendicular to the field

III only The magnetic force experienced by a particle in a magnetic field is given by F = qvB, where q denotes charge, v stands for velocity, and B gives the strength of the field. From this relationship, it is evident that a particle must possess both velocity and a charge (whether positive or negative) to be affected by a magnetic field. Only choice III includes both of these criteria.

Which of the following amino acid sequences would incur the greatest entropic penalty if it were used to replace Tyr-Cys-Met in the surface region of a protein?

Leu-Val-Phe C is correct. Entropic penalties are incurred when amino acids are exposed to an environment in which they are poorly soluble. In this case, the surface of a protein is likely to face an aqueous environment, so hydrophilic residues are favored and hydrophobic residues incur a penalty. Leucine, valine and phenylalanine are all hydrophobic amino acids.

Alfonso, a 15-year-old, has always been a good student and son. Lately, however, he has been exploring his identity and engaging in dangerous activities such as dirt biking. He even dyed his hair green! Which psychosocial crisis is Alfonso experiencing, and how can the resolution of this crisis contribute to the formation of his identity?

Identity versus role confusion; resolution of this crisis indicates that Alfonso has come closer to finding his unique identity. A is correct. Alfonso's behavior fits neatly into Erik Erikson's theory of adolescent development, which highlights "identity vs. role confusion" as a major conflict. Erikson believed that, during this stage, the adolescent will try different behaviors to either solidify or resolve uncertainty about his identity. The variation in Alfonso's behavior, such as his rebellious act of hair-dying, is his way of experimenting with himself. He is accepting or rejecting certain goals and values to become a unique individual.

Which of the following statements about boiling point is accurate?

If the vapor pressure of water is 38 mmHg in a near-vacuum environment with a Patm of 0.05 atm, it will boil. A liquid will boil when its vapor pressure equals the atmospheric pressure. Since one atmosphere is equal to 760 mmHg, 0.05 atm is equivalent to 38 mmHg.

A proton and another proton lay along a horizontal line. Which most accurately describes where the magnitude of the electric field will be nearly equal to zero?

In the middle of the horizontal line Electric field sum as vectors. Field lines point outwards from positive charges and in towards negative charges. At the point exactly midway between two positive charges, the vectors will be equal in magnitude and opposite in direction. Thus they will add up to zero.

Unlike humans, birds use a ZW sex-determination system in which males have two Z chromosomes and females have one Z and one W chromosome. What is the most likely regulatory mechanism for the potential imbalance in gene expression between male and female birds?

Inactivation of one Z chromosome in males In humans, females have two of the same sex chromosome (XX), while males have only one X and one Y. However, the X chromosome is much larger and carries significantly more genes than the Y version. To balance their genetic load with that of males, one of each female's X chromosomes is inactivated. As far as we know, birds likely use the same mechanism but for the opposite gender. The male, who carries two Z chromosomes, must then inactivate one to balance his female ZW counterpart.

According to Le Châtelier's principle, which glycolytic disturbances would force the metabolic system to reestablish equilibrium? I. Increased concentrations of glucose II. Decreased concentrations of pyruvate III. Increased production of hexokinase IV. Decreased availability of ATP

Increased concentrations of glucose, Decreased concentrations of pyruvate, and Decreased availability of ATP

Jessie's high school physics class is running a "potato cannon" competition. The goal is simple: shoot a potato the greatest possible horizontal distance. Right now, Jessie's cannon shoots potatoes at a 30° angle from the ground with a total velocity of 14 m/s. What change(s) can Jessie make to increase the distance her potatoes travel? I. Increasing the total velocity to 18 m/s while keeping all other factors constant II. Decreasing the masses of the potatoes to make them fall more slowly III. Changing the angle to 45° with respect to the ground IV. Changing the angle to 90° with respect to the ground Note that cos 30 = 0.87, sin 30 = 0.5, cos 45 = 0.71, and sin 45 = 0.71.

Increasing the total velocity to 18 m/s while keeping all other factors constant and Changing the angle to 45° with respect to the ground

The potential role of lipids in intracellular signaling has only recently become apparent; since most lipids are confined to a specific membrane, they were originally deemed unlikely signaling candidates. One lipid of considerable interest is ceramide, due in part to its proposed role in relaying apoptotic signals. Which statement, if true, would provide the strongest evidence AGAINST the hypothesis that ceramide acts as a proapoptotic signaling molecule?

Induced expression of Bax, a proapoptotic protein with a ceramide-binding domain, does not promote apoptosis in cells treated with a known ceramidase inhibitor. . A is correct. From the term "ceramidase," we can assume that this is the enzyme responsible for the breakdown of ceramide. Thus, cells treated with an inhibitor of this enzyme should display elevated ceramide levels. Concurrently, they have been induced to produce excess Bax, a known proapoptotic protein that apparently interacts with ceramide. If an increase in both ceramide and an apoptosis-inducing protein does not promote apoptosis, this strongly suggests against the hypothesis mentioned in the question stem. In fact, it seems more likely that ceramide inhibits apoptosis, possibly by binding to Bax through its aforementioned ceramide-binding domain.

The interaction between statins and grapefruit juice has long been an object of research, as certain components in grapefruit juice appear to inhibit cytochromes in the stomach. Why might this interaction be considered harmful?

Inhibition of the cytochromes reduces breakdown of the statin, leading to potential overdose. D is correct. Many oral drugs are metabolized significantly, often with only a small percentage of the drug actually reaching the bloodstream. Cytochrome P-450 is the enzyme responsible for metabolizing most types of statin. Irreversible inhibition by bergamottin, a compound found in grapefruit juice, results in less statin being metabolized than usual. This can lead to potential overdose due to increased bioavailability.

At 0.5 dB, a sound does not even stir the hair cells in an animal's inner ear. However, the sound intensifies, reaching 2 dB, at which point the associated signal is sent to the animal's central nervous system. Finally, the sound further loudens to 130 dB, causing the animal discomfort before gradually returning to 0.5 dB. The point at which the animal consciously perceives the sound is:

It cannot be determined from the given information. D is correct. Even though a signal is sent to the animal's central nervous system at 2 dB, we cannot be certain if it noticed the sound. The threshold must fall somewhere between 2 and 3 dB.

Two soccer players are looking at an old soccer ball with the classic black-and-white pattern. One of the players tells the other that the reason we see a ball (rather than just black objects in space) relates to the Gestalt principle of similarity. What is incorrect about his statement?

It is the principle of closure, not similarity, that explains this observation. The Gestalt principle of closure states that when a space is enclosed by an incomplete contour or line, it will generally be perceived as a closed figure. Here, the contours of the black squares are perceived as a round ball rather than as individual objects.

When the oocyte is released from the follicle, it is surrounded by a layer of maternal follicular cells known as the cumulus oophorus. Which of the choices below is a plausible function of the cumulus oophorus?

It produces vital amino acids and other molecules to support the fragile oocyte. This is the true function of the cumulus oophorus. Since metabolic processes generate free radicals and other toxic byproducts, the oocyte must remain as metabolically inactive as possible to protect its haploid genome. The cells described conduct most necessary metabolic processes and export the products to the oocyte.

An ice cube is floating partially submerged in a glass of water. What happens to the height of the water in the glass as the ice cube melts?

It stays the same. Archimedes' principle states that objects displace their weight in water. As long as objects are floating they displace enough water to support their mass. By turning from solid to liquid, the mass of water does not change so the cube will keep displacing the same amount of water. In other words, any volume displacement lost due to the cube's melting is offset by volume addition due to the meltwater. The volume of liquid water displaced before melting occurs has been replaced by the same volume of liquid water by the time all melting has occurred. Likewise, as the cube melts, displacement reduction is perfectly offset by the volume of new meltwater. Therefore, the height of the water in the glass will not change as the ice melts.

GABA is an inhibitory neurotransmitter that affects Cl- ion channels. When stimulated by GABA, what will most likely happen to a neuron's cell potential?

It will be hyperpolarized to -80 mV. D is correct. The typical neuron's resting potential is -70 mV. If the cell were depolarized, it could easily rise to the threshold level (approximately -55 mV), and an action potential would be triggered. Therefore, an inhibitory neuron must instead hyperpolarize its target neuron to prevent this response. GABA accomplishes this by opening Cl- channels, which lowers the resting potential (say, to -80 mV) and makes it more difficult for the excitatory entrance of Na+ ions to raise the cell potential to threshold level.

A circular segment of gold wire is positioned in an external magnetic field of strength B that points out of the page. If the strength of the field is increased to 2B, what will happen to the wire?A circular segment of gold wire is positioned in an external magnetic field of strength B that points out of the page. If the strength of the field is increased to 2B, what will happen to the wire?

It will experience a clockwise current. This scenario relates to electromagnetic induction and to Lenz's law. This law, which brings to mind conservation of energy and even Le Châtelier's principle, states that a change to a magnetic field will always generate a current that counteracts that change. Here, the external field is becoming stronger, or more heavily positioned out of the page. To resist this change, it will induce a current that promotes a field pointing the opposite direction, into the page. The right-hand rule tells us that this current must travel clockwise.

Of these statements about demographic shifts, which are most likely true of a wealthy, economically developed, and racially and ethnically diverse nation? I. It will have a higher rate of immigration than emigration. II. It will have a higher rate of emigration than immigration. III. It will exhibit a high fertility rate. IV. The approximate magnitude of its fertility rate cannot be determined without more information.

It will have a higher rate of immigration than emigration and The approximate magnitude of its fertility rate cannot be determined without more information.

By what factor does the period of oscillation of a pendulum change when its length is extended from 1 cm to 9 cm?

Its original period is multiplied by 3. C is correct. According to the equation , period will increase when a pendulum is lengthened. Specifically, since length is subject to a ninefold increase and "𝓁" is raised to the ½ power in the equation above, the period will increase by a factor of 3.

Which of these pairings correctly matches the theory of emotion with its definition?

James-Lange: physiological arousal happens first, followed by an unconscious interpretation of that experience as emotion. This description correctly fits the James-Lange theory of emotion.

A couple that has been married for forty years has spent the last decade deciding against divorce because they believe their marriage to be the admirable result of considerable effort. Which of these factors that may affect attitude change is most relevant to this example?

Justification of effort C is correct. Since so much time and effort has already been devoted to their marriage, this couple might justify continuing the relationship simply due to their high assessment of the work invested in it. This factor relates to cognitive dissonance and explains the disproportionate value that can be placed on such relationships, even when they are unhappy.

Consider the chemical reaction shown below. 2 A (aq) + B (aq) → C (s) + D2 (g) What is the correct equilibrium constant expression for the reaction?

Keq can be found by placing products over reactants, with each species raised to an exponent corresponding to its coefficient in the chemical reaction. (Note that all concentrations must be at equilibrium.) Solids and pure liquids are not included in this expression.

Which of the following proteins travel down microtubules towards the outside of the cell?

Kinesins B is correct. Microtubules originate from the MTOC, which is located close to the center of the cell. Because microtubules grow from the (+) end, that end is located towards the periphery of the cell while the (-) ends are found close to the center. Kinesins travel toward the (+) ends of microtubules, and therefore must carry cargo towards the outside of the cell.

Which of the following correctly matches a digestive enzyme with the site where it is active?

Lactase, small intestine Lactose is broken down in the small intestine.

Consider the figure below. The carbohydrate anomer shown above serves as a component of which larger sugar?

Lactose This molecule is a β anomer. Only the glucose monomer in lactose has a β acetal linkage (to galactose). Specifically, it is a β-1,4 linkage.

Vacuum distillation should be used to separate which of the following pairs of compounds?

Lauric acid (BP = 299°C) and propylene carbonate (BP = 240°C)

During the course of routine hospital blood work, a sample of whole blood is spun down in a laboratory centrifuge. The resulting centrifuge tube is shown below. If a laboratory researcher were hoping to isolate human ribosomes, which layer would be of most value?

Layer 2 Leukocytes, located in the "buffy coat" of Layer 2, are metabolically active and produce many proteins to fight pathogens and digest phagocytosed material. They would have abundant ribosomes that could be isolated for study. Erythrocytes are anucleate and therefore cannot produce RNA. These cells have no ribosomes and are unable to transcribe or translate.

Hemoglobin's oxygen binding affinity is affected by the pH of its environment and the concentration of carbon dioxide. This relationship is described as the Bohr effect. Which of these conditions would increase the affinity between hemoglobin and oxygen?

Low carbon dioxide and high pH The Bohr effect characterizes the inverse relationship between binding affinity and plasma carbon dioxide concentration. An increase in [CO2] generally correlates to an oxygen deficiency, as when an individual exercises. In such conditions, hemoglobin must be able to release oxygen when it reaches the tissues. As a result, when the concentration of carbon dioxide increases, hemoglobin decreases its affinity for oxygen. In contrast, low CO2 concentrations facilitate an increase in binding affinity. Since CO2 in the plasma directly leads to the formation of carbonic acid, low [CO2] correlates to alkaline, or basic, blood.

Reflex arcs, particularly those involving the hypothalamus and brain stem, are crucial to the maintenance of homeostasis. However, not all homeostatic processes involve input from these arcs. Which of the following processes does NOT directly involve a neural reflex arc?

Lung blood vessels with a low partial pressure of O2 constrict to divert blood to better-ventilated vessels with a high partial pressure of O2. D is correct. Dilation of lung blood vessels in response to O2 content is triggered by direct stimulation of the vasculature by O2; nerve cells are not involved.

Aspartic acid, lysine, and glutamine undergo cation-exchange chromatography. How should they be ordered from longest to shortest retention time?

Lysine, glutamine, aspartic acid Cation-exchange chromatography features a column with a negatively-charged stationary phase. This phase will interact with the positive lysine molecules, while repelling aspartic acid due to its negative carboxylate side chain. Thus, lysine will elute last, while aspartic acid will travel through the column the fastest. Glutamine, which has an uncharged polar side chain, will be neither attracted nor repelled to the stationary phase.

It is believed that a variety of factors, both genetic and environmental, interact to cause structural birth defects. These defects, such as spina bifida, oral clefts, and congenital heart defects, result from the complex influences that disrupt embryonic development. Which maternal factor(s) would increase the risk of developmental abnormalities in the embryo?

Maternal exposure to teratogens, maternal genotype, and maternal disease all are environmental and genetic factors that could predispose an embryo to developmental abnormalities. Teratogens are chemical agents that may disturb the development of an embryo, such as medications, drugs, or chemicals. More vulnerable maternal genotypes or existing maternal conditions, such as heart disease, may also increase the risk of pathological development.

A pharmaceutical intern exposes a sample of benzyl alcohol to a dilute solution of sodium methoxide in an attempt to produce the ether shown below. However, he anxiously realizes that this reaction yields almost none of the desired product. To actually form the ether shown above, which combination could be used as an alternative to the sodium methoxide?

Methyl iodide with sodium hydride Sodium hydride, while not a nucleophilic base, is extremely strong. Thus, its addition will facilitate the deprotonation of benzyl alcohol, permitting it to act as a nucleophile and attack methyl iodide. While the reaction mechanism is not analogous to the one originally intended by the intern, it will form the desired product.

An unknown element decays twice, leaving behind three final products: a β+ particle, a γ particle, and a Na23 nucleus. The element was:

Mg23. In β+ decay, also known as positron emission, a proton is converted into a neutron while a positron (or β+ particle) is emitted. Gamma decay doesn't change the original element. If both of these forms of radioactive decay occurred, our answer must be an element with one more proton and one fewer neutron than Na23.

At high temperatures, magnesium carbonate can be decomposed to produce a metal oxide used in various industrial applications. The decomposition of magnesium carbonate is given by which equation?

MgCO3 (s) --> MgO (s) + CO2 (g) When carbonate-containing compounds decompose, they form carbon dioxide and an oxide. Here, that oxide is MgO.

Valentine is an electrical engineer commissioned to designing an electrical supply system that will minimize the power lost during transmission through power lines. Which of the following design considerations is most appropriate?

Minimize current Electrical power is shown by the equations P = VI = V2/R = I2R. Since we wish to minimize power lost during transmission through the lines (i.e. the current carrying the power) we use the equation I2R which means she should minimize current through these wires. Note that this is how actual power transmission occurs over long distances — high voltage lines are used (those huge towers with 50,000 V lines) so that the electricity can be sent at very, very high voltage but relatively low current. With relatively small amounts of current flowing through the lines, less power is lost to resistance.

The endosymbiont theory states that mitochondria and chloroplasts were initially prokaryotes that were engulfed by unicellular organisms. If true, which of these statements would support this hypothesis?

Mitochondria and chloroplasts are surrounded by an additional plasma membrane. A is correct. When the mitochondria and chloroplasts were hypothetically engulfed by the larger cells, they became surrounded with an additional plasma membrane.

The formation of which of the following products serves as the main goal of the pentose phosphate pathway?

NADPH The primary function of the pentose phosphate pathway is to generate large quantities of NADPH for use in anabolic reactions. Without NADPH, numerous critical pathways would be hindered, including nucleic acid and fatty acid synthesis. This would directly impact the viability of the cell.

Julia's family suffers from MELAS, a mitochondrial disorder that affects many of the body's systems. Her father David has the disease, as do her aunt Helen and uncle Vincent. David has a severe form of the disease, while Helen and Vincent display many fewer symptoms. Aunt Helen is married with only one child, Charles. Uncle Vincent has three children from different marriages: Michael, Cindy and Sarah. Why might David suffer from a much more severe version of the disorder than either of his siblings?

Mitochondria divide randomly, so he may have inherited a larger percentage of those affected with the disease. Mitochondrial diseases are often associated with mitochondrial DNA, which is inherited through the maternal line. Thus, simply acquiring more mitochondria with the relevant mutation could result in a more severe form of the disease. Mitochondria divide and segregate to daughter cells randomly, so it is possible for some gametes to have more mutated mitochondria than others.

Which of these correctly describe the unique features of mitochondria when compared to other mammalian organelles? I. Mitochondria have their own set of genetic material. II. Mitochondria are able to self-replicate independent of the cell replication cycle. III. Mitochondria are found only in eukaryotes IV. Mitochondria are enclosed by a membrane bilayer.

Mitochondria have their own set of genetic material, Mitochondria are able to self-replicate independent of the cell replication cycle, and Mitochondria are enclosed by a membrane bilayer.

Propionic anhydride, shown below, is a molecule with a variety of synthetic applications. A team of scientists is focusing on the compound due to its role in the production of fentanyl, a powerful opioid used as an analgesic. Which of these reactions or multistep syntheses could produce the compound above?

Mix propionic acid with PCl5, then extract the product; finally, combine it with additional propionic acid and heat. C is correct. The process described here involves two steps. First, PCl5 is used to synthesize an acyl halide, a common first step in such reactions due to the high reactivity of these compounds. Next, the newly generated propionyl chloride is reacted with propionic acid, forming propionic anhydride and losing Cl- as a leaving group.

Several statisticians notice an interesting relationship between a high-carb diet and a particular visual deficiency. In all populations, consumption of large amounts of carbohydrates appears to worsen the deficiency. However, in Europe, a high-carb diet has a much stronger effect on disease outcome than in Mexico, where it only slightly appears to impair patients' vision. In this scenario, which type of variable is location?

Moderating This question is tricky! A moderating variable is one that affects the strength of a relationship between separate independent and dependent variables. Here, carbohydrate consumption (the independent variable) always seems to affect disease outcome (the dependent variable). However, this relationship may be strong or weak, depending on location. Thus, location (Europe, Mexico, etc.) moderates the relationship between the main variables in this study.

Two different compounds are allowed to travel across a plasma membrane. The graph below shows the rates of transport for these substances versus their concentrations outside the cell. What are the most likely identities of the compounds?

Molecule A is O2, while molecule B is glycine. C is correct. Molecule A is absorbed at a rate directly correlated with its concentration outside the cell, suggesting that it can easily diffuse through the membrane under any conditions. In contrast, molecule B seems to display Michaelis-Menten kinetics, with the concentration of the compound impacting rate at low, but not high, concentrations. This implies that a protein is required to transport the molecule inside the cell. Therefore, molecule A is likely small and nonpolar, while molecule B must be large, polar, or both.

A molecular biologist runs an SDS-PAGE on Protein A, using mercaptoethanol in the preparation of the protein sample. Protein A is known to possess quaternary structure. What will be the likely result after staining with Coomassie Blue?

Multiple bands that are further along the gel relative to Protein A's band in a native electrophoresis A is correct. Denaturation via SDS and mercaptoethanol effectively removes all forces that hold together secondary, tertiary, and quaternary structure. Therefore, the various subunits of Protein A would separate individually as they migrate along the gel. In a native gel, the protein is not denatured, and Protein A would migrate as a single, heavier molecule. This would result in slower travel.

Why must gluconeogenesis be localized to the kidney and liver and not other organs, such as the muscles or the heart?

Muscle cells need a continually high supply of ATP; therefore, they require a high glycolytic, not gluconeogenic, rate. D is correct. Only the liver and (to a lesser extent) the kidneys are able to carry out gluconeogenesis. Other organs, such as the muscles, are relatively unaffected by glucagon. Muscle cells must have a constant supply of ATP and are constantly undergoing glycolysis. If this were to happen at the same time as gluconeogenesis, it would create a futile cycle.

One method by which cells drive nonspontaneous reactions is to couple them to other processes that involve high-energy compounds. All of the following are species that contain high-energy bonds EXCEPT:

NAD+. ATP, creatine phosphate, and acetyl-CoA are all similar in that they contain high-energy phosphate or thioester bonds. In contrast, NAD+ is the oxidized version of NADH, an electron carrier. While it could be said that NADH possesses a high-energy bond, NAD+ itself represents the corresponding low-energy state. Alone, it is unable to provide the extremely negative ?G required to couple with a nonspontaneous process.

Which of the following molecules is a direct product of beta-oxidation?

NADH NADH is produced during beta-oxidation and is then used as an electron carrier for the electron transport chain. Note that many metabolic pathways are linked in the sense that the products of one may feed into another; beta-oxidation and the ETC are examples of this phenomenon.

Which of the following serve as electron donors or acceptors in the mitochondrial electron transport chain? I. NADH dehydrogenase II. Ubiquinone III. Cytochrome c IV. Plastocyanin

NADH dehydrogenase, Ubiquinone, and Cytochrome c

Although X-ray crystallography has proven indispensable in the characterization of biomolecules, it has a number of limitations. Chiefly, the necessary crystalline nature of the sample restricts the study of dynamic biological systems. Which of the following would be most appropriate for the analysis of a small organic ligand binding to its receptor?

NMR spectroscopy NMR spectroscopy is relatively non-invasive and is frequently used to observe chemical reactions. Since the sample remains in solution and is generally free to carry out processes as it would in vivo, the spectral changes can be observed in real time.

Copper (II) sulfide and nitric acid are reacted, yielding copper (II) nitrate, sulfur, water, and nitrogen monoxide. When two moles of copper (II) sulfide and twelve moles of nitric acid are mixed in a reaction vessel, which is the limiting reagent?

Neither reagent is limiting. C is correct. First, the balanced equation described by the question must be written: 3 CuS + 8 HNO3 🡪 3 Cu(NO3)2 + 3 S + 4 H2O + 2 NO. Since the 6:16 molar ratio of copper sulfide to nitric acid is equivalent to the 3:8 ratio in the balanced equation, the two reagents are present in stoichiometric quantities. Neither is limiting; both will be exhausted if the reaction runs to completion.

Although Creutzfeld-Jakob Disease is perhaps the best-known prion disease, recent hypotheses suggest that neurodegenerative diseases such as Alzheimer's disease and Parkinson's disease may have prion components. Which of the following, if true, would best refute the claim that prions contribute to the pathology of Parkinson's disease?

Neurons from Parkinson's patients show absolutely no increase in expression of heat shock proteins or cellular chaperones. Prions tend to induce misfolding and aggregation of endogenous cellular proteins, forming highly stable amyloid fibers. The natural cellular response to the presence of misfolded proteins is the production of heat shock proteins, which help to properly fold the defective protein molecules. The absence of heat shock activity suggests that the cell is not experiencing any problems with protein aggregation, which is one of the hallmarks of prion disease pathology.

Two automotive engineers are monitoring the movement of a car on a large circular track. They find that at t = 5 s, the car is traveling at 30 m/s due north. At t = 10 s, it has turned slightly and is moving at 35 m/s northwest, while at t = 40 s it is passing its starting position moving at 45 m/s due north. If we call movement in the northern direction positive, when is the distance traveled by the car negative?

Never; distance is a scalar quantity and cannot be given a negative sign D is correct. Distance, along with speed, is a scalar quantity, meaning that it measures only magnitude and not direction. Since the car continues to move throughout the process, distance is constantly increasing and is never negative.

Which of these molecules would display little to no observable activity on an IR spectrum?

Nitrogen C is correct. To be IR-active, a molecule must have an inducible dipole moment. (While a permanent dipole is not necessary, it also confers IR activity.) N2, a nonpolar diatomic molecule, has no dipole moment; electron density is shared equally between the two nitrogen atoms. When excited by infrared light, the molecule is only induced to stretch along its axis, which does not change its overall dipole moment. It exhibits no bending or asymmetric stretching vibrational modes.

A biological chemist is outlining a particular biological process that is still poorly understood by the scientific community. She hypothesizes that the process involves the molecule pictured below. A student of this chemist describes the molecule as a nucleotide. What error does this description entail?

No error; this molecule is a nucleotide. This molecule is deoxyadenosine triphosphate, or dATP. Like ATP itself, it is a nucleotide; it simply derives from deoxyribose, not ribose.

An optically active molecule that rotates plane-polarized light clockwise reacts with methanol in solution via an SN1 mechanism. In which direction will the product solution rotate plane-polarized light?

No rotation will be observed.

The Ksp of lead iodide is 7.1 × 10-9. A chemical engineer adds 0.0025 mol of KI to a solution of 0.00004 mol Pb(NO3)2 in 500 mL of water. Should the engineer expect to see a solid precipitate?

No, because the ion product is less than the Ksp of lead iodide. Lead iodide has a formula of PbI2. As such, its Ksp expression can be written as Ksp = [Pb2+][I-]2. Our lead and iodide concentrations are 0.00008 M and 0.005 M, respectively; these can be converted to 8 × 10-5 M and 5 × 10-3 M for easier calculations. Now, we simply need to evaluate whether the ion product (a quantity analogous to Q) exceeds the Ksp. (8 × 10-5)(5 × 10-3)2 = (8 × 10-5)(25 × 10-6). The product of these terms is 200 × 10-11, or 2.0 × 10-9. As this quantity is smaller than the Ksp, this substance will not precipitate.

Erik Erikson developed a series of stages of development that involve resolving various conflicts between needs and social demands. According to Erikson, must an individual successfully resolve a stage before moving on to the next?

No, one can progress to the next stage of development without resolving or obtaining mastery in the previous stage. C is correct. A person does not need to successfully master or resolve each conflict to enter the next stage of personality development. While resolution may be healthier in general, it is not required, either at that point or later in life.

The protein structure for transketolase is shown below, along with the molecular structure of its cofactor, thiamine pyrophosphate. Which phase of the pentose phosphate pathway would be most affected if this enzyme were to become nonfunctional?

Non-oxidative C is correct. Transketolase is active during the non-oxidative portion of the pentose phosphate pathway. It is responsible for catalyzing the production of glyceraldehyde 3-phosphate and fructose 6-phosphate.

A certain protein release factor functions to specifically recognize stop codons and terminate translation. How many tRNA molecules bind to the same codons as this factor?

None To ensure the proper termination of translation, stop codons are only recognized by protein release factors. In other words, no tRNA molecules bind to these codons, and they do not correspond to any amino acid residues. If tRNAs were able to recognize such codons, the translational machinery would be able to bypass them and produce inappropriately long protein chains.

Senator Jones belongs to the Democratic Party and attends a national rally. Because he wants to remain accepted and liked by the fellow members of his party, he takes a stronger Democratic stance on issues than he normally would. What type of influence does this demonstrate?

Normative influence This situation relates closely to group polarization. Two main forms of influence serve as components of this phenomenon: normative and informational influence. Normative influence refers to the pull of being socially desired, accepted, admired, or generally liked. Senator Jones experiences this pull, as he wants other Democrats to look upon him favorably.

A variety of experiments are being conducted in a large tank containing an ideal fluid. For a spherical object with a volume of 0.84 m3 and a specific gravity of exactly 1.05, which is true?

Not enough information is available to answer this question. D is correct. This question is trying to trick us into assuming that the liquid in the tank is water. Since we do not know the actual identity of the fluid, we have no idea whether this object will sink or float.

A particular system begins at an initial state and, after an exothermic process, eventually reaches equilibrium. Over this interval, how can you describe the energy change underwent by the system?

Not enough information is given to determine the change. We are told only that this system involves an exothermic process, not that this is the only process involved. We also have no idea whether any work was done on or by the system. For this reason, we cannot draw any conclusions regarding the change.

A particular form of vision impairment is inherited according to the pedigree below. Image adapted from Jerome Walker under CC BY 2.5. What type of inheritance is displayed by this trait?

Not enough information is given. It is "likely" that the condition is X-linked recessive, but it is still possible for the pedigree to be explained by an autosomal recessive trait. The question does not ask which is most likely, but asks to make a definitive decision. Therefore, as the question is currently written, there is not enough information to choose between options, so D is the correct answer choice here.

After spending hours accurately synthesizing propyl propanoate, you realize that you were actually supposed to create a different ester molecule. Which of the following reagents, if added to your current solution, has the best chance of fixing the issue via transesterification?

Octanol Transesterification involves the conversion of one ester to another via reaction with an alcohol. The -OH acts as a nucleophile and attacks the ester's carbonyl carbon, replacing the previous R group on the ester with its own. Here, only choices A and C are alcohols. However, the R group on choice C is an propyl group, which will simply react with propyl propanoate to produce the same product as before. Since we know that you originally synthesized the wrong ester, octanol is the best choice.

Altruism is often explained by the concept of the selfish gene. To what does this term refer?

Organisms display altruistic behavior toward other individuals that share the same genes; thus, altruism ultimately helps genes survive within a population. C is correct. The selfish gene, proposed by Richard Dawkins, expresses the idea that organisms display altruistic behavior to ensure the survival of genes that they have in common with the organism receiving the benefit of the behavior. In other words, while this selflessness does not directly appear to benefit the animal displaying it, it actually helps carry on that animal's genes through related organisms. The closer the relationship between the two individuals, the more likely altruism is to manifest.

Cooperative binding is an especially fascinating aspect of oxygen transport and regulation in the human body. Which statement best characterizes this feature?

Oxyhemoglobin that contains free heme groups is significantly more conformationally available for binding than is deoxyhemoglobin. The attachment of hemoglobin to a single oxygen molecule promotes a change in the position of the central iron ion in the heme group's porphyrin ring. This change facilitates the binding of additional oxygen molecules.

A mass of 10 kg is dropped from a height of 20 m. Ignoring air resistance, what is the maximum speed achieved by the mass? (assume g = 10 m/s2)

PE=mgh 10x10x20= 2000J KE=1/2mv^2 v=20

Which of the listed fatty acids can be synthesized in the human body?

Palmitic acid Excess carbohydrates in the body are converted to palmitic acid. Because of this, this fatty acid is extremely common in animal tissue. The only two essential fatty acids — ones that must be obtained from the diet because the human body cannot synthesize them are linolenic and linoleic acid.

Historically, the United States and Canada have been categorized as which type of governmental or social system?

Partial meritocracy A meritocracy is a system of social or governmental organization in which hard work and talent are rewarded with success. In such a system, those with the most money and power are those who have earned their positions. The U.S. and Canada are largely meritocratic societies, which explains why you often hear of "rags-to-riches" stories. However, virtually no true meritocracies exist, as those who are born to wealthy or politically powerful clans typically possess an advantage.

For an upcoming experiment, you need to obtain an ampicillin-resistant strain of Streptococcus pneumoniae. Assuming that you begin with a strain that is sensitive to this antibiotic, which of these protocols is most likely to accomplish your goal?

Plating Streptococcus on a gradient spanning from 1 μg/L to 1 g/mL of ampicillin Mutations always occur spontaneously, but we still must be able to detect cells that have mutated in the desired way. This can be accomplished by growing the bacteria on a gradient. 1 μg/L of ampicillin is extremely dilute, so colonies should grow regardless of resistance. In contrast, only those that are completely resistant to ampicillin will grow in media containing 1 g/ml.

Which part of the brain acts abnormally in those who suffer short periods of anxiety and elevated heart rate and respiration during periods of slow-wave sleep?

Pons

A young child eating dinner with his family hears his father say "yes, ma'am" to the waitress. The boy, thinking that this behavior is socially expected, answers yes-or-no questions with "ma'am" from then onward. This represents which type of socialization?

Primary socialization Primary socialization occurs when a child initially learns the norms and expected behaviors of his or her culture. Here, the young child learns a particular form of etiquette by observing his father.

How can one account for the increased prokaryotic ability to adapt to changing environments and toxins, in contrast with their eukaryotic counterparts?

Prokaryotes have a fast replication rate; therefore, advantageous mutations can be passed along more quickly than in eukaryotes. Prokaryotes reproduce at a much higher rate than eukaryotes. In addition, they possess fewer checkpoints to correct errors, making mutations more likely to be passed on. These mutations are predominantly deleterious, but some can confer an advantage to their hosts.

Proteins enter and exit the nucleus through the nuclear pore complex. How do proteins specify which direction to travel through this pore?

Proteins or macromolecules bind to Ran, which enters the nucleus in its GDP state and exits in its GTP state. Proteins have specific sequences that signal for nuclear import or export.

In which sleep stage, when analyzed with electroencephalography, would a patient display beta waves?

REM sleep D is correct. Beta waves are emitted both when someone is fully awake and when one is in REM sleep. The only difference between these two conditions is that the beta waves present during REM sleep display a less consistent frequency.

_________ is able to initiate the synthesis of polynucleotide strands.

RNA polymerase RNA polymerase initiates the synthesis of mRNA. Even DNA replication cannot be initiated directly by DNA polymerase; instead, an RNA primer is needed and is created by RNA polymerase.

RNA polymerase catalyzes the production of the mRNA transcript. How would this enzyme be adversely affected if it lacked its specific sigma factor?

RNA polymerase would be unable to bind to gene promoters. A is the correct answer. Sigma factors are initiation factors that enable RNA polymerase to bind to promoter sequences. This is necessary for the initiation of RNA synthesis. Sigma factors are highly specific, and any given RNA polymerase holoenzyme is associated with a single sigma factor.

Which of the following enzymes must be present within the viral capsid of a (-)RNA virus for successful infection and replication to take place?

RNA-dependent RNA polymerase Negative-sense RNA cannot be directly translated by host ribosomes to produce functional viral proteins. The (-) RNA must be used as a template strand for the production of (+)RNA by a viral RNA polymerase carried within the capsid. An enzyme that produces RNA from an RNA template is termed an RNA-dependent RNA polymerase.

Consider the following multistep reaction: A + B 🡪 D Step 1: A + A 🡪 B k1 Step 2: B 🡪 C k2 Step 3: C 🡪 D k3 If the unimolecular steps are extremely fast, what is the rate law for the overall reaction?

Rate = k1[A]2 B is correct. First, it must be determined which of the three steps is rate-limiting. Since the question stem states that both unimolecular steps (2 and 3) are "extremely fast," the bimolecular step (Step 1) must be the slowest, and therefore the rate-limiting, step. The rate law can thus be written directly from that step.

Given the following elementary reaction, what is the rate law? 3A + B → 2C

Rate = k[A]3[B] For elementary reactions, stoichiometric coefficients can be used to write the rate law. These coefficients become exponents according to the following theoretical example: for the reaction aA + bB → cC, rate = k[A]a[B]b.

The first step of fatty acid synthesis is depicted below. In this process, an activated acetyl-CoA molecule reacts with a unit of malonyl-CoA to form an intermediate product. Which sequence of reaction types, if catalyzed by appropriate enzymes, could convert this intermediate into the corresponding fatty acid product?

Reduction → dehydration → reduction This intermediate must first be reduced at the carbonyl farthest from the CoA substituent, producing a hydroxyl group. The resulting species will then be dehydrated, forming a double bond, which will finally be reduced in the last step. This is the exact opposite of the process by which fatty acids are oxidized.

A psychologist advises George to reframe the obstacles in his life so that they represent eustress. What is the psychologist telling George to do?

Reformulate his cognitive appraisal of these obstacles so that his reaction to them is positive and they affect him in a healthy way

Jade, a law student, successfully remembers all of the civil tax codes required for her morning exam. She then jumps on her motorcycle and rides home, where she relaxes by playing a video game in which she wins small tokens. Which choice correctly pairs each action with the type of memory or process involved?

Remembering tax codes - declarative; riding her motorcycle - procedural; playing the video game - operant conditioning B is correct. Facts and concepts are stored in semantic memory, which is a type of declarative or explicit memory. Completing an action that has become second nature, like riding a motorcycle, requires procedural memory. Finally, playing a video game in which certain actions are rewarded is not directly related to memory at all, but certainly involves operant conditioning.

A student researcher plans to study the effects of removing the endoplasmic reticulum from a cell. He discusses this idea with the professor overseeing his research, who informs him that the cells would almost immediately die. Why might the professor believe this?

Removal of the ER might disrupt the outer leaflet of the nuclear envelope, potentially releasing the contents of the nucleus into the cell and triggering apoptosis. The membrane of the endoplasmic reticulum is an extension of the outer leaflet of the nuclear envelope. Its removal would greatly diminish overall nuclear integrity, likely causing that organelle to lyse. As this would almost certainly trigger apoptosis, it is the most accurate and immediate of the choices given.

In a particular psychology lab, rabbits have recently learned to associate small gold coins with food. The sight of one coin makes them hop repeatedly and salivate, just as if food itself was presented. One scientist wants to teach the rabbits to hop and salivate when he plays a loud tone, so he decides to present a coin whenever the tone is played. What single change to this experimental procedure would maximize his chance of success?

Removing the coins from the experiment entirely, and instead pairing food with the tone D is correct. This is an example of second-order conditioning, in which a neutral event is paired with a previously conditioned stimulus instead of an unconditioned one. While this process can be effective, the question mentions that the rabbits have only recently formed an association between the coins and food. If so, the coins cannot possibly be a more effective reinforcer than food itself, and the scientist has no reason to use them.

Jenny, a premed student, forgot several Spanish verbs when she visited Europe and became fluent in French. Additionally, she learned many French customs that led her to have trouble recalling American traditions. In this scenario, which type(s) of interference have influenced Jenny?

Retroactive both when she forgot the Spanish verbs and when she couldn't remember the American customs D is correct. In both situations, Jenny learns new information, which impairs her retrieval of older, previously encoded memories. These are classic cases of retroactive interference.

In which phase of the cell cycle would it be reasonable to expect the highest activity of the pentose phosphate pathway?

S The pentose phosphate pathway produces ribose 5-phosphate, a compound that plays an important role in nucleic acid synthesis. During the S phase, DNA replication is ongoing, and the demand for nucleic acids is expected to be very high.

Which redox reaction best represents the combination of oxygen and solid sulfur into sulfur dioxide?

S + O2 + 4e- → SO2 + 4e- Like other redox reactions, this can be written as two separate half-reactions and then combined. The oxidation half is S → S4+ + 4e-, while the reduction half is O2 + 4e- → 2O2-. When these reactions are put together, we get S + O2 + 4e- → SO2 + 4e-.

Bacteria in the gut play an essential role in processing methane through the following reaction. CH4 (g) + SO42- (aq) 🡪 H2O (l) + HS- (aq) + HCO3- (aq) In a college biology experiment, 50 mg of methane and 60 mg of sulfate are mixed in a closed container. What is the limiting reagent, and how much excess reagent remains when that species has been exhausted?

SO4; 40 mg CH4 C is correct. To calculate the limiting reagent, we must use stoichiometry to determine how much sulfate is required to react with 50 mg of methane (or vice versa). However, we cannot do this before converting grams to moles. 0.05 g CH4 × 1 mol/16 g = 0.003 mol methane, which requires 0.003 mol SO42-, or 0.3 g, to fully react. Since we have only 0.06 g of SO42-, sulfate must be the limiting reagent. 0.06 g SO42- × 1 mol/96 g = approximately 0.0006 mol, which reacts with 0.0006 mol, or 0.01 g, of CH4. We will thus have 0.04 g of methane in excess once the sulfate has been exhausted.

In some gastropods, the direction of spiral coiling is dependent on allelomorphic genes, with a sinistral coil (S) being recessive to dextral (S+) coiling. However, coiling only occurs due to proteins already expressed in the female gamete; therefore, for the first generation, coiling is maternally derived. If you cross a homozygous sinistral female (SS) with a homozygous dextral male (S+S+), what will be the spin of the majority of the offspring in the first and second generation?

Sinistral, dextral B is correct. Coiling direction, as stated, is maternally inherited in the first generation. Therefore, all offspring in the F1 generation should have a sinistral coil with a heterozygous genotype (S, S+). When two individuals from the F1 generation later mate, note that both must be heterozygous for the dominant allele. Therefore, F2 offspring are more likely to have a dextral coil.

You need to memorize all of the amino acid structures by next week, but no matter how long you spend staring at the textbook, you can't remember any of them for longer than 45 seconds. Which study method would be LEAST effective?

Switching from using elaborative rehearsal six times per day to using maintenance rehearsal eight times per day D is correct. Maintenance rehearsal involves shallower processing than elaborative rehearsal, making it less effective as an encoding technique. Here, maintenance might consist of constant repetition of the same structural characteristics, while elaboration would involve forming a variety of connections between the amino acids and other information that has already been learned.

A number of diverse hypotheses have been proposed to explain the phenomenon of autoimmunity. One theory attributes it to defective clonal selection in the thymus, resulting in the inadvertent survival of autoreactive T cells. Which of the following, if true, would provide the strongest support for this hypothesis?

T cells isolated from patients with diabetes show strong affinity for HeLa cells induced to express insulin receptors and MHC. T cells should not recognize "self" antigens like the insulin receptor. The immune cells mentioned clearly did not undergo apoptosis during clonal selection, like they should have. This supports the hypothesis by implicating T cells in the process of autoimmunity

New research indicates that telomeres might play an integral part in the aging process. How may these structures contribute to aging?

Telomeres shorten with each round of DNA synthesis; as they degenerate, they may eventually become too short, and the cell cannot divide without losing useful DNA. Research has shown that telomeres, the repetitive sequences at the ends of chromosomes, shorten with each successive division. Eventually, this shortening can become so severe that the cell may lose genetic information or be unable to undergo division.

Name the part of the brain that is likely overstimulated in sleep-deprived individuals.

The anterior pituitary gland The anterior pituitary is likely be overstimulated, as it is responsible for releasing adrenocorticotropic hormone. ACTH effects the release of cortisol from the adrenal cortex.

A bus driver who was in a traumatic car accident as a teenager never exceeds 30 miles per hour, even on streets where the speed limit is 40 mph. Which component of his attitude toward driving is represented by the fact that he never travels faster than this speed?

The behavioral component As the term implies, the behavioral component of this man's attitude includes his behaviors and actions. Here, he is acting to ensure that he never surpasses 30 mph when driving a bus.

If trauma causes a cell to dramatically shorten the length of its S phase, what compensatory mechanism might occur?

The cell would spend a longer period of time in the G2 phase. The S phase is responsible for DNA replication. A drastically impaired S phase, then, is likely to yield a cell with an incomplete or inaccurately replicated genome. As a result, this cell would be prevented from passing the G2-M checkpoint, which ensures correctness in DNA replication. Instead of quickly entering mitosis, it will spend additional time in the G2 phase to correct the errors generated by the traumatic event.

Trichlorophenol can be synthesized by reacting phenol with:

chlorine and water. D is correct. In order to chlorinate the phenol, Cl2 and water (a polar solvent) should be used.

In Huntington's disease, the wild-type allele, h, is recessive to the disease allele, H. The persistence of an autosomal dominant allele that is fatal in 100% of cases is best explained by which of the following?

The disease has no effects until the individual has reached an age far past the normal age for reproduction. D is correct. The impact a trait has on an individual's fitness is determined only by its impact on the individual's reproductive success. If a trait has no effect until after the reproductive life is over, it won't change the fitness of an individual and can continue to exist in the population. Even in the case of Huntington's disease, which is both dominant and fatal, it can persist simply because the disease doesn't typically strike until the person is in late middle or old age.

In atherosclerosis, lipid deposits in the lining of arteries accumulate, inducing macrophages to attempt to absorb and "clean up" the deposits. If these cells are incapable of handling the volume, they die, inducing further immune cells to migrate to the deposit. Eventually, this cycle results in the formation of a plaque under the surface of the lining, which blocks the flow and reduces the flexibility of the artery. Under high mechanical stress, the plaque can rupture, creating a dangerous free-floating clot called a thrombus. A preventative treatment for the formation of thrombi would aim to strengthen which component of an artery?

The endothelium The endothelium is the name given to the epithelial lining of the vasculature surrounding the inner space or lumen. Underneath this lining lies vascular smooth muscle, which is encompassed by the outer lining or adventitia. To prevent rupturing of the plaque, it is best to strengthen the endothelium beneath which it is located.

Sociologists define self-concept as the way an individual evaluates and perceives him- or herself. Children begin to develop their self-concepts at a very young age. How can one accurately chronologize the development of the existential and categorical selves?

The existential self comes first, followed by the categorical self; the child realizes her existence as a separate entity before understanding that she is an "object" in the world and can be categorized. A is correct. The categorical self can only develop after the formation of the existential self. Together, these two aspects contribute to the existence of the self-concept. During the maturation of the existential self, the child discovers that she is distinct from others and that she can interact with the world throughout time and space. Following this revelation, she can develop the categorical self, in which she realizes that other entities may interact with her - in other words, that she also exists as an "object" in the world.

A researcher created a chromatogram to describe the elution of a mixture of two unknown compounds. If this chromatogram depicts the results of a reversed-phase chromatography process, and if the mixture that was analyzed includes methane and an unknown acid, which of these statements is accurate?

The first peak represents the unknown acid, since it is polar. In reversed-phase chromatography, the stationary phase of the column is hydrophobic and the mobile phase is hydrophilic. Thus, polar molecules will interact more strongly with the mobile phase and elute more quickly than nonpolar species. Since the unknown acid is more polar than methane (a nonpolar hydrocarbon), it will have a shorter retention time.

The signal to promote the release of cortisol begins in which of the following organs?

The hypothalamus Our bodies respond to stress through activation of the sympathetic nervous system. Part of this response involves the release of cortisol, a process that begins in the hypothalamus with the secretion of corticotropin-releasing hormone (CRH). This stimulates the anterior pituitary to release adrenocorticotropic hormone (ACTH), which in turn promotes the release of cortisol from the adrenal gland.

Give the correct sequence of the mechanism by which cortisol is released.

The hypothalamus releases corticotropin-releasing hormone, causing the anterior pituitary to release adrenocorticotropic hormone; the adrenal gland then releases cortisol. C is correct. Our bodies respond to stress through activation of the sympathetic nervous system. Part of this system involves the release of cortisol, a hormone with a number of effects, including a shift in metabolism from using sugar as an energy source to using fat. The cortisol release mechanism begins when the hypothalamus secretes corticotropin-releasing hormone (CRH), which stimulates the anterior pituitary to release adrenocorticotropic hormone (ACTH). Finally, in response, the adrenal cortex secretes cortisol into the bloodstream.

Unpolarized light with an intensity of 200 kW/m2 shines on a series of vertical polarizers, as shown below. The intensity of light incident on the eye after passing through both polarizers is recorded as the "reference intensity." If the second polarizer is then rotated by 90°, how much will the intensity of light reaching the eye change, compared to the reference intensity?

The intensity will decrease by 100 kW/m2. The intensity of unpolarized light will be cut in half as it passes through a single polarizing filter, leaving an intensity of 100 kW/m2. Originally, because the second polarizer is oriented in the same direction as the first, it will not change the intensity of the light that passes through it. Therefore, the intensity of the light that reaches the eye is 100 kW/m2. Then, turning the second polarizer by 90° will cause the filters to be perpendicular, blocking 100% of the light. The new intensity of the light reaching the eye will be 0 kW/m2, an intensity decrease of 100 kW/m2.

Variations on the traditional method of mass spectrometry have recently been employed to visualize large proteins. One such technique, electrospray ionization (ESI), achieves the required sample ionization by applying a high voltage across an aerosol of sample solution. Which of the following, if true, represents a reason why ESI should NOT be used to obtain data about transmembrane proteins in their native state?

The ionization process manufactures a highly charged environment that prevents cytosolic proteins from unfolding. Since transmembrane proteins contain a substantial amount of hydrophobic residues on their exterior, they require a similarly hydrophobic environment to retain their structure. In other words, while a charged environment prevents hydrophobic globular cytosolic proteins from unfolding, it is certainly possible that it would cause transmembrane proteins to denature.

Describe the potential energy curve for two hydrogen atoms as they are brought increasingly close together.

The potential energy begins at a baseline, then drops as the atoms move closer together. It reaches a trough and, as they are brought even closer, rises back above the baseline. The potential energy of the two hydrogen atoms begins at a baseline level. Due to the stability conferred by a bond, it then drops, reaching a minimum at the ideal bond length for H2. However, if they continue to move closer together, they will begin to repel each other and bring the potential energy back above the baseline.

The diagram below shows a type of epithelial layer. This type of epithelium would most likely be found in which of the following locations?

The lining of the small intestine B is correct. The diagram depicts a section of simple columnar epithelium. As a "simple," or one-layered, tissue, this epithelium is best suited for absorption or secretion. However, these functions are performed by a variety of organs, so we must look at the shapes of the individual cells. The column-shaped nature, as well as the cilia lining the top of each cell, are clear hallmarks of the intestinal lining.

A number of student researchers are investigating murine type II diabetes. They find that, despite consistently high insulin levels, the mice remain in a hyperglycemic state. How is this possible?

The mice have developed a resistance to insulin, lessening its systematic effects.

If a cell were infected and its NADPH degraded, all of the following cellular processes would be compromised EXCEPT:

The molecule would have a different name and serve an entirely distinct function. A is correct. Removing the phosphate group on the ribose-containing adenine would turn NADPH into NADH. These are two different molecules with very different functions. While NADPH is primarily used in anabolic pathways (including nucleic acid and lipid synthesis), NADH is an electron carrier in cellular respiration. Thus, removing the phosphate group would change both the name of NADPH and its function.

How do the (+) and (-) ends of actin differ?

The monomers on the (+) end are bound to ATP, while those on the (-) end are bound to ADP. The (+) end actin monomers have a greater binding affinity for each other than do those on the (-) end.

When an experimental relationship is depicted in graph form, which axis usually corresponds to the independent variable(s)?

The x-axis, and the dependent variable(s) are graphed on the y-axis A is correct. Since the values of an independent variable are controlled by the experimental procedure, it is typically plotted along the x-axis. In contrast, the quantity or extent of a dependent variable relies upon its relationship with the independent variable. As a result, it is most sensible to graph it along the y-axis, where a quick glance can compare the difference between higher and lower values.

A high-school biology student is describing the excretory system to her classmate. She mentions that, in healthy individuals, no glucose should be excreted in the urine. Instead, glucose monomers are secreted from the lumen of the nephron into the interstitium surrounding the proximal convoluted tubule for later return to the bloodstream. What mistake did this student make?

The movement of solute out of the lumen of the nephron for retention in the body is termed reabsorption, not secretion. Overall, the student did a good job explaining this vital physiological process. However, she incorrectly stated that glucose is secreted in the proximal tubule. Secretion, or the transport of solute from the bloodstream into the nephron, is the opposite of the process that she described.

The cells comprising a cancerous tumor frequently suffer progressive genetic damage, caused by uncontrolled proliferation due to deactivation of genes that suppress errors in DNA replication. This genetic damage can cause the cancerous cells to engage in normal activity, such as the synthesis and secretion of various biomolecules, but in an uncontrolled fashion. A biopsy of a tumor reveals pockets of a fluid with heavy concentrations of proteoglycans and glycoproteins. From which area do these cancerous cells most likely originate?

The mucosa of the small intestine

A neurologist isolates a sensory neuron from the left foot and places it in an in vitro environment similar to that found within the body. He then exposes the neuron to various stimuli. In one case (Trial 1), he pokes it with a small probe and observes an immediate depolarization to -40 mV. In a separate trial (Trial 2), he uses mild electric stimulation to provoke a change to -54 mV over the same short interval. If this neuron has a threshold of -55 mV, during which trial will this neuron fire more strongly?

The neuron will fire equally in both trials, as the nervous system operates under the "all-or-none" principle. The "all-or-none" principle states that a neuron will fire when it experiences a depolarization that surpasses its threshold, and will not fire when it does not. In other words, there is no "in-between" region in which the neuron fires more strongly or weakly. Since both stimuli surpass the threshold of -55 mV, this neuron should fire in an identical manner during both trials.

Two light bulbs are placed sequentially in a circuit. What will be the total current be if a third light bulb is wired into the circuit, parallel with the first two bulbs? (Note: Rbulb = 75 Ω)?

The new current will be triple the original current. Adding circuit components, like resistors (i.e. the light bulbs), in parallel is like adding roads in a city. With new paths for the current to take, this new "road" has less resistance, so it will draw more current through the circuit. V = IR. The original current (I) is V/R eq = V/(75+75) = V/150. Adding the third bulb in parallel means 1/R eq = 1/(75+75) + 1/75. 1/Req = 1/150 + 2/150 = 3/150. Req = 150/3 = 50 Ω. New current = V/50 = triple the original current.

Two charges in a medium with a permittivity ε experience an attractive electrostatic force, F. How would this force change if the dielectric constant were increased to 2ε and the distance were doubled?

The new force would be 0.125 times the original value. Coulomb's law is typically given as , where "r" represents the distance between the two charges. From this relationship, we can see that doubling "r" reduces the force by a factor of 4. However, it is slightly trickier to assess the role of the dielectric constant. Technically, though, Coulomb's constant "k" is equal to , so doubling ε halves the force generated. The product of one-fourth and one-half is one-eighth, or a change to the original force by a factor of 0.125.

Leptin is a hormone that is produced by adipose tissue and that regulates energy balance by triggering feelings of satiety (fullness). When tested, obese patients reported hunger at the end of a meal, despite displaying leptin levels much higher than their healthy-weight counterparts. Which of the following gives the best explanation for this observation?

The obese patients have developed a tolerance to high levels of leptin. B is correct. In a similar fashion to diabetes, obese patients tend to develop a resistance to leptin due to its perpetually high levels. This reduces its physiological effect on the individual.

Although in utero conditions can dramatically alter the course of fetal development, the rate of spontaneous DNA damage that occurs within the haploid oocyte is remarkably low. Which of the following, if true, constitutes a possible explanation for this finding? I. The oocyte travels along the tube encased in a layer of metabolically-active maternal cells derived from the ovarian follicle. II. The oocyte contains sufficient nutrients to nurture the embryo during the first zygotic divisions. III. Oocyte chromatin displays a higher degree of supercoiling than that of most other known mammalian cell types.

The oocyte travels along the tube encased in a layer of metabolically-active maternal cells derived from the ovarian follicle. Oocyte chromatin displays a higher degree of supercoiling than that of most other known mammalian cell types.

A heroin addict who decides to quit the drug immediately begins to experience leg tremors, trouble sleeping, and violent illness. Which theory of motivation is most commonly cited to explain this situation?

The opponent-process theory, which describes two contrasting emotional or physical components to many behaviors

Due to a sudden mutation, an important enzyme gains an unusually high affinity for numerous substrates in addition to its original one. If these new substrates bind at an allosteric site, and the enzyme is placed in a solution containing equal levels of the original substrate and the new substrates, how will this most likely affect the enzyme's reaction with its original substrate?

The original reaction will occur at a slower rate due to conformational changes of the enzyme. Allosteric inhibition involves the binding of a molecule at a site other than the active site. This binding causes a conformational change in the enzyme, rendering it unable to bind its original substrates. If the enzyme in the question stem is suddenly able to allosterically bind several new molecules, and those new substrates are present in the reaction environment along with the original substrate, then a significant proportion of the enzymes will be unable to carry out the original reaction.

All of the following are Freudian concepts EXCEPT: I. the latency stage. II. the death drive. III. fixation. IV. the ought self.

The ought self is part of self-discrepancy theory, which was originally put forth by Edward Higgins. While that specific name is unlikely to appear on the MCAT, you should understand the other components of this theory, which are the actual self and the ideal self. You should also know that this idea is not Freudian.

Diabetes insipidus (DI) is characterized by a deficiency of antidiuretic hormone. What effect would this have on a DI patient's urine?

The patient's urine would be hypotonic. Antidiuretic hormone (ADH) promotes the retention of water in the body. Without it, excess water is excreted, yielding hypotonic urine.

The peptide bond between which two amino acid residues is cleaved by HIV protease?

The peptide bond is cleaved between Phe on the N-terminus side and Pro on the C-terminus side.

A student is attempting to isolate several products from the organic layer of an extraction. Initially, her solution contains an alkene, a phenol, a secondary alcohol, and a tertiary alcohol. If she begins the separation process by adding a moderately strong base, which substance is most likely to move into the water layer?

The phenol During extractions, substances typically move into the water layer when they become charged. Since this student is adding a base to her mixture, she will likely deprotonate the strongest acid of the four substances, giving it a negative charge. The most acidic compound listed is the phenol. Its resonance delocalizes the negative charge on its conjugate base, making it more likely to lose a proton in the first place.

The lymphatic system is involved in monitoring the immunological condition of the body. Lymph nodes are lymphatic organs that house high amounts of T and B cells. The lymphatic system ultimately drains back into venous circulation at the left subclavian vein. A severely infected laceration on the right foot would most likely lead to significant swelling of which lymph node?

The popliteal lymph node, located behind the knee D is correct. Immune cells routinely sample antigens in the peripheral tissues and return to the lymph nodes through the lymphatic vessels to present them. Presentation will occur at the node that is most proximal to the site from which the migrating immune cell has come. The closest node to the foot is the popliteal lymph node, which will swell due to lymphocyte proliferation in response to infection.

While at the local amusement park, two young women notice that as the sun sets, the blue color of the roller coaster changes slightly due to differences in lighting. One woman tells the other that the Gestalt principle of similarity explains why our brain still interprets this as the same object. What is incorrect about her statement?

The principle of similarity states that similar objects tend to be grouped together by the brain. The phenomenon described is actually explained by the concept of perceptual constancy. C is correct. Perceptual constancy is the idea that, despite observed changes, our brain tends to interpret familiar objects as having the same size, shape, and brightness over time. It is this idea, not the law of similarity, that relates to the roller coaster.

Which choice is an accurate statement about electron affinity?

The process that it reflects is exothermic for bromine but endothermic for potassium. D is correct. To establish a complete octet, bromine needs only one additional electron. Therefore, when bromine adds an electron, it will become more stable and release energy. Potassium, in contrast, prefers to lose an electron to have a full octet. For this element, adding another electron is unfavorable and will require energy in an endothermic process.

All of the following statements about Ka and Kb are true EXCEPT:

The product of the Ka of HPO42- and the Kb of HPO42- is equal to the value of Kw at that temperature. When using Ka*Kb = Kw, remember that this relates the Ka of an acid with the Kb of its conjugate base, not the Ka and Kb of the same species! In other words, it is the product of the Ka of H2PO4- and the Kb of HPO42- that will be equal to the value of Kw at the relevant temperature.

In Anfisen's landmark experiment, the enzyme ribonuclease was denatured with 8 M urea and mercaptoethanol. The denaturing environment was subsequently removed via dialysis and the enzymatic activity of the protein was monitored after exposing the protein to the oxidizing environment of the atmosphere. What was the observed result?

The protein regained enzymatic activity. A is correct. Anfinsen's experiment supported the idea that a protein's primary structure, or amino acid sequence, contains the information that directs secondary, tertiary, and quaternary structure. The disulfides within the enzyme were reformed via oxidation, allowing the enzyme to fully regain its conformation and, consequently, its function.

A sample of decanoic acid is analyzed using nuclear magnetic resonance (NMR) spectroscopy. Which of this molecule's protons will exhibit a peak that is farthest downfield?

The proton on the -COOH group, which will display a shift of around 11.5 ppm With regard to NMR, "downfield" refers to the left-hand side of a spectrum. Here, signals appear if they correspond to nuclei that are "deshielded," or near electronegative atoms. In this case, the carboxylic acid proton is very close to two oxygen atoms, causing it to shift a large distance downfield. Specifically, -COOH signals typically appear between 10 and 13 ppm on an NMR spectrum.

A student notices that a flask containing several reagents forms ice crystals on its outer surface as they react. He places the flask in a warm bath to prevent further crystal formation. How would this bath affect the new concentration of product at equilibrium?

The reaction is endothermic, so product concentration should increase. The ice crystal formation indicates that this reaction requires heat energy to proceed. As such, it must be endothermic. Heat can thus be written as a reactant in this process. According to Le Châtelier's principle, addition of additional reactant shifts the reaction to the product side to reestablish equilibrium.

A biologist is making a variety of measurements for a sample of CO2 generated by a colony of aerobic bacteria. The sample contains exactly 1 mole of gas and is being held in a 0.05 L container. When the biologist calculates the PV/RT ratio of the gas, he finds that it is 1.05. How could this finding be explained?

The relatively high pressure within the flask is causing particle volume to exert a substantial effect. Since the sample contains 1 mole of gas, we can rearrange the ideal gas law to find our expected PV/RT ratio: 1. The given value is higher, implying that either pressure or volume is larger than expected. For the ideal gas law to work properly, two main assumptions are necessary: that the volume of particles themselves is negligible and that no intermolecular forces are present between particles. However, these assumptions become especially untrue at low temperature and high pressure. Given this information, D is the only sensible answer.

Which of the following is FALSE about ATP and its hydrolysis?

The ΔG for ATP hydrolysis is pH-dependent. ∆G is not affected by pH. The remaining choices are accurate - one high-energy bond can be hydrolyzed to yield ADP, and the second can be hydrolyzed to form AMP. Interestingly, the ∆G value for ATP hydrolysis is almost identical to that for ADP hydrolysis, with both reactions being highly exergonic (negative ∆G).

In humans, gluconeogenesis occurs in the:

kidneys. Gluconeogenesis mainly takes place in the liver, but does occur to a lesser extent in the kidneys. Remember, the liver plays an integral role in the maintenance of stable blood glucose levels.

An ecologist is making several measurements for a sample of methane gas produced by a herd of cattle. The sample contains exactly 1 mole of gas, but when the ecologist calculates its PV/RT ratio, he finds that it is 0.92. Without knowing the conditions under which the gas is held, what is the most likely explanation for these findings?

The relatively low temperature within the flask is causing intermolecular forces to exert a substantial effect. Since the sample contains 1 mole of gas, we can rearrange the ideal gas law to find our expected PV/RT ratio: 1. The given value is lower, implying that either pressure or volume is smaller than expected. For the ideal gas law to work properly, two main assumptions are necessary: that the volume of particles themselves is negligible and that no intermolecular forces are present between particles. However, these assumptions become especially untrue at low temperature and high pressure. Given this information, C is the only sensible answer.

Generally, cells tend to spend around 90% of their time in interphase. What would be the most likely result if a mutation removed the G1 checkpoint?

The resulting daughter cells would be smaller than those produced from the division of non-mutated cells.

The human genome contains a large number of repetitive sequences that have no known function. What happens to these sequences during the life of a cell?

The sequences are spliced out before translation takes place. D is correct. Splicing out introns to remove them from a mature transcript is one of the major post-transcriptional modifications. Other modifications include the addition of a 5' cap and a 3' poly(A) tail.

A scientist studying cell-to-cell communication discovers that a particularly biologically relevant signal acts in a paracrine manner. What must be true of the cell or cells involved?

The signal travels a very small distance from its origin to the target cell, but this response does not involve hormone movement through the bloodstream. Paracrine activity involves the secretion of local signaling molecules known as paracrine factors. These factors diffuse to nearby target cells and exert a variety of effects.

Caspases, also known as cysteine proteases, are instrumental in the progression of the apoptotic pathway. Prior to initiation of apoptosis, caspases exist as inactive pro-caspases in the cytosol. Caspase activation is tightly controlled and is actually mediated by the action of cytochrome c. Which of these targets is a likely substrate for cellular caspase?

The small nuclear protein ICAD, resulting in its dissociation from its binding partner, a DNase C is correct. In non-apoptotic cells, ICAD exists in a bound state with CAD, which is a DNase. It may help to know that CAD actually stands for "caspase-activated DNase," while ICAD stands for "inhibitor of caspase-activated DNase." When ICAD is cleaved by an effector caspase, it loses its structural integrity and can no longer remain associated with its partner. This relieves the inhibition of CAD and permits subsequent DNA degradation, a hallmark of apoptosis. Note that this answer can also be achieved by elimination, as much of the above information is too specific to be high-yield on the MCAT.

The cross-sectional area of the aorta is 4 cm2. The cross-sectional area of a capillary is 3 x 10-7 cm2. If there are 109 capillaries in the human body, what must be true of the speed of the blood in the capillaries compared to in the aorta?

The speed of flow must be lower in the capillaries since the total cross sectional area of all the capillaries is greater than the cross-sectional area of the aorta. Within a pipe system, the speed of flow and cross-sectional area is related by A1v1 = A2v2. The fact that there are 10 billion capillaries, all roughly in parallel with each other (bridging from arterial system to venous system) gives them a larger cross-sectional area for the blood to travel and thus, a lower velocity.

Consider the net reaction leading to formation of fructose 6-phosphate from glucose and ATP. ΔG° for the reaction equals:

The ΔG° for the formation of fructose 6-phosphate from glucose 6-phosphate and ATP is equal to -3,600 cal/mol. This is the sum of the ΔG° for the formation of glucose 6-phosphate and ADP from glucose and ATP (-4,000 cal/mol) and the ΔG° for the reaction to which it is coupled, the formation of fructose 6-phosphate from glucose 6-phosphate (+400 cal/mol). -3,600 cal/mol can be easily converted to -3.60 kcal/mol. ΔG° < 0 for a given reaction indicates that reaction is spontaneous under standard conditions, and that at equilibrium, the products of the reaction predominate over the reactants. This is indicated by Keq > 1 and can be clearly seen in the relationship between ΔG and Keq: ΔG = -ln Keq where Keq > 1 returns a value of ΔG < 0, and Keq < 1 returns a value of ΔG > 0. Here, then, since ΔG is less than 0, Keq for the reaction is greater than 1.

A hoop and a sphere, each of mass M and made of the same material, are both released from rest at the top of a ramp. The coefficient of friction between the ramp's rough surface and the objects is unknown. Both objects roll down the ramp without slipping. At the bottom of the ramp, which object will have the greater translational velocity, and why?

The sphere, because its moment of inertia is lower An object's Kinetic Energy (KE) is the sum of the KE from its rotational motion and the KE from its translational motion. Though the mass is the same for both the hoop and the sphere, the hoop's mass is distributed farther away from its center of rotation than is the sphere's mass. The hoop's mass therefore must move faster than the sphere's mass. Therefore: KErotation of hoop > KErotation of sphere But at any point along the ramp, because the hoop and the sphere have descended from the same height: KErotation of hoop + KEtranslation of hoop = KErotation of sphere + KEtranslation of sphere In order for both of the above equations to be valid, KEtranslation of hoop < KEtranslation of sphere Hence, since the common mass cancels out, the sphere must have a greater translational velocity. This is a consequence of its mass distribution. In other words, the sphere's translational velocity is greater than the hoop's because the sphere's moment of inertia is lower.

Which of these statements are true regarding sensation and perception? I. The two-point threshold defines the minimum distance between two regions that allows them to be perceived as distinct touch stimuli when simultaneously stimulated. II. Physiological zero refers to the normal temperature of skin. III. Smell is intensely linked to emotion because it is processed directly by the limbic system. IV. Taste receptors are grouped together on the tongue so that certain areas of the tongue only detect specific flavors.

The two-point threshold defines the minimum distance between two regions that allows them to be perceived as distinct touch stimuli when simultaneously stimulated. Physiological zero refers to the normal temperature of skin. Smell is intensely linked to emotion because it is processed directly by the limbic system.

A puck is on a frictionless tabletop. The puck is attached to a string, which is threaded through a small hole in the table such that it can be pulled on to be shortened. The puck is then swung in a circle with a velocity of v. If the string is loosened and allowed to extend to three times its original length, how will the puck's velocity change?

The velocity of the puck will decrease to 1/3 of its previous value. C is correct. The initial and final values for the puck's angular momentum should be identical. Angular momentum, or L, is given by the quantity mvR. From this expression, we see that v (velocity) must decrease by a factor of three to compensate for the increase in R (the length of the string).

3.75 moles of nitrogen gas is held in a rubber balloon with a current volume of 1 L. The balloon is placed in a room with an initial pressure of 1 atm and temperature of 25°C. If the pressure suddenly drops to 0.75 atm, what change in volume is necessary if the temperature is to remain the same?

The volume must be increased to 1.33 L. While we could plug all of the given values into the ideal gas law, we can make this problem much simpler by using Boyle's law, P1V1 = P2V2. Since initial pressure is 1 atm and initial volume is 1 L, we get (1 atm)(1 L) = (0.75 atm)(x L). The new volume must be approximately 1.33 L.

A clinical trial for an experimental form of birth control involves injecting patients with progestin, or synthetic progesterone. If a woman receives this injection during the early part of the follicular phase, and if its effects persist for approximately three weeks, what can be expected to occur?

The woman will not experience ovulation at the usual time. Typically, estrogen and progesterone negatively feed back on gonadotropin-releasing hormone (GnRH). As a result, high levels of these steroids lead to reduced release of LH and FSH. As a surge in LH is required to stimulate ovulation, persistently high progesterone levels near the beginning of the menstrual cycle will prevent this woman from ovulating - at least until the effects wear off.

Which of the following is true of both NADH and FADH2?

They are used as coenzymes and store the energy that is released in oxidation reactions. B is correct. Both NADH and FADH2 are formed when NAD+ and FAD, respectively, are reduced in redox reactions during processes such as glycolysis and the TCA cycle. Once in their reduced form, NADH and FADH2 carry electrons to the electron transport chain to generate ATP.

To win a card game, students must memorize all five of the playing cards shown below in a five-minute span. They must be able to correctly list the cards in order from left to right, and must accurately name the type of card (Jack, Ace, Queen, etc.), the color, and the symbol in the center. During the second round of the game, students still must remember the five cards shown above, but two other cards - a black queen and a gray jack - are shown immediately after the five minutes ends. When quizzed later, several students wrongly recall that those two cards were present in the original set of five. What role do the black queen and gray jack play in this situation?

They cause the students to experience retroactive interference. A is correct. The two additional cards serve as new information that interferes with the recall of old knowledge. This is the definition of retroactive interference.

Which of the following are true of Class I transposons? I. They involve the action of reverse transcriptase, which transcribes RNA back to DNA. II. They involve the normal transcription of DNA. III. They act via a "copy-and-paste" mechanism. IV. They act via a "cut-and-paste" mechanism.

They involve the action of reverse transcriptase, which transcribes RNA back to DNA, They involve the normal transcription of DNA, and They act via a "copy-and-paste" mechanism.

For two carbohydrates to be considered anomers, what must be true?

They need to have different stereochemistry solely at the anomeric carbon, which is the atom that serves as the carbonyl carbon in the molecule's straight-chain form. An anomer is a specific type of epimer that differs only at the anomeric carbon. This position, derived from the carbonyl carbon in the sugar's straight-chain aldehyde or ketone form, can be spotted in a cyclic structure as the only member of the ring that is bound to two oxygen atoms.

Consider a particular amino acid sequence taken from an enzyme: Pro-Leu-Asp-Cys-Arg-Ser-Tyr-His-Ser-Gly-Cys-Asp This catalyst is physiologically inactive until it is exposed to stomach acid. After exposure, it can be used to react with and activate digestive zymogens. How can this process be explained?

This sequence is part of an enzyme with a low optimal pH. Every enzyme has a specific temperature and pH at which it functions most effectively. Outside of these conditions, the enzyme will catalyze its reaction(s) less rapidly or even become entirely denatured. This enzyme is said to function only when in the presence of stomach acid, which has a pH of around 2.

A new species of bacteria infects the mitochondria of eukaryotic cells and releases a toxin into the mitochondrial matrix to inhibit glyceraldehyde-3-dehydrogenase. What effect will this have on glycolysis?

This will have no effect on glycolysis. C is correct. The question stem specifically states that the toxin is released within the mitochondrial matrix. Thus, it will not affect glycolysis, which takes place in the cell's cytosol.

The following set of Fischer projections depicts two aldohexose molecules in straight-chain form. If equal molar amounts of both of these molecules were dissolved in the same aqueous solution, how could they be separated?

Through fractional distillation, as they likely differ slightly in their boiling points These compounds are diastereomers, as their configurations appear identical with the exception of a single carbon. (Specifically, they are epimers, but this is not important for this question.) Diastereomers have distinct physical and chemical properties, including boiling point, melting point, and solubility. However, as both species are alcohols with similar molecular weights, their boiling points probably do not differ to a large degree.

A virus invades a cell and rapidly produces antisense RNA that is complementary to a gene's promoter sequence. What will happen to the transcription of the gene?

Transcription will be severely reduced. B is correct. This question requires understanding of the complementary DNA strands involved in transcription. One strand is the sense (non-template) strand, while the other is the antisense (template) strand. mRNA is transcribed directly from the antisense strand, giving it the same base sequence as the sense strand. Thus, if a virus produces antisense RNA complementary to a gene's promoter sequence, the viral RNA will hybridize that sequence, blocking transcription from occurring.

Which of the following is NOT required to develop a DNA fingerprint from a small sample of DNA?

Transformation Transformation is the genetic alteration of a cell from incorporating outside DNA. This would alter the DNA fingerprint and is not related to the actual analytical process of DNA fingerprinting.

Although eukaryotes differ from prokaryotes in many significant ways, their replication, transcription, and translation mechanisms are fairly similar. However, notable distinctions can be seen. Which of these aspects of translation is uniquely prokaryotic?

Translation requires multiple release factors (RFs). Prokaryotic translation does rely on the presence of several release factors. In contrast, eukaryotes need only one such factor: eukaryotic translation termination factor 1 (eRF1). This answer can also be found through elimination.

In the middle of a recital, a violinist notices with some annoyance that a rude individual is talking in the audience. Initially, she continues with her performance as usual; however, as the talking continues, she grows distracted and misses a note. Which model of attention best explains this phenomenon?

Treisman's attenuation model; a filter "turns down" the intensity of the annoying talking, but its intensity increases over time.

In the middle of her performance, a pianist notices with some annoyance that a cell phone is ringing in the audience. However, she still continues playing as usual. Which model of attention best explains this phenomenon?

Treisman's attenuation model; a filter lowers the intensity of, but does not fully eliminate, the distraction caused by the cell phone. C is correct. Because the pianist notices the ringing phone but is still able to focus on the task at hand, this situation best relates to Treisman's attenuation model. This theory proposes that we "turn down," or attenuate, the intensity of less important stimuli to focus on other tasks. Perhaps if the phone were to ring for the entire performance, the perceived intensity of that stimulus would increase and the pianist's playing might be affected.

Acidic and basic amino acids can be freely found in the bloodstream. How are these molecules involved in buffering the plasma at physiological pH?

Under low-pH conditions, acidic amino acids can decrease H+ concentration. Amino acids can buffer a solution either by gaining or releasing H+, depending on their abilities and on the condition of the environment. At physiological pH, acidic amino acids are negatively charged since their side chains contain a COO- group. If H+ concentrations increase, pH levels will decrease; at this time, acidic side chains can pick up some of the excess protons to re-establish a healthy level

A high-school chemistry student is given a flask containing n-octane (boiling point = 125 °C). If she must react this compound in such a way as to raise its boiling point above 145 °C, what should she do?

Use free-radical halogenation to add a chlorine atom to one of the compound's terminal carbons A is correct. Boiling point is mainly determined by intermolecular forces and molecular weight. Since unsubstituted alkanes are able to exert only London dispersion forces, they typically display low boiling points. However, if this student were able to halogenate n-octane, she would be giving it a polar bond between the chlorine atom and one carbon. This would allow the product to experience dipole-dipole forces, raising its boiling temperature.

Mrs. Jones, a middle-aged woman, is attempting to use biofeedback to deal with her stressful day-to-day life. What might this entail?

Using a machine to monitor heart and respiratory rate in order to train herself to control involuntary responses Biofeedback uses technology to monitor various bodily functions and display them to the participant. The goal of this technique is to allow people to maintain (or feel as though they are maintaining) more control over involuntary responses and develop better reactions to stress.

An MCAT instructor checks in on his students every one to three days, giving them praise if they've completed their assigned work. Which reinforcement schedule is involved in this situation?

Variable-interval B is correct. The instructor contacts his students after certain amounts of time, not a certain number of responses, making this either a fixed-interval or a variable-interval schedule. Since he checks in after inconsistent time periods ("every one to three days"), it must be variable-interval.

Of the situations below, which is LEAST likely to lead to total internal reflection?

Visible light travels from water (n = 1.34) to air (n = 1.0) at an angle of incidence of 46°. For total internal reflection to occur, two main criteria must be met. First, light must be traveling from a medium with a high index of refraction to one with a lower index, preferably with a large difference between the two. (Note that here, all four choices involve movement from a high to a low "n.") Second, the angle of incidence must be relatively large. Thus, as this is a LEAST question, we are looking for two things: a small difference between the two indices of refraction and a small angle of incidence. Choice D fits both of these conditions. With such a tiny difference between the two "n" values, as well as such a relatively small incident angle, it is highly unlikely that light will bend enough to exhibit total internal reflection.

A first-grade child struggles to write her name, but succeeds in doing so when a teacher guides her pencil. This situation best exemplifies which developmental theory?

Vygotsky's zone of proximal development Vygotsky proposed the idea that some tasks are easily accomplished by a child, while others are well outside the child's abilities. However, some skills fall in between those two categories and can be completed with help from a more knowledgeable other. In this way, the young person is able to learn to perform the action on his own.

Which changes are experienced by visible light as it moves from Medium 1 (n = 1.16) to Medium 2 (n = 1.68)?

Wavelength decreases while frequency remains constant. Here, light rays are moving from a material with a low index of refraction to one with a relatively high one. According to the equation n = c / v (which can be rearranged to v = c / n), the light will slow down during this transition. From here, it is helpful to remember this relationship: velocity = λf. Since the velocity of this light is decreasing, the product of wavelength and frequency must decrease in a corresponding manner. Furthermore, the frequency of light does not change when that light transitions between media. Thus, it must be wavelength that drops to yield the predicted decrease in speed.

Which of the following conditions could result in an inability to plan coherent sentences or understand syntax, without affecting an individual's ability to physically produce words? I. Wernicke's aphasia II. Broca's aphasia III. Temporal lobe damage

Wernicke's aphasia Temporal lobe damage

An aerobic organism placed in an oxygen-poor environment will begin to produce:

an excess of NADH. B is correct. In an oxygen-poor environment, the citric acid cycle will slow while glycolysis is activated further. Glycolysis utilizes ADP, NAD+, and glucose to form ATP and NADH. Note that aerobic organisms depend on the citric acid cycle and oxidative phosphorylation to produce the majority of their ATP; without these processes, ATP will not be abundant.

A perfectly round, transparent bucket is filled with equal volumes of distilled water (n = 1.33) and benzene (n = 1.50), which immediately separate into two layers. If light is then shone into this beaker from multiple directions, which of these statements is accurate?

When a ray initially moving through the water hits the benzene at a 76° angle to the normal, its angle of refraction will be smaller than 76° because light travels faster in water than in benzene. When light travels from a medium with a relatively low index (water) to one with a higher index (benzene), the ray will bend toward the normal. For this reason, its angle of refraction will certainly be smaller than 76°. Additionally, a lower value for "n" implies that light travels more quickly through the associated medium. Thus, light moves faster through water than through benzene.

Which scenario provides the strongest evidence for the concept of a sensitive period with regard to language development?

When a young girl is isolated from human interaction throughout her childhood, she is unable to fully acquire the syntax of the language spoken in her community later in her life. C is correct. The idea of sensitive developmental years is a part of the nativist theory of language acquisition. Specifically, the years in question span from an age of two until puberty. During this time, children are able to develop a mastery of language and the basic rules of syntax via exposure to human interaction. If a child was deprived of this social contact, he or she would never be able to gain a full grasp of language. In fact, this does occur in cases of extreme abuse or neglect.

The social desirability bias is a cognitive tendency that impacts the design of many research studies, particularly those which contain surveys or questionnaires. How can this bias best be described?

When asked personal questions, individuals typically respond in a way that exaggerates their own positive attributes. The social desirability bias refers to the tendency of individuals to respond to questions in a way that makes themselves look good. For example, a woman with a relatively low income might inflate this value when asked her salary, or a five-foot-eleven man may state that he is six feet tall.

Which of the following examples demonstrates the Gestalt principle of good continuation?

When observing two intersecting lines of different colors, instead of noticing four separate lines, we perceive two. The principle of good continuation explains that we tend to perceive two or more objects as separate, even when they intersect.

Which of the following scenarios does NOT produce a form of light interference?

White light that separates into its color components upon traveling through a glass prism When white light passes through a prism, the colors are simply spread out, or dispersed, according to their own refraction in the material. No interference occurs.

refers to the immune response that does not involve antibodies

cell-mediated immunity

A neuroscientist observes that treatment of temporal neurons with a certain toxin causes a depolarization of approximately 8 mV as compared to the resting value, but no action potential. He hypothesizes that an action potential would be caused by the opening of additional voltage-gated sodium channels. Is this explanation reasonable, and if not, why?

Yes; this explanation makes sense. The opening of voltage-gated Na+ channels would allow sodium ions to enter the neuron. Such an action certainly should depolarize the cell. Furthermore, if only a small number of channels were opened, this effect would likely not be large enough to reach the threshold value. (Note: Be very careful when reading this question stem! The question stem did NOT state that the membrane potential reached +8 mV; it only said that it depolarized by 8 mV compared to the resting value. For example, if the resting potential were -70 mV, this depolarization would bring the potential to -62 mV.)

Carbonic anhydrase catalyzes the conversion of water and carbon dioxide to carbonic acid. This process is a crucial part of the maintenance of a buffered blood pH. Which metal cofactor is necessary for carbonic anhydrase to function?

Zinc Carbonic anhydrase is a metalloenzyme that catalyzes the reversible reaction between carbon dioxide and water. The active site of this enzyme contains a zinc molecule.

What is the balanced equation for the oxidation of zinc by hydrochloric acid?

Zn (s) + 2HCl (aq) 🡪 ZnCl2 (aq) + H2 (g) C is correct. When zinc is oxidized, it loses two electrons to become Zn2+. Correspondingly, hydrogen is reduced from H+ to H in a process requiring two moles of H+ for each mole of Zn.

Zn has an oxidation potential of +0.76, while Al has an oxidation potential of +1.66. Which metal serves as the better oxidizing agent?

Zn2+, because it is more prone to gaining electrons The better oxidizing agent is the metal that is more likely to reduce itself (in other words, it oxidizes a different species). Here, Zn has a less positive oxidation potential; in other words, it is less likely to oxidize than Al. However, we need to switch the signs of the reactions to find their reduction potentials. Zn2+ thus has a reduction potential of -0.76, while Al has a reduction potential of -1.66. Zn2+ is the better choice for an oxidizing agent.

While the Nernst equation can be written in a variety of ways, one particularly biologically relevant iteration is shown below: Here, Vm represents the equilibrium membrane potential (in mV) for a particular ion, while z represents the valence of that ion. In neurons, the overall equilibrium potential is determined almost solely by potassium cations. Which set of conditions will yield a membrane potential closest to its actual resting value in humans?

[K+outside] = 10 mM; [K+inside] = 149 mM In humans, the resting membrane potential of a typical neuron is approximately -70 mV. For this reason, and since K+ has a valence of +1, we know that the log term in the above equation must be negative. Therefore, [K+inside] needs to be larger than [K+outside]. Furthermore, if the internal potassium concentration were exactly ten times the external [K+], the value for Vm would be -61 mV. As we are looking for a potential that is even more negative, [K+inside] must be relatively larger. Choice A displays a [K+inside] value that is around 15 times greater than [K+outside], which resembles values found in vivo.

In normal human extracellular fluid, the concentration of Na+ is approximately 140 mEq/L. A beaker of water is filled with a solution with exactly that sodium concentration. If a water-permeable synthetic cell is dropped in the beaker, what minimum NaBr concentration must it have to gain water from its environment?

[NaBr] > 70 mEq/L C is correct. For a water-permeable (but not solute-permeable) cell to gain water via osmosis, its contents must be hypertonic relative to its environment. In other words, it must include relatively more solute. Here, the extracellular fluid has a sodium ion concentration of 140 mEq/L, meaning that any cell with a larger ion concentration should absorb water. However, NaBr is virtually completely soluble in water, so 70 mEq/L of NaBr will dissociate into 140 mEq/L of Na+ and Br- ions.

In normal human extracellular fluid, the concentration of Na+ is approximately 140 mM. A beaker of water is filled with a solution with exactly that sodium concentration. If a water-permeable synthetic cell is dropped in the beaker, what NaCl concentration can it have and still lose water to its environment?

[NaCl] < 70 mM For a water-permeable (but not solute-permeable) cell to lose water via osmosis, its contents must be hypotonic relative to its environment. In other words, it must include relatively less solute. Here, the extracellular fluid has a sodium ion concentration of 140 mM, meaning that any cell with a smaller ion concentration should leak water. However, NaCl is virtually completely soluble in water, so 70 mM of NaCl will dissociate into 140 mM Na+ and Cl- ions.

In aqueous solution, copper ions can react with water molecules to form a vibrant blue complex. This takes place according to the reaction below: Cu2+ + 6 H2O → [Cu(H2O)6]2+ In this reaction, H2O acts as:

a Lewis base. Since this reaction involves the formation of a bond, we are dealing with the transfer of electrons, not the movement of a proton from one molecule to another. For this reason, the above reaction involves the Lewis definition of acids and bases, not the Brønsted-Lowry one. Specifically, water (which has two available lone pairs) is able to donate its electrons to copper (which, as a cation, is able to accept electrons). Electron-donating species are Lewis bases.

The ductus arteriosus is a fetal circulatory adaptation that shunts blood directly from the pulmonary artery to the aorta. In a fetus, this is beneficial, as it bypasses underdeveloped lungs that are not yet required for gas exchange. However, many babies are born with a condition known as "patent ductus arteriosus," in which this shunt remains open after birth. When examining a 3-month-old male with this condition, one would expect to observe:

a PCO2 in the aorta that is higher than expected. In adults and healthy infants, the pulmonary artery carries deoxygenated blood from the heart to the lungs to undergo gas exchange. If this low-O2, high-CO2 blood is allowed to mix with aortic blood, the main adverse consequence will be a lowering of oxygen concentration in the aorta. Additionally, the high levels of CO2 from the pulmonary artery will likely spread to the aorta, which normally contains minimal amounts of carbon dioxide.

A researcher wants to use ELISA to measure the amount of a certain antigen in a blood sample. This procedure will require: I. SDS. II. a reducing agent. III. a capture antibody. IV. an agarose gel.

a capture antibody. ELISA is a fairly simple procedure that tests for antibody-antigen interactions. Unlike SDS-PAGE, it does not involve sodium dodecyl sulfate as a reagent, and unlike regular gel electrophoresis, it does not involve an agarose gel.

Vacuum distillation aids in the separation of compounds because:

a compound will boil when its vapor pressure is equal to ambient pressure. B is correct. Boiling point is defined as the temperature at which a compound's vapor pressure is equal to the atmospheric (or ambient) pressure. We typically think of heating as the only way to cause a substance to boil, but according to this relationship, an alternate method involves lowering the ambient pressure. This can be achieved through vacuum distillation, and is typically used to separate compounds that would otherwise be very difficult to boil.

A separatory flask contains two layers: water and diethyl ether. Aniline, benzoic acid, and phenol are dissolved in the ether layer. To isolate the benzoic acid, a student should add:

a dilute solution of ammonia, dropped into the flask slowly. Benzoic acid is an acidic compound. To separate an acid from other molecules dissolved in the organic layer, we must use a base to deprotonate it. As a charged (and thus very polar) compound, our deprotonated acid will then move into the water layer where it can be isolated. However, both choices A and C include bases. Note, though, that phenol - another compound present in the organic layer - is weakly acidic. Adding large amounts of strong base risks deprotonating the phenol, allowing it to move into the water layer with benzoic acid. If we instead add a weak base very slowly, we minimize this risk and will end up with a more pure sample of our desired compound.

Nickel-cadmium (NiCd) batteries are rechargeable cells used in products ranging from remote-control toys to airplanes. An NiCd battery in the process of discharging is most analogous to:

a galvanic cell. In essence, rechargeable batteries work via reversible redox processes. When such a cell is discharging, it is releasing stored energy; this is similar to the spontaneous process seen in galvanic cells.

Dendritic cells are often credited as being the link between the innate and adaptive immune systems. These cells are capable of sampling the membranes of somatic cells using a process called "nibbling." The dendritic cell then carries the cellular sample to the nearest lymph node and presents the antigen to another immune cell, in a process that involves direct membrane contact and utilizes MHC. Assuming the sampled cell was subject to a novel intracellular infection, the dendritic cell most likely interacts with:

a helper CD4+ T cell. A is correct. Dendritic cells present antigens, in the context of MHC, to immature CD4+ T cells. The T cell will become activated if it has affinity for the antigen/MHC presented by the dendritic cell. An activated CD4+ T cell will produce a unique cytokine profile to regulate other immune cells and facilitate an effective response to the pathogen.

During the synthesis of a biologically relevant macromolecule, a chemist realizes that he has produced the thermodynamic enolate. In comparison to the kinetic enolate, this molecule has:

a higher activation energy, but is more stable overall. A is correct. This question deals with kinetic vs. thermodynamic control, two competing pathways by which certain reactants can produce multiple distinct products. The thermodynamic product is the one that is more "thermodynamically" stable, but is more difficult to synthesize due to a higher activation energy. This product is favored by energy-rich conditions, such as high temperature. In general, the thermodynamic enolate is that which is more substituted - it is more stable once produced, but steric hindrance gives its synthesis reaction a high Ea.

A long-standing pharmaceutical challenge has been the effect of so-called "first pass metabolism" on oral medications. Blood from the small intestine travels directly to the liver via the hepatic portal system, and many drugs are extensively modified before they can reach their targets and exert desirable therapeutic effects. To circumvent this physiological roadblock, an intrepid pharmaceutical chemist might elect to cross-link an oral drug to:

a long-chain fatty acid incorporated into a triglyceride. B is correct. The question states that blood from the intestine travels to the liver, where it is subject to enzymatic modification. Amino acids, carbohydrates, and most other small molecules enter portal circulation after absorption in the small intestine. Triglycerides, in contrast, are absorbed via intestinal lacteals. Free fatty acids enter the lymphatic system directly, and are therefore exempt from first pass metabolism. A drug attached to a fatty acid is more likely to maintain its molecular integrity long enough to have a therapeutic effect.

The following image depicts an alcohol thought by researchers to have potential pharmaceutical applications. Treatment of this molecule with methanol under acidic conditions would most likely yield:

a mixture of diastereomers. D is correct. The molecule shown has two chiral centers, but only one (the carbon bound to the -OH group) represents a viable site for nucleophilic substitution. Therefore, the other center will remain unchanged under the described reaction conditions. Since the question stem mentions a weak nucleophile (methanol) while the diagram shows a potential tertiary carbocation, we can conclude that this reaction will occur via an SN1 mechanism, yielding products that are racemic with respect to that chiral center. However, since the other stereocenter remains unaltered by the reaction, the two resulting products will be diastereomers rather than enantiomers.

The following image depicts an alcohol thought by researchers to have potential pharmaceutical applications. Treatment of this compound with sulfuric acid and methanol should result in:

a mixture of products, not all of which are isomers. Treatment of a tertiary alcohol with a weak nucleophile under acidic conditions is a perfect method by which to form a carbocation. However, the carbocation derived from this particular molecule can potentially undergo either elimination or substitution. Elimination will produce multiple products that are structural isomers, while substitution will form products that are stereoisomers. However, the elimination and substitution products will not be isomeric with respect to one another.

The molecule shown below is treated with a peroxyacid. Assuming that the starting material is completely enantiopure, subsequent treatment with potassium tert-butoxide would yield:

a mixture of two diastereomers. This reaction first involves the formation of an epoxide. Such reactions proceed without stereochemical bias, yielding an equal mixture of stereoisomeric products with respect to the site of epoxide formation. The carboxylic acid is then deprotonated and is subsequently able to open the epoxide ring by attacking the less sterically hindered side. Here, the opening of the epoxide leaves an intact chiral center that was not present in the original molecule. Since the epoxidation resulted in the formation of two diastereomers, and since the ring-opening reaction did not change any stereochemistry, the final products are best described as a mixture of two stereoisomers. Note that the single chiral center originally present in the molecule remains untouched.

Ion-exchange chromatography with a positively-charged stationary phase is used to separate two polypeptides, A and B. Which of the below amino acids is LEAST likely to abound in the polypeptide that elutes first?

a negatively charged amino acid D

Though Anna has been diagnosed with a psychological disorder, she feels completely normal and even perceives some of her symptoms as "just how she is." Anna most likely has:

a personality disorder, since many tend to be ego-syntonic. A is correct. Ego-syntonic disorders are those that align with the patient's self-concept. In other words, instead of feeling sick or abnormal, the patient feels good and may even enjoy her disease, considering it to be part of her usual behavior. A number of personality disorders, as well as anorexia nervosa, are classic examples of ego-syntonic conditions.

The following reaction is useful for the decarboxylation of esters under certain synthetic circumstances. Treatment of the starting material with sodium hydride would most immediately yield:

a resonance-stabilized, tertiary carbanion. Sodium hydride is most notable as a strong base. The most acidic site on the starting molecule is the tertiary carbon between the two carbonyl groups, a position that serves as the alpha carbon for two distinct carbonyl functionalities. Resonance stabilization of the conjugate base renders this site much more acidic than would normally be expected of a carbon atom. Sodium hydride would be more than adequate for the abstraction of a proton from this position.

If you wish to promote an E1 reaction, you should use:

a tertiary substrate, a mild base, heat, and a polar protic solvent. E1 reactions are unimolecular eliminations. These reactions are analogous to SN1 mechanisms, as both are first-order and involve the formation of a carbocation. As such, both are heavily favored by tertiary reagents. Additionally, use of a mild or weak base tends to push a reaction toward E1, as E2 reactions (which are bimolecular) require moderately strong basic species. Finally, like SN1 processes, E1 reactions prefer protic solvents for carbocation stabilization.

A salt solution with a density of 1075 kg/m3 is moving through two consecutive closed vessels. Both vessels are positioned at the same height, but the solution travels twice as quickly through one (Vessel A) as the next (Vessel B). If the pressure exerted by the fluid on the walls of Vessel A is 6000 Pa, the pressure exerted on Vessel B is:

a value that cannot be determined. D is correct. Let's use Bernoulli's equation for this problem: PA + ρghA + ½ ρv2A = PB + ρghB + ½ ρv2B. Since both vessels are placed at the same height, we can drop the ρgh terms to obtain 6000 Pa + ½ ρv2A = PB + ½ ρv2B. Even if we simplify further and plug in our solution density, we see that we must know at least one of the velocity values to properly find this answer. Velocities of 2 m/s and 1 m/s yield a very different value for PB than velocities of 2000 m/s and 1000 m/s, for example.

Ras is a small protein with a structure similar to the alpha subunit of a typical heterotrimeric G protein. Ras, which is involved in cellular growth and cell cycle procession, is regulated in part by the actions of NF-1. NF-1 is a tumor suppressor and a member of a class of molecules known as GTPase-activating proteins (GAPs). Loss-of-function mutations in NF-1 are associated with the development of von Recklinghausen syndrome, a tumor disorder. In patients with von Recklinghausen syndrome, Ras activity is most likely:

abnormally high, because NF-1 normally promotes Ras inactivation. A is correct. Since we are told that Ras resembles a typical G protein alpha subunit, it is reasonable to assume that it binds and hydrolyzes GTP in a similar manner. In short, Ras is active when GTP is bound and inactive when attached to GDP. Since NF-1 is a GTPase activator, it must trigger the hydrolysis of GTP and the subsequent inactivation of Ras. Therefore, if NF-1 is inactivated, Ras will remain active longer than it normally should.

A complex multi-ring compound is generated during the course of a synthetic pathway. Suppose the chemist responsible for synthesizing this species is concerned about possible side reactions involving the naked alcohol moieties. While leaving the rest of the molecule untouched and causing limited complications, protection of these groups would be most effectively achieved with:

acetyl chloride and pyridine. B is correct. Acetyl chloride, though still not perfect, is the most viable option for a protecting group presented. Importantly, acetylation of the free alcohol groups can be achieved without the need for acid catalysis that could damage the ketal. Generation of acidic conditions in situ should be effectively prevented by the presence of pyridine, which forms an insoluble precipitate with chloride.

In gluconeogenesis, all of the following molecules can be directly utilized EXCEPT:

acetyl-CoA. Acetyl-CoA cannot be directly incorporated into gluconeogenesis, nor can it be converted to oxaloacetate in human cells.

Small intracellular G proteins, such as Ras and Rho, are regulated by a complex system of upstream signaling proteins. One such protein is called Son of Sevenless (SOS), which acts as a guanine nucleotide exchange factor (GEF) for Ras, promoting the ejection of GDP. Co-localization of Ras and SOS via the action of Grb2 will likely result in:

activation of Ras. G proteins are only active when GTP is bound, a state that necessitates the release of GDP. Since we are told that SOS promotes GDP release, it must activate Ras. In many signaling cascades, co-localization alone is sufficient to promote interaction between relevant proteins. Though we do not need to know this, Grb2 is purely a scaffolding protein that facilitates the interaction of the other two molecules.

The first IE value is very small, indicating that it is very easy for this element to lose a single electron. However, the second ionization energy is higher than any other, implying that removing a second electron is energetically unfavorabl

alkali metals

on a graph low first and second ionization energies then increase

alkaline earth metals

Ketals make excellent protecting groups. This is true in part because they are extremely resistant to nucleophilic attack under basic or neutral conditions. The best explanation for this property is that:

alkoxide ions constitute relatively poor leaving groups. Ketal hydrolysis under basic conditions would necessitate the loss of an alkoxide leaving group, which is relatively unfavorable. For this reason, acid-catalyzed ketal hydrolysis is much more rapid. Alcohols make much more suitable leaving groups than unstable, negatively-charged alkoxide ions.

A protein's sedimentation coefficient, measured in Svedbergs, depends upon:

all the levels of protein structure. C is correct. Sedimentation coefficient basically measures the efficiency of a particle's migration down a tube during centrifugation. Therefore, both the shape and mass of a particle are significant. This explains why sedimentation coefficients are not additive. A protein that is comprised of two subunits does not necessarily have a sedimentation coefficient that is the sum of the subunits' coefficients.

The figure below gives the structure of linolenic acid. According to IUPAC rules, an alternative name for this fatty acid is:

all-cis-9,12,15-octadecatrienoic acid. When naming a fatty acid, number the carbon atoms starting at the carbonyl. We can find the prefix from the total number of carbons in the chain (18 carbons = "octadec-"), while "-oic acid" is always used as the suffix. For unsaturated fatty acids, we must specify both the nature (cis vs. trans) and the location of the double bond(s); since all bonds span two carbons, remember to use the lower number. Finally, "-trien-" is inserted in between the prefix and suffix to designate the three alkenes shown

Scientists at a clinical research lab are particularly interested in the following carbohydrate. Immediately following an initial proton transfer, acid-catalyzed hydrolysis of this disaccharide will proceed through:

an sp2-hybridized oxonium intermediate. B is correct. After protonation of the bridging oxygen atom, electrons from the left-hand ring oxygen will move down to displace this protonated atom. This creates an intermediate featuring a positively-charged oxygen with a bond order of three. By definition, this is an oxonium ion.

Consider the course of a typical SN1 reaction. The intermediate generated by the rate-limiting step is best described as:

an unstable intermediate with vacant orbitals. The intermediate mentioned in the question stem is the carbocation, a species with a positively-charged carbon atom. Carbocations, though unstable, are intermediates, not transition states; as such, they can be observed using typical chemical means. Finally, remember that carbon atoms tend to bind to four species. As a carbocation has only three substituents and no lone pairs, it does not have full orbitals.

Some bacterial toxins are able to bind with high affinity to MHC, displacing whatever peptide was previously bound. As a result, the immune response is systemically activated without a clear target. The resulting pathology would most resemble:

anaphylaxis. The question describes a systemic reaction in which the immune system responds inappropriately to all cell types that express MHC (essentially, all somatic cells). This closely resembles anaphylaxis, an extreme systemic allergic reaction that can be fatal.

is the process by which a person practices for future social relationships. This form of socialization is not relevant to the scenario described.

anticipatory socialization

Which of the following will NOT cause precipitation to occur in a saturated solution of lead (II) chloride?

anything that does not share common ions will not effect the solution

A is correct. The titration curve displays three pKa values (the points halfway along the flat parts of the curve), representing three groups that can potentially be deprotonated. This indicates that the amino acid in question must be either acidic or basic. Note that pKa1 and pKa2 are both in the acidic range. As an amine-based group could not lose its proton at such a low pH, the amino acid's side chain must be a carboxylic acid. Our answer, then, can only be

aspartic or glutamic acid

Imipenem is a β-lactam antibiotic with a number of uses, including the treatment of Pseudomonas infections. The structure of imipenem must contain:

at least one heterocyclic ring. C is correct. Since this compound is likely unfamiliar, we must use its name to discern its chemical structure. The described antibiotic is said to be a lactam, or cyclic amide. Thus, its structure contains at least a single ring with an incorporated nitrogen atom.

A student in an organic chemistry lab grabs a bottle that supposedly contains an aqueous solution of ammonium phosphate. Unfortunately, it is not until later that he notices a thick precipitate lining the sides of the container. The solution was most likely contaminated with:

barium nitrate. Since ammonium always forms soluble salts, we're looking for a substance that forms a precipitate with phosphate. Barium cation is a perfect example of such a substance, as it would create barium phosphate, an insoluble compound.

In Huntington's disease, cells produce a defective form of protein known as huntingtin (Htt) that contains a long chain of glutamine amino acids. This chain interferes with the normal folding of the protein and results in the formation of Htt plaques in many neurons, causing cell death; however, certain neuron types are more vulnerable than others. One of the first areas affected is the striatum, which is part of the darker central structure pictured below. Early signs of Huntington's disease include jerky, random motions known as chorea, which later progress to more severe writhing motions and difficulties with eating, chewing, and speaking. From this information, the striatum is most likely part of the:

basal ganglia. D is correct. The description of the symptoms of Huntington's disease includes impairment in the smoothness of movement, a process in which the basal ganglia are heavily involved.

A chemistry student notices that, at 273 K and 1.01 × 105 Pa of pressure, the Ksp value for copper (II) hydroxide is 2.2 × 10-20. If he wished to decrease this value, he could: I. add more Cu(OH)2 to his reaction vessel. II. add sodium hydroxide, a strong base, to the solution. III. change the temperature of his flask to 220 K. IV. lower ambient pressure to 1 atm.

change the temperature of his flask to 220 K.

In fluid dynamics, the continuity equation relates the cross-sectional area of a vessel to the speed of fluid moving through it. According to this equation:

blood travels slower in capillaries than in arterioles. The continuity equation is often written as A1v1 = A2v2. This shows us that fluids travel slower through vessels with larger areas. However, we cannot compare the area of a single capillary to that of a single artery or vein, since one artery branches off into many arterioles, which then each branch to form countless capillaries. Thus, it is the combined area of all capillaries - a huge value - that helps us realize that capillaries experience the slowest blood flow of all circulatory vessels. Alternatively, we could simply remember that blood must travel slowly through capillaries to maximize gas and nutrient exchange.

Synaptotagmin is a presynaptic protein that has been found to be essential in the fusion of intracellular neurotransmitter-laden vesicles with the presynaptic membrane. Synaptotagmin directly responds to the specific changes in local cytoplasmic conditions that occur when an action potential reaches the axon terminal, triggering the rapid and reliable fusion of docked vesicles with the plasma membrane. Structurally, synaptotagmin is most likely analogous to:

calmodulin. The arrival of an action potential at the axon terminal triggers a rapid influx of calcium ions, which then initiates a number of critical neurotransmission events. This increase in intracellular calcium is the "change in cytoplasmic conditions" to which the question is referring. If synaptotagmin must directly respond to the presence of intracellular calcium, it must contain some sort of calcium-binding domain. The only protein listed in the answer choices that also binds calcium is calmodulin. It is most reasonable to assume that these two proteins may share structural similarities.

Operant and classical conditioning differ in that classical conditioning:

can convert a neutral stimulus into a conditioned one. A is correct. Classical conditioning involves involuntary actions, while operant conditioning involves conscious, voluntary ones. Specifically, classical conditioning pairs a neutral stimulus with an unconditioned one, like pairing the ringing of a bell with the presentation of food. Over time, the neutral stimulus becomes "conditioned" and elicits the same response as the unconditioned stimulus (e.g., dogs begin salivating when the bell is rung).

When an individual quickly moves from a warm to a cold environment, the body will cope in all of the following ways EXCEPT:

capillaries in the skin will constrict to reduce the flow of blood near the cold environment. While vasoconstriction is certainly a mechanism by which the body aims to remain warm, the actual constriction is performed by arteries and arterioles, not capillaries. Remember, capillary walls consist only of a single endothelial layer. Since they lack smooth muscle entirely, they cannot possibly constrict or expand.

Mr. Jones achieves a good position as a municipal service worker. However, no matter how hard his children try, they are only allowed to reach the same level of seniority that he has attained. In fact, if they try to climb further up the social ladder, society will disallow them from seeking further education. Mr. Jones and his family probably live in a:

caste system. C is correct. A caste system is marked by strict constraints on the movement between social classes. In the vast majority of cases, then, one is born into the same class that one will die in. If it is true that, no matter how hard Mr. Jones' children work, they can only ever reach a certain social standing, this society is almost certainly a caste system.

Succinylcholine, a charged, organic ester that is structurally analogous to acetylcholine, binds competitively to the nicotinic acetylcholine receptor without antagonistic activity. It is used in a hospital setting as a paralytic agent to facilitate procedures such as endotracheal intubation. When administered intravenously, succinylcholine most likely:

causes measurable postsynaptic depolarization and is degraded by acetylcholinesterase.

An opiate overdose slows the respiratory drive, resulting in decreased gas exchange across the alveoli. This can damage tissues or cause death from:

cell damage resulting from reduced ATP levels. If the respiratory rate slows, the transport of oxygen to tissues will decrease as well. In such conditions, cells cannot generate ATP through mitochondrial respiration, though they can still use anaerobic processes such as glycolysis and fermentation. Therefore, opiate overdose leads to a decrease in ATP concentration in the cells of vital organs, inhibiting their function.

A genetic disorder that causes a deficiency in cell membrane cholesterol levels would result in:

cell membranes that are too rigid and lack fluidity. In eukaryotic cells in vivo, the role of cholesterol in the membrane is to provide fluidity within the otherwise rigid phospholipid structure.

p-chlorophenol can most easily be synthesized by reacting phenol with:

chlorine in carbon disulfide. The process described is the halogenation of a phenol. When deciding which solvent to use for such a procedure, make sure to distinguish between monohalogenation and polyhalogenation. Here, since we are attempting monohalogenation, we should use a nonpolar solvent, such as carbon disulfide.

An ether is synthesized according to the diagram below. If the product of this reaction is then treated with hydrochloric acid, the resulting products will be:

chloromethane and benzyl alcohol. The acid will protonate the ether oxygen atom, allowing the chloride anion to attack the terminal carbon. The subsequent substitution reaction will displace the protonated oxygen along with the benzyl group to which it is bound. This process yields free benzyl alcohol and chlorinated methane.

Nicotinamide adenine dinucleotide (NAD) plays an important role in cellular respiration. In its reduced form, this species can transfer hydrogen to protein complexes that generate the H+ gradient required for ATP synthesis. NAD can be most specifically described as a:

coenzyme. Coenzymes are a subset of cofactors that tend to bind loosely totheir associated enzymes. This type of molecule is also known fortransferring functional groups between species. NAD serves this functionby donating its hydrogen to Complex I, which is one of the four proteincomplexes responsible for generating the proton gradient during aerobiccellular respiration.

She would begin to understand the perspectives of other people.

concrete operational stage

Isopentane and n-pentane are best described as:

configurational isomers. A is correct. Configurational and structural isomers are synonymous. Both isopentane and n-pentane contain five carbons and twelve hydrogens, but these atoms are bonded differently.

A researcher is interested in discovering how patients' varying access to social and material resources affects their decisions about healthcare. Furthermore, she wishes to examine how these choices result in positive or negative outcomes for groups of patients with similar resources. This study's approach seems most closely related to:

conflict theory. Conflict theory focuses on the resources that groups in society (like races or classes) have, as well as the material and social effects of these disparities on their lives. In this case, the researcher appears to be examining the effect of socioeconomic status on the decision-making process and outcomes of healthcare recipients.

Aplysia californica is a species of sea hare that was extensively studied by Eric Kandel in his experiments on habituation. One member of this species is tapped repeatedly with a pencil; each time, it pulls both its gills and siphon (a body part resembling a tube) away from the researcher. If the scientists wish to diminish this response, they should:

continue tapping the animal after fixed intervals of time. The scientists want to diminish, or reduce, the animal's response to the tapping stimulus. Habituation is a decrease in response after the same stimulus is administered multiple times. To accomplish their goal, the scientists simply need to continue tapping the sea hare in exactly the same way until the response becomes habituated.

Reducing enzyme concentration will

decrease the Vmax of the enzyme-catalyzed reaction

Absolute thresholds:

depend on the energy necessary to stimulate a sensory receptor to trigger an action potential. Absolute threshold is the smallest level of energy required by an external stimulus to be detectable by the human senses, including vision, hearing, taste, smell and touch.

Thyroxine is a hormone produced by the thyroid in a reaction that uses a tyrosine substrate. This hormone affects virtually every cell in the body by binding to its cytosolic receptor and translocating into the nucleus. Compared to steroid hormones, thyroxine:

differs in its tissue of origin, but shares a similar mechanism of action. As far as the scientific community is aware, the thyroid does not produce any steroid hormones. However, both steroid hormones and thyroxine act by binding cytosolic receptors and entering the nucleus to alter gene transcription.

All of the below conditions are reasons for a cell to enter the G0 phase

lack of growth factors a decreased presence of cyclin-dependent kinases a state of terminal differentiation

Anticholinergic drugs serve to block parasympathetic activity in the body. A patient on several powerful anticholinergic medications would be expected to experience:

dry mouth. Parasympathetic innervation is involved in "rest and digest" functions. Logically, blocking such eating-related responses would dry the mouth out by inhibiting salivation.

Narcissistic personality disorder is characterized by a tendency toward self-aggrandizing behavior designed to feed the sufferer's need for admiration. Individuals with this disorder have trouble seeing the impact of their behavior on others. Relationships are mainly viewed in terms of their effect on the narcissist's self-image, as opposed to genuine interest in the needs or wants of the other person. In other words, those with NPD tend to deal with others in way that lacks:

empathy

In the order that they occur, the three main stages of memory are:

encoding, storage, retrieval. D is correct. The first stage of memory is encoding, or the initial adaptation of a piece of information into a memory. The second stage, storage, involves the preservation of that encoded fragment in a memory system (short-term, long-term, etc.). The final stage is retrieval, in which these stored memories are accessed again by the person who formed them.

The G2-M checkpoint:

ensures that the DNA has been replicated accurately.

A young boy has a genetic defect that impairs the synthesis of keratin filaments. This condition is likely to most severely impact the structural integrity of:

epithelial tissue Epithelial tissue is linked by intermediate filaments (specifically keratin) that pass through the cells from side-to-side. The filaments of one cell are indirectly linked to those of another in structures known as desmosomes. This network of fibers helps to distribute mechanical stress among the cells. Other tissues do not rely on keratin to alleviate stress in this manner, and would not have their structural integrity disrupted so heavily.

The loop of Henle functions to:

form a solute gradient in the medulla of the kidney, allowing for the later movement of water through channels in the collecting duct. The loop of Henle is likely one of the most confusing aspects of MCAT-level human physiology. While many students think that the loop actually concentrates urine itself, it simply prepares the medulla of the kidney for the later absorption of water. Specifically, the antiparallel nature of the loop establishes a countercurrent multiplier system that makes the inner medulla very solute-rich. Interestingly, the collecting duct also runs parallel to this long structure. When ADH is present, as in periods of dehydration, the collecting duct is made water-permeable through the introduction of aquaporin channels. Since the medulla is now so salty, water can passively exit the nephron and remain in the body.

After digestion of a piece of cake that contains flour, milk, and sucrose as its major ingredients, the primary carbohydrate products entering the blood are:

glucose, fructose, and galactose. Sucrose is composed of glucose and fructose, and lactose (from the milk) consists of glucose and galactose.

A bird wing and a human arm are an example of _____ structures.

homologous Homologous structures are similar in anatomy, but may differ in function.

Which of these are classifications of water-soluble vitamins? I. Prosthetic groups II. Coenzymes III. Antioxidants IV. Carotenoids

prosthetic groups, coenzymes, and antioxidants

In regions of the world where malaria is endemic, it has been observed that the frequency of the sickle cell allele is significantly higher than in other regions, despite the reduced survival rate for individuals born homozygous recessive (with the disease) in these regions. The persistence of the allele in these regions is likely due to:

hybrid vigor, or heterosis. A is correct. While sickle cell disease is a dangerous affliction that can be deadly, sickle cell trait (being heterozygous for the trait) confers some survival advantages by increasing resistance to malaria. With malaria present as a selection pressure, those with resistance will be more likely to survive and pass on their trait. Thus, a recessive gene that is dangerous (or even fatal) in homozygotes can persist due to the advantage of being heterozygous. This is called hybrid vigor or heterosis.

The structure of sucrose, a dimer of glucose and fructose, is shown in the following figure. The unique nature of the glycosidic linkage present in sucrose renders it resistant to hydrolysis, unlike that of other common disaccharides. Sucrase, a brush-border enzyme, rapidly increases the ordinarily sluggish degradation of sucrose into its component monosaccharides. Alternatively, sucrose may be hydrolyzed using:

hydrochloric acid. B is correct. The glycosidic linkage shown above is an acetal. All acetals are acid-sensitive.

A student needs to make a buffer solution at a pOH of 4.80. Of the following acids, the student should use:

hydrocyanic acid (Ka = 6.2 × 10-10). To manufacture a buffer, an acid should be selected with a pKa as close to the desired pH as possible. However, watch out - this question asks about a pOH of 4.80, which is a pH of 9.20. Also, the given values are Kas, not pKas. To convert, remember that pKa = -log(Ka). We can estimate negative logs by finding the values that each of these Kas falls between. For hydrocyanic acid, 6.2 × 10-10 is between 1 × 10-10 and 1 × 10-9, making its pKa between 9 and 10.

Of the following choices, the most effective buffer solution could be formed by mixing equal moles of:

hydrofluoric acid and potassium fluoride. Hydrofluoric acid (HF) is a weak acid, and F- is its conjugate base. Equimolar amounts of a weak acid and its conjugate will form an ideal buffer.

Alpha helices display a measurable molecular dipole along their long axes. This property results from:

hydrogen bonding between backbone carbonyl oxygen atoms and protonated nitrogen atoms separated in sequence by several amino acid residues.

One of the roles of the kidneys is the maintenance of acid-base balance. All of the following statements regarding pH regulation by the kidneys are inaccurate EXCEPT:

hydrogen ions are secreted into the urine while bicarbonate is reabsorbed. This is an accurate statement. The kidneys have two major roles in the maintenance of acid-base balance: the reabsorption of bicarbonate as well as the excretion of hydrogen ions.

With regard to human skin, another name for the subcutaneous layer is the:

hypodermis. The name given in the question stem provides a major hint. Remember, the prefix "sub" means "below," so we are looking for a lower, or more interior, layer of the skin. The hypodermis is the most interior layer and consists mostly of fat deposits.

According to Freud, the energy from life instincts that drives personality is called the:

ibido. Libido is a term used by in psychoanalytic theory to describe the energy created by the survival and sexual instincts. According to Sigmund Freud, the libido is part of the id and is the driving force of all behavior.

Epigenetic modification is one method by which environment can influence the behavior of an organism. A way in which such a modification could influence behavior is:

in response to stress, certain genes are methylated and "turned off." C is correct. DNA methylation represents one of the most well-studied epigenetic modifications. The term "epigenetics" refers to non-heritable changes in expression that do not change the sequence of DNA itself.

During UV-Vis spectroscopy, an electron that has absorbed a photon of sufficient energy will be found:

in the LUMO only. When an electron absorbs a photon of sufficient energy, it moves from the highest occupied molecular orbital (HOMO) to the lowest unoccupied molecular orbital (LUMO), a higher-energy position.

Monoamine oxidase inhibitors (MAOIs), an older class of antidepressant medications, have largely been replaced by more modern selective serotonin reuptake inhibitors (SSRIs), which act on a specific serotonin transporter. However, MAOIs are still used in the treatment of hypokinesia associated with Parkinson's disease, a condition caused by the loss of dopaminergic neurons in the CNS. Both SSRIs and MAOIs:

increase extracellular concentrations of monoamines, such as dopamine and serotonin.

The reaction below is part of the citric acid cycle and is mediated by the enzyme aconitase. Michaelis-Menten kinetics can be used to determine initial reaction velocity. All of the following changes will result in a decreased initial velocity for this reaction

increase in Km decrease in Vmax decreased concentration of citrate

Inhibin is a small peptide hormone secreted by the gonads that inhibits the production of FSH via direct action on the pituitary. Males with temporarily elevated inhibin levels are most likely to display:

increased GnRH production. The question states that inhibin acts directly on the pituitary, meaning that it does not decrease FSH production by inhibiting GnRH. Therefore, FSH levels in the blood will be low, which will cause an increase in GnRH secretion due to negative feedback.

The internal region of the bacterial K+ channel contains a large amount of water. Additionally, crystallographic measurements indicate that its dimensions are sufficient to permit potassium ions to pass through most of the channel in a hydrated state until they reach the extracellular side. This feature most likely:

increases the speed of channel conduction by reducing the energetic penalty associated with dehydration. Only a small section of the channel is narrow enough to strip the ions of their hydration shells. This reduces the energetic penalty associated with passing a charged particle through a hydrophobic environment, such as the interior of the bilayer.

A congenital defect in the synthesis of the proteins cloudin and occludin is most likely to impact the function of the:

intestinal lining. Cloudin and occludin are the proteins that form tight junctions between epithelial cells. Therefore, any organ that relies on tight junctions will be negatively affected if their synthesis is impaired. If the intestinal lining is to properly absorb nutrients while leaving behind undesired materials, free diffusion cannot be permitted between the digestive ECM and the lumen of the digestive tract. Tight junctions seal the gaps between epithelial cells in the intestine, allowing cells to selectively control what passes through the epithelium through the use of transmembrane transport proteins. Without tight junctions, nutrient concentrations would equalize due to diffusion through the epithelium, vastly decreasing the efficiency of the GI tract.

A well-fed person with type 1 diabetes receives an insulin injection in the hospital. As a result of this treatment, levels of all of the following will decrease EXCEPT:

intracellular pyruvate. B is correct. Insulin, a peptide hormone, helps cells take up glucose from the bloodstream. Type 1 diabetics lack the ability to properly produce this hormone. Thus, giving a dose of insulin to this patient would promote an influx of glucose into cells that were previously unable to gain it from the blood. Because these cells were improperly acquiring and utilizing glucose before, we can infer that they were not producing ATP at a maximal rate. The insulin treatment allows them to take in glucose and produce ATP through glycolysis, while also stimulating the citric acid cycle and the electron transport chain. Since pyruvate is a glycolytic product, its levels should increase after insulin administration.

When an alkene is exposed to HBr, the anti-Markovnikov product:

involves the bromine atom adding to the less substituted end of the alkene, and occurs during radical reactions. C is correct. When an alkene is reacted with hydrobromic acid, the Markovnikov product typically forms. Since this product includes the halogen bound to the more substituted end of the bond, it tends to be especially stable. Radical reactions, particularly those initiated by peroxides, form an exception to this rule. In these processes, hydrogen adds to the less substituted end, while the bromine ion adds to the other position.

IR spectroscopy involves the exposure of a sample to relatively low-energy infrared rays. Absorbance is monitored as a function of the incident energy. In this technique, the incident light:

is absorbed directly into chemical bonds. IR involves absorption of incident light directly into chemical bonds, a process that occurs at specific wavelengths depending on the chemical environment of the bond

The amino acid with the chemical formula C5H11NO2S:

is not able to form disulfide bonds with other residues, because it does not possess a free thiol group. The given formula is that of methionine, a residue that contains a thioether functionality as part of its side chain. Since it does not possess a free -SH group, it cannot establish disulfide linkages with the R groups of other amino acids.

Elemental nitrogen does not exhibit an IR spectrum because it:

is symmetrical. While infrared spectroscopy is an enormously complex technique, it basically measures the way in which the dipole moment of a bond vibrates over time. Since a diatomic molecule that is composed of two identical atoms (like N2) does not have a dipole, it gives no signal.

The three techniques used for brainwashing identified by McConnell are:

isolation, dependency, and reward. McConnell recognized that brainwashing techniques resembled the process of shaping used in operant conditioning. The target behavior is for the subject of brainwashing to do what the authorities say. For example, if prisoners are kept from each other to prevent the psychological support that enables resistance to propaganda, and if they are systematically rewarded for small changes in behavior, many can be led to change their ways in order to achieve the ultimate objective of rejoining society.

Consider the figure below. Curve A most likely represents:

isothermal compression. Here, the volume is decreasing, so Curve A must relate to some form of compression. Isothermal curves (which denote processes conducted under constant temperature) are generally less steeply-sloping than adiabatic curves.

Eukaryotic DNA polymerase requires a free 3' -OH group, which is provided by a short RNA strand synthesized by primase enzymes. This -OH group is necessary because:

it binds to DNA polymerase, which otherwise cannot initiate synthesis. DNA polymerase is incapable of starting initiation, and instead is only able to elongate existing strands. Without a primer, DNA polymerase would have no initial position to bind to.

genetic drift truths

it can lead to loss of alleles from the population, it can lead to alleles becoming fixed in a population, and it is the result of random allele segregation from parents.

Which of the following statements are true of SiO2? I. It is a molecule. II. It is a compound. III. 7 moles of it contains 3.5 moles of oxygen. IV. When mixed with water, one mole of it yields 3 ions.

it is a molecule it is a compound

All of the following statements regarding reverse transcriptase are false EXCEPT:

it is carried within the retroviral capsid and released into the cytosol following viral penetration. Reverse transcriptase must be carried intact within the capsid in order to process the RNA genome. Without reverse transcriptase, the RNA genome cannot be converted into DNA.

The conversion of pyruvate to acetyl-CoA is mediated by the enzyme pyruvate dehydrogenase. Which of the following equations relates to the rate in which the ES complex is formed? (Note: [PD]t = total concentration of pyruvate dehydrogenase, [PD-Y] = concentration of the pyruvate dehydrogenase complex bound to pyruvate, [Y] = concentration of pyruvate, [A] = concentration of acetyl-CoA, k1= rate constant for PD-Y formation from pyruvate and the pyruvate dehydrogenase complex, k-1= rate constant of the reverse reaction)

k1([PD]t - [PD-Y])[Y] C is correct. The rate of the reaction is dependent on the concentration of the reactants and the rate constant, which in this case is k1. As the pyruvate dehydrogenase complex is created and bound with substrate, the amount of unbound enzyme and substrate decreases. Therefore, [PD]t - [PD-Y] is equal to the concentration of unbound enzyme [PD]. The rate at which [PD-Y] forms, then, is equal to k1([PD]t - [PD-Y])[Y].

In his classic studies of conformity, Asch demonstrated that:

lack of unanimity greatly reduces the pressure to conform. Studies show that when there is little to no consensus (unanimity) in a group, it is easier to form an independent opinion.

Compared to its analogous acyl chloride, a carboxylic acid is considered:

less electrophilic due to resonance. Both chlorine and oxygen are technically capable of electronic delocalization. However, the resonance structure in which the oxygen atom forms a double bond with the carbonyl carbon is more significant than the corresponding structure in which Cl forms the same double bond. Thus, although both atoms are very electronegative, the dominant resonance contribution is more significant for the oxygen species. The resulting increase in electron density around the carbonyl carbon confers decreased electrophilicity.

A chemist fills two separate beakers, one with 70% ethanol and the other with 70% ethanethiol. If subject to standard conditions, both beakers will have a pH of:

less than 7, with ethanethiol having the lower pH of the two. Both ethanol (a two-carbon alcohol) and ethanethiol (a two-carbon sulfur-containing molecule) are acidic. In fact, the only difference between the two is that ethanethiol has an S atom bound to its acidic proton. Since sulfur is much larger than oxygen, the conjugate base of ethanethiol is better able to delocalize negative charge, increasing its stability. Remember, the more stable the conjugate base, the stronger the acid.

on a run process graph what is the intermediate

less than a transition state but greater than the product

During the course of routine hospital blood work, a sample of whole blood is spun down in a laboratory centrifuge. The resulting centrifuge tube is shown below. An inquiring physician notices enlarged, abnormally-shaped cells in Layer 2. The patient from whom this blood was drawn is most likely suffering from:

leukemia, a cancer affecting stem cells responsible for leukocyte production.

While standing in a packed bus, a child in the back thinks it is funny to nudge the person in front of him. That person stumbles forward a bit, then returns to his previous position. However, the damage is done; when the passenger fell forward, he nudges the person in front of him, and in this way, a wave of stumbling proceeds to the front of the bus. This wave could best be characterized as a:

longitudinal wave. This wave's oscillation is forward-and-backwards in amplitude, while its overall direction of propagation is forward. Because the amplitude of oscillation is parallel to the direction of the wave's motion, it is a longitudinal wave.

A.K., an individual described in an advanced neurobiology textbook, can easily detect color and the shapes of objects, but not light and dark nor motion. Of the choices below, the most logical conclusion is that A.K. has an impairment of his:

magnocellular cells and rods. D is correct. Rods are rhodopsin-containing photoreceptors responsible for perceiving light and dark, while magnocellular cells are neurons in the lateral geniculate nucleus (LGN) that detect motion. Of course, photoreceptors are found in the retina while the LGN is part of the thalamus, so few patients would suffer damage to both; even so, D is the best answer.

All of the following statements regarding sects are true EXCEPT:

members must practice their beliefs in controlled, isolated settings. A sect is a collection of individuals with distinct, often extreme religious beliefs. Typically, sects arise by "splitting off" from a more mainstream, larger religion. While we tend to think of sects as similar to cults, which often exist in isolation and may involve coercion, this is not necessarily the case.

In an ideal study, the experimental protocol should be designed to:

minimize confounding variables. Confounding variables are factors that stem from the failure to properly control an experiment. For example, imagine that researchers are examining the effect of Thing A on Thing B. If they completely neglect to consider Thing C, which also has an effect on Thing B, it may lead them to an incorrect conclusion regarding Thing A. Therefore, a protocol should always strive to remove or lessen the effect of these variables.

Typically, the dissolution of a lipid in water is highly unfavorable. With this in mind, the entropy change associated with dissolving a phospholipid in aqueous solution is:

more positive than the entropy associated with dissolving a cholesterol molecule. Phospholipids are amphipathic, meaning that they contain both hydrophobic and hydrophilic regions. For this reason, a phospholipid will interact more favorably with water than will entirely hydrophobic molecules such as cholesterol or triacylglycerols. A positive (or less negative) change in entropy is a hallmark of a more favorable dissolution. In other words, since the solvation of a phospholipid is more favorable than the solvation of a cholesterol molecule, it will have a more positive entropy.

Jon conducts an experiment in his biochemistry class. First, he denatures Enzyme A under stringent conditions; afterwards, he removes these conditions and observes that the enzymatic activity of the protein is regained. Jon concludes that protein conformation is:

more than one of the above. Jon's procedure parallels Anfisen's famous experiment with ribonuclease. Even though the protein is first denatured, causing it to lose its higher-level structure, its enzymatic activity is able to be restored once harsh conditions are removed. This indicates that a protein's conformation may be recovered even when only its primary structure is still present. However, C also represents a true statement. This protein was denatured in a certain environment (likely involving exposure to urea or guanidinium hydrochloride) and renatured in another.

Following fertilization of an ovum by a sperm cell, four cell divisions occur with virtually no cell growth. The resulting sixteen-cell ball is known as the:

morula. The morula is a spherical collection of sixteen cells that is the same size as the original zygote. Further division results in the blastula, a hollow ball of cells. The blastula then develops into the gastrula, which is an invaginated structure containing three germ layers.

nondeclarative memory can relate to the

motor cortex

When beginning your first day of surgical rotation, both you and a colleague answer one of the attending physician's questions incorrectly. All day, you remain frustrated at yourself for being "too stupid" to answer, but feel terrible for your colleague because she "just happened to get the hardest question." In making these judgments, you are falling victim to:

neither the fundamental attribution error nor the self-serving bias. The fundamental attribution error refers to the common tendency to blame others' actions on dispositional, rather than situational, factors. For example, if you had concluded that your colleague hadn't studied enough and was therefore a lazy person, that would constitute a dispositional attribution. Interestingly, the self-serving bias is the virtual opposite of this tendency. When bad things happen to us, we generally attribute them to situational factors ("bad luck," etc.) as opposed to dispositional ones. Here, you cited a situational factor to explain another person's actions while attributing your own failure to a dispositional one; this is the exact opposite of what both of these biases would predict.

Tuberculosis begins with colonization of the lungs by the bacterium M. tuberculosis. The lungs contain billions of alveolar macrophages that normally control the spread of airborne pathogens. M. tuberculosis lives within these macrophages, but is unable to enter alveolar epithelial cells. Consequently, a rapid and robust macrophage-mediated response accelerates the spread of the infection. In vitro, the bacterium would be most infectious to:

neutrophils. If the bacterium can enter macrophages but not other somatic cells, it likely capitalizes on the natural entry mechanism offered by phagocytosis. Although T cells and endothelial cells can phagocytize under certain circumstances (as can alveolar epithelial cells), neutrophils are professional phagocytes and would be rapidly infected by the bacterium.

All of the following are true regarding the relationship between carbon and nitrogen EXCEPT:

nitrogen has a greater electron affinity than carbon. A is correct. Electron affinity relates to an element's desire to gain an electron. For the majority of elements, this quality increases as you move upwards and to the right on the periodic table. However, the nitrogen-containing group is an exception to this rule. Carbon has an electron configuration of [He]2s22p2, so the addition of another electron would give carbon a half-filled p orbital and is thus highly favorable. Nitrogen, on the other hand, has an electron configuration of [He]2s22p3. Addition of another electron would alter nitrogen's half-filled valence state and produce a partially-filled orbital. Therefore, nitrogen will have a lower electron affinity than expected, making this choice more likely than the others.

When trying to retrieve the name "Abraham Lincoln" from memory, we might first think of "government," then "president," then "George Washington," etc. Within the concept of spreading activation, the word "government" in this example is a:

node. . A is correct. Long-term memories are thought to be organized within interconnected webs called semantic networks. In these networks, each distinct concept or term exists as a node. When information needs to be retrieved, certain nodes are activated first, and this "memory cascade" spreads to adjacent nodes in a process known as spreading activation.

Jackie M., a patient in a case study, has significant trouble locating the sources of sounds. Additionally, she cannot focus her vision on a specific point in space while rotating her head. Which of these statements is likely true of Jackie M.? I. Her auditory cortex is damaged, since she cannot localize sounds. II. Her superior olive is damaged, since she cannot fixate on a point when rotating her head. III. Her inferior colliculus is damaged, leading to her inability to detect the locations of sounds. IV. Her semicircular canals are damaged, creating the issue with sound localization.

none are true

A molecular biologist discovers an enzyme in insects, which she calls Protein F. She runs a sample through an SDS-PAGE gel and observes a single dark band. This biologist is able to deduce that:

none of the above. The presence of a single dark band could be consistent either with the presence of multiple subunits with equal masses (which would then be impossible to distinguish on a gel) or with the idea that Protein F is composed of a single subunit. The first of these conclusions is represented in choice C, and the second is described in choice A. Since both of these scenarios are possible, the biologist cannot draw any one conclusion for certain.

An individual hears a loud tapping noise due to construction outside his building. At first, he jumps each time he hears the noise, and his heart rate and breathing increase. After two days, he barely responds to the noise at all. This process is:

not a form of associative learning. B is correct. The process occurring here is habituation, or the decrease in response during repeated exposures to the same stimulus. Both habituation and its opposite, sensitization, are types of nonassociative learning.

A chiral molecule in solution is found to rotate light in the clockwise direction. This species successfully undergoes a unimolecular nucleophilic substitution reaction. The product solution will rotate plane-polarized light:

not at all. In an SN1 reaction, the original stereochemistry of the molecule is lost when the carbocation intermediate (a planar structure) is formed. Thus, the product is present as a racemic mixture. Such a mixture includes each enantiomer in a 1:1 ratio, meaning that it promotes no net rotation of polarized light.

A seven-year-old child loves playing with her older brother's science kit, which she found on her own and experiments with for hours each day. If her parents want to reinforce this behavior, they should:

offer occasional praise, but mainly leave her alone with the kit. A is correct. This child is already intrinsically motivated to play with the science kit, as evidenced by the fact that she does so for hours on her own with no outside rewards. Research has shown that offering rewards (extrinsic motivation) to people who are already intrinsically motivated actually makes them less willing to engage in the task. For this reason, leaving the child alone is preferable to rewarding her in any predictable way.

Part of the replication fork on a eukaryotic dsDNA molecule is shown below. At which point does the lagging strand of the new DNA molecule begin to form?

on the ope 3' end

Part of the replication fork on a eukaryotic dsDNA molecule is shown below. At which position could the leading strand of the new DNA molecule be found?

on the open 5' end

If spatial inequality exists between two areas of the same city:

one area must have more accessible resources (such as stores, clean water, and medical facilities) than the other. Spatial inequality refers to the uneven distribution of resources between particular areas. High-income areas typically have easy access to grocery stores, educational facilities, hospitals, and businesses in general, while low-income neighborhoods lack many of these institutions. This cycle perpetuates itself, as wealthier individuals who move to the area naturally settle in the regions with more resources available.

While carbon dioxide exhibits four vibrational modes in space, its experimentally-obtained IR spectrum shows no more than two distinct peaks. This is partly because:

one of the vibrational modes is not observable by IR spectroscopy. C is correct. The symmetric stretching mode of this molecule does not induce a dipole. It is thus effectively invisible to IR spectroscopy.

At a particular instant in time, an electron that is moving toward the right is held in equilibrium while experiencing two forces: one due to gravity and pointing straight downward, and one due to an external magnetic field. For this scenario to be logical, this field must point:

out of the page. For this question, use the right-hand rule. Since the electron is traveling with a rightward velocity vector, first point your thumb toward the right. Next, because the magnetic force exists in equilibrium with the downward-pointing gravitational force, the magnetic force must be exerted in the upward direction. To account for this, ensure that your palm is facing upward. At this point, your fingers should point into the page; however, this rule is typically used for positive charges, so you must switch this final direction. The magnetic field must point out of the page instead.

Unlike SN1 reactions, which require at least two steps, SN2 reactions can proceed through a single transition state. This high-energy species is best described as:

pentavalent, because the nucleophile approaches from the side opposite the leaving group. A is correct. As a result of steric constraints imposed upon the geometry of the transition state, the nucleophile always approaches the electrophile from the side opposite the leaving group. (This is why you may have heard an SN2 reaction described as a "backside attack.") Since none of the orbitals are empty (all are occupied by atoms or lone pairs), the transition state reaction center is transiently coordinated by five different substituents.

Monobromination of an aromatic ring is best facilitated by using:

phenol and a nonpolar solvent. Benzene, alone, does not react well with Br2. In contrast, phenol contains an activating -OH group. Presence of this substituent favors the formation of a brominated product. Additionally, nonpolar solvents are more effective when attempting monosubstitution, while polar solvents are typically used for polysubstitution.

During the first weeks after fertilization, hCG levels can increase by a factor of over 10,000. However, excessively high hCG has been associated with multiple complications. Endometrial cells treated with high concentrations of the hormone display excessive apoptosis of CD8+ T cells. Pregnant females with abnormally high hCG are most likely at risk for the development of:

placenta percreta, a condition in which the placenta invades the uterine myometrium.

A fatty acid that contains three carbon-carbon double bonds, as well as one carbon-oxygen double bond, is termed a:

polyunsaturated fatty acid. A is correct. An unsaturated fatty acid is one that contains one or more multiple bonds, meaning that it is "unsaturated" with hydrogen atoms. Since this molecule includes three double bonds, it is polyunsaturated.

The neuronal voltage-gated sodium channel has been extensively studied. A diagram of this transmembrane protein in its resting state is shown below. The channel has four separate transmembrane domains, each consisting of six membrane-spanning helices. Specific motifs are highly conserved and functionally relevant, namely the P-loops, S-loops, and I-loops. Elimination by hydrolytic cleavage of the S-loops permits free passage of sodium ions through the channel even in the absence of any change in membrane voltage. In contrast, substitution by mutagenesis of S-loop residues with alanine results in a channel that does not respond to voltage changes, but that also cannot be induced to conduct ions. Based on this information, the amino acid residues in the S-loop are most likely:

positively charged, which results in a conformational change during depolarization that shifts the S-loop toward the extracellular side. A is correct. The figure shows that the S-loop enters the channel from the extracellular side, which necessitates movement back in the extracellular direction in order to prevent occlusion of the pore. In other words, depolarization must cause the loop to shift outward and expose the open channel. Since depolarization involves a positive shift in transmembrane voltage, positive residues will be repelled away from the cytoplasm. For this reason, the S-loop must be primarily positively charged.

The reaction of ammonia with butanoic acid produces a(n):

positively-charged amine. This is simply an acid-base reaction. Butanoic acid will lose its proton to ammonia, which will become protonated to form an ammonium ion.

modification refers to chemical alterations made to completed proteins. Novel antibody sequences are determined well before translation occurs.

post translational modification

A member of the species Aplysia californica is being observed by a team of scientists. They poke the animal in its siphon, causing it to pull both its gill and siphon away. After twenty such stimuli, the animal no longer responds. The researchers could restore the gill- and siphon-withdrawal responses by:

presenting the animal with a mild shock, causing dishabituation. D is correct. Aplysia californica is a species of sea hare studied by Eric Kandel and his team in their famous experiments on non-associative learning. Habituation refers to a decrease in response after the same stimulus is administered multiple times. Since we wish to restore the original withdrawal response, we want dishabituation to occur. Shocking the animal, or presenting another new stimulus, can bring the previously-habituated response back to a stronger level.

Two years ago, a biology student had a terrible professor who taught him a large number of incorrect facts. As a result, he currently struggles to learn the enzymes involved in glycolysis. This situation exemplifies:

proactive interference, since previously learned information is impairing his ability to learn new facts. Proactive interference occurs when information from the past interferes with the encoding and retrieval of new memories. Here, the old poorly-taught material is hindering this student's ability to memorize new concepts.

is a means to induce an individual with added stimuli to perform a desired behavior.

prompting

In contrast to conventional X-ray crystallography, which provides information about inert systems, variations of NMR spectroscopy are frequently used to follow molecular processes in real time. For example, heteronuclear single quantum coherence (HSQC) may be employed to examine the binding of a small organic molecule to its protein target. HSQC probes the nuclear spin of a heteronucleus, typically nitrogen, after magnetic excitation of a nearby hydrogen nucleus. This interaction between these two nuclei, known as the nuclear Overhauser effect, is most likely:

propagated through space. C is correct. We are told that HSQC is useful for exploring ligand-binding interactions, which typically do not involve the formation of new covalent bonds. In other words, the ligand and protein remain chemically separate. Nevertheless, HSQC is able to return usable data concerning the process being measured; this is only possible if the interaction occurs through space.

Endomitosis is a phenomenon in which cells properly replicate their chromosomes but experience prematurely-terminated mitosis. As a result, the replicated chromosomes are retained within the original nucleus. In these cases, mitosis must arrest during:

prophase. The question states that the replicated chromosomes are retained in the original nucleus. For this to be possible, either the nucleus was always intact, or the chromosomes have not migrated and it can easily re-form. The only mitotic phase that occurs before the chromosomes begin to migrate is prophase.

A urological researcher is able to extract minute samples from particular regions of a mammalian nephron. One particular sample, Sample A, contains cuboidal epithelial cells that appear to be lined with microvilli. The most likely origin of Sample A is the:

proximal convoluted tubule. Simple cuboidal epithelia are well-suited for solute transport and absorption. In addition, the presence of microvilli should remind you of the small intestine, where nutrient absorption is a main function. Of the choices given, the only region of the nephron that is heavily involved in absorption is the PCT. Cells lining this structure must reabsorb glucose, sodium, amino acids, and other vital solutes while also permitting the secretion of ions and toxins.

When adenylate cyclase reacts with ATP, cyclic AMP is formed. What is/are the other product(s) of this reaction?

pyrophosphate If it is known that ATP goes to AMP, two phosphate groups must be lost; thus a pyrophosphate (or double phosphate group) is the side product of this reaction.

converts pyruvate into acetyl-CoA. This process occurs in the mitochondria.

pyruvate dehydrogenase

Which molecule is produced by RNA polymerase I?

rRNA RNA pol I transcribes certain genes into rRNA (ribosomal RNA).

A mixture of reagents K, L, and M react in one bimolecular step to yield product P. The rate law for this reaction CANNOT be:

rate = k[L][K][M]. C is correct. The question stem mentions that this reaction is bimolecular, which means that it must have an overall reaction order of 2. The rate law given in choice C is trimolecular, which would make it third order overall. Note that to find overall reaction order, simply add the exponents of all reactants involved in the rate law.

Compared to UV-Vis spectroscopy, IR spectroscopy is:

reliant on the use of lower-energy radiation. Read this question carefully to note that it is asking for a trait of IR spec, not UV-Vis. As their names imply, IR spectroscopy utilizes infrared radiation, while UV-Vis uses ultraviolet light. Infrared radiation has a lower frequency, longer wavelength, and lower energy than UV rays.

An optometrist wishes to construct the most powerful converging lens that he can. In other words, he wants to create a lens that refracts, or bends, rays toward each other as efficiently as possible. To do so, he should:

replace a typical glass lens (n = 1.5) with a transparent material (n = 1.7) that is slightly thicker. Though lenses can be intimidating, the basic principle is simple: they operate by refracting light. The greater the difference between the index of refraction of the lens material and that of the surrounding environment, the more the rays will bend. Here, originally, the optometrist used a glass lens in what we can assume is air, meaning that the respective refractive indices of the two media were 1.5 and 1.0. Raising the "n" of the lens to 1.7 will improve the strength of the apparatus, as will increasing its thickness, which will simply give light more medium through which it must travel at its bent angle.

A judge who is caught cheating on his income taxes will likely experience:

role conflict. Role conflict arises when a situation occurs in which a person is expected to play two incompatible roles, such as criminal and judge.

is the act of learning a subset of values and norms that apply to a specific group within society, like the military, a particular college, or prison. In this example, the child is not learning a behavior that applies only to a small group.

secondary socialization

A child is born to religious parents. During his childhood, he attends "Sunday school" seminary meetings, where he learns that his religion sprang from one that is currently endorsed by his nation's government. Eventually, he and his parents move to a small rural community in which all residents adhere to this religion. It is explained that, although their beliefs are not dangerous or even frowned upon by society, residents will be better off living in a community in which they will fit in. This child's religion is most likely a:

sect. A sect is a collection of individuals with distinct, often extreme religious beliefs. Typically, sects arise by "splitting off" from a more mainstream, larger religion. This description perfectly matches that which is given by the question stem.

Spaced repetition would most likely be used to encode information into:

semantic memory, since it involves the storage of facts and pieces of knowledge.

Anthony, a member of a college fraternity, tends to be impulsive and often earns poor grades. However, he believes that all of his friends see him as a successful, outgoing role model. According to Cooley's concept of the looking-glass self, Anthony will:

shape a more confident and positive sense of self, as he believes that others view him positively The concept of the looking-glass self states that we shape our identities based on our perception of the way others view us. In other words, regardless of Anthony's actual personality or of his friends' actual thoughts about him, since he believes that his friends view him as successful, he will feel like a successful person.

Recoding, chunking, and rehearsal are especially important for the improvement of:

short-term memory efficiency. These processes all involve combining smaller bits of information into larger and more meaningful units (chunking), practicing a behavior until it is held in memory (rehearsal), or rewriting the brain with new information; all of these strategies serve to increase the amount of information held in short term memory.

The golden ratio refers in part to:

the body proportions that human beings find most attractive. A is correct. Studies of human attraction have found that we gravitate toward people who possess the golden ratio, or the body and facial proportions that relate in a particular numerical way.

The figure below was shown to a number of participants in a psychology experiment. Identifying a square in the middle of this array of shapes demonstrates the Gestalt principle of:

similarity. The principle of similarity states that people tend to group objects that look similar into a pattern. In this case, though objects are scattered throughout the image, we perceive the hearts as forming a square due to their similar shape.

While the heart pumps blood to both the systemic and pulmonary circuits, coronary arteries supply blood to the heart tissue itself. A cardiothoracic surgeon isolates a sample of myocytes from the inner lining of one of these vessels. This sample most likely contains:

smooth muscle. The term "myocyte" means "muscle cell," so we simply need to identify the type of muscle that lines coronary vessels. Arteries (as well as arterioles, veins, and venules) contain a layer of smooth muscle in their walls. This allows for processes such as vasoconstriction and vasodilation.

A sample of non-coding RNA is isolated from a cell's nucleolus. The sample most likely contains:

snoRNA. Small nucleolar RNAs (or snoRNAs) are involved in the modification of rRNA. As such, they would be located in the nucleolus, where ribosomes are assembled.

Although white and minority populations are slowly trending towards equal pay, sociologists argue that the vast disparity in asset wealth in America will continue to define the wealth gap between white and most other populations. This pervasive lack of access to asset wealth is one example of:

social reproduction. D is correct. Social reproduction is the tendency for demographic facts to repeat themselves throughout successive generations. For example, if Jane's parents are poor and have unhealthy spending habits, she will more than likely learn the same behaviors and perpetuate a loop of poverty. The idea that certain minority groups will continue to possess less overall wealth than the white population relates perfectly to this intergenerational trend.

Consider the following reaction scheme. The reaction proceeds in three experimentally-observable steps. Note that stereochemistry is deliberately omitted from the figure and need not be considered. The second step of this process results in the formation of a high-energy intermediate. The hybridization and molecular geometry of this species can best be described as:

sp2 and trigonal planar. C is correct. This reaction utilizes an SN1 mechanism, which involves the formation of a high-energy carbocation intermediate. Since the carbocation formed during this particular process is substituted by three alkyl groups, it is tertiary; since it lacks lone pairs, we must only consider these three substituents when determining both its hybridization and its geometry. Therefore, this carbocation is sp2-hybridized with trigonal planar geometry.

Which of the following correctly describes the orbital hybridization of XeF4 and NH3, respectively?

sp3d2, sp3

increasing polarity on a TLC is shown by

spots closest to the top

In a certain population of wild coyotes, brown fur (B) is dominant over spotted fur (b) and approximately 80% of a given breeding population has brown fur. Over the course of two decades, a genetic change occurs such that nearly the entire population expresses the spotted fur phenotype. In this population:

spotted fur is the wild type. The term "wild type" refers to the traits an animal typically possesses when found in nature. This usually refers to a dominant trait, but not always. If nearly all of the coyotes now have spotted fur, then spotted fur is the wild type.

The cells and cell layers comprising the human epidermis can accurately be described as: I. simple. II. cuboidal. III. columnar. IV. stratified.

stratified

found in the citric acid cycle which takes place in the mitochondria.

succinate dehydrogenase

do not contain free carbonyl groups (either aldehydes or ketones), they are unable to undergo the nucleophilic attack by the lysine residue that is displayed above.

sucrose

Cancer cells frequently exist in an immunologically privileged environment within the body. This is accomplished by production of high levels of cytokines, including transforming growth factor (TGF). Of the following, the cells that are most likely activated by TGF are:

suppressor T cells. Activation of suppressor T cells triggers production of a cytokine cocktail that inactivates other immune cells, including the cytotoxic T cells and NK cells that are vital for tumor elimination.

Of the following, the strongest base is:

tert-butoxide. Tert-butoxide, the highly substituted conjugate of tert-butanol, is a very strong base. (Remember, the weaker the original acid, the stronger its conjugate base, and tert-butanol is notably weak.) This idea stems from the fact that the negative charge is fully concentrated on tert-butoxide's lone oxygen atom. Additionally, its alkyl substituents donate added electron density to that position, localizing the charge further and resulting in high reactivity.

Ba(NO3)2 (aq) + H2SO4 (aq) 🡪 BaSO4 (s) + HNO3 (aq) All of the following are true of this reaction EXCEPT:

that it is a redox reaction. B is correct. This is not a redox reaction, as no atoms change oxidation states at all. Barium has a +2 state throughout the reaction, nitrogen is +5, hydrogen is +1, oxygen is -2, and sulfur is +6.

The three major components of the self-discrepancy theory are:

the actual self, the ideal self, and the ought self. C is correct. The self-discrepancy theory includes the actual self (how we perceive ourselves to be in reality), the ideal self (how we wish we could perceive ourselves), and the ought self (how we think others wish they could perceive us).

Facial expressions appear to be more highly conserved between species than body language. One behavior that exemplifies this trend is:

the baring of teeth conveying aggression and imminent attack in a number of species. D is correct. The question stem asks for two things: a facial expression and a behavior conserved across species. The baring of teeth fits both of these requirements, as it is an expression that many species use as a sign of aggression.

Antidiuretic hormone serves to increase water reabsorption by inducing translocation of aquaporins into certain cells. A scientist is monitoring the change in number of aquaporins during this process. He would most likely focus his study on:

the collecting duct

After a stroke, a formerly confident and decisive woman has difficulty making even the simplest choices, such as deciding which pair of socks to wear in the morning, though she otherwise seems to have no other psychological deficits. This stroke most likely affected:

the frontal lobe of her brain. A is correct. The frontal lobes are involved in executive functions like decision-making.

In a certain individual, a genetic error led to a translocation in which a segment of the p arm on chromosome 14 was broken off and attached to the q arm of chromosome 16. The translocation created no observable changes in the individual's traits. For the genes located on the translocated segment:

the loci have changed but the alleles have remained the same. A is correct. A locus (plural: loci) is simply a location where a gene is found. Moving the genes from one chromosome to another will change their loci. An allele is simply a given variant of a given gene (e.g. H versus h). Nothing in the question suggests the genes themselves were changed.

Cholesterol biosynthesis is tightly regulated by the action of SREBP; when cellular cholesterol levels are high, this species exists in complex with two other proteins, SCAP and INSIG-1. Upon dissociation from its binding partners, SREBP is able to enter the nucleus and act as a transcription factor. SREBP most likely activates transcription of the gene(s) for:

the low-density lipoprotein (LDL) receptor, which has a high affinity for serum LDL.

The auto-ionization of water is an endothermic process. With this in mind, if the temperature of a beaker of pure water is increased:

the pH of the water will decrease. Think of auto-ionization in terms of equilibrium: heat + 2 H2O → H3O+ + OH-. The reaction will thus shift toward the product side in response to increased temperature. Interestingly, both [H+] and [OH-] will rise as a result, decreasing the pH and pOH to the same extent.

A psychologist tells her patient that he often employs the defensive impression management strategy of self-handicapping. The psychologist means that:

the patient creates excuses and obstacles in order to anticipate and mitigate poor performance. B is correct. Self-handicapping refers to a cognitive defense strategy in which people create excuses and "put themselves down" in an attempt to reduce the disappointment of themselves and others when (or if) they fail.

According to Piaget's stages of cognitive development, the emergence of symbolic thinking is a hallmark of:

the preoperational stage. . D is correct. Symbolic thinking is the ability to use symbols or other indirect representations to describe objects or ideas. This mode of thinking is closely related to imaginative play (for example, consider a child who uses pieces of paper to represent food while playing "house"). The preoperational stage, which lasts from around ages two to seven, is marked by the development of symbolic thinking. It is also qualified by egocentrism and centration, or the tendency to focus on one aspect of a concept and neglect all others.

A protein from the urine of a chronically ill patient was isolated and run on an SDS-PAGE gel. The result displayed on the gel was a long smear instead of a crisp band. These results can most likely be attributed to the fact that:

the protein was partially degraded. Only partial degradation would account for the smear described by the question stem. This would break the protein into a very large number of lighter-colored bands, which would appear to blend together on the polyacrylamide gel.

A daycare teacher keeps all of her fun toys and colorful blocks in a closet near the back of the room. When playtime is about to start, she turns around abruptly to go open the closet door. After two months, the toddlers in the class look up and become more active whenever the teacher turns around for any reason. During at least part of this scenario:

the sight of the teacher turning around is a conditioned stimulus.

A series of scans revealed a correlation between the development of a specific region of the brain and a child's ability to understand syntax. This finding would best support the notion of a language acquisition device as described by the:

the social interactionist theory of language development.

A student has a sample of an undetermined solid hydrocarbon. He observes that increasing the temperature by 12 K converts the sample to liquid, while a 110-Pa decrease in pressure causes it to transition to vapor. The original conditions of the sample were likely closest to:

the triple point, since all three phases meet under those conditions.

A chef is attempting to listen to a football game while simultaneously cooking during peak dinner hours. The chef will succeed in dividing his attention if:

this combination of tasks does not require more attentional resources than he has. C is correct. The resource model of attention proposes that we are capable of dividing our attention (or "multitasking") as long as our total attentional resources exceed those required by the combined tasks at hand.

Shortly after engulfing bacteria, macrophages undergo a "respiratory burst" characterized by substantial ATP production in a matter of milliseconds. This metabolic activity is so intense that macrophages usually die after engulfing 40-50 bacteria. The purpose of the respiratory burst is most likely:

to provide energy for the rapid acidification of phagosomes via proton pumps. Bacteria must be promptly exposed to acidic conditions following phagocytosis. Otherwise, they could simply replicate within the phagosome and cause lysis of the immune cell. Macrophages can acidify their phagosomes to a pH approaching 1.5, which is extremely energetically demanding. As protons are pumped into the vesicles, the increased concentration of positive charge makes subsequent loading more difficult.

An unknown molecule, which has the ability to form multiple stable oxidation states, tends to exist in brightly-colored compounds. This species is most likely a:

transition metal. D is correct. Two of the most well-known chemical hallmarks of transition metals are their ability to possess multiple oxidation numbers and their tendency to form brightly-colored compounds. This second characteristic stems from the arrangement of their d electrons.

The molecular geometry of carbonate ion is:

trigonal planar. First, draw the Lewis structure of the carbonate (CO32-) anion. From this, we see that the central carbon forms one C=O and two C-O bonds with surrounding oxygen atoms. Since there are three substituents and zero lone pairs, the geometry of this molecule is trigonal planar.

A decline in the agricultural sector of a coastal African country will most probably result in:

urbanization and the formation of slums as citizens migrate to cities. C is correct. As the country's agricultural industry suffers, more unemployed citizens will be forced to leave rural areas and move to cities in search of work. This is the very definition of urbanization. Unfortunately, as larger and larger numbers of relatively impoverished people move in from the farmland, limited resources will necessitate the formation of slums. These are simply the poorer, more run-down and generally crowded regions of large cities.

If a student working with this mechanical system needed to halve the puck's velocity without lengthening the string, she should:

use a heavier puck. C is correct. Angular momentum, or mvR, should be conserved in this situation. Here, we are already told that changing R (the string's length) is not an option. For this reason, we must turn to m, or the puck's mass. If the student doubles the mass of the puck, she will decrease its velocity by a factor of 2.

A salesman hopes to use impression management techniques to increase his sales of a probiotic pill. If he specifically decides to utilize altercasting, he could:

use a statement like "As a health-conscious person, you should be very interested in probiotics." The altercasting technique involves projecting an identity on another individual in a way that is congruent with one's own goals. This is a manipulative method that increases the likelihood that the listener will act to match the projected identity. Here, the salesman projects a "health-conscious" identity onto the buyer, who may or may not care about health at all.

To win a card game, students must memorize all five of the playing cards shown below in a five-minute span. They must be able to correctly list the cards in order from left to right, and must accurately name the type of card (Jack, Ace, Queen, etc.), the color, and the symbol in the center. John spends the entire five minutes repeating the details of the cards to himself over and over. He later finds that he can hardly remember a single detail. John could have encoded this information more efficiently by doing all of the following EXCEPT:

using elaborative rehearsal to commit the information to prospective memory. D is correct. While elaborative rehearsal is an effective method to aid encoding, this information is not being committed to prospective memory. Prospective memory involves remembering that you need to do something in the future, and has nothing to do with memorizing details or facts.

Suppose the ego were a hungry person in a crowded cafeteria. The ego would most likely:

wait in line. The ego operates according to the reality principle, working out realistic ways of satisfying the id's demands, often postponing satisfaction to avoid negative consequences of society. The ego considers social realities and norms, etiquette, and rules in deciding how to behave.

isolation, dependency, and reward. McConnell recognized that brainwashing techniques resembled the process of shaping used in operant conditioning. The target behavior is for the subject of brainwashing to do what the authorities say. For example, if prisoners are kept from each other to prevent the psychological support that enables resistance to propaganda, and if they are systematically rewarded for small changes in behavior, many can be led to change their ways in order to achieve the ultimate objective of rejoining society.

with an envious stereotype. The stereotype content model sorts relationships along two scales: competence and warmth. When two individuals are part of different in-groups, they tend to perceive each other with either low or high competence, meaning that they either consider them incapable and worthy of pity or capable and competitive. As Jackson is actively fighting with Jay on the tennis court, he certainly would view him as highly competent. Additionally, since the two boys are no longer close friends, we can assume that Jackson sees Jay with low, not high, warmth. These factors combine to produce an envious stereotype.

Suppose the electrical potential of a certain membrane were determined entirely by the relative concentrations of potassium ions on either side. If the value of [K+] both inside and outside the cell were cubed, in comparison to before, the membrane potential:

would increase by a factor of 3. The cell membrane potential relates to the ion concentrations by way of the Nernst equation, which can be written as Vm = 2.3026 [(RT / zF) * (log (Pc[Co+] + PA[Ai-]) / (Pc[Ci+] + PA[Ao-])], where R is the gas constant, T is temperature, z is the effective charge of the ions involved (usually 1), F is Faraday's constant, and P is the permeability of the relevant type of ion. Note also that [Co+] represents the concentration of various cation species outside the cell, while [Ai-] denotes the concentration of anions inside. Assuming that the membrane potential is determined by a single species of cation (K+), the anion terms cancel, as does the permeability factor for K+. Finally, cubing the concentrations inside and outside the cell allows us to resolve the problem using log identities. Log ([Ko+]3 / [Ki+]3) = log [([Ko+] / [Ki+])3] = 3*log ([Ko+] / [Ki+]). The value of the potential must therefore be tripled compared to its prior value.

A chemist attempting a complex synthesis procedure succeeds in creating the molecule shown below. The chemist then treats the compound with methyl iodide in the presence of sodium hydride, effectively protecting the naked hydroxyl groups. She then combines the compound, under sufficiently basic conditions, with another modified sugar that is globally protected except at its 6-position. Following displacement of the azide group, the resulting glycosidic linkage between the two sugars will be:

β-1,6. The linkage will result from the SN2 displacement of the azide group by the naked hydroxyl on the other sugar molecule. This constitutes a bond between carbon 1 on the illustrated sugar and carbon 6 on the sugar that is not shown. Since SN2 reactions proceed with an inversion of stereochemistry, the β anomer will predominate. Note that, as this reaction is conducted under basic conditions, anomerization is unlikely to occur spontaneously.


Set pelajaran terkait

HS 10: Enlightenment/enlightened despotism

View Set

AP World History: Reading Quiz (pgs. 50-72)

View Set

Entire Audit Set - Just one answer

View Set

Unit 17 Human Rights and Fair Housing

View Set

Sat Reading type questions/strategies

View Set

Writing Process & Unit Assignment

View Set